BOC practice test

अब Quizwiz के साथ अपने होमवर्क और परीक्षाओं को एस करें!

d (Morphological identifiable perinuclear halo.)

1) The light-colored zone adjacent to the nucleus in a plasmacyte is the: a. ribosome b. chromatin c. mitochondria d. Golgi area

d (Calculation of RBC indices.)

107) Which of the following is the formula for MCH? a. Hct / (RBC x 1000) b. Hgb / Hct c. RBC / Hct d. (Hgb x 10) / RBC

d (Normal bone marrow differential.)

133) In normal adult bone marrow, the most common granulocyte is the: a. basophil b. myeloblast c. eosinophil d. metamyelocyte

c (Hypersegmented neutrophils in pernicious anemia.)

144) The white cell feature most characteristic of pernicious anemia is: a. eosinophilia b. toxic granulation c. hypersegmentation d. atypical lymphocytes

d (Immunoglobulin structure; IgM is a pentamer,)

146) The IgM molecule is a: a. dimer b. trimer c. tetramer d. pentamer

d (Classic feature discriminates AML from other types of erythroid/ myeloid metaplasia.)

150) Auer rods are most likely present in which of the following? a. chronic myelocytic leukemia b. myelofibrosis with myeloid metaplasia c. erythroleukemia d. acute myelocytic leukemia

b (This is a ratio calculation. 3/100 = 6/x. Solve for x.)

157) How many mL of a 3% solution can be made if 6 grams of solute are available? a. 100mL b. 200mL c. 400mL d. 600mL

a (Lab findings are reflective of erythroid rather that myeloid metaplasia.)

168) Biochemical abnormalities characteristic of polycythemia vera include: a. increased serum B12 binding capacity b. hypouricemia c. hypohistaminemia d. decreased leukocyte alkaline phosphatase activity

a (Visual vs ultraviolet wavelength.)

192) Which of the following wavelengths is within the ultraviolet range? a. 340 nm b. 450 nm c. 540 nm d. 690 nm

a (Nanometer is the unit for wavelength.)

196) The nanometer is a measurement of: a. wavelength of radiant energy b. specific gravity c. density d. intensity of light

a (Tau transferrin is found only in CSF.)

198) The tau isoform of transferrin is a carbohydrate deficient protein found only in: a. CSF b. sweat c. amniotic fluid d. semen

a (Glycated hemoglobin directly related to life of RBC.)

20) A patient with hemolytic anemia will: a. show a decrease in glycated Hgb value b. show a increase in glycated Hgb value c. show little or no change in glycated Hgb value d. demonstrate an elevated Hgb A1

b (Morphology of a lymphocyte.)

231) In the image below, the small nucleated cell seen in the lower left corner is a: a. polychromatophilic normoblast (rubricyte) b. mature lymphocyte c. plasma cell d. lymphoblast

a (Morphology of reactive lymphocytes.)

237) The most characteristic morphologic features of atypical lymphs include: a. course nuclear chromatin and basophilic cytoplasm b. blue-grey cytoplasm, fine nuclear chromatin c. nucleoli and deep blue RNA-rich cytoplasm d. a stretched nucleus and cytoplasmic indentations

c (Match the color to the pigment. Bilirubin is amber. Myoglobin is red, like hemoglobin. Homogenistic acid will produce a dark urine on standing.)

24) Red urine may be due to: a. bilirubin b. excess urobilin c. myoglobin d. homogenistic acid

d (A goal targets the purpose of an educational unit. The objective specifies what a learner is expected to know or do.)

240) In planning an instructional unit, the term goal has been defined as a: a. plan for reaching certain objectives b. set of specific tasks c. set of short and long term plans d. major purpose or final desired result

d (Association of traumatic arthritis with macrophages containing hemosiderin.)

241) In synovial fluid, the most characteristic finding in traumatic arthritis is: a. monosodium urate crystals b. cartilage debris c. calcium pyrophosphate dihydrate crystals d. hemosiderin laden macrophages

b (Tumor markers are useful for monitoring therapy, detecting recurrence and aiding in prognosis of tumors but are not useful for screening the general population for cancer.)

248) Clinical assays for tumor markers are most important for: a. screening for the presence of cancer b. monitoring the course of a known cancer c. confirming the absence of disease d. identifying patients at risk for cancer

d (Polyagglutination is a property of the red blood cells. Structures on the red cells are altered due to bacterial enzymes or a somatic mutation, so crypt antigens not normally exposed on cells are now present. Antibodies to the exposed structures are naturally occurring in adult plasma.)

248) Which one of the following is an indicator of polyagglutination? a. RBCs typing as weak D+ b. presence of red cell autoantibody c. decreased serum bilirubin d. agglutination with normal adult ABO compatible sera

a (Diagnosis of lactase deficiency.)

25) What is the best method to diagnose lactase deficiency? a. H2 breath test b. plasma aldolase level c. LDH level d. d-xylose test

d (Brucella and Pasteurella are aerobic, gram-negative bacilli, and Actinomyces is an anaerobic gram-positive bacillus. Bacteroides species are anaerobic, gram-negative bacilli.)

250) Which of the following genera include anaerobic gram-negative non-sporulating bacilli? a. Brucella b. Pasteurella c. Actinomyces d. Bacteroides

b (HIPAA patient confidentiality requirements state that patient data is secure from unauthorized access at all levels of communication of that data.)

251) The use of security systems such as firewalls and data encryption for electronic transmission of patient data from a laboratory information system to a remote location are required for: a. LOINC b. HIPAA c. ICD-9 d. CLIA

c (Since fungi grow more slowly than bacteria, a medium with antimicrobials is included to assist in the recovery of fungi. Chloramphenicol is an antibacterial agent active against Klebsiella and most other bacteria.)

260) A sputum specimen from a patient with a known Klebsiella pneumoniae infection is received in the laboratory for fungus culture. The proper procedure for handling this specimen is to: a. reject the current specimen and request a repeat culture when the bacterial organism is no longer present b. incubate culture tubes at room temperature in order to inhibit the bacterial organism c. include media that have cycloheximide and chloramphenicol added to inhibit bacterial organisms and saprophytic fungi d. perform a direct PAS stain; if no fungal organisms are seen, reject the specimen

c (Diseases causing respiratory acidosis.)

28) The following blood gas results were obtained: The patient's results are compatible with which of the following? a. fever b. uremia c. emphysema d. dehydration

a (A high-fat diet increases the serum concentrations of triglycerides. Fasting overnight for 10-14 hours is the optimal time for fasting around which to standardize blood collections, including lipids.)

282) Blood was collected in a serum separator tube on a patient who has been fasting since midnight. The time of collection was 7 am. The lab test which should be recollected is: a. triglycerides b. iron c. LD d. sodium

a (Bleeding time [BT] assesses platelet activity.)

294) A bleeding time is used to evaluate the activity of: a. platelets b. prothrombin c. labile factor d. Factor VIII

a (Diseases causing respiratory acidosis.)

30) An emphysema patient suffering from fluid accumulation in the alveolar spaces is likely to be in what metabolic state? a. respiratory acidosis b. respiratory alkalosis c. metabolic acidosis d. metabolic alkalosis

d (Once thawed, FFP is stored at 1-6C for up to 24 hours.)

30) Once thawed, FFP must be transfused within: a. 4 hrs. b. 8 hrs. c. 12 hrs. d. 24 hrs.

d (Treatment of acute hemolytic transfusion reactions focuses on supportive measures and control of DIC, hypotension, and acute renal failure.)

302) For a patient who has suffered an acute hemolytic transfusion reaction, the primary treatment goal should be to: a. prevent alloimmunization b. diminish chills and fever c. prevent hemoglobinemia d. reverse hypotension and minimize renal disease

a (Inhalation is a common mode of transmission for fungal spores. Specimens for fungal culture should be processed in a biological safety cabinet to minimize the potential for aerosol spread.)

303) In processing clinical specimens and fungal isolates, laboratory workers may contract systemic fungal infections through: a. inhalation b. ingestion c. skin contact d. insect vector

b (Normal ratio bicarbonate/carbonic acid.)

31) At blood pH 7.40, what is the ratio of bicarbonate to carbonic acid? a. 15:1 b. 20:1 c. 25:1 d. 30:1

a (Febrile nonhemolytic transfusion reactions are defined as fever of 1C or greater [over baseline temperature] during or after transfusion, with no other reason for the elevation than transfusion, and no evidence of hemolysis in the transfusion reaction investigation.)

310) A temperature rise of 1 C or more occurring in association with a transfusion, with no abnormal results in the transfusion reaction investigation. usually indicates which of the following reactions? a. febrile b. circulatory overload c. hemolytic d. anaphylactic

c (Protein electrophoresis pH.)

315) What is the proper pH for the buffered solution used to perform serum protein electrophoresis? a. 5.6 b. 7.6 c. 8.6 d. 9.6

d (Warfarin is a Vitamin K antagonist; coagulation Factors II, VII, IX, X, Protein C, and S are reduced as nonfunctional molecules are produced. The rate of reduction is based on the half-life of the factors.)

318) The results on a patient are: These results reflect: a. thrombophilia b. Factor IX deficiency c. heparin d. warfarin

d (Mycobacteria form stable complexes with Kinyoun carbol fuchsin.)

318) Tubercle bacilli are specifically stained by: a. crystal violet b. 1% acid fuchsin c. methylene blue d. carbol fuchsin

b (Dissolved solids, including salt, sugar, urea, etc., contribute to specific gravity. Turbidity is caused by cells and crystals, which do not dissolve, and do not contribute to specific gravity.)

32) A urine's specific gravity is directly proportional to its: a. turbidity b. dissolved solids c. salt content d. sugar content

c (Arterial pH reference range.)

32) The reference range for the pH of arterial blood measured at 37C is: a. 7.28-7.34 b. 7.33-7.37 c. 7.35-7.45 d. 7.45-7.50

c (HIPAA protects health insurance coverage for workers and their families when they change or lose their jobs, and also addresses the security and privacy of health data.)

32) Your friend calls and asks you to access his test results. Which of the following does this violate? a. CAP b. The Joint Commission c. HIPAA d. CLIA

c (Rnp/Sm extractable nuclear antigens show coarse nuclear speckles.)

33) In an anti-nuclear antibody indirect immunofluorescence test, a sample of patient serum shows a positive, speckled pattern. Which would be the most appropriate additional test to perform? a. antimitochondrial antibody b. immunoglobulin quantitation c. screen for Sm and RNP antibodies d. anti-DNA antibody using C luciliae

a (Albumin method.)

330) Bromcresol purple at a pH of 5.2 is used in a colorimetric method to measure: a. albumin b. globulin c. Bence Jones protein d. immunoprotein

d (The major cause of transfusion associated fatalities is transfusion of blood to the wrong patient.)

330) The most important step in the safe administration of blood is to: a. perform compatibility testing accurately b. get an accurate patient history c. exclude disqualified donors d. accurately identify the donor unit and recipient

b (Prolonged drying of slides will produce erythrocyte distortion [crenated] on microscopic examination.)

378) If a blood smear is dried too slowly, the RBCs are often: a. clumped b. crenated c. lysed d. destroyed

d (The ISI represents the international sensitivity index. The most responsive reagents have an ISI of 1.)

388) The ISI in the INR represents the reagents: a. activator b. specificity c. phospholipids d. sensitivity

c (Chemical cause of alkalosis and acidosis.)

39) Acidosis and alkalosis are best defined as fluctuations in blood pH and CO2 content due to changes in: a. Bohr effect b. O2 content c. bicarbonate buffer d. carbonic anhydrase

c (Osmolality empirical calculation.)

43) A patient had the following serum results: Which osmolality is consistent with these results? a. 188 b. 204 c. 270 d. 390

c (Major component of CO2 in blood.)

50) Most of the carbon dioxide present in blood is in the form of: a. dissolved CO2 b. carbonate c. bicarbonate ion d. carbonic acid

d (The entire set of HLA antigens located on one chromosome is a haplotype.)

59) The linked HLA genes on each chromosome constitutes a (an): a. allele b. trait c. phenotype d. haplotype

a (Direct ISE method.)

59) Which of the following is true about direct ion selective electrodes for electrolytes? a. whole blood specimens are acceptable b. elevated lipids cause falsely decreased results c. elevated proteins cause falsely decreased results d. elevated platelets cause falsely increased results

a (Urine in the bladder is normally sterile. Suprapubic aspiration removes urine directly from the bladder and yields a specimen free of urethral contamination.)

6) The expected colony count in a suprapubic urine from a healthy individual is: a. 0 CFU/mL b. 100 CFU/mL c. 1,000 CFU/mL d. 100,000 CFU/mL

b (Interferences with indirect ISE methods.)

60) Sodium determination by indirect ion selective electrode is falsely decreased by: a. elevated chloride levels b. elevated lipid levels c. decreased protein levels d. decreased albumin levels

c (The A and B structures cannot be developed since there is no H precursor substance due to the lack of the H gene in the blood donor.)

61) A blood donor has the genotype: hh, AB. what is his RBC phenotype? a. A b. B c. O d. AB

b (Regulation of Ca++.)

63) Calcium concentration in the serum is regulated by: a. insulin b. parathyroid hormone c. thyroxine d. vitamin C

c (When 2 drops of urine are added to 10 drops of water, it is a 2/12 proportion. 12 is the total volume. This is the same as a 1:6 dilution.)

65) A urine tested with Clinitest exhibits a pass-through reaction and is diluted by adding 2 drops of urine to 10 drops of water. This dilution is: a. 1:4 b. 1:5 c. 1:6 d. 1:8

a ( The most common genotype in Rh-negative individuals is rr. Anti-e would not be formed because the recipient's red cells contain the e antigen. The first antibody most likely to develop would be anti-c.)

65) In an emergency situation, Rh-neg. red cells are transfused into a Rh-pos person of the genotype CDe/CDe. the first antibody most likely to develop is: a. anti-c b. anti-d c. anti-e d. anti- E

d (Blood group genes are autosomal; they are not carried on the sex gene. Whenever the gene is inherited, the antigen is expressed on the red blood cells, which is known as codominant.)

66) Most blood group systems are inherited as: a. sex-linked dominant b. sex-linked recessive c. autosomal recessive d. autosomal codominant

d (A 4-fold or greater increase in antibody titer from 2 serum specimens taken from a patient during the acute and convalescent phases of an infection are considered to be diagnostic.)

67) Blood is drawn from a patient for serological tests for viral disease at the time of onset and again 4 wks. later. The results of the tests are considered diagnostic if the: a. first antibody titer is 2x the second b. first and second antibody titers are equal c. first antibody is 4x the second d. second antibody titer is a least 4x the first

c (Flocculation tests for syphilis are nontreponemal tests that detect antibody specific for cardiolipin antigen. When this antibody, called reagin, combines with the fine cardiolipin particles, small clumps are formed in a reaction called flocculation.)

84) Flocculation tests for syphilis use antigen composed of: a. Treponema pallidum b. reagin c. cardiolipin and lecithin d. charcoal

c (Fucose is the immunodominant sugar for H.)

89) The enzyme responsible for conferring H activity on the red cell membrane is alpha_: a. galactosyl transferase b. n-acetylgalactosaminyl tranferase c. L-fucosyl transferase d. N-acetylglucosaminyl transferase

a (Hyaline casts may be excreted by healthy people following strenuous exercise or normal condition that produces decreased urine flow. Red cell, white cell, and waxy casts indicate a pathogenic condition within the nephron.)

92) Which of the following casts is most likely to be found in healthy people? a. hyaline b. RBC c. waxy d. WBC

c (In beta-thalassemia major, reduced synthesis of beta chains affects the production of Hgb A. Hgb A2 and Hgb F are increased and Hgb A decreased.)

97) Which pattern is consistent with beta-thalassemia major? a. pattern A b. pattern B c. pattern C d. pattern D

a (The correct answer is a, biohazard.)

99) This symbol represents a: a. biohazard b. radiation hazard c. chemical hazard d. environmental hazard

c (Dolichos biflorus plant seed extract forms complexes with N-acetylgalactosamine. When properly diluted, it can distinguish between A1 donor cells and all other subgroups of A)

253) A sample gave the following results: cells: anti-A = 3+ | anti-B = 4+ serum: A1 cells = 2+ | B cells = 0 Which lectin should be used first to resolve the discrepancy? a. Ulex europaeus b. Arachis hypogaea c. Dolichos biflorus d. Vicia graminea

d (The serum of a group O individual contains anti-A, anti-B and anti-A,B. To prepare a suitable reagent, the ABO antibodies must be removed and anti-D left in the serum. The serum would need to be adsorbed with cells of the A1R cde/ cde phenotype.)

254) The serum of a group O, Cde/Cde donor contains anti-D. In order to prepare a suitable anti-D reagent from this donor's serum, which of the following cells would be suitable for the adsorption? a. group O. Cde/Cde b. group O, Cde/cde c. group A2B, CDe/cde d. groupA1B, cde/cde

c (In a flow cytometer, side scatter provides an estimate of a cell complexity or granularity.)

257) In flow cytometric analysis, right angle of side scatter of a laser light beam provides information that pertains to the cell's: a. volume b. viability c. granularity d. lineage

c (Antibody-antigen complexes are dependent upon a neutral pH. Extremes in pH causes dissociation. Both auto and alloantibodies are recovered in eluates prepared by reagent kits that alter the pH.)

260) One of the most effective methods for the elution of warm autoantibodies from RBCs utilizes: a. 10% sucrose b. LISS c. change in pH d. distilled water

c (Protein electrophoresis migration.)

313) When separating serum proteins by cellulose acetate electrophoresis, using Veronal buffer at pH 8.6. beta globulin migrates: a. faster than albumin b. slower than gamma globulins c. faster than gamma globulin d. faster than alpha-2 globulin

c (PCO2 electrode.)

328) Which blood gas electrode is composed of silver/silver chloride reference electrode and glass? a. PO2 b. pH c. PCO2 d. HCO3

d (Total protein method.)

329) Most chemical methods for determining total protein utilize which of the following reactions? a. molybdenum blue b. ferri-ferrocyanide c. resorcinol-HCL d. biuret

d (Immunoglobulin function; IgE is the immunoglobulin involved with allergy.)

138) The immunoglobulin class associated with immediate hypersensitivity or atopic reactions is: a. IgA b. IgM c. IgD d. IgE

b (Phagocytosis is performed by granulocytes.)

141) Phagocytosis is a function of: a. erythrocytes b. granulocytes c. lymphocytes d. thrombocytes

d (The intact structure of the glomerular membrane does not permit passage of high-molecular-weight substances, such as protein molecules.)

152) Which of the following components are present in serum but not present in the glomerular filtrate? a. glucose b. amino acids c. urea d. large molecular weight proteins

a (Warm autoantibodies often exhibit Rh specificity.)

206) Autoantibodies demonstrating blood group specificity in warm autoimmune hemolytic anemia are associated more often with which blood group system. a. Rh b. I c. P d. Duffy

a (WBC correction for nRBC.)

223) An automated leukocyte count is 22.5 x 10^9/mL (22.5 x 10^9/L). The differential reveals 200 normoblasts/100 leukocytes. What is the actual leukocyte count per microliter? a. 7,500/mL (7.5 x 10^9/L) b. 11,500/mL (11.5 x 10^9/L) c. 14,400/mL (14.4 x 10^9/L) d. 22,300/mL (22.3 x 10^9/L)

b (WBC correction for nRBC.)

224) A total leukocyte count is 10.0 x 10^3/mL (10.0 x 10^9/L) and 25 NRBCs are seen per 100 leukocytes on the differential. What is the corrected leukocyte count? a. 2,000/mL (2.0 x 10^9/L) b. 8,000/mL (8.0 x 10^9/L) c. 10,000/mL (10.0 x 10^9/L) d. 12,000/mL (12.0 x 10^9/L)

c (WBC correction for nRBC.)

225) If the total leukocyte count is 20.0 x 10^3/mL (20.0 x 10^9/L) and 50 NRBCs are seen per 100 leukocytes on the differential, what is the corrected leukocyte count? a. 6,666/mL (6.666 x 10^9/L) b. 10,000/mL (10.0 x 10^9/L) c. 13,333/mL (13.333 x 10^9/L) d. 26,666/mL (26.666 x 10^9/L)

b (Aldosterone is a hormone produced by the adrenal cortex.)

227) Which of the following steroids is an adrenal cortical hormone? a. angiotensinogen b. aldosterone c. epinephrine d. growth hormone

d (CLIA was established to ensure quality standards for laboratory testing.)

23) CLIA was established to provide oversight to: a. research labs b. point of care testing by nonlaboratory personnel c. CAP-accredited labs d. any lab performing patient testing

a (Morphology of a myelocyte.)

230) The large nucleated cell in the lower right-hand side of the image below is a: a. myelocyte b. metamyelocyte c. basophil d. plasma

b (Angiotensin II is a vasoconstrictor and stimulates the adrenal cortex to produce aldosterone.)

236) The major action of angiotensin II is: a. increase pituitary secretion of vasopressin b. increase vasoconstriction c. increase parathormone secretion by the parathyroid d. decrease adrenal secretion of aldosterone

a (Cryoprecipitated AHF is stored at -18C or lower.)

24) Optimum storage for Cryo AHF is: a. -20 C b. -12 C c. 4 C d. 22 C

a (The 24-hour urine free cortisol is the most sensitive and specific screen for hypercortisolism.)

241) The screen for adrenal cortical hyperfunction with the greatest sensitivity and specificity is: a. 24 hr. urine free cortisol b. plasma cortisol c. urinary 17-hydroxycoricosteroids d. plasma corticosterone

a (The only reagent listed is in a. The other answers are forms of ketones.)

48) Which of the following reagents is used to react with ketones in the urine? a. sodium nitroprusside b. acetoacetic acid c. acetone d. beta-hydroxybutyric acid

c (Due to circadian variation, the 24-hour UFC is an accurate measurement of active forms of cortisol.)

244) Which of the following sample collections would give an accurate assessment of potential excess cortisol production? a. collect a plasma sample as a baseline, and another one 1 hr. after administration of metyrapone b. collect a plasma sample at 8 am only c. collect a 24 hr. urine free cortisol d. collect a plasma sample at 8 am and at 8 am the next day

c (Calculation of RBC indices.)

113) Which of the following is the formula for MCV? a. (Hgb x 10) / RBC b. Hgb / Hct c. (Hct x 10) / RBC d. RBC / Hct

b (Calcium oxide monohydrate crystals are most frequently seen following ingestion of ethylene glycol/antifreeze. Unlike the more commonly seen envelope-shaped dihydrate crystals, they are oval or dumbbell shaped.)

114) Following ingestion of ethylene glycol, numerous crystals are found in the urine. The shape of these crystals is: a. flat with notched corners b. oval/dumbbell c. coffin-lid d. rosettes/rhomboid

b (HBeAg is present in patient serum during periods of active HBV replication, and is therefore a marker of high infectivity.)

114) The antigen marker most closely associated with transmissibility of HBV infection is: a. HbsAg b. HBeAg c. HBcAg d. HBV

c (Lymphocyte function.)

130) Cells that produce antibodies and lymphokines are: a. erythrocytes b. granulocytes c. lymphocytes d. thrombocytes

a (A secretory IgA molecule is composed of 2 units of 2 heavy chains and 2 light chains. These chains are joined by the J chain, and are protected from the harsher environment where there are secreted by an additional chain called the secretory piece.)

135) Which of the following immunoglobulin classes is associated with a secretory component: a. IgA b. IgD c. IgE d. IgG

a (Complement C1q is the recognition piece.)

162) Which of the following is the "recognition unit" in the classical complement pathway? a. C1q b. C3a c. C4 d. C5

b (Accuracy or trueness is the closeness of agreement with the true value.)

17) The extent to which measurements agree with the true value of the quantity being measured is known as: a. reliability b. accuracy c. reproducibility d. precision

c (Presence of agglutination with A1 cells, screening cells and autocontrol at IS and RT is indicative of a cold autoantibody.)

180) Consider the following ABO typing results: What is the most likely cause of this discrepancy? a. A2 with anti-A1 b. cold alloantibody c. cold autoantibody d. acquired-A phenomenon

a (Hypergammaglobulinemia such as a polyclonal increase is associated with autoimmune disorders. Hypergammaglobulinemia such as a monoclonal increase is associated with such disease states as multiple myeloma, lymphomas, etc.)

19) Positive rheumatoid factor is generally associated with: a. hyperglobulinemia b. anemia c. decreased ESR d. azotemia

c (May be referred to as hyperchromic because of decreased MCHC.)

19) What cell shape is most commonly associated with an increased MCHC? a. teardrop cells b. target cells c. spherocytes d. sickle cells

c (Initial result was most likely a false-negative result due to the omission of patient serum. This would explain the initial negative result followed by the subsequent positive result.)

193) a patient received 2 units of RBCs and had a delayed transfusion reaction. Pretransfusion antibody screening records indicate no agglutination except after the addition of IgG sensitized cells. Repeat testing of the pretransfusion specimen detected an antibody at the antiglobulin phase. what is the most likely explanation for the original results? a. red cell were overwashed b. centrifuge time was prolonged c. patient's serum was omitted from the original test d. antiglobulin reagent was neutralized

d (Discrimination of CLL from other major mature B cell neoplasms)

194) Chronic lymphocytic leukemia is defined as: a. malignancy of the thymus b. accumulation of prolymphocytes c. accumulation of hairy cells in the spleen d. accumulation of monoclonal B cells with a block in cell maturation

b (Other distractors do not describe double beam-in-space.)

197) In a double-beam photometer, the additional beam is used to: a. compensate for variation in wavelength b. correct for variations in light source intensity c. correct for changes in light path d. compensate for variation in slit-widths

d (The WHO rating for sperm motility is: 0 - no movement; 1.0 = no forward movement; 2.0 = slow forward/lateral movement; 3.0 = faster speed, some lateral movement; 4.0 = rapid, straight line movement.)

202) Rapid forward progression of sperm is rated as: a. 1.0 b. 2.0 c. 3.0 d. 4.0

d (Discrimination of anomaly from others involving nuclear hyposegmentation; large granules in leukocytes; lipid storage disease.)

210) Of the following, the disease most closely associated with mucopolysaccharidosis is: a. Pelger-Huet b. Chediak-Higashi c. Gauchers disease d. Alder-Reilly

c (Both Neisseria meningitidis and N lactamica produce acid from maltose and grow on modified Thayer martin agar. N lactamica ferments lactose, N meningitidis does not.)

213) Which of the following is the most reliable test to differentiate Neisseria lactamica from Neisseria meningitidis? a. acid from maltose b. growth on modified Thayer-Martin agar c. lactose degradation d. nitrite reduction to nitrogen gas

c (Correlation of gout with sodium urate crystals.)

217) In synovial fluid , the most characteristic microscopic finding in gout is: a. calcium pyrophosphate crystals b. cartilage debris c. monosodium urate crystals d. hemosiderin laden macrophages

b (Hemacytometer calculation .)

221) If a WBC count is performed on a 1:10 dilution and the number of cells counted in 8 squares is 120, the total WBC count is: a. 1,200/mL (1.2 x 10^9/L) b. 1,500/mL (1.5 x 10^9/L) c. 12,000/mL (12.0 x 10^9/L) d. 15,000 /mL (15.0 x 10^9/L)

b (Polyspecific AHG contains anti-IgG and anti-C3d.)

224) Polyspecific reagents used in the DAT test should have specificity for: a. IgG and IgA b. IgG and C3d c. IgM and IgA d. IgM and C3d

b (The total number of lymphocytes can be calculated by multiplying the WBC [5 x 10^3, or 5,000] by the % of lymphocytes [15% or 0.15] to get 750. This number is then multiplied by the % of CD4+ cells [8% or 0.08] to get an absolute CD4 cell count of 60.)

230) Calculate the absolute CD4 a. 40 b. 60 c. 400 d. 750

a (Kanamycin-vancomycin laked blood agar is a selective medium used for the isolation of Bacteroides or Prevotella.)

233) Which of the following pairs of organisms usually grow on kanamycin, vancomycin, laked blood agar? a. Bacteroides and Prevotella b. Mobiluncus and Gardnerella c. Porphyromonas and Enterococcus d. Veillonella and Capnocytophaga

a (Correlation of sepsis with a leukemoid reaction.)

236) A patient diagnosed as having bacterial septicemia. Which of the following would best describe the expected change in his peripheral blood? a. granulocytic leukemoid reaction b. lymphocytic leukemoid reaction c. neutropenia d. eosinophilia

d (Lithium [carbonate] is used to treat manic depression or bipolar disorder.)

270) Lithium therapy is widely used in the treatment of: a. hypertension b. hyperactivity c. aggression d. manic-depressive disorder

c (Symptoms of hemolytic transfusion reactions are fever, chills, flushing, chest and back pain, hypotension, nausea, dyspnea, shock, renal failure, and DIC. Circulatory overload, allergic, and anaphylactic reactions are not characterized by fever.)

280) Hypotension, nausea, flushing, fever and chills are symptoms of which of the following transfusion reactions? a. allergic b. circulatory overload c. hemolytic d. anaphylactic

d (Definition of increased platelet count.)

280) Thrombocytosis would be indicated by a platelet count of: a. 100 x 10^3/mL (100 x 10^9/L) b. 200 x 10^3/mL (200 x 10^9/L) c. 300 x 10^3/mL (300 x 10^9/L) d. 600 x 10^3/mL (600 x 10^9/L)

b (Several monomorphic molds resemble the filamentous phase of dimorphic molds, so conversion to yeast phase must be performed for identification of dimorphic molds.)

280) Which of the following procedures should be performed to confirm that an unknown mold is one of the pathogenic dimorphic fungi: a. animal inoculation b. culture conversion to yeast form c. demonstration of sexual and asexual reproduction d. serological studies

c (Iron studies: microcytic hypochromic anemia results in a decrease in serum and storage iron, but an increase in TIBC.)

29) A patient is admitted with a history of chronic bleeding secondary to peptic ulcer. Hematology workup reveals a severe microcytic, hypochromic anemia. Iron studies were requested. Which of the following would be expected in this case? a. result A b. result B c. result C d. result D

c ("GEN 41770: Are there appropriate documented procedures for handling and cleaning glassware, including methods for testing for detergent removal?")

29) CAP requires that glassware cleaning practices include periodic testing for: a. chemical residues b. silicates c. detergents d. heavy metals

a (Lactophenol cotton blue is used as the mounting medium, since lactic acid acts as a clearing agent, phenol acts as a killing agent, glycerol prevents drying, and cotton blue gives color to the structures.)

291) Which of the following is most often used to prepare a slide from a plate culture of a dermatophyte for microscopic observation? a. lactophenol cotton blue b. potassium hydroxide c. iodine solution d. Gram stain

a (BT assesses platelet function; PT, TT and PTT do not assess platelet function.)

297) Which of the following detects or measures platelet function? a. bleeding time b. prothrombin time c. thrombin time d. partial thromboplastin time

b (Early-morning sputum specimens are optimal for AFB culture.)

304) A sputum specimen received at 8 AM for an Acid Fast smear reveals acid-fast bacilli. An additional sputum is submitted that afternoon. This specimen was concentrated by the NALC-sodium hydroxide method and inoculated on 2 Lowenstein-Jensen slants and held for 8 weeks at 35C in 5%-10% CO2. No growth occurs. The best explanation is that: a. the hypochlorite technique was not used b. an improper specimen was submitted for culture c. improper media was used for culture d. cultures were held for an insufficient period of time

c (Leukocyte-Reduced RBCS and Platelets can be used to prevent further nonhemolytic transfusion reactions.)

304) The use of Leukocyte-Reduced Red Blood Cells and Platelets is indicated for which of the following patient groups? a. CMV-seropositive postpartum mothers b. victims of acute trauma with massive bleeding c. patients with history of febrile transfusion reactions d. burn victims with anemia and low serum protein

c (Warfarin interferes with the carboxylation of vitamin K factors by interrupting the enzymatic phase of the reaction. Factors are inhibited according to their half-life, VII having the shortest [4-5 hours] and II the longest [2-3 days].)

305) Coagulation factors affected by warfarin (Coumadin) drugs are: a. VIII, IX, and X b. I, II, V, and VII c. II, VII, IX and X d. II, V, and VII

a (Mycobacteria use glycerol as a carbon source in the presence of mineral salts.)

305) The preferred carbon source for mycobacteria is: a. glycerol b. glucose c. fatty acids d. casein hydrolysate

c (Anaphylactic transfusion reactions are attributed to anti-IgA in IgA- deficient recipients.)

306) Posttransfusion anaphylactic reactions occur most often in patients with: a. leukocyte antibodies b. erythrocyte antibodies c. IgA deficiency d. Factor VIII deficiency

c (Two distinguishing features or anaphylactic transfusion reactions are that symptoms occur with transfusion of only small amounts of blood, and the patient has no fever.)

307) Which of the following transfusion reactions occurs after infusion of only a few milliliters of blood and gives no history of fever? a. febrile b. circulatory overload c. anaphylactic d. hemolytic

d (Atomic absorption spectrophotometry [AAS] measures calcium by detecting its atomic absorption by electromagnetic radiation. One limitation of this method is the nonspectral interference which occurs when phosphates are present and complex with calcium. The use of lanthanum chloride with the method has prevented the interference. Lanthanum chloride competes for the phosphate.)

336) Which of the following calcium procedures utilizes lanthanum chloride to eliminate interfering substances? a. o-cresolphthalein complexone b. precipitation with chloranilic acid c. chelation with EDTA d. atomic absorption spectrophotometry

c (The APTT reagent contains phospholipid and a negatively-charged particulate activator such as kaolin, ellagic acid, or Celite@ . Ionic calcium and phospholipid are supplied as reagents)

337) In the APTT test, the patient's plasma is mixed with: a. ADP and calcium b. tissue thromboplastin and collagen c. phospholipid and calcium d. tissue thromboplastin and calcium

c (Azobilirubin is the chromophore measured in the Jendrassik-Grof reaction. Azobilirubin is formed by bilirubin in the presence of diazotized-sulfanilic acid.)

341) In the Jendrassik-Grof reaction for total bilirubin, bilirubin reacts with diazotized sulfanilic acid to form: a. diazo bilirubin b. biliverdin c. azobilirubin d. bilirubin glucuronide

d (A prolonged thrombin time can indicate diminished or abnormal fibrinogen, the presence of FDPs, paraproteins, and heparin. Reptilase is insensitive to the effects of heparin, and sensitive to dysfibrinogenemia. Therefore, when the TT is prolonged and the reptilase test is normal, this confirms the presence of heparin.)

342) A prolonged thrombin time and a normal reptilase-R time are characteristic of : a. dysfibrinogenemia b. increased D-Dimer c. fibrin monomer-split product complexed d. therapeutic heparinization

a (LDH chemical reaction.)

342) In the assay of lactate dehydrogenase, which of the following products is actually measured? a. NADH b. ATP c. lactic acid d. pyruvic acid

a (Chlamydia trachomatis is a well-known cause of sexually transmitted infections, including urethritis and cervicitis, as well as inclusion conjunctivitis and pneumonia in neonates. It also causes trachoma and lymphogranuloma venereum.)

362) Chlamydial infections have been implicated in: a. urethritis and conjunctivitis b. gastroenteritis and urethritis c. neonatal pneumonia and gastroenteritis d. neonatal meningitis and conjunctivitis

c (The heel is the preferred site for drawing capillary blood from the newborn. the posterior curvature of the heel should never be used.)

376) The ideal capillary blood collection site on a newborn is: a. tip of the thumb b. ear lobe c. plantar surface of the heel d. the great toe

d (Respiratory alkalosis caused by hyperventilation)

38) A common cause of respiratory alkalosis is: a. vomiting b. starvation c. asthma d. hyperventilation

b (Fresh Frozen Plasma [FFP] must be separated and frozen within 8 hours of Whole Blood collection.)

38) Which of the following blood components must be prepared within 8 hrs. after phlebotomy: a. RBCs b. FFP c. Frozen rbcs d. Cryo AHF

c (Fluorochrome is a labeled anti- human immunoglobulin for indirect immunofluorescent assays.)

41) In the indirect immunofluorescence method of antibody detection in patient serum, the labeled antibody is: a. human anti-goat immunoglobulin b. rheumatoid factor c. goat anti-human immunoglobulin d. complement

a (Albumin is the most abundant plasma protein and it is relatively small. In nephropathy, albumin will be the most abundant protein in the urine. This test is done to look for nephropathy [i.e. kidney disease].)

46) The protein section of the urine reagent strip is most sensitive to: a. albumin b. mucoprotein c. Bence Jones protein d. globulin

c (Describes the overall interaction of an antibody with its antigen.)

48) The strength of a visible reaction is known as: a. prozone reaction b. absorption c. avidity d. elution

a (Skirrow medium is an enriched selective blood agar medium used for the isolation of Campylobacter from specimens with mixed flora. CIN and bismuth sulfite agars are selective and differential for Yersinia enterocolitica and Salmonella, respectively. CNA agar is selective for gram- positive organisms.)

52) Which of the following media can be used to culture Campylobacter jejuni? a. Skirrow medium b. CIN agar c. anaerobic CNA agar d. bismuth sulfite

d (Zone of antigen excess is depicted on the figure as D.)

57) The curve below was obtained by adding increasing amounts of a soluble antigen to fixed volumes of monospecific antiserum: The area on the curve where soluble antigen-antibody complexes have begun to form is: a. A b. B c. C d. D

b (Largest solute in osmolality.)

57) The solute that contributes the most to the total serum osmolality is : a. glucose b. sodium c. chloride d. urea

a (Continuous coalescing precipitin lines form when the 2 antigens are identical.)

59) Which of the above figures demonstrates a reaction pattern of identity? a. Figure #1 b. Figure #2 c. Figure #3 d. Figure #4

d (The patient lacks E. Since C and c are alleles, C is inherited from one parent and c from the other. Since the person is homozygous for e, one of the genes needs to code for ce [RHce] and the other Ce [RHCe]. The RHD gene is more likely inherited with Ce than ce, so the person's most probable genotype is DCe/dce. This genotype is found in 31% of the white and 15% of the black populations.)

60) An individual's red blood cells give the following reactions with Rh antisera: The individual's most probable genotype is: a. DCe/DcE b. DcE/dce c. Dce/dce d. DCe/dce

b (Two nonrelated antigens form independent precipitin lines that cross over each other.)

60) Which of the above figures demonstrates a reaction pattern of nonidentity? a. Figure #1 b. Figure #2 c. Figure #3 d. Figure #4

c (Interferences with indirect ISE methods.)

61) A physician requested that electrolytes on a multiple myeloma patient specimen be run by direct ISE and not indirect ISE because: a. excess protein binds Na in indirect ISE b. Na falsely increased by indirect ISE c. Na is falsely decreased by indirect ISE d. excess protein reacts with diluent in indirect ISE

b (The correct answer is b, hallways and corridors should be clear and free of obstruction at all times. For distractor a, fire drills should not be announced or practiced in advanced. For distractor c, laboratories larger than 1,000 square feet and/or contain explosion hazards require 2 exits.)

61) For fire safety and prevention: a. fire drills should be announced and practiced in advance b. hallways and corridors should be clear and free of obstruction at all times c. only one exit is necessary in laboratories that contain an explosive hazard d. hazard evaluations only need to be done prior to initiation of clinical operations

c (The bilirubin dipstick pad can show a false positive from a colored urine. All positives should be confirmed with an ictotest.)

63) A positive result for bilirubin on a reagent strip should be followed up by: a. notifying the physician b. requesting a new specimen c. performing an Ictotest d. performing a urobilinogen

d (Development time line of production of Hgb delta chains.)

8) Which curve represents the production of delta polypeptide chains of hemoglobin? a. B b. C c. D d. E

c (Lewis antigens are found soluble in saliva. If saliva containing Lewis substance is added to a sample with anti-Lea, then neutralization occurs. Le[a+] indicator cells added to the test system would be nonreactive. A proper control system is required whenever neutralization studies are performed.)

98) Inhibition testing can be used to confirm antibody specificity for which of the following antibodies? a. anti-Lua b. anti-M c. anti-Lea d. anti-Fya

b (Demonstrates CML as having predominant myeloid cell line.)

146) The M:E ratio in chronic myelocytic leukemia is usually: a. normal b. high c. low d. variable

b (ANA detects circulating antibodies to nuclear antigens in systemic rheumatic diseases.)

1) Antinuclear antibody tests are performed to help diagnose: a. acute leukemia b. lupus erythematosus c. hemolytic anemia d. Crohn's disease

a (Primary functions of red blood cells.)

17) Cells for the transport of O2 and CO2 are: a. erythrocytes b. granulocytes c. lymphocytes d. thrombocytes

c (GGT levels are elevated in alcoholism. The levels may range from 2-3 times the upper limit of normal. ALT, AST, and ALP may be increased depending on the alcohol damage to the liver.)

188) The most sensitive enzyme indicator for liver damage from ethanol intake is: a. ALT b. AST c. GGT d. alkaline phosphatase

b (To calculate the dilution, take the amount of sample and divide it by the total of the new solution. So the dilution is 50 mL divided by 500 + 50 mL. This is a 1/11 dilution. To correct the final answer, multiply by the inverse of the dilution, or 11.)

194) A CSF was hazy and the WBC was too high to perform undiluted. The technologist took 50 mL of sample and added 500 mL of saline. The cell count on the diluted sample was 200 WBC/mL. This should be multiplied by: a. 10 b. 11 c. 1/10 d. 1/11

b (The absence of agglutination at the AHG phase with screening cells and agglutination with one of 3 donor units is most likely due to an antibody to a low-incidence antigen.)

194) At the indirect antiglobulin phase of testing there is no agglutination between patient serum and screening cells. One of 3 donor units was incompatible. The most probable explanation for these findings is that the: a. patient has an antibody directed against a high incidence antigen b. patient has an antibody directed against a low incidence antigen c. donor has an antibody directed against donor cells d. donor has a positive antibody screen

a (Differentiation of lipid storage disease from other anomalies of leukocytes.)

211) Of the following, the disease most closely associated with glucocerbrosidase deficiency is: a. Gauchers disease b. Chediak-Higashi c. Pelger- Huet d. May-Hegglin

b (TSH produced by pituitary gland.)

211) TSH is produced by the: a. hypothalamus b. pituitary gland c. adrenal cortex d. thyroid

a (Contact dermatitis is a delayed type hypersensitivity reaction due to T-cell responses to environmental chemicals or metals. The other conditions are examples of other types of hypersensitivity.)

217) Delayed hypersensitivity may be induced by: a. contact sensitivity to inorganic chemicals b. transfusion reaction c. anaphylactic reaction d. bacterial septicemia

b (TSH is the American Thyroid Association's recommended screening test.)

217) The recommended initial thyroid function test for either a healthy, asymptomatic patient, or a patient with symptoms which may be related to a thyroid disorder is: a. free thyroxine (free T4) b. thyroid stimulating hormone (TSH) c. total thyroxine (T4) d. triiodothyronine (T3)

a (All 3 antibodies can cause HDFN and delayed transfusion reactions. Anti-Jka is associated with showing dosage.)

217) Which characteristics are true of all 3 of the following antibodies: anti-Fya, anti-Jka and anti-K? a. detected at IAT phase and may cause HDFN and transfusion reactions b. not detected with enzyme treated cells; may cause delayed transfusion reactions c. requires the IAT technique for detection; usually not responsible for causing HDFN d. may show dosage effect; may cause severe hemolytic transfusion reactions

b (Neisseria sicca is the only Neisseria species of those listed that ferments glucose maltose and sucrose.)

218) An aspirated specimen of purulent material was obtained from a brain abscess. After 24 hours incubation, pinpoint colonies grew on sheep blood and small, yellowish colonies grew on chocolate. Gram stain of the organism showed gram-negative cocci. Results of carbohydrate degradation studies were as follows: - glucose: acid - maltose: acid - sucrose: acid - lactose: negative Additional testing revealed that the organism was oxidase positive and beta-galactosidase negative. The organism is most likely Neisseria: a. meningitidis b. sicca c. lactamica d. gonorrhoeae

c (Calculation of absolute from relative % and WBC.)

218) What is the absolute lymphocyte count? a. 170/mL (0.17 x 10^9/L) b. 510/mL (0.51 x 10^9/L) c. 2,550/mL (2.55 x 10^9/L) d. 4,760/mL (4.76 x 10^9/L)

b (Chemiluminescent labels are based on the emission of light produced during a chemical reaction. These labels are very useful because they provide very low levels of detection [2 x 10^-20 mol/L] with little or no background interference.)

226) Which of the following immunoassay labels offer the greatest detection limit? a. fluorescence b. electrochemiluminescence c. radioactivity d. chemiluminescence

a (Reduced excretion of acids.)

26) The expected blood gas results for a patient in chronic renal failure would match the pattern of: a. metabolic acidosis b. respiratory acidosis c. metabolic alkalosis d. respiratory alkalosis

a ("GEN 41042: Are refrigerator/freezer temperatures checked and recorded daily?")

27) CAP requires refrigerator temperature to be recorded: a. daily b. weekly c. monthly d. periodically

a (Per AABB standards, thawed FFP should be stored at 1-6 C for no more than 24 hours.)

28) According to AABB standards FFP must be infused within what period of time following thawing: a. 24 hrs. b. 36 hrs. c. 48 hrs. d. 72 hrs.

b (Delayed hemolytic transfusion reactions are caused by a secondary anamnestic response in a previously alloimmunized recipient. Unlike a primary response, a secondary response is rapid. Antibody may be detectable 3-7 days from the time of transfusion.)

285) In a delayed transfusion reaction the causative antibody is generally too weak to be detected in routine compatibility testing and antibody screening tests, but is typically detectable at what point after transfusion? a. 3-6 hours b. 3-7 days c. 60-90 days d. after 120 days

d (Failure to adequately chill blood gas specimens if not immediately analyzed will allow glucose metabolism, which increases carbon dioxide and lowers pH.)

285) Unless blood gas measurements are made immediately after sampling, in vitro glycolysis of the blood causes a: a. rise in pH and PCO2 b. fall in pH and a rise in PO2 c. rise is pH and a fall in PO2 d. fall in pH and a rise in PCO2

b (There is no indication that any part of the procedure has been done incorrectly.)

286) A phase-platelet count is performed using a platelet Unopette. 155 platelets are counted on one side of the hemacytometer in the center square millimeter, and 145 are counted on the other side in the same area. After making the appropriate calculations, the next step would be to: a. repeat the procedure, using a 1:20 dilution with acetic acid b. report the calculated value c. collect a new specimen d. repeat the procedure, using a 1:200 dilution with saline

a (Antibodies in the ABO system may activate complement and cause immediate intravascular hemolysis if incompatible blood is transfused. Antibodies in the Rh, Duffy, and MN systems typically cause extravascular hemolysis, which is usually less severe.)

286) The most serious hemolytic transfusion reactions are due to incompatibility in which of the following blood group systems? a. ABO b. Rh c. MN d. Duffy

d (Glucose decreases at a rate of 5%-7% per hour in whole blood at room temperature. Glycolysis will continue until the specimen is processed by centrifugation, and serum and plasma is separated from the cellular components of blood.)

286) Which of the following serum constituents is unstable if a blood specimen is left standing at room temp for 8 hrs. before processing: a. cholesterol b. triglycerides c. creatinine d. glucose

d (The use of an incorrect tube type or collection of a specimen at an inappropriate time may also require specimen recollection.)

287) An arterial blood specimen submitted for blood gas analysis was obtained at 8:30 am but was not received in the lab until 11 am. The technologist should: a. perform test immediately upon receipt b. perform the test only if the specimen was submitted in ice water c. request a venous blood specimen d. request a new arterial specimen be obtained

a (ABO antibodies activate complement and may cause intravascular hemolysis. Rh, Kell, and Duffy antibodies are primarily associated with extravascular hemolysis.)

287) Severe intravascular hemolysis is most likely caused by antibodies of which blood group system? a. ABO b. Rh c. Kell d. Duffy

c (Standard calculation for hemacytometer cell count.)

288) Blood is diluted 1:200, and a platelet count is performed. 180 platelets were counted in the center square millimeter on one side of the hemacytometer and 186 on the other side. The total platelet count is: a. 146 x 10^3/mL (146 x 10^9/L) b. 183 x 10^3/mL (183 x 10^9/L) c. 366 x 10^3/mL (366 x 10^9/L) d. 732 x 10^3/mL (732 x 10^9/L)

b (Antibodies in the Kidd system activate complement and may cause intravascular hemolysis. The antibodies often decline in vivo, are weak, show dosage, and are difficult to detect in vitro, making them prime candidates for causing anamnestic delayed hemolytic transfusion reactions.)

288) Which of the following blood group systems is most commonly associated with delayed hemolytic transfusion reactions? a. Lewis b. Kidd c. MNS d. I

d (Mucormycosis is an aggressive infection where biopsy, curettage or fine-needle aspiration is essential for diagnosis.)

296) The appropriate specimen for the diagnosis of mucormycosis is: a. nasal swab b. sputum c. sinus washing d. eschar biopsy

b (In a Clauss fibrinogen, a standard amount of thrombin is added to diluted plasma, and the time required for clot formation is recorded.)

345) In the Clauss fibrinogen method, the time to clot formation in plasma is measured after the addition of: a. calcium b. thrombin c. phospholipids d. kaolin

a (Platelet clumping is expected if smears are made directly from the finger. Smears must be made promptly before any clotting begins. It is important to examine the edges for platelet clumping.)

385) On a smear made directly from a finger stick, no platelets were found in the counting area. The first thing to do is: a. examine the slide for clumping b. obtain another smear c. perform a total platelet count d. request another finger stick

b (Sodium citrate is the only anticoagulant used for hemostasis testing. Sodium citrate hinds free calcium ions. EDTA inhibits the thrombin-mediated conversion of fibrinogen to fibrin and binds reagent calcium added to initiate clot based tests. Heparin inhibits Ila, Xa, XIIa, XIa and IXa.)

386) The anticoagulant of choice for routine coagulation procedures is: a. sodium oxalate b. sodium citrate c. heparin d. sodium fluoride

b (The image displays the trophozoite form of Giardia lamblia, a flagellated protozoan parasite.)

387) The organism depicted is a(n): a. amoeba b. flagellate c. filaria d. sporozoan

c (The 9;22 translocation creates the BCR/ABL1 fusion gene in CML.)

41) A translocation in which chromosome pair creates a BCR/ABL1 fusion gene product associated with CML? a. 11;18 b. 14;18 c. 9;22 d. 9;14

d (Kidney concentration determination.)

44) The degree to which the kidney concentrates the glomerular filtrate can be determined by: a. urine creatine b. serum creatinine c. creatinine clearance d. urine to serum osmolality ratio

d (Morphology in Hgb C disease.)

49) A characteristic morphologic feature in hemoglobin C disease is: a. macrocytosis b. spherocytosis c. rouleaux formation d. target cells

b (Development time line of production of Hgb alpha chains)

5) Which curve represents the production of alpha polypeptide chains of hemoglobin? a. A b. B c. C d. D

c (Protein-bound Ca++.)

62) Which percentage of total serum calcium is nondiffusible protein bound? a. 80% - 90% b. 51% - 60% c. 40% - 50% d. 10% - 30 %

b (The most likely haplotype is DCe/dce.)

77) A donor is tested with Rh antisera with the following results: What is his most probable Rh genotype? a. R1R1 b. R1r c. R0r d. R2r

a (Increase in both myeloid and erythroid precursors)

79) The M:E ratio in polycythemia vera is usually: a. normal b. high c. low d. variable

a (By definition, precipitation involves combination of antigen and soluble antibody to form insoluble complexes that fall out of solution.)

80) What kind of antigen-antibody reaction would be expected if soluble antigen is added to homologous antibody? a. precipitation b. agglutination c. complement fixation d. hemagglutination

d (Diagnosis of hypoglycemia in adults.)

1) Following overnight fasting, hypoglycemia in adults is defined as a glucose of: a. <70 mg/dL (<3.9 mmol/L) b. <60 mg/dL (<3.3mmol/L) c. <55 mg/dL (<3.0mmol/L) d. <45mg/dL (<2.5mmol/L)

d (Chocolate agar and chocolate agar-based selective media should be used for recovery of Neisseria gonorrhoeae from urethral discharge. Chocolate agar provides the nutrients required by N gonorrhoeae and selective media contains antimicrobial agents that inhibits other organisms and permits recovery of pathogenic Neisseria.)

1) Proper media for culture of a urethral discharge from a man include: a. sheep blood and phenylethyl alcohol agars b. eosin methylene blue and sheep blood agars c. thioglycollate broth and chocolate agar d. chocolate agar and modified Thayer Martin agar

b (All donors, regardless of sex, require a minimum hemoglobin of 12.5 g/dL [125 g/L]. The value must not be performed on an earlobe stick.)

1) The minimum Hgb concentration in a fingerstick from a male blood donor is: a. 12.0 g/dL (120 g/L) b. 12.5 g/dL (125 g/L) c. 13.5 g/dL (135 g/L) d. 15.0 g/dL (150 g/L)

b (Larger sediment constituents sink to the bottom of the specimen container. Without prior mixing, the sediment in the container may not be poured into the centrifuge tube.)

10) Failure to observe RBC casts in a urine specimen can be caused by: a. staining the specimen b. centrifuging an unmixed specimen c. mixing the sediment after decantation d. examining the sediment first under low power

d (Source for latex agglutination of RF is gamma globulin pools from humans, rabbits, or cattle.)

17) Rheumatoid factor reacts with: a. inert substances such as latex b. Rh-pos erythrocytes c. kinetoplasts of Crithidia luciliae d. gamma globulin-coated particles

b (Recognizes myeloid predominance in AML would increase normal [2:1 to 4:1] myeloid:erythroid ratio.)

171) The M:E ratio in acute myelocytic leukemia is usually: a. normal b. high c. low d. variable

d (Specimen requirements for analyte stability.)

106) Erroneous ammonia levels can be eliminated by all of the following except: a. assuring water and reagents are ammonia-free b. separating plasma from cells and performing test analysis as soon as possible c. drawing the specimen in a pre-chilled tube and immersing the tube in ice d. storing the specimen protected from light until the analysis is done

a (Elevations of serum LD4 and LD5 fractions are seen in liver and skeletal muscle diseases because the isoenzymes are located in the tissues. LD1 and LD2 are elevated in hemolytic anemia and myocardial infarction. Increased levels of LD3 are observed in pulmonary edema.)

172) Of the following diseases, the one most often associated with elevations of LD isoenzymes 4 and 5 on electrophoresis is: a. liver disease b. hemolytic anemia c. myocardial infarction d. pulmonary edema

a (Antibodies in the Kidd blood group system are IgG and react best at the antiglobulin phase. These antibodies are associated with delayed hemolytic transfusion reactions and reactivity can be enhanced by testing with enzyme pretreated cells.)

111) The antibodies of the Kidd blood group system: a. react best at the IAT b. are predominately IgM c. often cause allergic transfusion reactions d. do not generally react with antigen-positive, enzyme-treated RBC's

b (Calculation of RBC indices.)

115) What is the MCV if the hematocrit is 20%, the RBC is 2.4 x 10^6/mL (2.4 x 10^12/L) and the hemoglobin is 5 g/dL (50 g/L)? a. 68 pg b. 83 pg c. 100 pg d. 120 pg

d (Questions that could be discriminatory in nature [eg, nationality, marital status, dependents, religion, affiliations, sexual orientation, or physical or mental disabilities] should be avoided on the employment application or during the interview process, unless the question is relevant or pertinent to the particular job; otherwise, it can be considered illegal.)

117) Legal pre-employment questions on an application are: a. medical history of an employee b. place of birth c. felonies unrelated to job requirements d. name and address of person to notify in case of emergency

b (Triple phosphates crystals, nicknamed "coffin lids" are nonpathologic. They are found in alkaline urines, usually urines with bacterial overgrowth.)

117) The following crystal is found in: a. acid pH and is nonpathologic b. alkaline pH and is nonpathologic c. acid pH and is pathologic d. alkaline pH and is pathologic

c (IgM is the first immunoglobulin to be produced during an immune response, and is produced by infants with congenital infections. IgG in the blood of a newborn infant is primarily of maternal origin, since it can cross the placenta, while IgA would be acquired through mother's breast milk.)

117) The initial immune response following fetal infection with rubella is the production of which class of antibodies? a. IgG b. IgA c. IgM d. both IgG and IgA

a (The principle screening test would be a peripheral smear and RBC indices. The confirmatory osmotic fragility is frequently a reference lab only test.)

117) The primary confirmatory test for hereditary spherocytosis is: a. osmotic fragility b. sucrose hemolysis c. heat instability test d. Kleihauer-Betke

a (Staphylococcus aureus produces an enterotoxin that is associated with food poisoning. Symptoms typically appear within 2 - 6 hours.)

122) How many hrs. after eating contaminated food do initial symptoms of staphylococcal food poisoning typically occur? a. 2 - 6 hrs. b. 12 - 18 hrs. c. 24 - 48 hrs. d. 72 hrs.

b (These are starch crystals, a contaminant from powder.)

122) The presence of this element in urine indicates the presence of: a. a UTI b. powder c. carbohydrate deficiency d. high urine amylase

c (Salmonella is the only distractor that produces H2S. Also, Klebsiella and E coli produce acid/acid reactions in TSI.)

135) An organism was inoculated to a TSI tube and gave the following reactions: alkaline slant/acid butt, H2S, gas produced This organism most likely is: a. Klebsiella pneumoniae b. Shigella dysenteriae c. Salmonella typhimurium d. Escherichia coli

c (Definition of leukocytosis.)

135) Elevation of the total white cell count above 12 x 10^3 is termed: a. relative lymphocytosis b. absolute lymphocytosis c. leukocytosis d. relative neutrophilic leukocytosis

b (Systemic lupus erythematosus [SLE] is an autoimmune disease. The ANA procedure is a screening tool for SLE.)

124) A patient has pancytopenia, decreased total serum iron, decreased serum iron binding capacity, and show a homogeneous fluorescence pattern with a high titer on a fluorescent anti-nuclear antibody test. This is suggestive of : a. polycythemia vera b. SLE c. iron deficiency anemia d. hemoglobin SC disease

a (Patients with recent streptococcal infections produce antibodies to several enzymes produced by Streptococcal bacteria. The antibodies detected most commonly in the laboratory are those directed against streptolysin O or DNase B.)

124) The most commonly used serological indicator of recent streptococcal infection is the antibody to: a. streptolysin "O" b. hyaluronidase c. NADase d. DNA

c (Iron deficiency anemia laboratory features include: decreased RBC, hemoglobin, MCV, MCH, MCHC, serum iron, serum ferritin % saturation, bone marrow iron stores; and increased RDW, TIBC, FEP and serum soluble transferrin receptor levels.)

125) In an uncomplicated case of severe iron deficiency anemia, which of the following sets represents the typical pattern of results? a. A b. B c. C d. D

d (Based on the time between collection and analysis of a urine specimen and the method of preservation, bacteria can be a heavy contaminant of urine. A positive LE test indicating the presence of WBCs confirms the actual presence of an infection.)

127) Bacteria are considered significant in the urine sediment when the: a. nitrite is positive b. protein is positive c. specimen is cloudy d. leukocytes is positive

b (Biological functions of lymphocytes; B cells make antibody, become plasma cells.)

127) Cells that are precursors of plasma cells and also produce immunoglobulins are: a. macrophages b. B lymphs c. T lymphs d. monocytes

c (Definition of leukoerythroblastosis.)

129) A term that means varying degrees of leukocytosis with a shift to the left and occasional nucleated red cells in the peripheral blood is: a. polycythemia vera b. erythroleukemia c. leukoerythroblastosis d. megaloblastoid

c (Yeast cells are commonly seen in urine specimens from persons with diabetes mellitus, because the high glucose content provides an excellent growth media for yeast.)

129) Which of the following positive chemical reactions is most closely associated with the presence of yeast in the urine sediment? a. nitrite b. protein c. glucose d. blood

d (Major site of destruction of senescent red blood cells.)

13) In the normal adult, the spleen acts as a site for: a. storage of RBCs b. production of RBCs c. synthesis of erythropoietin d. removal of imperfect and aging cells

a (Comparative mobilities due to structures of Hgb variants.)

130) Hemoglobin S can be separated from hemoglobin D by: a. electrophoresis on a different medium and acidic pH b. hemoglobin A2 quantitation c. electrophoresis at higher voltage d. Kleihauer-Betke acid elution

c (Sorbitol replaces lactose in MacConkey. E coli 0157:H7 does not ferment sorbitol, whereas other species of E coli are positive for fermentation of sorbitol. This makes the media a good screen for 0157:H7.)

130) MacConkey media for screening suspected cases of hemorrhagic E coli O157:H7 must contain: a. indole b. citrate c. sorbitol d. lactose

d (Many bacteria produce nitrites from nitrate. If the biochemical strip is positive for nitrite, you should find bacteria in the sediment.)

130) What of the following is consistent with this urine microscopic finding? a. ketone b. glucose c. specific gravity d. nitrate

a (Glucose in the urine of a person with a normal blood glucose is indicative of renal tubule damage or the inherited disorder, renal glycosuria, in which transport receptors are absent in the proximal convoluted tubules.)

131) A 21 yr. old woman had glucose in her urine with a normal blood sugar. These findings are most consistent with: a. renal glycosuria b. diabetes insipidus c. diabetes mellitus d. alkaline tide

d (Pyelonephritis is an infection involving the renal tubules. Therefore, the presence of WBC casts and bacteria aids in the diagnosis. Cystitis is an infection of the bladder and does not affect the tubules. RBC casts are the prominent finding with glomerulonephritis.)

135) The following urine results were obtained on a 25-year-old female: These results are most compatible with: a. glomerulonephritis b. renal calculus c. cystitis d. pyelonephritis

d (The presence of RBC casts is consistent with glomerulonephritis. WBCs, RBCs and protein are present as a result of the glomerular damage.)

136) A urinalysis performed on a 27-year-old woman yields the following results: These findings are most consistent with: a. yeast infection b. pyelonephritis c. bacterial cystitis d. glomerulonephritis

c (Definition of relative neutrophilic granulocytosis.)

136) Elevation of the granulocyte percentage above 75% is termed: a. absolute lymphocytosis b. leukocytosis c. relative neutrophilic leukocytosis d. absolute neutrophilic leukocytosis

d (TdT staining in lymphoblasts.)

138) Terminal deoxynucleotidyl tranferase is a marker found on: a. hairy cell b. myeloblasts c. monopolists d. lymphoblasts

b (Light-exposed bilirubin is oxidized [structurally altered].)

139) Serial bilirubin determinations are charted below. The best explanation for the results is: a. sample hemolysis and hemoglobin deterioration b. sample exposure to light c. sample left in warm location d. reagent deterioration

b (The probability of an observation having a value within +/- 2 standard deviations of the mean in a normal distribution is 95.5%. Therefore, 5 control values out of 100 would be out of control due to random error.)

139) The acceptable limit of error in the chemistry laboratory is 2 standard deviations. If you run the normal control 100 times, how many of the values would be out of the control range due to random error? a. 1 b. 5 c. 10 d. 20

b (The probability of an observation having a value within +/- 2 standard deviations of the mean in a normal distribution is 95.5%. Therefore, 100 +/- 2 x 1.8 = 95% confidence interval.)

140) A mean value of 100 and a standard deviation of 1.8 mg/dL were obtained from a set of glucose measurements on a control solution. The 95% confidence interval in mg/dL would be: a. 94.6 - 105.4 b. 96.4 - 103.6 c. 97.3 - 102.7 d. 98.2 - 101.8

c (Acute interstitial nephritis is caused by an allergic reaction, resulting in inflammation, not infection, of the renal tubules. Bacteria are not present in an inflammation.)

140) A specimen with a negative nitrate reaction and a positive leukocytes reaction that has WBCs, WBC casts and no bacteria in the sediment will be seen in cases of: a. cystitis b. pyelonephritis c. acute interstitial nephritis d. acute glomerulonephritis

a (Philadelphia chromosome composition.)

140) The Philadelphia chromosome is formed by a translocation between the: a. long arm of chromosome 22 and long arm of chromosome 9 b. long arm of chromosome 21 and long arm of chromosome 9 c. long arm of chromosome 21 and short arm of chromosome 6 d. long arm of chromosome 22 and short arm of chromosome 6

a (Blood selected for exchange transfusion should be antigen-negative and ABO compatible with the mother and baby. Red Blood Cells are usually less than 7 days old, CMV-, hemoglobin S-, and irradiated.)

141) Blood selected for exchange transfusion must: a. lack RBC antigens corresponding to maternal antibodies b. be <3 days old c. be the same Rh as the baby d. be ABO compatible with the father

b (The biochemical characteristic that best fits Shigella is that it is nonmotile. Shigella are urease negative and oxidase negative. Shigella are lactose nonfermenters.)

141) Shigella species characteristically are: a. urease positive b. nonmotile c. oxidase positive d. lactose fermenters

b (The presence of heavy bacteria with a negative leukocyte esterase and normal WBC numbers indicates the specimen has been collected >2 hours before being tested. Testing a fresh specimen will determine if bacterial multiplication has occurred in the first specimen.)

141) Urinalysis results on a female patient who brings a urine specimen to the physician's office for her annual physical are: What action should be taken? a. place the patient on a broad spectrum antibiotic for 7 days b. ask the patient to collect another specimen at the office c. tell the patient she will be called when the culture and sensitivity reports are back d. have the patient return in a week with a new specimen

d (Simple dilutions are ratios of 2 volumes, which involve a single substance diluted with one other substance. In this case, 0.25 mL solution A is added to 19.75 mL solution B [ratio 0.25/19.75], for a total volume of 20 mL. This represents a dilution of 0.25/20. To convert a 0.25/20 dilution to a 1-in-something dilution, set up a ratio" proportion calculation: 0.25 is to 20 as 1 is to x, and solve for x.)

141) When 0.25 mL is diluted to 20 mL, the resulting dilutions is: a. 1:20 b. 1:40 c. 1:60 d. 1:80

d (Some Shigella produce capsular antigen that mask the cell wall and boiling removes the capsule.)

142) A gram-negative bacillus has been isolated from feces, and the confirmed biochemical reactions fit those of Shigella. The organism does not agglutinate in Shigella antisera. What should be done next? a. test the organism with a new lot of antisera b. test with Vi antigen c. repeat the biochemical test d. boil the organism and retest with the antisera

b (The prefix oligo- means scanty. Knowing that the normal daily urine volume is around 1200 mL, 300 mL is scanty. The prefix an- means not, poly- means many, and dys- means pain.)

146) The volume of urine excreted in a 24 hr. period by an adult patient was 300 mL. This condition would be termed: a. anuria b. oliguria c. polyuria d. dysuria

b (The combination of these results is consistent with diabetes mellitus. The polyuria occurs due to the need to excrete the excess dissolved glucose in the urinary filtrate. Hyperglycemia is not present with renal glucosuria. Both stress and a heavy meal can cause transient glucosuria.)

147) A patient has glucosuria, hyperglycemia and polyuria. These findings are most consistent with: a. renal glucosuria b. diabetes mellitus c. emotional stress d. eating a heavy meal

a (This is a ratio calculation. 1:300 dilution equals 1 antigenic unit in 0.5mL. Therefore 2 antigenic units in 0.5mL equals a 1:150 dilution.)

147) If 0.5 mL of a 1:300 dilution contains 1 antigenic unit, 2 antigenic units would be contained in 0.5 mL of a dilution of: a. 1:150 b. 1:450 c. 1:500 d. 1:600

b (IgG highest in concentration in normal sera.)

147) Which of the following immunoglobulins is present in the highest concentration in normal human serum? a. IgM b. IgG c. IgA d. IgE

b (Although the normal glomerular filtration rate is 120 mL/min, tubular reabsorption returns normally all but 1 mL to the plasma.)

148) A normal glomerular filtration rate is: a. 1 mL/min b. 120 mL/min c. 660 mL/min d. 1200 mL/min

c (Normal plasma constituents that can be filtered by the glomerulus are water, urea and sodium chloride. Protein molecules are too large to normally pass the glomerulus. Bilirubin is not a normal constituent of plasma.)

149) Normal urine primarily consists of: a. water, protein, and sodium b. water, urea, and protein c. water, urea, and sodium chloride d. water, urea and bilirubin

d (Formed with oxidization of Fe and formation of methemoglobin.)

15) Heinz bodies are: a. readily identified with polychrome stains b. rarely found in glucose-6-phosphate dehydrogenase deficient erythrocytes c. closely associated with spherocytes d. denatured hemoglobin inclusion that are readily removed by the spleen

c (Definition of glycated hemoglobin.)

15) The glycated hemoglobin value represents the integrated values of glucose concentration during the preceding: a. 1-3 weeks b. 4-5 weeks c. 6-8 weeks d. 16-20 weeks

d (Cefoxitin is used as a surrogate for mecA-mediated oxacillin resistance in S aureus. S aureus with cefoxitin MICs >4 mg/mL are considered oxacillin resistant. The Clinical Laboratory Standards Institute [CLSI] recommends addition of 2% NaCl, incubation at 35 +/- 2 C, and incubation for 24 hours when performing susceptibility testing of S aureus against oxacillin.)

15) The procedure that assures the most accurate detection of mecA-mediated oxacillin resistance in routine broth microdilution susceptibility testing against S aureus is: a. addition of 4% NaCl b. incubation at 30 C c. incubation for 48 hrs. d. use of cefoxitin for testing

b (One dose of RhIg will protect the mother from a bleed of 30 mL. The bleed was 35 mL, 2 vials of RhIg will be needed)

150) The results of a Kleihauer-Butke stain indicate a fetomaternal hemorrhage of 35 mL of WHOLE BLOOD. How many vials of Rh immune globulin would be required. a. 1 b. 2 c. 3 d. 4

d (One vial of Rh immune globulin protects against a fetomaternal hemorrhage of 15 mL of red cells, or 30 mL of Whole Blood. Divide the volume of fetomaternal hemorrhage [35 mL] by 15; round down to 2, then add 1 extra vial = 3 vials total.)

151) A fetal maternal hemorrhage of 35 mL of fetal Rh-pos PACKED RBCs has been detected in an Rh-neg woman. How many vials of Rh immune globulin should be given a. 0 b. 1 c. 2 d. 3

b (Galactosuria is an inborn error of metabolism, resulting in the failure to inherit the one of the enzymes needed to metabolize dietary galactose to glucose. Byproducts of this metabolic failure are toxic, and can result in severe mental retardation. Early detection and dietary changes can prevent the toxicity.)

155) The reason for performing a Clinitest on a newborn's urine is to check for: a. fructose b. galactose c. glucose d. lactose

b (The nitroprusside/ferricyanide reagent strip reaction reacts with acetoacetic acid. Glycine must be present for the reaction to include acetone. beta-hydroxybutyric acid is present in the largest amount, but does not react with nitroferricyanide.)

157) Reagent strip tests for ketones measure primarily: a. acetone b. acetoacetic acid c. cholesterol d. beta-hydroxbutyric acid

d (Of the organisms listed only Proteus mirabilis is phenylalanine deaminase positive)

157) The stock cultures needed for quality control testing of phenylalanine deamination activity are: a. Escherichia coli / Klebsiella pneumoniae b. Salmonella typhimurium / Escherichia coli c. Escherichia coli / Pseudomonas aeruginosa d. Proteus mirabilis / Escherichia coli

a (The rosette test is a sensitive method to detect FMH of 10 mL or more.)

158) The rosette test will detect a fetomaternal hemorrhage as small as: a. 10 mL b. 15 mL c. 20 mL d. 30 mL

b (Klebsiella is the only distractor that is VP positive, and the other biochemical reactions are typical for K pneumoniae.)

134) A sputum culture from an alcoholic seen in the ER grows gray, mucoid, stringy colonies on SBA. The isolate grows readily on MacConkey agar and forms mucoid, dark pink colonies. The colonies yield the following test results: ONPG - positive indole - negative glucose - positive oxidase - negative citrate - positive VP - positive The organism is most likely: a. Edwardsiella tarda b. Klebsiella pneumoniae c. Escherichia coli d. Proteus vulgaris

d (Definition of absolute neutrophilic leukocytosis.)

134) Elevation of the total granulocyte count above 7.7 x 10^3 is termed: a. relative lymphocytosis b. leukocytosis c. relative neutrophilic leukocytosis d. absolute neutrophilic leukocytosis

a (Helicobacter pylori produces large amounts of extracellular urease and is positive for urea within 2 hours.)

167) A gastroenterologist submits a gastric biopsy from a patient with a peptic ulcer. To obtain presumptive evidence of Helicobacter pylori, a portion of the specimen should be added to which media? a. urea broth b. tetrathionate c. selenite d. tryptophan

d (White blood cells are often attached to the cast matrix as well as being imbedded in the matrix. White blood cells frequently occur in clumps and could can resemble a cast, but no cast matrix is observed. WBC casts indicate a more serious tubular infection, whereas WBC clumps can be seen in cystitis.)

100) To distinguish between a clump of WBCs and a WBC cast it is important to observe: a. the presence of free floating WBCs b. a positive leukocyte reaction c. a positive nitrite reaction d. the presence of a cast matrix

d (Organisms that used to be categorized as nutritionally variant or deficient streptococci have been reclassified into the genera Abiotrophia and Granulicatella.)

101) "Nutritionally deficient" streptococci are: a. enterococci b. group D nonenterococci c. cell wall-deficient streptococci d. in the genera Granulicatella and Abiotrophia

c (Calculating RBC indices.)

101) A patient has the following laboratory results: The mean corpuscular volume (MCV) of the patient is: a. 35 mm3 (35 fL) b. 83 mm3 (83 fL) c. 120 mm3 (120 fL) d. 150 mm3 (150 fL

b (Spherical urothelial cells appear similar to round tubular epithelial cells. The eccentric placement of the nucleus in renal tubular cells differentiates them from spherical urothelial cells, which have a centrally-placed nucleus.)

101) Spherical urothelial cells may be confused with: a. oval fat bodies b. renal tubular epithelial cell c. glitter cells d. lymphocytes

b (T cells and cell-mediated immunity are important in defense against intracellular pathogens such as viruses, fungi, and parasites. Patients with T-cell deficiencies will therefore exhibit increased parasitic infections.)

208) A patient with a T-cell deficiency will most likely exhibit: a. increased immune complex formation b. increased parasitic infections c. decreased IgE-mediated responses d. decreased complement levels

b (RBC production is inversely regulated by 02 levels. 02 would decrease in emphysema [hypoxia]; therefore, RBC levels would compensate, i.e., increase.)

182) Which of the following types of polycythemia is most often associated with emphysema? a. polycythemia vera b. polycythemia, secondary to hypoxia c. relative polycythemia associated with dehydration d. polycythemia associated with renal disease

d (Discriminates between faulty function of platelets, which would inhibit clotting and other PCV characteristics.)

183) Hemorrhage in polycythemia vera is the result of: a. increased plasma viscosity b. persistent thrombocytosis c. splenic sequestration of platelets d. abnormal platelet function

d (Obstructive jaundice is characterized by an increased ALP 3 times the upper limit of normal and a marked increase in GGT. The aminotransferases are slightly elevated owing to the fact that they are sensitive for acute hepatocellular conditions.)

183) This is most consistent with: a. acute hepatitis b. osteitis fibrosa c. chronic hepatitis d. obstructive jaundice

a (Hypochromic, microcytic RBC is most commonly associated with iron deficiency, which would most likely result from repeated therapeutic phlebotomies.)

184) A patient diagnosed with polycythemia vera 5 yrs. ago has a normal hematocrit, decreased hemoglobin, and microcytic, hypochromic red cells. What is the most probable cause for the current blood situation? a. phlebotomy b. myelofibrosis c. proleukemia d. aplastic anemia

d (Accumulation of fluid in the peritoneal cavity is called ascites, and it is collected by paracentesis.)

184) Ascites is collected by: a. thoracentesis b. lumbar puncture c. amniocentesis d. paracentesis

b (A method used for screening must have a high degree of sensitivity to detect everyone with the disease.)

184) The reliability of a test to be positive in the presence of the disease it was designed to detect is known as: a. accuracy b. sensitivity c. precision d. specificity

b (C5a is a chemotaxin for neutrophils, basophils, mast cells and monocytes.)

189) Potent chemotactic activity is associated with which of the following components of the complement system: a. C1q b. C5a c. C3b d. IgG

b (Agglutination at AHG phase indicates the presence of clinically significant antibody, indicating the need for antibody identification.)

189) Review the following schematic diagram: patient serum + reagent group O cells --> incubate --> read for agglutination --> wash --> add AHG --> agglutination observed --> ? The next step would be: a. add check cells as a confirmatory measure b. identify the cause of the agglutination c. perform an elution technique d. perform a DAT

a (Basis of chromatography.)

208) Chromatography is based on the principle of: a. differential solubility b. gravity c. vapor pressure d. temperature

a (Antibodies to antigens on cells 2, 3, 4, and 5 can be ruled out in tubes II and III, in which there was no reaction between patient serum and cells.)

209) An antibody identification study is performed with the 5-cell panel shown below: An antibody against which of the following antigens could not be excluded? a. 1 b. 2 c. 3 d. 4

a (A patient with blunt trauma, such as a car accident, may have internal bleeding. Peritoneal lavage introduces a fixed volume of saline into the peritoneal cavity, and withdraws an aliquot. RBCs are counted. Counts greater than 100,000/mL indicate blunt trauma.)

211) Peritoneal lavage is used to: a. detect intra-abdominal bleeding in blunt injury b. dialyze patients with end stage renal disease c. replace ascites with saline d. perform therapeutic thoracentesis.

b (Clinical history does not distinguish Neisseria gonorrhoeae from Chlamydia trachomatis. Because of the presence of nonpathogenic Neisseria in the female genital tract, Gram stain does not differentiate these organisms from N gonorrhoeae. Culture of an appropriate genital tract specimen is required to confirm identification.)

211) The diagnosis of Neisseria gonorrhoeae in females is best made from: a. clinical history b. an endocervical culture c. a gram stain of cervical secretions d. examination for clue cells

b (Increased TSH, decreased free T4 and total T4, positive microsomal Ab consistent with primary hypothyroidism.)

212) A patient has the following thyroid profile: total T4 = decreased free T4 = decreased thyroid peroxisase antibody = positive TSH = increased This patient most probably has: a. hyperthyroidism b. hypothyroidism c. a normal thyroid d. Graves disease

c (An ABO discrepancy in an A1 individual, manifested by agglutination in the serum grouping with A2 cells, is most likely due to anti-H. The greatest concentration of H substance is found on O cells, followed by A2 cells. The least amount of H substance is found on Al and AIB cells.)

215) The ABO discrepancy seen above is most likely due to: a. anti-A1 b. rouleaux c. anti-H d. unexpected IgG antibody present

a (Copper reduction reaction detects many reducing substances.)

23) An infant with diarrhea is being evaluated for a carbohydrate intolerance. His stool yields a positive copper reduction test and a pH of 5.0. It should be concluded that: a. further tests are indicated b. results are inconsistent-repeat both tests c. the diarrhea is not due to carbohydrate intolerance d. the tests provided no useful information

c (Rheumatoid factor is an autoantibody to the Fc portion of the immunoglobulin molecule.)

23) Rheumatoid factors are defined as: a. antigens found in the sera of patients with rheumatoid arthritis b. identical to the rheumatoid arthritis precipitin c. autoantibodies with specificity for the Fc portion of the immunoglobulin molecule d. capable of forming circulating immune complexes only when IgM-type autoantibody is present

b (A probable anaerobe is the only organism growing so the microbiologist can proceed with identification.)

230) An aspirate of a deep wound was plated on blood agar plates aerobically and anaerobically. At 24 hrs. there was growth on the anaerobic plate only. The next step in the evaluation of this culture is to: a. reincubate for another 24 hrs. b. begin organism identification c. issue the final report d. set up a Bauer-Kirby sensitivity

a (The normal variation of serum cortisol is higher at 8 AM than 4 PM .)

230) Plasma for cortisol determinations were collected at 7 am, after waking the patient, and at 10 pm that evening. The cortisol level of the morning sample was higher that the evening sample. This is consistent with: a. a normal finding b. Cushing syndrome c. Addison disease d. hypopituitarism

c (The patient's percent of CD4+ cells [8%] is well below the normal range of 50%-60%, and the patient's absolute number of CD4+ cells [5,000 x 0.15 x 0.08 = 60] is also far below the normal range of 500-1,300 cells/mL peripheral blood. These findings are consistent with AIDS, which is caused by HIV.)

231) Which of the following is the correct interpretation? a. CD4% and absolute CD4 normal b. consistent with an intact immune system c. consistent with a viral infection such as HIV d. technical error

d (A deficiency of thiamine [vitamin B1] known as beriberi, may be seen with chronic alcoholism in the U.S.)

232) Beriberi is associated with deficiency of vitamin: a. A b. C c. niacin d. thiamine

c (Patient education can alleviate anxiety.)

232) Several complaints have been received from parents of children in the pediatric wing about the anxiety that venipuncture causes their children. An informal staff meeting with the phlebotomists reveals that they feel both parents and pediatric nurses are less than supportive and frequently make the task of venipuncture in children worse with their own anxiety. The best course of action would be to: a. have pediatric nurses do venipuncture on children as they are more familiar with the children b. limit physicians to only one draw per day on children c. prepare written pamphlets for parents and in-service education for nursing personnel d. take no action as parents will always overreact where their children are concerned

b (Second and third generation cephalosporins react when the drug is present in vitro. When serum, drug, and red cells are present, direct or indirect agglutination or lysis may be observed.)

232) The drug cephalosporin can cause a positive DAT with hemolysis by which of the following mechanisms? a. drug dependent antibodies reacting with drug treated cells. b. drug dependent antibodies reacting in the presence of a drug c. drug independent with autoantibody production d. nonimmunologic protein adsorption with positive DAT

b (A deficiency of ascorbic acid [vitamin C] is called scurvy)

233) Scurvy is associated with deficiency of which of the following vitamins? a. A b. C c. niacin d. thiamine

d (A deficiency of vitamin D in children leads to rickets.)

234) Rickets is associated with deficiency of which of the following vitamins? a. B1 b. C c. niacin d. D

c (Parathyroid hormone [PTH] and the hormone vitamin D play a dominant role in calcium regulation.)

238) Which of the following hormones regulates normal blood calcium levels? a. thyroxine b. estriol c. parathyroid hormone d. growth hormone

d (Lead interferes with heme synthesis, which on a CBC may present as a decreased hemoglobin, with basophilic stippling of the red blood cells. These findings in a child may indicate lead toxicity. Whole blood lead is the recommended test; but urine delta-aminolevulinic acid and REC zinc protoporphyrin are also useful assays.)

263) A 3 yr. old child was evaluated for abdominal pain and anorexia by a physician. A CBC revealed a hemoglobin of 9.8 g/dL and basophilic stippling of RBCs. The doctor should order further tests to check for poisoning from: a. arsenic b. iron c. mercury d. lead

c (Nuclear maturation and division occurs first, and is largely complete before cytoplasmic maturation begins.)

263) Which of the following is characteristic of cellular changes as megakaryoblasts mature into megakaryocytes within the bone marrow? a. progressive decrease in overall cell size b. increasing basophilia of cytoplasm c. nuclear division without cytoplasmic division d. fusion of the nuclear lobes

b (Each unit of RBCs is expected to increase the hemoglobin level by 1-1.5 g/dL [10-15 g/L].)

264) A 42 year old male of average body mass has a history of chronic anemia requiring transfusion support. 2 units of RBCs are transfused. If the pretransfusion hemoglobin was 7.0 g/dL, the expected posttransfusion hemoglobin concentration should be: a. 8.0 g/dL (80 g/L) b. 9.0 g/dL (90 g/L) c. 10.0 g/dL (100 g/L) d. 11.0 g/dL (110 g/L)

b (Salicylate levels are used to determine if aspirin [acetylsalicylic acid] toxicity is present. Toxic serum or plasma levels are generally >300 mg/mL.)

269) A salicylate level is performed to detect toxicity caused by ingestion of excess: a. acetaminophen b. aspirin c. ibuprofen d. pseudoephedrine

d (The required temperature for storage of platelets is 20-24 C.)

27) Optimum storage temperature for platelets: a. -20 C b. -12 C c. 4 C d. 22 C

b (HLA antibodies are formed in response to pregnancy, transfusion or transplantation and are therefore not naturally occurring. They are associated with febrile nonhemolytic transfusion reactions and TRALI. They are directed against antigens found on granulocytes and other cells such as platelets.)

94) HLA antibodies are: a. naturally occurring b. induced by multiple transfusions c. directed against granulocyte antigen only d. frequently cause hemolytic transfusion reactions.

c (Optochin susceptibility is used to differentiate S pneumoniae, which are susceptible, from other alpha-hemolytic streptococci, which are resistant.)

94) The optochin (ethylhydrocupreine hydrochloride) disk is used for the identification of: a. Haemophilus influenzae b. group A beta-hemolytic streptococci c. Streptococcus pneumoniae d. alpha-hemolytic streptococci

d (MHC consists of both class I and class II HLA antigens. Discrimination of self from nonself is the primary function of the HLA system and involves many immune responses.)

95) Genes of the major histocompatibility complex: a. code for HLA-A,HLA-B, and HLA-C antigens only b. are linked to genes in the ABO system c. are the primary genetic sex-determinants d. contribute to the coordination of cellular and humoral immunity

d (Group O have the most H substance in their saliva. The person must also be a secretor of ABH substances. Due to gene interaction between the secretor gene and Lewis gene, people who are Le[a-b+] assures H in their saliva.)

97) Saliva from which of the following individuals would neutralize an auto anti-H in the serum of a group A, Le(a-b+) patient: a. group A, Le(a-b-) b. group A, Le(a+b-) c. group O, Le(a+b-) d. group O, Le(a-b+)

c (Staphylococcus aureus produces an enterotoxin that is associated with short- incubation food poisoning.)

97) The enterotoxin produced by certain strains of hemolytic, coagulase positive Staphylococcus aureus: a. is destroyed by boiling for 15-30 minutes b. is identical to the dermonecrotic toxin c. causes one type of bacterial food poisoning d. is highly antigenic

a (Electrophoretic pattern for sickle cell trait on cellulose acetate.)

98) Which electrophoresis pattern is consistent with sickle cell trait? a. pattern A b. pattern B c. pattern C d. pattern D

c (P02 electrode.)

319) An electrode has a silver/silver chloride anode and a platinum wire cathode. It is suspended in KCl solution and separated from the blood to be analyzed by a selectively permeable membrane. Such an electrode is used to measure which of the following? a. pH b. PCO2 c. PO2 d. HCO3

d (The total iron binding capacity [TIBC] is the amount of iron that transferrin and other minor iron binding proteins are capable of binding. In the measure of the TIBC, the molecules are saturated with iron [since they are only 30% saturated normally]. Magnesium carbonate is used to remove the excess unbound by adsorption.)

331) Magnesium carbonate is added in an iron binding capacity determinations in order to: a. allow color to develop b. precipitate protein c. bind with hemoglobin iron d. remove excess unbound iron

a (The laboratory profile for a patient with acute DIC is: increased PT, APTT, D-dimer and a decrease in platelets and hypofibrinogenemia.)

332) Acute DIC is characterized by: a. hypofibrinogenemia b. thrombocytosis c. negative D-dimer d. shortened thrombin time

b (Mycobacterium leprae cannot be cultured in vitro. A clinical diagnosis is made, supported by the presence of AFB in a biopsy specimen.)

334) In a suspected case of Hansen disease (leprosy), a presumptive diagnosis is established by: a. isolation of organisms on Lowenstein-Jensen medium b. detection of weakly acid-fast bacilli in infected tissue c. isolation of organisms in a cell culture d. detection of niacin production by the isolated bacterium

c (The laboratory profile for a DIC workup includes increased PT, APTT and D-dimers, with decreased fibrinogen and platelets.)

334) Patient results are as follows: This workup suggests: a. blood dot b. hemorrhage c. DIC d. HUS

d (PT thromboplastin reagents are prepared from recombinant or affinity purified tissue factor suspended in phospholipid mixed with a buffered O.025 M solution of calcium chloride.)

336) The prothrombin time test requires that the patient's citrated plasma be combined with: a. platelet lipids b. thromboplastin c. Ca++ and platelets and lipids d. Ca++ and thromboplastin

d (Unconjugated bilirubin solvent.)

337) Before unconjugated bilirubin can react with Ehrlich diazo reagent, which of the following must be added? a. acetone b. ether c. distilled water d. caffeine

b (Pigment production is controlled by an oxygen-dependent, light-inducible enzyme in photochromogenic mycobacteria.)

338) Photochromogens produce pigment when: a. kept in the dark at 22 C b. exposed to light for 1 hour c. grown in the presence of CO2 d. incubated with x-ray film

a (HCO3 and TCO2 in metabolic alkalosis.)

34) A patient is admitted to the emergency room in a state of metabolic alkalosis. Which of the following would be consistent with this diagnosis? a. high PCO2, increased HCO3 b. low PCO2, increased HCO3 c. high PCO2, decreased HCO3 d. low PCO2, decreased HCO3

d (The correct answer is d, flammability. For distractor a, reactivity is the right yellow quadrant. For distractor b, special reactivity is the lower white quadrant. For distractor c, health is the left blue quadrant.)

35) Using a common labeling system for hazardous material identification such as HMIS or NFPA 704, the top red quadrant represents which hazard? a. reactivity b. special reactivity c. health d. flammability

b (The indole test is used for determining an organism's ability to produce indole from deamination of tryptophan by tryptophanase.)

35) When performing a Kovac indole test, the substrate must contain: a. indole b. tryptophan c. ornithine d. paradimethylaninobenzaldehyde

a (The chromogenic anti-Xa assay is the only assay available to monitor LMWH; it may also be used to measure UFH. The APTT cannot be used to monitor low-molecular-weight heparin therapy.)

353) Low molecular weight heparin is monitored by a: a. anti-Xa assay b. APTT c. PT d. anti-IIa assay

a (Osmometry of serum and other body fluids is commonly measured by freezing-point depression, using a freezing point osmometer.)

353) The osmolality of a urine or serum specimen is measured by a change in the: a. freezing point b. sedimentation point c. midpoint d. osmotic pressure

b (Appearance of WBC histogram.)

371) Which area in the automated cell counter histogram indicates the non-lymphocyte curve? a. B b. C c. D d. E

a (Tubes that remain at room temperature for >5 hours have unacceptable blood cell artifacts. This will affect the Hct and the MCV.)

380) Blood collected in EDTA undergoes which of the following changes if kept at room temp for 6 - 24 hrs. a. increased Hct and MCV b. increased ESR and MCV c. increased MCHC and MCV d. decreased reticulocyte count and Hct

d (Falsely prolonged results will occur if there is too much anticoagulant for the plasma; this also occurs with an increased Hct. Conversely, if a tube is overdrawn, there may be too little anticoagulant, resulting in a clot.)

387) A blue top tube is drawn for coagulation studies, the sample is a short draw results may be: a. falsely shortened b. correct c. unable to be obtained d. falsely prolonged

b (The ova of Enterobius vermicularis cannot be demonstrated in a routine ova and parasite examination. The adult female Enterobius worm migrates out of the anus, and lays her eggs in the perianal folds. A scotch tape preparation of the skin of the perianal folds is used to collect ova.)

392) The best method to demonstrate the ova of Enterobius vermicularis is: a. acid-ether concentration b. cellophane tape preparation c. formalin-ether concentration d. zinc sulfate flotation

c (The ova of Enterobius vermicularis cannot be demonstrated in a routine ova and parasite examination. The adult female Enterobius worm migrates out of the anus, and lays her eggs in the perianal folds. A scotch tape preparation of the skin of the perianal folds is used to collect ova.)

393) Proper collection of a sample for recovery of Enterobius vermicularis includes collecting: a. a 24-hour urine collection b. a first morning stool collection with proper preservative c. a scotch tape preparation from the perianal region d. peripheral blood from a finger

a (Humans may become infected with Taenia solium by either ingesting the larval form or ova. If ova are ingested the parasite cannot complete the life cycle, and cysticerci encyst in various tissues including the brain.)

398) The causative agent of cysticercosis is: a. Taenia solium b. Taenia saginata c. Ascaris lumbricoides d. Trichuris trichiura

b (Normal 2 hour postprandial value.)

4) If a fasting glucose was 90 mg/dL, which of the following 2 hr. postprandial glucose result would most closely represent normal glucose metabolism? a. 55 mg/dL (3.0 mmol/L) b. 100 mg/dL (5.5 mmol/L) c. 180 mg/dL (9.9 mmol/L) d. 260 mg/dL (14.3 mmol/L)

a (The scolex of Taenia saginata is square with 4 suckers and no hooks.)

400) The scolex of Taenia saginata has: a. 4 suckers b. no suckers and 14 hooklets c. 24 hooklets d. 26 - 28 sucking discs

d (The disk diffusion procedure will not differentiate S aureus strains with reduced susceptibility to vancomycin [MICs 4 -8 mg/mL] from susceptible stains even when incubated for 24 hours.)

42) Which of the following methods is inadequate for the detection of vancomycin-intermediate S aureus? a. broth macrodilution b. agar dilution c. gradient diffusion d. disk diffusion

c (Ion selective electrode standard Na K.)

47) Quantitation of Na+ and K+ by ion selective electrode is the standard method because: a. dilution is required for flame photometry b. there is no lipoprotein interference c. of advances in electrochemistry d. of the absence of an internal standard

a (Hereditary spherocytosis-lab results.)

52) Lab findings in hereditary spherocytosis do not include: a. decreased osmotic fragility b. increased autohemolysis corrected by glucose c. reticulocytosis d. shortened erythrocyte survival

b (As more antigen is added, the reaction moves to antigen excess. Precipitation does not occur due to lack of free antibody. Upon the addition of antigen to a fixed quantity of antibody, immune complexes start forming immediately.)

52) Soluble immune complexes are formed under the condition of: a. antigen deficiency b. antigen excess c. antibody excess d. complement

d (Maintenance of buffering capacity blood.)

53) The buffering capacity of blood is maintained by a reversible exchange process between bicarbonate and: a. sodium b. potassium c. calcium d. chloride

d (Cation in osmotic pressure.)

55) Which of the following electrolytes is the chief plasma cation whose main function is maintaining osmotic pressure? a. chloride b. calcium c. potassium d. sodium

b (Some bacteria such as Enterococcus sp., H influenzae and Neisseria gonorrhoeae continually produce beta-lactamase. S aureus produces beta-lactamase only after exposure to an inducing agent [such as penicillin]. C diphtheriae and S pyogenes do not produce beta-lactamase.)

64) Production of beta-lactamase is inducible in which of the following: a. Haemophilus influenzae b. Staphylococcus aureus c. Corynebacterium diphtheriae d. Streptococcus pyogenes

d (The correct answer is d, use only UL or other safety agency-rated electrical equipment. For distractor a, multiple outlet or gang plug adapters are not to be used. For distractor b, there is no requirement to change circuit breakers annually. For distractor c, worn wiring is to be replaced and not taped over.)

65) To help prevent electrical fires in healthcare facilities: a. use multiple outlet or gang plug adapters b. change circuit breakers annually c. tag over worn wiring with certified electrical tape d. use only UL or other safety-agency-rated electrical equipment

c (The correct answer is c, remove persons from immediate danger. The other distractors are correct, but are secondary actions. The sequence of actions after removing persons from immediate danger is to pull the fire alarm, contain the fire and evacuate the area, if required.)

68) A fire occurs in the laboratory. the first course of action is to: a. evacuate the area b. pull the fire alarm box c. remove persons from immediate danger d. contain the fire by closing doors

b (Reciprocal relationship of Ca++ and phosphate.)

68) A reciprocal relationship exists between: a. sodium and potassium b. calcium and phosphate c. chloride and CO2 d. calcium and magnesium

c (Newborn infants have elevated reticulocyte counts.)

71) The mean value of a reticulocyte count on a specimen of cord blood from a healthy, full term newborns is approximately: a. 0.5 % b. 2.0 % c. 5.0 % d. 8.0 %

d (Laboratory results in hyperthyroidism.)

71) These results are most compatible with: a. multiple myeloma b. milk-alkali syndrome c. sarcoidosis d. primary hyperparathyroidism

a (Anti-f will react with cells that carry c and e on the same Rh polypeptide. No other listed genotypes produce an Rh polypeptide that carries both c and e.)

71) Which of the following phenotype will react with anti-f: a. rr b. R1R1 c. R2R2 d. R1R2

b (Direct exclusion of paternity is established when a genetic marker is present in the child but is absent from the mother and the alleged father.)

85) In a relationship testing case the child has a genetic marker that is absent in the mother and cannot be demonstrated in the alleged father. What type of paternity exclusion is this: a. indirect b. direct c. prior probability d. Hardy-Weinbery

a (The colony description and biochemical results presented describe Streptococcus agalactiae. The identification of this organism is confirmed by streptococcus antigen typing.)

102) After 24 hrs. a blood culture from a newborn grows catalase-negative, gram-positive cocci. The bacterial colonies are small, translucent and beta-hemolytic on a blood agar plate. Biochemical test results of a pure culture are: bacitracin: resistant CAMP reaction: positive bile esculin: not hydrolyzed 6.5% NaCl broth: no growth Assuming that all controls react properly and reactions are verified, the next step would be to: a. perform a Streptococcus group typing b. report the organism as Streptococcus pneumoniae c. report the organism as Staphylococcus aureus d. report the organism as Staphylococcus epidermidis

a (Before reporting a red blood cell cast, it is essential to observe free-floating RBCs in the sediment. A coarsely granular cast may sometimes resemble a red blood cell cast. Without the presence of free red blood cells, a red blood cell cast could not have formed in the tubules.)

102) Prior to reporting a RBC cast, it is important to observe: a. free floating RBCs b. hyaline casts c. granular casts d. increased WBCs

b (Arterial vs venous glucose values.)

12) In the fasting state, the arterial and capillary blood glucose concentration varies from the venous glucose concentration by approximately how many mg/dL (mmol/L) a. 1 mg/dL (0.05 mmol/L) b. 5 mg/dL (0.27 mmol/L) c. 10 mg/dL (0.55 mmol/L) d. 15 mg/dL (0.82 mmol/L)

c (The creatinine clearance is a filtration rate, and is reported in minutes. The sample is a 24-hour urine, which is entered into the calculation as the volume in milliliters, and the factor of 1,440 minutes per 24 hours is applied.)

12) The creatinine clearance is reported in: a. mg/dL b. mg/24hrs c. mL/min d. mL/24hrs

b (Structural definition of a peptide bond.)

121) Which of the following is an example of a peptide bond? a. A b. B c. C d. D

c (RhIg should be given to nonimmunized D- females who are pregnant or have delivered a D+ infant.)

152) Criteria determining Rh immune globulin eligibility include: a. mother is Rh-pos b. mother is Rh-neg c. mother has not been previously immunized to the D antigen d. infant has a positive DAT

b (All of the biochemical and serological reactions listed are consistent with an identification of Shigella flexneri, with the exception of motility. All Shigella are nonmotile.)

16) When performing a stool culture a colony type typical of an enteric pathogen is subcultured on a blood agar plate. The resulting pure culture is screened with several tests to obtain the following results: The serological typing is verified with new kit and controls. The best course of action would be to: a. report the organism as Shigella flexneri without further testing b. verify reactivity of motility medium with positive and negative controls c. verify reactivity of the TSI slants with positive and negative controls for H2S production d. verify reactivity of phenylalanine deaminase with positive and negative controls

b (Identifies classic cellular findings in CML.)

160) Increased numbers of basophils are often seen in: a. acute infections b. chronic myelocytic leukemia c. chronic lymphocytic leukemia d. erythroblastosis fetalis (hemolytic disease of the newborn)

b (The transferases, alanine aminotransferase [ALT] and AST are located primarily in the liver. Elevated serum levels of the enzymes are seen in hepatocellular disorders. The levels may be 100 times the upper limit of normal. The ALT level is usually higher than AST. Increased levels of AST are also seen in infectious mononucleosis and muscular dystrophy, but ALT is not elevated in the clinical disorders.)

162) Aspartate aminotransferase and alanine aminotransferase are both elevated in which of the following diseases? a. muscular dystrophy b. viral hepatitis c. pulmonary emboli d. infectious mononucleosis

b (Growth only on chocolate agar is typical for Haemophilus influenzae, which is a gram-negative coccobacillus that causes upper respiratory infections.)

163) A small, gram-negative bacillus is isolated from an eye culture. It grows only on chocolate agar and is oxidase-variable. The most likely organism is: a. Acinetobacter lwoffii b. Haemophilus influenzae c. Stenotrophomonas maltophilia d. Pseudomonas aeruginosa

b (HLA antigen typing is important to consider before organ transplantation.)

163) HLA antigen typing is important in screening for: a. ABO incompatibility b. a kidney donor c. Rh incompatibility d. a blood donor

a (Demonstrates difference between acute vs chronic leukemias; intermediate cell maturity seen in chronic myelogenous, but would not predominate in erythroleukemia.)

178) All stages of neutrophils are most likely to be seen in the peripheral blood of patient with: a. chronic myelocytic leukemia b. myelofibrosis with myeloid metaplasia c. erythroleukemia d. acute myelocytic leukemia

b (The CSF/serum albumin ratio with a value of <9 indicates an intact blood-brain barrier. The other distractors have different purposes: a the IgG index compares IgG in CSF and serum to determine if IgG is being synthesized in the CNS; c the fluid/serum LD ratio is used to determine if a body fluid is a transudate or an exudate; d the albumin gradient is used to determine if an effusion is of hepatic origin.)

216) What calculation is used to determine if there is a breach in the blood brain barrier? a. IgG index b. CSF / serum albumin index c. fluid / serum LD ratio d. albumin gradient

b (Each unit of platelets should increase the count 5,000-10,000/uL [5,000-10,000/L].)

272) What increment of platelets/uL in the typical 70-kg human, is expected to result from each single unit of platelets transfused to a non-HLA sensitized recipient? a. 3,000 - 5,000 b. 5,000 - 10,000 c. 20,000 - 25,000 d. 25,000 - 30.000

d (Factor VII [proconvertin] is a single- chain glycoprotein that is Vitamin K dependent and remains stable 4-5 hours in blood. Produced in the liver, it has the shortest half-life; therefore, it is the first factor affected when a Vitamin K antagonist such as warfarin is administered)

307) Which of the following is Vitamin-K dependent? a. Factor XII b. fibrinogen c. antithrombin III d. Factor VII

c (A delay of more than 15 minutes between placing the disks on an inoculated plate and incubation permits excess prediffusion of the antimicrobial agent from the disk. This would result in a larger than expected zone diameter.)

43) In a disk diffusion susceptibility test, which of the following can result if disks are placed on the inoculated media and left at room temp for an hour before incubation? a. the antibiotic would not diffuse into the medium, resulting in no zone b. zones of smaller diameter would result c. zones of larger diameter would result d. there would be no effect on the final zone diameter

a (Folate deficiency-peripheral smear will show abnormally large RBCs.)

43) The characteristic morphologic feature in folic acid deficiency is: a. macrocytosis b. target cells c. basophilic stippling d. rouleaux formation

d (Clots in the unit may indicate contamination.)

43) Upon inspection, a unit of platelets is noted to have visible clots, but otherwise appears normal. the tech should: a. issue without concern b. filter to remove clots c. centrifuge to express off the clots d. quarantine for gram stain and culture

b (The change in pH is due to breakdown of urea to ammonia by urease producing bacteria, and loss of CO2.)

43) Upon standing at room temp, a urine pH typically: a. decreases b. increases c. remains the same d. changes depending on bacterial concentration

c (Susceptibility testing should be performed when the susceptibility of the organism cannot reliably be predicted and resistance is known or suspected. Susceptibility testing of penicillins for treatment of S pyogenes does not need to be performed routinely since resistance has not been documented.)

47) Which of the following organisms does not require susceptibility testing to the antimicrobial indicated when isolated from a clinically significant source? a. Staphylococcus Aureus - clindamycin b. Proteus mirabilis - gentamicin c. Streptococcus pyogenes - penicillin d. Escherichia coli - levofloxacin

d (Reference range sweat chloride.)

58) A sweat chloride result of 55 mEq/L (55 mmmol/L) and a sweat sodium of 52 mEq/L (52 mmol/L) were obtained on a patient who has a history of respiratory problems. The best interpretation of these results is: a. normal b. normal sodium and an abnormal chloride test should be repeated c. abnormal results d. borderline results, the test should be repeated

b (The correct answer is b, contain the fire by closing the doors. For distractor a, it is the last action to be taken and only if indicated. For distractor c, a type A extinguisher is for paper, wood and plastics fires, not electrical fires. Also, fire extinguishers only should be used by those who have received appropriate training. For distractor d, it is unnecessary and unacceptable to lock windows and especially doors, which would block access to firefighters.)

69) An electrical equipment fire breaks out in the lab. Personnel have been removed from immediate danger, the alarm has been activated. What is the next action to be taken? a. evacuated the facility b. contain the fire by closing doors c. extinguish fire with type A extinguisher d. lock all windows and doors in the immediate area

d (The prefix py- means pus [leukocytes]. The suffix -uria means pertaining to urine.)

76) The presence of leukocytes in the urine is known as: a. chyluria b. hematuria c. leukocytosis d. pyuria

c (Consider whether a substance can increase or decrease outside the body. No more blood can be produced. Although the RBC may rupture, they will still make a positive result on a biochemical strip. The amount of solutes won't change, so specific gravity won't change. pH is affected by metabolism of the urine components by bacteria, and room temperature is warm enough for this to occur. Protein will not increase or decrease.)

8) Which of the following would be affected by allowing a urine specimen to remain at room temp for 3 hrs. before analysis: a. occult blood b. specific gravity c. pH d. protein

d (The correct answer is d, perform frequent and appropriate hand hygiene. The other distractors are methods for preventing spread of infection but are not the most effective method.)

86) What is the single most effective method to prevent nosocomial spread of infection? a. wear mask, gown and gloves b. require infectious patients to mask c. wear a N95 respirator mask d. perform frequent and appropriate hand hygiene

d (Testing tor syphilis was the first mandated donor screening test for infectious disease and is still part of donor screening.)

9) All donors blood testing must include: a. complete Rh phenotype b. anti-CMV testing c. DAT d. serological test for syphilis

d (Measure the total volume of the sample before removing an aliquot. To calculate the total protein, measure the protein of an aliquot to learn the mg/dL. Then multiply that answer by the number of dL in the 24-hour collection.)

1) After receiving a 24 hr. urine for quantitative total protein analysis the technician must first: a. subculture the urine for bacteria b. add the appropriate preservative c. screen for albumin using a dipstick d. measure the total volume

a (Factors contributing to PBS.)

10) A 45 yr. old woman has a fasting serum glucose concentration of 95 mg/dL (5.2 mmol/L) and a 2 hr. postprandial glucose concentration of 105 mg/dL (5.8 mmol/L). The statement which best describes this patient's fasting serum glucose concentration is: a. normal; reflecting glycogen breakdown by the liver b. normal; reflecting glycogen breakdown by skeletal muscle c. abnormal; indication diabetes mellitus d. abnormal; indicating hypoglycemia

b (Platelets are prepared and stored at 20-24 C for optimum function.)

10) During the preparation of Platelet Concentrates from whole blood, the blood should be: a. cooled towards 6 C b. cooled towards 20-24 C c. warmed to 37 C d. heated to 57 C

d (Functional form of Fe [reduced].)

10) In order for hemoglobin to combine reversibly with oxygen, the iron must be: a. complexed with haptoglobin b. freely circulating in the cytoplasm c. attached to transferrin d. in the ferrous state

d (Standard deviation is a measure of the dispersion of data around the mean.)

10) The mean value of a series of hemoglobin controls was found to be 15.2 g/dL, and the standard deviation was calculated at 0.20. Acceptable control range for the laboratory is +/- 2 standard deviations. Which of the following represents the allowable limits for the control? a. 14.5 - 15.5 g/dL b. 15.0 - 15.4 g/dL c. 15.2 - 15.6 g/dL d. 14.8 - 15.6 g.dL

a (Columbia CNA agar contains colistin and nalidixic acid, which inhibit most gram-negative organisms. Eosin methylene blue is selective and inhibits gram-positive organisms and modified Thayer Martin is selective and inhibits gram-positive organisms, gram-negative bacilli and yeast.)

10) Which selective medium is used for the isolation of gram-positive microorganisms? a. Columbia CNA with 5% sheep blood b. trypticase soy agar with 5% sheep blood c. eosin methylene blue d. modified Thayer-Martin

a (Bile esculin, PYR, bacitracin and hippurate are biochemicals/tests used in the presumptive or definitive identification of beta-hemolytic streptococci such as S pyogenes, S agalactiae and Enterococcus.)

100) Gram-positive cocci in chains are seen on a Gram stain from a blood culture. The organism grows as a beta-hemolytic colony. Further tests that could be performed include: a. bile esculin, PYR, bacitracin, and hippurate b. catalase and coagulase c. oxidase and deoxyribonuclease d. Voges-Proskauer and methyl red

d (Principle of method of analysis.)

100) The first step in the quantitation of serum iron is: a. direct reaction with appropriate chromogen b. iron saturation of transferrin c. free iron precipitation d. separation of iron from transferrin

a (Using RBC indices to classify anemia.)

100) The lab findings on a patient are as follows: MCV = 55 um MCHC = 25% MCH = 17 pg A stained blood film of this patient would most likely reveal a red cell picture that is: a. microcytic, hypochromic b. macrocytic, hypochromic c. normocytic, normochromic d. microcytic, normochromic

b (IgM is the first antibody to be produced during an immune response, and levels decline within 6-12 months; it is therefore an indicator of a current infection.)

101) Which of the following is the best indicator of an acute infection with the hepatitis A virus? a. the presence of IgG antibodies to hepatitis A virus b. the presence of IgM antibodies to the hepatitis A virus c. a sharp decline in the level of IgG antibodies to hepatitis A virus d. a rise in both IgM and IgG levels of antibody to hepatitis A virus

a (Patients with the autoimmune disease, lupus erythematosus, frequently produce antibody against cardiolipin, the same antibody that is used to screen for syphilis by the VDRL test.)

102) Biological false-positive VDRL reactions are frequently encountered in patients with: a. lupus erythromatosus b. AIDS c. gonorrhea d. tertiary syphilis

d (Any turbidity in the cyanmethemoglobin reagent- patient specimen will result in falsely elevated values.)

102) The following results were obtained from a post-surgical patient receiving total parenteral nutrition: The most consistent explanation for the above data is: a. acute surgical bleeder b. specimen on day 19 from wrong patient c. improperly mixed specimen on day 19 d. lipid interference on days 17 and 18

a (Of the biochemicals listed, only growth in 6.5% NaCl will aid in the identification of Enterococcus, which has the ability to grow in the presence of high salt concentrations.)

103) A beta-hemolytic streptococcus that has been isolated from an ear culture grows up to the edge of a 0.04 unit bacitracin disk. Which of the following tests would help to determine if the organism is Enterococcus? a. growth in 6.5% NaCl broth b. growth in the presence of penicillin c. optochin susceptibility d. fermentation of 10% lactose

d (Autoagglutination of anticoagulated blood can occur at room temperature in patients with a cold autoagglutinin. The MCV will be falsely elevated and the count falsely decreased, resulting in an elevated MCHC. The blood sample should be warmed to 37 C and rerun.)

103) A patient has a high cold agglutinin titer. Automated cell counter results reveal an elevated MCV, MCH and MCHC. Individual erythrocytes appear normal on a stained smear, but agglutinates are noted. The appropriate course of action would be to: a. perform the RBC, Hgb and Hct determinations using manual methods b. perform the RBC determination by a manual method; use the automated results for the Hgb and Hct c. repeat the determination using a microsample of diluted blood d. repeat the determinations using a prewarmed microsample of diluted blood

d (The renal tubular cells lining the tubules absorb the urinary filtrate, and therefore will appear bile-stained.)

103) In a specimen with a large amount of bilirubin, which of the following sediment constituents would be most noticeably bile-stained? a. squamous epithelial cells b. WBC casts c. cystine crystals d. renal tubular epithelial cells

a (The M and N antigens are found on glycophorin A.)

103) Which of the following red cell antigens are found on glycophorin-A: a. M,N b. Lea, Leb c. S, s d. P, P1,pk

c (Antibody to the surface antigen of hepatitis B virus [anti-HBs] appears after the acute stage of infection during convalescence and is a marker of recovery and immunity, while the other markers listed are components of the virus itself.)

103) Which serological marker of HBV infection indicates recovery and immunity? a. viral DNA polymerase b. HBe antigen c. anti-HBs d. HbsAg

d (Autoagglutination of anticoagulated blood can occur at room temperature in patients with a cold autoagglutinin. The MCV will be falsely elevated and the RBC count falsely decreased, resulting in an elevated MCHC. The blood sample should be warmed to 37 C and rerun.)

104) A blood sample from a patient with a high titer cold agglutinin, analyzed at room temp, with an electronic particle counter would cause an error in the: a. Hgb and MCV b. MCHC and WBC c. WBC and RBC d. MCV and MCHC

c (Autoanti-P, a cold-reactive IgG autoantibody described as a biphasic hemolysin, is associated with paroxysmal cold hemoglobinuria.)

104) Paroxysmal cold hemoglobinuria is associated with antibody specificity toward which of the following: a. Kell system antigens b. Duffy system antigens c. P antigen d. I antigen

c (All of the organisms listed are potential causes of meningitis. Group B Streptococcus is associated with neonatal meningitis and meningitis in the elderly.)

104) The organism most commonly associated with neonatal purulent meningitis is: a. Neisseria meningitidis b. Streptococcus pneumoniae c. group B streptococci d. Haemophilus influenzae

a (RBC casts, protein and RBCs together are indicative of glomerulonephritis. RBC casts form when there is bleeding in the glomerulus and tubules.)

104) What is the most likely diagnosis given this microscopic finding? a. glomerulonephritis b. pyelonephritis c. nephrotic syndrome d. cystitis

c (Amorphous phosphates are found in alkaline urine. Under conditions such as refrigeration, they produce a white precipitate. Urates produce a pink precipitate, and WBCs and bacteria do not precipitate.)

105) A white precipitate in a urine specimen with a pH of 7.5 would most probably be caused by: a. amorphous urates b. WBCs c. amorphous phosphates d. bacteria

d (Streptococcus pyogenes is the cause of exudative pharyngitis, commonly called strep throat.)

105) An important cause of acute exudative pharyngitis is: a. Staphylococcus aureus (beta-hemolytic) b. Streptococcus pneumoniae c. Streptococcus agalactiae d. Streptococcus pyogenes

c (Treponemal tests, such as the FTA-ABS, remain positive throughout the course of syphilis [except for IgM, which is only positive in the early stages], while nontreponemal tests are generally nonreactive in the late stages of disease.)

105) For diagnosis of late latent or tertiary syphilis the most appropriate assay is: a. RPR b. VDRL c. FTA-ABS d. FTA-ABS IgM

c (Specimen requirements for analyte stability.)

105) To assure an accurate ammonia level result, the specimen should be: a. incubated at 37 C prior to testing b. spun and separated immediately, tested as routine c. spun, separated, iced, and tested immediately d. stored at room temp. until tested

a (Autoagglutination of anticoagulated blood can occur at room temperature in patients with a cold autoagglutinin. The MCV will be falsely elevated and the RBC count falsely decreased, resulting in an elevated MCHC. The blood sample should be warmed to 37 C and rerun.)

105) When using an electronic cell counter, which of the following results can occur in the presence of a cold Agglutinin: a. increased MCV and decreased RBC b. increased MCV and normal RBC c. decreased MCV and increased MCHC d. decreased MCV and RBC

b (Patients with infectious mononucleosis often demonstrate potent examples of anti-i that are transient in nature.)

105) Which of the following is a characteristic of anti-i? a. associated with warm autoimmune hemolytic anemia b. found in the serum of patients with infectious mononucleosis c. detected at lower temps in the serum of normal individuals d. found only in the serum of group O individuals

b (The nurse's history and symptoms suggest that she has hepatitis. The liver enzymes, alanine aminotransferase [ALT] and aspartate aminotransferase [AST] are elevated in hepatitis as general indicators of liver inflammation.)

106) A 26 yr. old nurse developed fatigue, a low grade fever, polyarthritis and urticaria. 2 months earlier she had cared for a patient with hepatitis. Which of the following findings are likely to be observed in this nurse? a. a negative hepatitis B surface antigen b. elevated AST and ALT levels c. a positive rheumatoid factor d. a positive Monospot test

c (Anti-I is commonly found in all individuals, but when it causes hemolysis, the titer may be high and react at all temperatures. Cold agglutinin syndrome is mainly found in lymphoproliferative diseases.)

106) In a case of cold autoimmune hemolytic anemia, the patient's serum would most likely react 4+ at immediate spin with: a. group A cells, B cells and O cells, but not his own cells b. cord cells but not his own or other adult cells c. all cells of a group O cell panel and his own cells d. only penicillin treated panel cells, not his own

a (LAP activity may be increased in polycythemia Vera.)

106) In polycythemia vera, the leukocyte alkaline phosphatase activity is: a. elevated b. normal c. decreased

d (Staphylococcus epidermidis is the most common cause of prosthetic valve endocarditis.)

106) Of the following bacteria, the most frequent cause of prosthetic heart valve infections occurring within 2-3 months after surgery is: a. Streptococcus pneumoniae b. Streptococcus pyogenes c. Staphylococcus aureus d. Staphylococcus epidermidis

a (Cystine crystals appear as hexagonal plates, frequently in dumps. Tyrosine crystals are needle-shaped, leucine crystals are round, and cholesterol crystals are flat with notched corners.)

106) Which of the following is an abnormal crystal described as a hexagonal plate? a. cystine b. tyrosine c. leucine d. cholesterol

c (Anti-I is associated with cold agglutinin syndrome.)

107) Cold agglutinin syndrome is associated with an antibody specificity toward which of the following: a. Fy:3 b. P c. I d. Rh:1

a (The immune response to HAV follows the classic pattern for an antibody response, with IgM appearing first, followed by a decline in IgM and appearance of IgG.)

107) The classic antibody response pattern following infection with hepatitis A is: a. increase in IgM antibody; decrease in IgM antibody, increase in IgG antibody b. detectable presence of IgG antibody only c. detectable presence of IgM antibody only d. decrease in IgM antibody, increased in IgG antibody of the IgG3 subtype

c (The number of colonies isolated is multiplied by 100 when a 0.01 mL loop is used for inoculation. Gram-positive, catalase negative cocci are indicative of streptococci.)

108) A catheterized urine is inoculated onto blood and MacConkey agar using a 0.01 mL loop. After 48 hrs., 68 colonies of a small translucent nonhemolytic organism grew on blood agar but not MacConkey. Testing reveals small gram-positive, catalase-negative cocci. The preliminary report and follow-up testing would be: a. growth of 680 colonies/mL of gram-positive cocci, optochin and bacitracin susceptibility tests to follow b. Growth of 6,600 colonies/mL of a Streptococcus species, coagulase test to follow c. growth of 6,800 colonies/mL of a Streptococcus species, esculin hydrolysis and NaCl growth test to follow d. growth of 6,800 colonies/mL of a Streptococcus species, no tests to follow

b (Creatinine measurement. Kidney function test.)

108) A serum sample demonstrates an elevated result when tested with the Jaffe reaction. This indicates: a. prolonged hypothermia b. renal functional impairment c. pregnancy d. arrhythmia

b (Calculation of RBC indices.)

112) What is the MCHC if the Hct is 20%, the RBC is 1.5 x 10^6/mL (1.5 x 10^12/L) and the Hgb is 6 g/dL (60 g/L)? a. 28% b. 30% c. 40% d. 75%

a (Amorphous urates will dissolve when the specimen is briefly warmed. Amorphous phosphates are dissolved by the addition of acid, which will also destroy other sediment constituents.)

108) After warming, a cloudy urine clears. This is due to the presence of: a. urates b. phosphates c. WBCs d. bacteria

a (Hepatitis B surface antigen [HbsAg] is located in the outer envelope of the virus.)

108) Refer to the following illustration of the hepatitis B virus: Select the corresponding lettered component indicated on the diagram for surface antigen. a. A b. B c. C d. D

a (Calculation of RBC indices.)

108) What is the MCH if the Hct is 20%, the RBC is 2.4 x 10^6/mL (2.4 x 10^12/L) and the Hgb is 5 g/dL (50 g/L)? a. 21 mm3 (21 fL) b. 23 mm3 (23 fL) c. 25 mm3 (25 fL) d. 84 mm3 (84 fL)

b (Anti-i is an IgM antibody that reacts with cord cells and i adult cells. It is not associated with hemolytic disease of the newborn since IgM antibodies do not cross the placenta.)

108) Which of the following is a characteristic of anti-i? a. often associated with HDFN b. reacts best at room temp or 4 C c. reacts best at 37 C d. is usually IgG

b (Noninfectious sequelae associated with infection with Streptococcus pyogenes are glomerulonephritis and rheumatic fever.)

109) Children who have infections with beta-hemolytic streptococci can develop: a. acute pyelonephritis b. acute glomerulonephritis c. chronic glomerulonephritis d. nephrosis

d (Hepatitis Be antigen [HBeAg] is a soluble component in the core of the hepatitis B virus.)

109) Refer to the following illustration of the hepatitis B virus: Select the corresponding lettered component indicated on the diagram for e antigen. a. A b. B c. C d. D

b (The Kell antigen is highly immunogenic. It is present on the red cells of up to 9% of adults and neonates, and is not affected by enzymes.)

109) The Kell antigen is: a. absent from the red cells of neonates b. strongly immunogenic c. destroyed by enzymes d. has a frequency of 50% in the random population

b (Calcium carbonate crystals are small dumbbell-shaped or round crystals often seen in clumps. With careful examination, dumbbell-shaped forms can be distinguished.)

109) Tiny colorless dumbbell shaped crystals were found in an alkaline urine sediment. They most likely are: a. calcium oxalate b. calcium carbonate c. calcium phosphate d. amorphous phosphate

c (Calculation of RBC indices.)

109) What is the MCH if the Hct is 20%, the is 1.5 x 10^6/mL (1.5 x 10^12/L) and the Hgb is 6 g/dL (60 g/L)? a. 28 mm3 (28 fL) b. 30 mm3 (30 fL) c. 40 mm3 (40 fL) d. 75 mm3 (75 fL)

d (RBC morphology; disease state identification.)

11) In which of the following disease states are teardrop cells and abnormal platelets most characteristically seen? a. hemolytic anemia b. multiple myeloma c. G-6-PD deficiency d. myeloid metaplasia

d (Gestational diabetes.)

11) Pregnant women with symptoms of thirst, frequent urination or unexplained weight loos should have which of the following tests performed? a. tolbutamide test b. lactose tolerance test c. epinephrine tolerance test d. glucose tolerance test

d (The chromogenic cephalosporin test using nitrocefin is the most sensitive and specific test for detection of beta lactamase. Acidimetric tests employing penicillin are less expensive, but not as sensitive, as the nitrocefin assay.)

11) The most sensitive substrate for the detection of beta-lactamases is: a. penicillin b. ampicillin c. cefoxitin d. nitrocefin

d (Streptococcus agalactiae is catalase and bile esculin hydrolysis negative and bacitracin and optochin resistant. Streptococcus agalactiae hydrolyzes hippurate.)

110) A gram-positive coccus isolated from a blood culture has the following characteristics: optochin susceptibility: negative bacitracin susceptibility: negative bile esculin hydrolysis: negative hippurate hydrolysis: Positive catalase: negative This organism is most likely: a. Staphylococcus aureus b. Streptococcus pneumoniae c. Streptococcus pyogenes d. Streptococcus agalactiae

c (The hepatitis B core antigen [HBc] is a protein in the nucleocapsid of HBV.)

110) Refer to the following illustration of the hepatitis B virus: Select the corresponding lettered component indicated on the diagram for core antigen. a. A b. B c. C d. D

b (Calcium oxalate crystals are found in acidic and neutral urine, but not in alkaline urine.)

110) Which of the following crystals may be found in acidic urine: a. calcium carbonate b. calcium oxalate c. calcium phosphate d. triple phosphate

a (Calculation of RBC indices.)

110) Which of the following is the formula for MCHC? a. (Hgb x 100) / Hct b. Hgb / RBC c. RBC / Hct d. (Hct x 1000) / RBC

b (Group A streptococci [Streptococcus pyogenes] are susceptible to bacitracin and CAMP test negative.)

111) A beta-hemolytic streptococci that is bacitracin-sensitive and CAMP-negative is: a. group B b. group A c. beta-hemolytic, not group A, B or D d. beta-hemolytic, group D

b (Double-stranded DNA is present in the core of the virus.)

111) Refer to the following illustration of the hepatitis B virus: Select the corresponding lettered component indicated on the diagram for viral DNA. a. A b. B c. C d. D

b (Calculation of RBC indices.)

111) What is the MCHC if the Hct is 20%, the RBC is 2.4 x 10^6/mL (2.4 x 10^12/L) and the Hgb is 5 g/dL (50 g/L)? a. 21% b. 25% c. 30% d. 34%

c (Group B streptococci [Streptococcus agalactiae] are resistant to bacitracin and CAMP test positive.)

112) A beta-hemolytic Streptococcus that is bacitracin-resistant and CAMP-positive is: a. group A or B b. group A c. group B d. beta-hemolytic, group D

c (The Fya and Fyb antigens are sensitive to denaturation by proteolytic enzymes. Serum containing anti-Fya reacts with untreated Fy[a+] cells, but not with enzyme treated Fy[a+] cells.)

112) Proteolytic enzyme treatment of red cells usually destroys which antigen? a. Jka b. E c. Fya d. k

d (Tyrosine crystals are fine needles often seen in clumps. Leucine crystals are spherical with concentric striations. Cholesterol crystals are flat plates with notched corners. Hemosiderin granules are dark, and often clumped.)

112) Which of the following crystals appear as fine, silky needles? a. cholesterol b. leucine c. hemosiderin d. tyrosine

c (Anti-Fya is an IgG antibody that reacts best at the AHG phase, does not react with enzyme-treated red cells, is capable of causing hemolytic disease of the newborn, and is not known to be an auto agglutinin.)

113) Anti-Fya is: a. usually a cold reactive agglutinin b. more reactive when tested with enzyme treated RBCs c. capable of causing hemolytic transfusion reactions d. often an auto agglutinin

d (Glomerular filtration rate is estimated by filtration markers such as creatinine clearance.)

113) Creatinine clearance is used to estimate the: a. tubular secretion of creatinine b. glomerular secretion of creatinine c. renal glomerular and tubular mass d. glomerular filtration rate

a (Growth in 6.5% NaCl, growth in bile esculin medium and susceptibility are not used in the routine identification of Streptococcus agalactiae. Polysaccharide typing for group B antigen is routinely used for identification of S agalactiae.)

113) Group B, beta-hemolytic streptococci may be distinguished from other hemolytic streptococci by which of the following procedures? a. latex antigen typing b. growth in 6.5% NaCl broth c. growth on esculin medium d. bacitracin susceptibility

c (The ELISA for HbsAg is a sandwich technique in which HbsAg in patient serum binds to anti-HBs on a solid phase; the HbsAg is then detected by the addition of an anti-HBs labeled with an enzyme.)

113) The ELISA technique for the detection of HbsAg: a. requires radiolabeled C1q b. is quantitated by degree of fluorescence c. uses anti-HBs linked to horseradish peroxidase d. uses beads coated with HbsAg

c (Urinary crystals associated with liver disorders include bilirubin, tyrosine, and leucine.)

113) Which of the following crystals is seen in an amber urine with a positive bilirubin? a. ammonium biurate b. cystine c. tyrosine d. uric acid

a (Questions that could be discriminatory in nature [eg, nationality, marital status, dependents, religion, affiliations, sexual orientation, or physical or mental disabilities] should be avoided on the employment application or during the interview process, unless the question is relevant or pertinent to the particular job; otherwise, it can be considered illegal.)

113) Which of the following questions can be legally asked on an employment application? a. are you a US citizen b. what is you date of birth c. is your wife/husband employed full time d. do you have any dependents

c (Abnormal results correlate with abnormal kidney function.)

114) A blood creatinine value of 5.0 mg/dL (442.0 umol/L) is most likely to be found with which of the following blood values? a. osmolality: 292 mOsm/kg b. uric acid: 8 mg/dL c. urea nitrogen: 80 mg/dL d. ammonia: 80 ug/dL

d (Calculation of RBC indices.)

114) Given the following data: Hgb: 8 g/dL (80 g/L) Hct: 28% RBC: 3.6 x 10^6/uL (3.6 x 10^12/L) The MCV is: a. 28 mm3 (28 fL) b. 35 mm3 (35 fL) c. 40 mm3 (40 fL) d. 77 mm3 (77 fL)

d (Enterococcus species are more resistant to antimicrobial therapy than group D streptococci such as S bovis.)

114) It is important to differentiate between Enterococcus and group D streptococci because: a. viridans streptococci are often confused with enterococci b. several enterococci cause severe puerperal sepsis c. group D streptococci are avirulent d. enterococci often show more antibiotic resistance than group D streptococci

d (The Duffy glycoprotein on red cells is a receptor for the malarial parasite Plasmodium vivax. Red cells with the phenotype Fy[a-b-] are resistant to invasion by P vivax.)

114) Resistance to malaria is best associated with which of the following blood groups: a. Rh b. I/i c. P d. Duffy

b (HBeAg is present in patient serum during periods of active HBV replication, and is therefore a marker of high infectivity.)

114) The antigen marker most closely associated with transmissibility of HBV infection is: a. HbsAg b. HBeAg c. HBcAg d. HBV

b (Increased lipids in the urine is a characteristic of the nephrotic syndrome. Massive amounts of protein also are associated with the nephrotic syndrome. The crystals form more readily in urine that has been refrigerated.)

115) Cholesterol crystals will most likely be observed in urine that contains: a. 3+ glucose b. 4+ protein c. WBC casts d. triple phosphate crystals

b (Some patients who become infected with HBV do not develop immunity and become long-term carriers of the virus who can transmit the infection to others.)

115) Chronic carriers of HBV: a. have chronic symptoms of hepatitis b. continue to carry HBV c. do not transmit infection d. carry HBV but are not infectious

d (Bile solubility testing of alpha-hemolytic streptococci differentiates S pneumoniae [soluble] from other alpha-hemolytic streptococci such as viridans streptococci [insoluble].)

115) Streptococcus pneumoniae can be differentiated best from the viridans group of streptococci by: a. Gram stain b. the type of hemolysis c. colonial morphology d. bile solubility

b (Enterococci are bile esculin-positive, hippurate-negative and have the ability to grow in 6.5% NaCl. Enterococci are relatively resistant to penicillin and require combination therapy to treat serious infections.)

116) Characteristically, enterococci are: a. unable to grow in 6.5% NaCl b. relatively resistant to penicillin c. sodium hippurate positive d. bile esculin negative

d (Calculation of RBC indices.)

116) What is the MCV if the hematocrit is 20%, the RBC is 1.5 x 10^6/mL (1.5 x 10^12/L) and the hemoglobin is 6 g/dL (60 g/L)? a. 68 pg b. 75 pg c. 115 pg d. 133 pg

d (Staphylococcus aureus are usually beta-hemolytic. Some strains may not produce bound coagulase detected by the slide coagulase test. A tube coagulase is performed to detect free coagulase and should be performed on colonies with typical S aureus morphology that are slide coagulase- negative.)

117) A beta-hemolytic, catalase-positive, gram-positive coccus is coagulase negative by the slide coagulase test. Which of the following is the most appropriate action in identification of this organism? a. report a coagulase-negative Staphylococcus b. report a coagulase-negative Staphylococcus aureus c. reconfirm the hemolytic reaction on a fresh 24-hr culture d. do a tube coagulase test to confirm the slide test

d (After performing rule outs, the most likely antibody is anti-c. To form anti-c, the patient would need to inherit a gene from both parents that does not produce the c antigen. The most common gene that codes for no antigen is denoted as R1.)

118) A 25-year-old Caucasian woman, gravida 3, para 2, required 2 units of Red Blood Cells. The antibody screen was positive and the results of the antibody panel are shown below: What is the most probable genotype of this patient? a. rr b. r'r' c. R0r d. R1R1

a (In serological assays for rubella, a titer of 1:8 or greater indicates presence of protective antibodies. An acute infection would be indicated by a rise in antibody titer between 2 serum samples collected 2 or more weeks apart, while susceptibility to rubella would be indicated by a negative antibody result)

118) A maternal serum rubella titer that is equal to or greater than 1:8 indicates: a. probable immunity to rubella b. evidence of acute rubella infection c. susceptibility to rubella infection d. absence of acute rubella

b (The sucrose hemolysis test is still sometimes used for screening; however, the most accurate measurement is immunophenotyping.)

118) A screening test for paroxysmal nocturnal hemoglobinuria is: a. heat instability test b. sucrose hemolysis c. osmotic fragility d. dithionite solubility

a (Uric acid has many shapes, is found in acid urine, and is nonpathologic. The reader should become familiar with the different shapes.)

118) Identify this crystal: a. uric acid b. calcium phosphate c. calcium carbonate d. triple phosphate acid

c (Of the biochemicals listed only hydrolysis of sodium hippurate will differentiate Streptococcus agalactiae [positive] from S pyogenes [negative].)

118) Which of the following would best differentiate Streptococcus agalactiae from Streptococcus pyogenes? a. ability to grow in sodium azide broth b. a positive bile-esculin reaction c. hydrolysis of sodium hippurate d. beta-hemolysis on sheep blood agar

d (The crystals are amorphous phosphates. These can be dissolved in dilute acetic acid, in order to view other formed elements that are obscured.)

119) Alkaline urine showed this microscopic finding. The technologist should: a. dilute with saline b. request a new sample c. culture for bacteria d. dissolve with acetic acid

b (Sideroblasts and siderocytes may be identified with Perl Prussian blue iron stain.)

119) The Prussian blue staining of peripheral blood identifies: a. Howell-Jolly bodies b. siderotic granules c. reticulocytes d. basophilic stippling

c (Coagulase production is the primary biochemical used to differentiate S aureus from other coagulase-negative staphylococci.)

119) The most critical distinction between Staphylococcus aureus and other Staphylococcus is: a. phosphatase reaction b. DNA production c. coagulase production d. hemolysis

b (Western blot uses purified HIV proteins to confirm reactivity in samples whose screening test for anti-HIV is positive.)

12) The western blot is a confirmatory test for: a. CMV antibody b. anti-HIV-1 c. HbsAg d. serum protein abnormality

c (Repeating a QC measurement on a new sample of QC material may establish that the alert was caused by a deteriorated QC material rather than a method problem.)

12) Upon completion of a run of cholesterol tests, the technician recognizes that the controls are not within the 2 standard deviations confidence range. What is the appropriate course of action? a. report the results without any other action b. run a new set of controls c. run a new set of controls and repeat specimens d. recalibrate instrument and run controls

a (Contaminants frequently contain substances capable of polarizing light. None of the other listed constituents are capable of polarizing light.)

120) Polarized light can often be used to differentiate between: a. fibers and mucus clumps b. hyaline and waxy casts c. squamous and transitional epithelial cells d. RBCs and WBCs

a (Using a supravital stain, residual ribosomal RNA is precipitated within the reticulocytes.)

120) Supravital staining is important for reticulocytes since the cells must be living in order to stain the: a. remaining RNA in the cell b. iron before it precipitates c. cell membrane before it dries out d. denatured hemoglobin in the cell

c (The only treponemal test listed is the PTA-ABS [fluorescent treponemal antibody absorbed test], while the others are all nontreponemal tests that detect antibody to cardiolipin.)

120) Which of the following is a treponemal test? a. RST b. RPR c. FTA-ABS d. VDRL

c (Haemophilus influenzae and Neisseria gonorrhoeae do not have predictable susceptibility to penicillin. Corynebacterium diphtheriae is not universally susceptible to penicillin. To date, no penicillin resistance has been demonstrated in Streptococcus pyogenes.)

120) Which of the following organisms is, to date, considered universally susceptible to penicillin: a. Haemophilus influenzae b. Neisseria gonorrhea c. Streptococcus pyogenes d. Corynebacterium diphtheriae

c (The most likely organism isolated from this specimen is Streptococcus agalactiae. Polysaccharide antigen typing will confirm the identification of S agalactiae and differentiate it from other beta-hemolytic streptococci.)

121) A beta-hemolytic gram-positive coccus was isolated from the cerebrospinal fluid of a 2-day-old infant with signs of meningitis. The isolate grew on sheep blood agar under aerobic conditions and was resistant to a bacitracin disc. Which of the following should be performed for the identification of the organism? a. oxidase production b. catalase formation c. latex antigen typing d. esculin hydrolysis

a (Patients with severe hemophilia A may have spontaneous hemorrhages that are treated with Factor VIII concentrate.)

121) A blood component used in the treatment of hemophilia A is: a. Factor VIII concentrate b. FFP c. Platelets d. Whole Blood

b (It is recommended that the VDRL and RPR tests be performed at a temperature range between 23 C and 29 C; optimal agglutination does not occur at temperatures below that range, resulting in false negative tests.)

121) The air temperature throughout the serology lab is 20 C. How will this affect VDRL and RPR test results? a. no effect-the acceptable test range is 20 - 24 C b. weaken reactions so that false negatives occur c. strengthen reactions so that positive titers appear elevated d. increase the number of false positives from spontaneous clumping

d (Manageable and unmanageable cost components for a laboratory manager. While it is under a manager's control to schedule and train employees, "acts of God" [eg, military leave, pregnancy, injury] are not under their control and affect their ability to schedule employees.)

121) Which of the following activities is not under the direction or control of the lab manager? a. number of employees b. direct test costs c. skill mix d. military leave

a (Starch granules are very refractile and produce a Maltese cross under polarized light. Oil, air bubbles and pollen grains do not polarize.)

121) Which of the following contaminants has a dimpled center and will polarize? a. starch b. oil droplets c. air bubbles d. pollen grains

c (Using a supravital stain [new methylene blue], residual ribosomal RNA is precipitated within the reticulocytes.)

121) Which of the following is used for staining reticulocytes? a. Giemsa stain b. Wright stain c. new methylene blue d. Prussian blue

b (Function of ceruloplasmin.)

122) 90% of the copper present in the blood is bound to: a. transferrin b. ceruloplasmin c. albumin d. cryoglobulin

a (Nontreponemal tests are used to screen for syphilis and monitor syphilis patients during therapy. These tests, the VDRL and RPR, are based on the principle of flocculation, created by the clumping of the fine cardiolipin particles used in the tests, after binding to patient's antibody.)

122) Which lab technique is most frequently used to diagnose and follow the course of therapy of a patient with secondary syphilis? a. flocculation b. precipitation c. complement fixation d. indirect immunofluorescence

d (Prussian blue stain is used for assessing iron stores in bone marrow.)

122) Which of the following stains is used to demonstrate iron, ferritin and hemosiderin? a. peroxidase b. Sudan black B c. periodic acid-Schiff d. Prussian Blue

c (Appropriate skin antisepsis is the most important factor in preventing contaminated blood cultures. Staphylococcus epidermidis is a common blood culture contaminant because it is a common inhabitant of the skin.)

123) During the past month, Staphylococcus epidermidis has been isolated from blood cultures at 2 - 3 times the rate from the previous year. The most logical explanation for the increase in these isolates is that: a. the blood culture media are contaminated with the organism b. the hospital ventilation system is contaminated with Staphylococcus epidermidis c. there has been a break in proper skin preparation before drawing blood for culture d. a relatively virulent isolate is being spread from patient to patient

a (Cloth fiber is a contaminant. It should not be confused with a cast. Fiber can have a rough and stringy appearance. Vegetable fibers have intricate repeating detail.)

123) Identify the formed element in this photomicrograph: a. cloth fiber b. hyaline cast c. granular cast d. waxy cast

b (Prussian blue stain is used to differentiate siderotic granules [Pappenheimer bodies] from basophilic stippling.)

123) Which of the following stains can be used to differentiate siderotic granules from basophilic stippling? a. Wright b. Prussian Blue c. crystal violet d. periodic acid-Schiff

b (Acetic acid lyses red blood cells, but not oil droplets and yeast. Acetic acid will also lyse other formed elements and should be added to an aliquot of the sediment.)

124) A technologist is having trouble differentiating between RBCs, oil droplets and yeast cells on a urine microscopy. Acetic acid should be added to the sediment to: a. lyse the yeast cells b. lyse the RBCs c. dissolve the oil droplets d. crenate the RBCs

a (Yeast cells are commonly seen in urine specimens from persons with diabetes mellitus, because the high glucose content provides an excellent growth media for yeast.)

125) A urine specimen is tested and has the following results: This is indicative of: a. diabetes mellitus b. contamination c. pyelonephritis d. diabetes insipidus

c (Micrococcus and Staphylococcus can be differentiated by susceptibility to furazolidone [100 mg/disk]. Staphylococcus is susceptible and Micrococcus is resistant.)

125) A yellow colony from a wound culture tested catalase-positive and coagulase-negative. The organism stained as gram-positive cocci in clusters. Which of the following tests should differentiate between a coagulase-negative Staphylococcus and Micrococcus? a. novobiocin susceptibility b. leucine aminopeptidase production c. furazolidone (100 ug/disk) susceptibility d. bile esculin

c (Biological functions of immunoglobulins; IgG crosses the placenta.)

125) Antibodies composed of IgG immunoglobulin: a. occur during the primary response to antigen b. are larger molecules that IgM antibodies c. can cross the placenta from mother to fetus d. can be detected in saline crossmatches

d (HDFN is caused by maternal antibody crossing the placenta and destroying fetal antigen-positive red cells. Unlike ABO antibodies, which are naturally-occurring and can affect the first pregnancy, Rh antibodies are not produced until the mother has been exposed to Rh-positive red cells, usually during delivery of the first Rh-positive child Once immunized, subsequent pregnancies with Rh-positive infants are affected, usually with increasing severity.)

126) An obstetrical patient has had 3 previous pregnancies. her 1st baby was healthy, the 2nd was jaundiced at birth and required and exchange transfusion, while the 3rd was stillborn. Which of the following is the most likely cause: a. ABO incompatibility b. immune deficiency disease c. congenital spherocytic anemia d. Rh incompatibility

d (Morphology of Dohle bodies.)

126) Inclusions in the cytoplasm of neutrophils as shown in the figure below are known as : a. Auer bodies b. Howell-Jolly bodies c. Heinz bodies d. Dohle bodies

b (Biological functions of immunoglobulins; IgE is the antibody involved in multiple allergies.)

126) Measurement of serum levels of which of the following immunoglobulin can serve as a screening test for multiple allergies? a. IgA b. IgE c. IgG d. IgM

b (Urine crystal formation is associated with the optimal pH needed for their formation [acid, alkaline or neutral].)

126) When identifying urinary crystals, which reagent strip result is most important? a. protein b. pH c. specific gravity d. nitrite

c (To correct for having used a dilution, multiply the answer obtained times the reciprocal of the dilution made.)

127) An automated CK assay gives a reading that is above the limits of linearity. A dilution of the serum sample is made by adding 1 mL of serum to 9 mL of water. The instrument now reads 350 U/L. The correct report on the undiluted serum should be: a. 2,850 U/L b. 3,150 U/L c. 3,500 U/L d. 3,850 U/L

b (An increased amount of cytoplasmic basophilia in a blood cell indicates decreased cytoplasmic maturation.)

127) An increased amount of cytoplasmic basophilia in a blood cell indicates: a. increased cytoplasmic maturation b. decreased cytoplasmic maturation c. reduction in size of the cell d. decreased nuclear maturation

d (The question describes the CAMP test, which is positive for Streptococcus agalactiae.)

127) An isolate of an unknown beta-hemolytic Streptococcus is streaked perpendicular to a streak of beta-lysin-producing Staphylococcus aureus. After incubation a zone of arrowhead hemolysis is noted at the interface of the 2 streaks. What is the name of the test and the presumptive identification of the unknown Streptococcus? a. hippurate hydrolysis and S agalactiae b. CAMP test and S pyogenes c. hippurate hydrolysis and S pyogenes d. CAMP test and S agalactiae

d (ABO HDFN is a mild disease, not usually requiring transfusion. It may occur in any pregnancy in which there is ABO incompatibility. High-titered IgG antibodies are more frequently seen in group O mothers than in A or B mothers.)

128) ABO hemolytic disease of the newborn: a. usually requires an exchange transfusion b. most often occurs in 1st born children c. frequently results in stillbirth d. is usually seen only in the newborn of group O mothers

d (Biological functions of immunoglobulins; IgM binds complement well and is hemolytic.)

128) IgM antibodies are frequently hemolytic because of: a. their dimeric structure b. the molecules 5 antigen binding sites c. their sedimentation coefficient of 7-15 S d. their efficient ability to fix complement

b (Definition of left shift.)

128) The term "shift to the left" refers to: a. a microscope adjustment b. immature cell forms in the peripheral blood c. a trend on a levy-jennings chart d. a calibration adjustment on an instrument

d (Commonly used units for standard solutions are mg/mL.)

128) The unit of measure for a standard solution is: a. g/L b. % c. mg/% d. mg/mL

b (The flagellate Trichomonas vaginalis moves rapidly through the sediment. If not moving, it may resemble a WBC, and careful examination of phase microscopy is needed to visualize the flagellum. Urine is toxic to spermatozoa.)

128) Which of the following exhibits rapid motility in urine sediment? a. spermatozoa b. Trichomonas vaginalis c. Gardnerella vaginalis d. Enterobius vermicularis

d (Enterococcus is positive for the bile esculin test while Streptococcus pyogenes and staphylococci are negative.)

128) Which of the following may be used as a positive quality control organism for the bile esculin test? a. Staphylococcus epidermidis b. Staphylococcus aureus c. Streptococcus pyogenes d. Enterococcus faecalis

a (The value of the unknown is a ratio of the absorbance reading of the unknown to the absorbance reading of the standard.)

129) A glucose determination was read on a spectrophotometer. The absorbance reading of the standard was 0.30. The absorbance reading of the unknown was 0.20. The value of the unknown is: a. 2/3 of the standard b. 3/5 of the standard c. the same as the standard d. 1.5 x the standard

b (>80% of uncomplicated UTIs are caused by E coli.)

129) Infection of the urinary tract is most frequently associated with: a. Staphylococcus aureus b. Escherichia coli c. Enterococcus faecalis d. Serratia marcescens

c (Biological functions of immunoglobulins; IgG antibody crosses the placenta and is involved in hemolytic disease of the newborn.)

129) To which of the following classes do the antibodies that cause hemolytic disease of the newborn belong? a. IgA b. IgE c. IgG d. IgD

d (HDFN is caused by maternal IgG antibodies. Outside the Rh system, the most clinically significant antibody for HDFN is anti-K. IgM antibodies do not cross the placenta.)

129) Which of the following antigens in most likely to be involved in HDFN. a. Lea b. P1 c. M d. Kell

a (Systemic enterococcal infections, such as endocarditis, are commonly treated with a cell-wall-active agent and an aminoglycoside. These agents act synergistically to kill the organism. If the organism is resistant to one or both, there is no synergy, and the combination will fail. It is important to detect aminoglycoside and beta-lactam resistance in these cases. Enterococci have intrinsic moderate level resistance to aminoglycosides. Acquired resistance corresponds to very high MICs [greater than 500 mg/mL] for gentamicin and is termed high level resistance.)

13) An Enterococcus isolated from multiple blood cultures in a patient with endocarditis should be: a. screened for high level aminoglycoside resistance b. checked for tolerance c. assayed for serum antimicrobial activity d. tested for beta-lactamase production

b (Patients with hypertension and diabetes mellitus are at risk for kidney disease. Detection of small amounts of albumin in the urine predict eventual kidney disease. The advantage of this sensitive detection is that patients with microalbuminuria can be treated with anti-hypertensive medications and followed up more intensely to delay nephropathy.)

13) Microalbumin can be measured by a random urine collection. An increased microalbumin is predictive of: a. diabetes mellitus b. nephropathy c. hypertension d. nephrotic syndrome

d (Definition of glycolysis)

13) The conversion of glucose or other hexoses into lactate or pyruvate is called: a. glycogenesis b. glycogenolysis c. gluconeogenesis d. glycolysis

c (Standard deviation is a measure of the dispersion of data around the mean.)

13) The following data was calculated on a series of 30 determinations of serum uric acid control: mean=5.8 mg/dL, 1 standard deviation = 0.15 mg/dL. If the confidence limits are set at =-2 standard deviations, which of the following represents the allowable limits for the control? a. 5.65 - 5.95 mg/dL b. 5.35 - 6.25 mg/dL c. 5.50 - 6.10 mg/dL d. 5.70 - 5.90 mg/dL

b (The causative agent for AIDS is the human immunodeficiency virus types 1 and 2.)

13) The test that is currently used to detect donors who are infected with the AIDS virus is: a. anti-HBc b. anti-HIV-1,2 c. HbsAg d. ALT

b (Volumetric flasks are used to measure exact volumes and are primarily used in preparing solutions of known concentrations.)

130) A technician is asked by the supervisor to prepare a standard solution from the stock standard. What is the glassware of choice for this solution? a. graduated cylinder b. volumetric flask c. acid-washed beaker d. graduated flask

a (ABO HDFN is a mild disease that may occur in any ABO-incompatible pregnancy, including the first, since the antibodies are naturally occurring Rh HDFN does not occur until the mother has become immunized. Once this happens, subsequent pregnancies may be quite severely affected. The DAT is typically weak or even negative in ABO HDFN, and strongly positive in Rh HDFN.)

130) ABO HDFN differs from Rh HDFN in that: a. Rh HDFN is clinically more severe that ABO HDFN b. the DAT test is weaker in Rh HDFN than ABO c. Rh HDFN occurs in the 1st pregnancy d. the mother's antibody screen is positive in ABO HDFN

c (Babies' IgG comes from their mother, but if they have elevated IgM, an in utero, or neonatal infection is indicated.)

130) It is important to note when an infant is born, levels of specific antibody of the following class are used to indicate neonate infection: a. IgA b. IgG c. IgM d. IgD

c (The most commonly used equation for preparing suspension solutions is V1 x C1 = V2 x C2. A 4% red cell suspension contains 4 mL of red cells per 100 mL [1 dL] of solution. Therefore, [25] x [4] = [100] x [X]. Solve for x.)

131) How many mL of red blood cells are to be used to make 25 mL of a 4% red cell suspension? a. 0.25 mL b. 0.5 mL c. 1 mL d. 2 mL

d (Members of Enterobacteriaceae are oxidase negative, ferment glucose, and reduce nitrate to nitrite.)

131) Members of the family Enterobacteriaceae share which one of the following characteristics? a. produce cytochrome oxidase b. ferment lactose c. produce beta-hemolysis d. reduce nitrate to nitrite

c (Comparative mobilities due to structures of Hgb variants.)

131) On electrophoresis at alkaline pH, which of the following is the slowest migrating hemoglobin? a. Hgb A b. Hgb S c. Hgb C d. Hgb F

a (ABO HDFN occurs most commonly in group A babies born to group O mothers and usually has a mild course. The DAT is typically weak or negative and jaundice develops 12-48 hours after birth. The mother and baby are both Rh-positive.)

132) A group A, Rh positive infant of a group O, Rh positive mother has a weakly positive DAT and moderately elevated bilirubin 12 hours after birth. the most likely cause is: a. ABO incompatibility b. Rh incompatibility c. blood group incompatibility due to and antibody to a low frequency antigen d. neonatal jaundice not associated with blood group

d (The function of B cells is to produce antibodies.)

132) Antibodies are produced by: a. killer cells b. marrow stem cells c. mast cells d. B cells

a (Secondary granule appearance in neutrophils.)

132) Specific granules of the neutrophilic granulocyte: a. appear first at the myelocyte stage b. contain lysosomal enzymes c. are formed on the mitochondria d. are derived from azurophilic granules

c (The most commonly used equation for preparing suspension solutions is V1 x C1 = V2 x C2. In this case, [100] x [5] = [x] x [25]. Solve for x.)

132) The volume of 25% stock sufosalicylic acid needed to prepare 100mL of 5% working solutions is: a. 1.25 mL b. 5 mL c. 20 mL d. 50 mL

d (Shigella is lactose negative, most species do not produce gas, are VP, urea, lysine decarboxylase and citrate negative, and they are nonmotile.)

132) Which one of the following genera is among the least biochemically reactive members of the Enterobacteriaceae? a. Proteus b. Pseudomonas c. Citrobacter d. Shigella

a (The area on the immunoglobulin molecule that is the antigen-specific region is the Fab region [this is the amino terminal end]. Fab stands for fragment antigen binding, and indicates a fragment after papain cleavage. The biological function of the immunoglobulin, i.e., whether it goes to the placenta, whether it binds complement, and what effector cells it binds, resides in the Fc region, on the carboxy terminal end. The Fc stands for fragment crystallizable, and indicates the part of the immunoglobulin molecule after papain cleavage that has structural identity, and thus can be crystallized.)

133) Antibody class and antibody subclass are determined by major physiochemical differences and antigenic variations found primarily in the: a. constant region of heavy chain b. constant region of light chain c. variable regions of heavy and light chains d. constant regions of heavy and light chains

c (Light-exposed bilirubin is oxidized [structurally altered].)

133) The following bilirubin results are obtained on a patient: Given that the controls were within range each day, what is a probable explanation for the result on day 4? a. no explanation necessary b. serum, not plasma, was used for testing c. specimen had prolonged exposure to light d. specimen was hemolyzed

d (Uric acid crystals are seen in acid urine. The reagent strip pH may have been recorded wrong. Uric acid crystals have many shapes, and an artifact may have been mistaken for the uric acid crystals.)

133) The results of a urinalysis on a first morning specimen are: The next step is to repeat the: a. microscopic examination b. protein and glucose c. specific gravity d. pH and microscopic examination

d (Alcaligenes, Pseudomonas and Acinetobacter are all nonfermenters; Yersinia is a member of the Enterobacteriaceae and, by definition, ferments glucose.)

133) Which one of the following gram-negative bacilli ferments glucose? a. Alcaligenes faecalis b. Pseudomonas cepacia c. Acinetobacter lwoffii d. Yersinia enterocolitica

c (This is a ratio calculation. 0.9% normal saline contains 0.9 grams NaCl in 100 mL solution. 0.9/100 = x/1,000 Solve for x to determine how much NaCl is needed to prepare 1 L [1,000 mL].)

134) How many grams of sodium chloride are needed to prepare 1 L of 0.9% normal saline? a. 0.9 g b. 1.8 g c. 9.0 g d. 18.0 g

b (The change in optical density [absorbance] of amniotic fluid measured spectrophotometrically at 450 nm is calculated and plotted on the Liley graph according to the weeks gestation. The graph is divided into 3 zones, which predict the severity of HDFN and the need for intervention and treatment.)

134) The Liley method of predicting the severity of HDFN is based on the amniotic fluid: a. bilirubin concentration by standard methods b. change in optical density measured at 450nm c. Rh determination d. ratio of lecithin to sphingomyelin

c (The reagent strip glucose test using glucose oxidase is specific for glucose; therefore, glucose must be present in the sample. The copper reduction test is positive with many sugars, including glucose. The copper reduction test has a lower sensitivity than the reagent strip; therefore, the higher reading on the copper reduction test indicates the presence of an additional sugar. In the case of a nursing mother, the most likely additional sugar is lactose.)

134) The following urinalysis results were obtained from an 18-year-old woman in labor: Which of the following is the most likely explanation for the patient's positive copper reduction test? a. only glucose is present b. only lactose is present c. glucose and possibly other reducing substances/ sugars are present d. results are false positive due to the presence of protein

d (Biochemical pathway of bilirubin.)

134) Urobilinogen is formed in the: a. kidney b. spleen c. liver d. intestine

c (A positive DAT on cord blood demonstrates the presence of maternal antibody coating the baby's red cells and indicates hemolytic disease of the newborn. Normal cord hemoglobin in newborns ranges from 14-20 g/L. A cord hemoglobin value of 10 g/L indicates anemia and supports the diagnosis of HDFN.)

135) These lab results were obtained on maternal and cord blood samples: Mother: A- baby: AB+ DAT 3+ cord hemoglobin 10 g/dL Does the baby have HDFN? a. no, as indicated by the cord hemoglobin b. yes, although the cord hemoglobin is normal, the DAT indicates HDFN c. yes, the DAT and cord hemoglobin level both support HDFN d. no, a diagnosis of HDN cannot be established without cord bilirubin levels

a (Shigella has colorless colonies on both MacConkey and Hektoen agars. Yersinia is lactose negative, but Hektoen agar [has both lactose and sucrose] produces yellow colonies from the fermentation of sucrose. V parahaemolyticus needs at least 1% NaCl to grow and Campylobacter does not grow on MacConkey or Hektoen agars.)

136) An isolate from a stool culture gives the following growth characteristics and biochemical reactions: These screening reactions are consistent with which of the following enteric pathogens? a. Yersinia enterocolitica b. Shigella sonnei c. Vibrio parahaemolyticus d. Campylobacter jejuni

b (Immunoglobulin A [IgA] is the most abundant immunoglobulin in saliva, tears, and other mucosal secretions and plays an important role in mucosal immunity.)

136) The immunoglobulin class typically found to be present in saliva, tears, and other secretions is: a. IgG b. IgA c. IgM d. IgD

a (Damage to the electrical charges of the glomerular membrane, allowing the passage of high molecular-weight proteins and lipids occurs in nephrotic syndrome. This results in markedly increased urine protein levels, and the appearance of fatty casts and oval fat bodies that are characteristic of nephrotic syndrome.)

137) A 62 yr. old patient with hyperlipoproteinemia has a large amount of protein in his urine. Microscopic analysis yields moderate to many fatty, waxy, granular and cellular casts. Many oval fat bodies are also noted. This is most consistent with: a. nephrotic syndrome b. viral infection c. acute pyelonephritis d. acute glomerulonephritis

d (Salmonella produce H2S in TSI and Yersinia produces an acid slant and acid butt. Shigella fits this biochemical profile.)

137) A TSI tube inoculated with an organism gave the following reactions: alkaline slant, acid butt, no H2S, no gas produced This organism is most likely: a. Yersinia enterocolitica b. Salmonella typhi c. Salmonella enteritidis d. Shigella dysenteriae

a (Definition of relative lymphocytosis.)

137) Elevation of the lymphocyte percentage above 47% is termed: a. relative lymphocytosis b. absolute lymphocytosis c. leukocytosis d. absolute neutrophilic leukocytosis

a (Bleeding from upper GI.)

146) A stool specimen that appears black and tarry should be tested for the presence of : a. occult blood b. fecal fat c. trypsin d. excess mucus

b (the area on the immunoglobulin molecule that is the antigen-specific region is the Fab region [called the amino terminal end]. Fab stands for fragment antigen binding, and indicates a fragment after papain cleavage. The biological function of the immunoglobulin, i.e., whether it goes to the placenta, whether it binds complement, and what effector cells it binds, resides in the Fc region, on the carboxy terminal end. The Fc stands for fragment crystallizable, and indicates the part of the immunoglobulin molecule after papain cleavage that has structural identity, and thus can be crystallized.)

137) Treatment of IgG with papain results in how many fragments from each immunoglobulin molecule? a. 2 b. 3 c. 4 d. 5

a (Blood for an exchange transfusion should lack the antigen to any maternal antibodies that have entered the infant's circulation and are reactive at 37 C or AHG.)

137) Which unit should be selected for exchange transfusion if the newborn is group A, Rh pos. and the mother is Group A, Rh pos. with anti-c? a. A, CDe/CDe b. A, cDE/cDE c. O, cde/cde d. A, cde/cde

a (A specimen with a specific gravity of 1.004 is very dilute. This will result in the concentration of urine constituents being too low, below the ability to be detected by chemical and microscopic examination.)

138) A patient has 2 separate urinalysis reports, which contain the following data: Which of the following statements best explains these results? a. protein, glucose and microscopy of A are false negatives because of the specific gravity b. protein and glucose are false positives in B due to the specific gravity c. microscopic of A is false negative because of the pH d. microscopic of B is false positive because of the pH

b (Light-exposed bilirubin is oxidized [structurally altered].)

138) A serum sample was assayed for bilirubin at 10 AM, and the result was 12 mg/dL (205.6 umol/L). The same sample was retested at 3 PM. The result now is 8 mg/dL (136.8 umol/L). The most likely explanation for this discrepancy is: a. the reagent has deteriorated b. the sample was exposed to light c. a calculation error in the first assay d. the sample was not refrigerated

c (These biochemicals are characteristic for E coli. Klebsiella pneumoniae is indole negative and nonmotile. Shigella dysenteriae is nonmotile and Enterobacter cloacae is indole negative.)

138) An organism gave the following reactions: This organism most likely is: a. Klebsiella pneumoniae b. Shigella dysenteriae c. Escherichia coli d. Enterobacteria cloacae

b (Standard deviation is a measure of the dispersion of data around the mean. It is calculated by taking the square root of the summation of the squared differences of the observed x values from their mean divided by n.)

138) Which of the following is the formula for standard deviation? a. square root of the mean b. square root of (sum of squared differenced)/(N-1) c. square root of the variance d. square root of (mean)/(sum of squared differences)

d (Definition of multipotent stem cell.)

139) Multipotent stem cells are capable of producing: a. daughter cells of only one cell line b. only T-lymphs and B-lymphs c. erythopoeitin, thrombopoietin and leukopoietin d. lymphoid and myeloid stem cells

c (Immunoglobulin function; IgM best at agglutination.)

139) Which of the following immunoglobulins is the most efficient at agglutination? a. IgG b. IgA c. IgM d. IgE

b (E coli can produce several different types of toxins that result in different gastroenteritis manifestations.)

139) Which of the following organisms can grow in the small bowel and cause diarrhea in children, travelers' diarrhea, or a severe cholera-like syndrome through the production of enterotoxins? a. Yersinia enterocolitica b. Escherichia coli c. Salmonella typhi d. Shigella dysenteriae

d (The enzyme-labeled immunosorbent assay [ELISA] method is a very sensitive method employed to screen donors for markers of transfusion-transmitted viruses.)

14) A commonly used screening method for anti-HIV detection is: a. latex agglutination b. RIA c. TLC d. ELISA

c (Systemic lupus is characterized by an association of autoantibody production; generalized multisystem.)

14) A patient has the following test results: The above results could be seen in patients with: a. rheumatic fever b. rheumatoid arthritis c. lupus erythematosus d. glomerulonephritis

a (The high specific gravity is due to the glucose in the urine. Patients with diabetes mellitus have polyuria, so that the volume of urine dilutes the urochrome [color], making the urine pale.)

14) A patient with uncontrolled Diabetes mellitus will most likely have: a. pale urine with a high specific gravity b. concentrated urine with a high specific gravity c. pale urine with a low specific gravity d. dark urine with a high specific gravity

b (Normal degradation products of red blood cells.)

14) After the removal of RBCs from the circulation hemoglobin is broken down into: a. iron, prophyrin, amino acids b. iron, protoporphyrin, globin c. heme, protoporphyrin, amino acids d. heme, hemosiderin, globin

d (Precision is the closeness of agreement among replicate measurements, or reproducibility. The coefficient of variation, a more useful measure of reproducibility, is the measure of relative random error expressed as a percentage.)

14) An index of precision is statistically known as the: a. median b. mean c. standard deviation d. coefficient of variation

c (Variations in the concentrations of divalent cations primarily calcium and magnesium affect the results of aminoglycoside, tetracycline and colistin tests with P aeruginosa isolates. A cation concentration that is too high results in smaller zone sizes, and a concentration that is too low increases zone sizes.)

14) In the Kirby-Bauer disk diffusion susceptibility test, which variable is critical when testing Pseudomonas species for antibiotic susceptibility to aminoglycosides? a. incubation temp b. duration of incubation c. cation content of media d. depth of agar

b (Diagnosis of diabetes mellitus)

14) Which of the following values obtained during a glucose tolerance test are diagnostic of diabetes mellitus? a. 2 hr. specimen = 150 mg/dL (8.3 mmol/L) b. fasting plasma glucose = 126 mg/dL (6.9 mmol/L) c. fasting plasma glucose = 110 mg/dL (6.1 mmol/L) d. 2 hr. specimen = 180 mg/dL (9.9 mmol/L)

d (Of the organisms listed only Klebsiella pneumoniae is nonmotile.)

155) The stock cultures needed for quality control testing of motility are: a. Salmonella typhimurium / Escherichia coli b. Escherichia coli / Pseudomonas aeruginosa c. Serratia marcescens / Escherichia coli d. Klebsiella pneumoniae / Escherichia coli

a (Blood selected for exchange transfusion should be ABO-compatible with the mother and baby, and antigen-negative. Prenatal antibody titers above 16 or 32 are considered significant, and the condition of the fetus should be monitored.)

140) A blood specimen from a pregnant woman is found to be group B, Rh neg. and the serum contains anti-D with a titer of 512. What would be the most appropriate type of blood to have available for a possible exchange transfusion for her infant? a. O, Rh-neg b. O, Rh-pos c. B, Rh-neg d. B, Rh-pos

b (Definition of kernicterus.)

142) Kernicterus is an abnormal accumulation of bilirubin in: a. heart tissue b. brain tissue c. liver tissue d. kidney tissue

c (Immunoglobulin subclasses differ from each other in their Fc regions; this is the reason that the different classes have different biological function. The Fc region is the region that is crystallizable after papain cleavage. It varies in sequence in the different classes of immunoglobulin.)

142) The key structural difference that distinguishes immunoglobulin subclasses is the: a. number of domains b. stereometry of the hypervariable region c. the sequence of the constant regions d. covalent linkage of the light chains

a (Definition of a standard.)

142) When the exact concentration of the solute of a solution is known and is used to evaluate the concentration of an unknown solution, the known solution is: a. standard b. normal c. control d. baseline

c (For exchange transfusion, antigen- negative Red Blood Cells are typically resuspended in ABO-compatible thawed Fresh Frozen Plasma.)

142) When the main objective of an exchange transfusion is to remove the infant's antibody-sensitized RBCs and to control hyperbilirubinemia, the blood product of choice is ABO compatible: a. FFP b. RBCs washed c. RBC suspended in FFP d. heparinized RBCs

c (To correct for having used a dilution, multiply the answer obtained times the reciprocal of the dilution made.)

143) A serum glucose sample was too high to read, so a 1:5 dilution using saline was made (dilution A). Dilution A was tested and was again too high to read. A further 1:2 dilution was made using saline (dilution B). To calculate the result, the dilution B value must be multiplied by: a. 5 b. 8 c. 10 d. 20

d (Boiling removed the capsule so that the antiserum could react with cell wall antigen. Group D Shigella is S sonnei.)

143) Biochemical reactions of an organism are consistent with Shigella. A suspension is tested in antiserum without resulting agglutination. However, after 15 mins. of boiling, agglutination occurs in group D antisera. The Shigella species is: a. dysenteriae b. flexneri c. boydii d. sonnei

c (The ideal conditions for the growth of Candida albicans are an acid pH and the presence of glucose. Candida is a frequent cause of urinary tract infections in diabetic patients.)

143) The sediment of a urine specimen with a reagent strip glucose of 250 mg/dL and a pH of 5.5 is ideal for the presence of: a. cystine crystals b. Trichomonas vaginalis c. Candida albicans d. thorny apple crystals

b (Blood selected for intrauterine transfusion and transfusion to premature infants should be irradiated to prevent graft-vs-host disease.)

143) To prevent graft vs host disease, RBCs prepared for infants who have received intrauterine transfusions should be: a. saline washed b. irradiated c. frozen and deglycerolized d. group-and Rh-compatible with the mother

b (Morganella and Providencia do not produce H2S; the indole reaction differentiates P mirabilis and P vulgaris.)

144) >100,000 CPU/mL of a gram-negative bacilli were isolated On MacConkey from a urine specimen. Biochemical results are as follows: The organism is most likely: a. Morganella morganii b. Proteus mirabilis c. Proteus vulgaris d. Providencia stuartii

b (To correct for having used a dilution, multiply the answer obtained times the reciprocal of the dilution made.)

144) In performing a spinal fluid protein determination, the specimen is diluted 1 part spinal fluid to 3 parts saline to obtain a result low enough to measure. To calculate the protein concentration, the result must be: a. multiplied by 3 b. multiplied by 4 c. divided by 3 d. divided by 4

b (Serratia can produce a red pigment; Proteus mirabilis swarms, is TDA positive and produces H2S.)

145) A urine culture had the following culture results: - sheep blood: swarming - Columbia CNA: no growth - MacConkey: 1. >100,000 CFU/mL nonlactose-fermenter - 2. >100,000 CFU/mL nonlactose-fermenter with red pigment The isolates from MacConkey agar had the following biochemical reactions: The organisms are most likely: a. Proteus vulgaris and Enterobacter cloacae b. Proteus mirabilis and Serratia marcescens c. Morganella morganii and Klebsiella pneumoniae d. Providencia stuartii and Serratia liquefaciens

b (Care must be taken so that fetal Rh-positive RBCs in the maternal circulation are not interpreted as maternal, because the mother would be assumed erroneously to be weak D+.)

145) Rh-Immune Globulin is requested for an Rh-negative mother who has the following results: What is the most likely explanation? a. mother is a genetic weak D b. mother had a fetomaternal hemorrhage of D+ cells c. mother's red cells are coated weakly with IgG d. anti-D reagent is contaminated with an atypical antibody

c (The renal threshold is the plasma level at which a substance, such as glucose, is no longer reabsorbed by the proximal convoluted tubules. The plasma level for glucose ranges from 160 - l80 mg/dL.)

145) The normal renal threshold for glucose in the adult is approximately: a. 50 mg/dL b. 100 mg/dL c. 160 mg/dL d. 300 mg/dL

d (Differentiates anemia due to conditioned nutritional deficiency from those of bone marrow dysfunction.)

145) Which of the following conditions is not associated with a high incidence of leukemia? a. paroxysmal nocturnal hemoglobinuria b. Fanconi anemia c. aplastic anemia d. megaloblastic anemia

d (The presence of D+ infant's red cells in the mother's circulation can cause the weak D test to show mixed-field agglutination. Care must be taken so that fetal Rh-positive RBCs in the maternal circulation are not interpreted as maternal, because the mother would be assumed erroneously to be weak D+.)

146) The following results are seen on a maternal postpartum sample: The most appropriate course of action is to: a. report the mother as Rh-negative b. report the mother as Rh-positive c. perform an elution on mother's RBCs d. investigate for a fetomaternal hemorrhage

b (The rosette test is a qualitative test. When enzyme-treated cells are used as indicator cells, a negative test [indicating there was not an excessive bleed] can have up to 1 rosette per 3 fields. The mother needs to receive 1 vial of RhIg for a normal bleed.)

147) What is the most appropriate interpretation for the laboratory data given below when an Rh-neg woman has an Rh-pos child? mother: 1 rosette/3 fields Positive Control.: 5 rosettes/3 fields Negative Control: no rosettes a. mother is not a candidate for RhIg b. mother need 1 vial RhIg c. mother need 2 vials RhIg d. the fetal maternal hemorrhage needs to be quantified

c (Differentiates this from other myeloid leukemias or other red cell abnormalities.)

148) Abnormalities found in erythroleukemia include: a. rapid DNA synthesis b. marrow fibrosis c. megaloblastoid development d. increased erythrocyte survival

c (The weak D result is most likely due to excessive bleed of fetal cells. Rosette results indicate a quantitative test for approximate volume of fetal-maternal bleed should be performed.)

148) What is the best interpretation for the laboratory data given above? a. mother is Rh-positive b. mother is weak D+ c. mother has had a fetal-maternal hemorrhage d. mother has a positive DAT

c (Immunoglobulin structure; IgA is in serum and secretions.)

148) Which of the following statements about immunoglobulins is true? a. immunoglobulins are produced by T lymphs b. IgA class is determined by the gamma heavy chain c. IgA class exists as serum and secretory molecules d. there are only 2 subclasses of IgG

b (Recognizes multiple causes impacting granulocyte production.)

149) Neutropenia is not usually associated with: a. viral infections b. Hodgkin disease c. select antibodies d. chemotherapy

d (Precision is the reproducibility of analytical results, or the degree to which results of multiple analyses of the same specimen agree.)

15) The term used to describe reproducibility is: a. sensitivity b. specificity c. accuracy d. precision

a (Phenylketonuria is a genetic disorder that results in a urine with a mousy odor. Acetone has a fruity odor. Bacteria can produce an ammonia odor. Porphyrin has no odor, but a characteristic red color.)

15) While performing an analysis of a baby's urine, the technologist notices the specimen to have a "mousy" odor. Of the following substances that may be excreted in urine, the one that most characteristically produces this odor is: a. phenylpyruvic acid b. acetone c. coliform basilli d. porphyrin

d (Yersinia pestis is classically described as having a "safety pin" appearance on Wayson stain. This patient's presentation is classic for bubonic plague.)

150) A 25 yr. old man who had recently worked as a steward on a transoceanic grain ship presented to the ER with high fever, diarrhea and prostration. Axillary lymph nodes were hemorrhagic and enlarged. A Wayson stain of the aspirate showed bacilli that were bipolar, resembling safety pins. The most likely ID of this organism is: a. Brucella melitensis b. Streptobacillus moniliformis c. Spirillum minus d. Yersinia pestis

c (Urine is the only body fluid containing large amounts of the waste products urea and creatinine. These 2 constituents are used to determine if an unknown fluid is urine.)

150) An abdominal fluid is submitted from surgery. The physician wants to determine if this fluid could be urine. The technologist should: a. perform a culture b. smell the fluid c. test for urea and creatinine d. test for protein, glucose and pH

c (Immunoglobulin structure, idiotype is in variable regions of heavy and light chains.)

151) Antibody idiotype is dictated by the: a. constant region of the heavy chain b. constant region of the light chain c. variable regions of heavy and light chains d. constant regions of heavy and light chains

a (Lab findings, increased WBC but with increased LAP; negative Philadelphia chromosome differentiates leukemoid response from leukemia.)

151) The following results were obtained on a 45-year-old man complaining of chills and fever: These results are consistent with: a. neutrophilic leukemoid reaction b. polycythemia Vera c. chronic myelocytic leukemia d. leukoerythroblastosis in myelofibrosis

c (The most common equation for preparing dilutions is V1 x C1 = V2 x C2, where V1 is the volume, C1 is the concentration of solution 1, and V2 and C2 are the volume and concentration of the diluted solution.)

151) Which of the following is the formula for calculating the dilution of a solution. a. V1 + C1 = V2 + C2 b. V1 + C2 = V2 + C1 c. V1 x C1 = V2 x C2 d. V1 x V2 = V1 x C2

c (The correct quantitation on a urine sample is obtained by counting the colonies and multiplying them by the dilution factor, which in this case is 1000 because a .001 mL loop was used for culture. The biochemicals are characteristic of Enterobacter cloacae.)

152) A clean catch urine sample from a nursing home patient is cultured using a .001mL loop. It grows 67 colonies of a lactose fermenter that has the following biochemical reactions: TSI: Acid/Acid oxidase: negative motility: positive indole: negative citrate: positive VP: positive lysine decarboxylase: negative ornithine decarboxylase: positive urea: negative What should the microbiologist report? a. 670 CFU/mL Serratia marcescens species b. 6,700 CFU/mL Providencia stuartii c. 67,000 CFU/mL Enterobacter cloacae d. 67,000 CFU/mL Klebsiella oxytoca

d (Simple dilutions are ratios of 2 volumes, which involve a single substance diluted with one other substance. In this case, 0.1 mL solution A is added to 9.9 mL solution B [ratio 0.1/9.9], for a total volume of 10 mL. This represents a dilution of 0.1/10. To convert a 0.1/10 dilution to a 1-in-something dilution, set up a ratio proportion calculation: 0.1 is to 10 as 1 is to x, i.e., 0.1/10=1/x, and solve for x.)

152) A colorimetric method calls for the use of 0.1 mL of serum, 5 mL of reagent and 4.9 mL of water. What is the dilution of the serum in the final solution? a. 1:5 b. 1:10 c. 1:50 d. 1:100

d (Immunoglobulin allotype is in constant regions of heavy and light chains.)

152) Antibody allotype is determined by the: a. constant region of heavy chain b. constant region of light chain c. variable regions of heavy and light chains d. constant regions of heavy and light chains

b (Identifies predominant cell line impacted in viral response.)

152) In an uncomplicated case of infectious mononucleus, which of the following cells are affected? a. erythrocytes b. lymphocytes c. monocytes d. thrombocytes

c (Simple dilutions are ratios of 2 volumes, which involve a single substance diluted with one other substance. In this case, 1 mL solution A is added to 4 mL solution B [ratio 1/4], for a total volume of 5 mL. This represents a dilution of 1/5.)

153) 4 mL of water are added to 1 mL of serum. This represents which of the following serum dilutions? a. 1:3 b. 1:4 c. 1:5 d. 1:6

a (Enterobacteriaceae ferment glucose and are oxidase negative. Plesiomonas was a member of the Vibrio family in part because it is oxidase positive. However, it was moved to the Enterobacteriaceae family despite its positive oxidase reaction.)

153) Plesiomonas shigelloides is a relatively new member of the family Enterobacteriaceae. What characteristic separates it from other members of the Enterbacteriaceae? a. it is oxidase positive b. it ferments glucose c. it produces pyocyanin d. it requires 10% CO2 for growth

b (The increased plasma glucose seen in diabetes mellitus results in excess glucose in the glomerular filtrate. Increased amounts of water are required for excretion of the excess glucose in the filtrate. As a result, increased fluid intake is characteristic of persons with diabetes mellitus.)

153) Polyuria is usually correlated with: a. acute glomerulonephritis b. diabetes mellitus c. hepatitis d. tubular damage

b (Delineates pathophysiology of initial B-cell proliferation in EBV infection.)

153) The atypical lymphocyte seen in the peripheral smear of patients with infectious mono is probably derived from which of the following: a. T lymphs b. B lymphs c. monocytes d. mast cells

a (Immunoglobulin IgG subclass function, IgG1 goes through placenta best.)

153) Which IgG subclass is most efficient at crossing the placenta? a. IgG1 b. IgG2 c. IgG3 d. IgG4

d (The prefix an- means not, or without. The prefix azo- stands for nitrogenous, dys- means pain, and di- means double.)

154) Cessation of urine flow is defined as: a. azotemia b. dysuria c. diuresis d. anuria

c (RhIg is of no benefit once a person has been actively immunized and has formed anti-D)

154) Rh immune globulin administration would not be indicated in an Rh-neg woman who has a: a. first trimester abortion b. husband who is Rh-pos c. anti-D titer of 1:4,096 d. mother having a positive DAT

a (Immunoglobulin structure, J chain associated with IgA.)

154) The J chain is associated with which of the following immunoglobulins? a. IgA b. IgG c. IgE d. IgD

a (T-cell activation follows during second week of IM in response to the EBV induced B-cell infection and activation.)

154) Which of the following cells is the atypical lymphocyte seen on the peripheral blood smear of patients with infectious mononucleosis? a. T lymphocytes b. B lymphocytes c. monocytes d. mast cells

a (The most frequently used expression, concentrations of m/v are reported as grams percent [g%] or g/dL, as well as mg/dL and ug/dL. When percent concentration is expressed without a specified form, it is assumed to be weight per unit volume.)

154) Which of the following is the formula for calculating a percent (w/v) solution? a. grams of solute/volume of solvent x 100 b. grams of solute x volume of solvent x 100 c. volume of solvent/grams of solute x 100 d. (grams of solute x volume of solvent)/ 100

b (The formula to calculate the percentage assumes the mother's blood volume as 5,000 mL. 0.003 x 5,000 mL = l5 mL.)

155) A Kleihauer-Betke stain of a postpartum blood film revealed 0.3 fetal cells. What is the estimated volume of the fetomaternal hemorrhage expressed as whole blood: a. 5 b. 15 c. 25 d. 35

b (The concentration of a weight/unit volume solution is expressed as grams/100 mL. Therefore, 20 grams in 0.5 L = 4 grams in 100 mL = 4% solution.)

155) A solution contains 20 g of solute dissolved in 0.5 L of water. What is the percentage of this solution? a. 2% b. 4% c. 6% d. 8%

b (Differentiates CMV as most common of viral diseases that lacks serological evidence of infection.)

155) The disease most frequently present in patients with atypical lymphocytosis and persistently negative tests is: a. toxoplasmosis b. cytomegalovirus infection c. herpes virus infection d. viral hepatitis

d (There is an increase in the serum levels of amylase and lipase in acute pancreatitis. However, the elevated level of lipase persists longer than amylase. Elevated levels of lipase and amylase are seen in other intra- abdominal conditions, but the frequency of elevations is less with lipase than amylase.)

155) The most specific enzyme test for acute pancreatitis is: a. acid phosphatase b. trypsin c. amylase d. lipase

b (The percentage of fetal cells/100, the mother's volume is assumed to be 5,000 mL. The percentage must be multiplied by 50 to determine total volume.)

156) Based upon Kleihauer-Betke test results, which of the following formulas is used to determine the volume of fetomaternal hemorrhage expressed in mL of whole blood. a. % of fetal cell present x 30 b. % of fetal cell present x 50 c. % of maternal cells x 30 d. % of maternal cells x 50

b (The concentration of a weight/unit volume solution is expressed as grams/100 mL. A 3% [w/v] solution contains 3 grams in 100 mL therefore 1 L contains 30 grams)

156) How many grams of sulfosalicylic acid (MW=254) are required to prepare 1 L of a 3% (w/v) solution? a. 3 b. 30 c. 254 d. 300

b (Ketones are intermediate components of fat metabolism. When access to carbohydrates normally broken down to supply energy is limited, fats are broken down for energy, and the intermediate ketone products [acetone, acetoacetic acid and beta-hydroxybutyric acid] can be detected in the urine.)

156) Ketones in urine are due to: a. complete utilization of fatty acids b. incomplete fat metabolism c. high carbohydrate diets d. renal tubular dysfunction

c (Of the organisms listed only Pseudomonas aeruginosa is oxidase positive.)

156) The stock cultures needed for quality control testing of oxidase production are: a. Escherichia coli / Klebsiella pneumoniae b. Salmonella typhimurium / Escherichia coli c. Escherichia coli / Pseudomonas aeruginosa d. Proteus mirabilis / Escherichia coli

a (Amylase and lipase are hydrolases involved in the breakdown of starch and glycogen, and lipid metabolism, respectively. Both enzymes are primarily located in the pancreas. Disorders of the pancreas are characterized by elevated levels of the enzymes. 5`-NT, GGT, AST and LD are elevated in liver and hepatobiliary diseases.)

156) Which of the following enzymes are used in the diagnosis of acute pancreatitis? a. amylase (AMS) and lipase (LPS) b. aspartate aminotransferase (AST) and alanine aminotransferase (ALT) c. 5'-nucleotidase (5'N) and gamma-glutamyl transferase (GGT) d. aspartate aminotransferase (AST) and lactate dehydrogenase (LD)

c (Use the formula: [fetal cells counted/ cells counted] x [maternal blood volume]. Assume the mother's blood volume is 5,000 mL. In this example, 30 fetal cells/2,000 cells counted x 5,000 mL = 75 mL. RhIg protects against 30 mL. So 2.5 vials are needed, rounded up to 3 full vials. Add 1 vial for hospital policy and 4 vials are needed.)

157) An acid elution stain was made using a 1-hour post-delivery maternal blood sample. Out of 2,000 cells that were counted, 30 of them appeared to contain fetal hemoglobin. It is the policy of the medical center to add 1 vial of Rh immune globulin to the calculated dose when the estimated volume of the hemorrhage exceeds 20 mL of whole blood. Calculate the number of vials of Rh immune globulin that would be indicated under these circumstances. a. 2 b. 3 c. 4 d. 5

a (Macrophage phagocytosis enhanced by opsonins.)

157) Macrophage phagocytosis of bacteria is enhanced by which of the following: a. opsonin b. antigen c. hapten d. secretory piece

b (In the amyloclastic, saccharogenic and chromogenic methods for measurement of amylase, the substrate, starch is converted to glucose and maltose.)

157) Which of the following enzymes catalyzes the conversion of starch to glucose and maltose? a. malate dehydrogenase b. amylase c. creatine kinase d. isocitric dehydrogenase

c (Differentiates leukemia with classic granulocyte anomaly.)

157) Which of the following is associated with pseudo-Pelger-Huet anomaly? a. aplastic anemia b. iron deficiency anemia c. myelogenous leukemia d. Chediak-Higashi syndrome

c (Bilirubin is a product of hemoglobin degradation. It occurs in both unconjugated and conjugated forms. Only conjugated bilirubin can pass through the glomerulus, because unconjugated bilirubin is bound to albumin. Conjugation of bilirubin takes place in the liver, and liver damage interferes with the continued degradation to urobilinogen.)

158) Bilirubinuria may be associated with: a. strenuous exercise b. increased destruction of platelets c. viral hepatitis d. hemolytic anemia

d (Of the organisms listed only Serratia marcescens is DNase positive.)

158) The stock cultures needed for quality control testing of deoxyribonuclease (DNase) production are: a. Salmonella typhimurium / Escherichia coli b. Escherichia coli / Pseudomonas aeruginosa c. Proteus mirabilis / Escherichia coli d. Serratia marcescens / Escherichia coli

c (Haptens initiate the alternative pathway.)

158) Which of the following is most likely to activate the alternative pathway of complement activation? a. lipopolysaccharides b. glycoproteins c. haptens d. IgG complexed with antigen

d (Amylase is present primarily in the pancreas. Pancreatitis results in the release of the enzyme into the serum. Creatinine is a nonprotein nitrogenous substance and is measured for renal function. Beta- hydroxybutyrate is measured for diabetic acidosis and LD isoenzymes are evaluated for disorders involving the heart and liver.)

159) A physician suspects his patient has pancreatitis. Which test would be most indicative of this disease? a. creatinine b. LD isoenzymes c. beta-hydroxybutyrate d. amylase

d (Quality control of indole requires both a positive and a negative control. E coli and E cloacae respectively produce a positive and negative reaction with indole.)

159) Quality control of the spot indole test requires the use of ATCC cultures of: a. Pseudomonas aeruginosa / Proteus mirabilis b. Salmonella typhi / Shigella sonnei c. Escherichia coli / Proteus vulgaris d. Escherichia coli / Enterobacter cloacae

a (The mean = sum of values/number of values; standard deviation = square root of the sum of [observed values - mean] squared / number of samples; coefficient of variation = [standard deviation/ mean] x 100. Therefore, [2.64/139] x 100 = 1.9%.)

159) The following 5 sodium control values in unit were obtained: 140, 135, 138, 140, 142 Calculate the coefficient of variation: a. 1.9% b. 2.7% c. 5.6% d. 6.1%

b (C3 breaks down to a small C3a, which floats away and is an anaphylatoxin, and the larger C3b, which lands on the target surface and becomes part of C5 convertase, is also a powerful opsonin.)

159) Which of the following is the larger residual split portion of C3? a. C3a b. C3b c. C4 d. C1q

c (This is a ratio calculation. 100 [g of NaCl] to 58.5 [gram equivalent weight of NaCl] = 1.709. 23 [gram equivalent weight of Na] x 1.709 = grams of Na in 100g NaCl.)

167) The sodium content (in grams) in 100 grams of NaCl is approximately: (atomic weights: Na=23, Cl=35.5) a. 10 b. 20 c. 40 d. 60

d (Sterile docking devices allow entry into donor units without affecting the expiration date of the product.)

16) A unit of packed cells is split into 2 aliquots under closed sterile conditions at 8 am. The expiration time for each aliquot is: a. 4 pm the same day b. 8 pm the same say c. 8 am the next morning d. the original date of the unsplit unit

d (Average glucose over time is best predictor.)

16) Monitoring long term glucose control in patients with adult onset diabetes mellitus can best be accomplished by measuring: a. weekly fasting 7 am serum glucose b. glucose tolerance testing c. 2 hr. postprandial serum glucose d. hemoglobin A1c

a (Specificity is defined as negativity in the absence of disease.)

16) The ability of a procedure to measure only the component it claims to measure is called: a. specificity b. sensitivity c. precision d. reproducibility

a (Nephelometry and latex agglutination are used to detect rheumatoid factor. The sensitivity of detection of immune complexes containing rheumatoid factor has been increased by nephelometry over the sensitivity of manual latex agglutination methods.)

16) Which of the following is the most sensitive and appropriate method for the detection of rheumatoid factor? a. nephelometry b. immunofixation electrophoresis c. immunofluorescence d. manual latex agglutination

a (Haemophilus influenzae requires X and V factors. Sheep blood agar supplies X factor, and the staphylococci produce V factor, so colonies grow around staph colonies.)

160) An organism that exhibits the satellite phenomenon around colonies of staphylococci is: a. Haemophilus influenzae b. Neisseria meningitidis c. Neisseria gonorrhoeae d. Klebsiella pneumoniae

a (Aspartate aminotransferase [AST] is involved in the transfer of an amino group between aspartate and alpha-keto acids. AST is present in several tissues, with its highest concentrations in cardiac tissue, liver and skeletal muscle. Depending on the type of liver disease, the levels may be 100x the upper limits of normal [ULN].)

160) Aspartate amino transferase (AST) is characteristically elevated in diseases of the: a. liver b. kidney c. intestine d. pancreas

c (The mean [often called the average] is the most widely recognized descriptive statistic. The mean of a set of data can be calculated in several ways, but in each case the result is, in general, an indication of the central point of the data.)

160) The statistical term for the average value is the: a. mode b. median c. mean d. coefficient of variation

a (C3 breaks down to a small C3a, which floats away and is an anaphylatoxin, and the larger C3b, which lands on the target surface and becomes part of C5 convertase, is also a powerful opsonin.)

160) Which of the following activities is associated with C3b? a. opsonization b. anaphylaxis c. vasoconstriction d. chemotaxis

d (Transfusion should generally be avoided except in cases of life-threatening anemia. A hemoglobin of 10.8 g/dL [108 g/L] is not life-threatening, especially if the patient is not actively bleeding.)

161) A 40 year old man with autoimmune hemolytic anemia due to anti-E has a hemoglobin level of 10.8 g/dL. This patient will most likely be treated with: a. whole blood b. RBCs c. FFP d. no transfusion

d (Biological functions of immunoglobulins, IgE mediates immediate type hypersensitivity seen in penicillin allergy.)

161) After a penicillin injection a patient rapidly develops respiratory distress, vomiting and hives. This reaction is primarily mediated by: a. IgG b. IgA c. IgM d. IgE

b (Aspartate aminotransferase [AST] belongs to the class of transferase enzymes. Specifically, AST catalyzes the transfer of an amino group from aspartate to alpha-ketoglutarate forming oxaloacetate and glutamate.)

161) Amino tranferase enzymes catalyze the: a. exchange of amino groups and sulfhydydryl groups between alpha-amino and sufur-containing acids b. exchange of amino and keto groups between alpha-amino and alpha-keto acids c. hydrolysis of amino acids and keto acids d. reversible transfer of hydrogen from amino acids to coenzyme

d (Pseudomonas aeruginosa produces the blue-green pigment, pyocyanin.)

161) An organism isolate from the surface of a skin burn is found to produce a diffusible green pigment on a blood agar plate. Further studies of the organism would most likely show the organism to be: a. Staphylococcus aureus b. Serratia marcescens c. Flavobacterium meningosepticum d. Pseudomonas aeruginosa

c (The majority of the filtered glucose is reabsorbed by active transport in the proximal convoluted tubules [PCT]. Damage to the PCTs results in glycosuria and a normal plasma glucose.)

161) Glycosuria may be due to: a. hypoglycemia b. increased renal threshold c. renal tubular dysfunction d. increased glomerular filtration rate

a (The mode is the most frequently occurring value in a set of data.)

161) The most frequent value in a collection of data is statistically known as: a. mode b. median c. mean d. standard deviation

d (Pseudomonas aeruginosa often has a sweet odor that smells like grapes.)

162) A nonfermenting gram-negative bacillus is isolated from a wound. The nitrate and oxidase are strongly positive. The growth on SBA has a grape-like odor. The organism is: a. Burkholderia cepacia b. Moraxella lacunata c. Chryseobacterium (Flavobacterium) meningosepticum d. Pseudomonas aeruginosa

d (Bone marrow transplant patients are at risk for transfusion-associated graft-vs-host disease [TA-GVHD] and therefore should receive irradiated blood products.)

162) A patient in the immediate post bone marrow transplant period has a hematocrit of 21%. The red cell product of choice for this patient would be: a. packed b. saline washed c. micro aggregate filtered d. irradiated

c (In immunoelectrophoresis, first the serum is separated in an agarose gel by electrophoresis, then in a trough that is cut parallel to the plane of the electrophoresis, antiserum is placed. The antibody diffuses toward the serum proteins and arcs of antibody antigen precipitation occur.)

163) In immunofixation electrophoresis: a. the antibody reacts with the antigen and them to complex is electrophoresed b. the antigen is electrophoresed into an antibody containing gel c. the antigen is electrophoresed and then monospecific antisera is reacted with it d. the antigen is eletrophoresed. transferred to nitrocellulose and then antibody reacts with it and an EIA is performed

d (Thrombocytes [platelets] participate in several aspects of hemostasis.)

270) Cells involved in hemostasis are: a. erythrocytes b. granulocytes c. lymphocytes d. thrombocytes

a (AST and ALT levels are the highest in acute hepatocellular conditions, specifically acute viral hepatitis. The levels may be 100 times the upper limit of normal. Slight increases of the aminotransferases are seen in cirrhosis and metastatic hepatic carcinoma.)

163) The greatest activities of serum AST and ALT are seen in which of the following? a. acute viral hepatitis b. primary biliary cirrhosis c. metastatic hepatic cirrhosis d. alcoholic cirrhosis

d (The juxtaglomerular nephron have long loops of Henle, and the urinary filtrate passes through the renal medulla with its high osmotic gradient, causing reabsorption of water in the descending loop of Henle. Cortical nephrons are located in the renal cortex and have short loops of Henle that do not reach the medulla.)

163) The urinary tract structures responsible for renal concentration are the: a. renal pelvis b. cortical nephrons c. renal papillae d. juxtamedullary nephrons

d (The arithmetic mean is the quantity that is most familiar and is ordinarily meant when we refer to the mean or average of a set of data. It is calculated as the sum of values divided by the number of values.)

163) Which of the following is the formula for arithmetic mean? a. square root of the sum of values b. sum of values x number of values c. number of values/sum of values d. sum of values/number of values

b (The mean = sum of values/ number of values.)

164) Given the following values: 100, 120, 150, 140, 130 What is the mean? a. 100 b. 128 c. 130 d. 640

a (Osmolarity measures the number of particles in a solution, whereas specific gravity is influenced not only by the number of particles but also their density. Renal concentration is concerned with smaller molecules, such as sodium and chloride. Each of these molecules will contribute the same to an osmolarity reading as a large molecule of glucose.)

164) The most accurate test to determine renal concentration is: a. osmolarity b. glomerular filtration rate c. specific gravity d. tubular reabsorption rate

d (Complement, C5 - C9, is the membrane attack unit.)

164) Which of the following is the "membrane attack complex" of complement activation? a. C1 b. C3 c. C4, C2, C3 d. C5b, C6, C7, C8, C9

a (Negative check cells means the results of tubes with the negative reactions are invalid. The reactivity of the check cells should be verified with anti-IgG since anti-E was detected, indicating the anti-IgG was reactive. All tests that were nonreactive with the check cells requires repeat test performance.)

165) Anti-E is identified in a panel at the antiglobulin phase. When check cells are added to the tubes no agglutination is seen. The most appropriate course of action would be to: a. quality control the AHG reagent and check cells and repeat the panel b. open a new vial of check cells for subsequent testing that day c. open a new vial of AHG for subsequent testing that day d. record the check cell reactions and report the antibody panel result

c (Leukopenia with immature granulocytes in all stages, including blast with markedly decreased LAP and Philadelphia chromosome, positively identify CML. Leukopenia is consistent with engorged marrow space.)

165) The following results were obtained on a 35-year-old woman complaining of fatigue and weight loss: These results are consistent with: a. neutrophilic leukemoid reaction b. idiopathic thrombocythemia c. chronic myelocytic leukemia d. leukoerythroblastosis in myelofibrosis

d (Campylobacter jejuni/coli grow better at 42 C than 37 C and other organisms in the colon are inhibited at this high temperature)

165) The optimal incubator temp. for isolation of Campylobacter jejuni/coli group is: a. 4 C b. 20 C c. 25 C d. 42 C

c (Imprecision is the measure of random error commonly expressed as the standard deviation. A more useful measure, the coefficient of variation [CV], is the measure of relative random error usually expressed as a percentage: CV [%] = [standard deviation/ mean] x 100.)

165) Which of the following is the formula for coefficient of variation? a. (standard deviation x 100)/standard error b. (mean x 100)/standard deviation c. (standard deviation x 100)/mean d. (variance x 100)/mean

a (C3a, C4a and C5a are anaphylatoxins and cause release of histamine from basophils and mast cells.)

165) Which of the following releases histamine and other mediators from basophils? a. C3 b. properdin factor B c. C1q d. C4

b (The coefficient of variation % - [standard deviation/mean] x 100.)

166) A cholesterol QC chart has the following data for the normal control: The coefficient of variation for this control is: a. 1.14 % b. 2.19 % c. 4.38 % d. 9.49 %

d (The lumbar tap may be traumatic, which will produce blood. Blood cells in the CSF will not be due to a central nervous system defect in that case. Do the cell count on the last tube; it is the least likely to be contaminated by a bloody tap.)

166) To avoid falsely elevated spinal fluid cell counts : a. use an aliquot from the first tube collected b. use only those specimens showing no turbidity c. centrifuge all specimens before counting d. select an aliquot from the last tube collected

d (Elevated serum levels of LD up to 50 times the upper limit of normal are seen with pernicious anemia. The ineffective erythropoiesis results in the release of large quantities of LDI and LD2. Increased levels of LDI and LD2 may be seen in renal disease, but the increase is not as great as for pernicious anemia. Slight increases of LD3 are seen in pulmonary conditions and pancreatitis.)

166) Which of the following clinical disorders is associated with the greatest elevation of lactate dehydrogenase isoenzyme? a. pneumonia b. glomerulonephritis c. pancreatitis d. pernicious anemia

c (Demonstrates nature of basophils.)

167) In chronic myelocytic leukemia, blood histamine concentrations tend to reflect the : a. number of platelets present b. serum uric acid concentrations c. number of basophils present d. the total number of granulocytes

d (Creatine kinase [CK] catalyzes the reversible phosphorylation of creatine. The highest levels of the enzyme are found in skeletal muscle, heart muscle and brain tissue. Increased serum enzyme activity is present in diseases involving the listed muscles and tissue. Lipase is measured for acute pancreatitis; the transaminase and lactate dehydrogenase [LD] are not markedly increased in muscular dystrophy.)

167) The enzyme which exists chiefly in skeletal muscle, heart, and brain, is grossly elevated in active muscular dystrophy, and rises early in myocardial infarction is: a. lipase b. transaminase c. lactate dehydrogenase d. creatine kinase

a (Samples must be labeled with 2 independent patient identifiers and the date of collection. This information should be identical to that on the patient's identification band and request.)

167) Which of the following represents an acceptably identified patient for sample collection and transfusion a. a handwritten band with patients name and hospital identification number is affixed to the patients leg b. the addressographed hospital band is taped to the patients bed c. an unbanded patient responds positively when his name is called d. the chart transported with the patient contains his armband not yet attached

b (Haemophilus influenzae was previously the most common cause of bacterial meningitis in young children. However, the Haemophilus influenzae type B vaccine has been in use for several years, resulting in a low incidence of H influenzae causing meningitis. This patient has not had most childhood vaccinations, so he is susceptible to H influenzae.)

168) A 4 yr. old boy is admitted to the hospital with suspected meningitis. He has not had most of the childhood vaccines. The suspected pathogen is: a. Listeria monocytogenes b. Haemophilus influenzae c. Streptococcus agalactiae d. Neisseria meningitidis

d (Babies' IgG comes from their mother, but if they have elevated IgM, an in utero, or neonatal infection is indicated.)

168) A single reliable screening test for detecting neonatal infection in the absence of clinical signs is: a. serum immunoelectrophoresis b. differential leukocyte count c. CD4 cell counts d. quantitative serum IgM determination

d (Results of ABO and Rh testing on a current specimen must always be compared to that of a previous transfusion record. Errors in typing or patient identification may be detected when discrepancies are found. Collection of a new sample allows determination of which sample was incorrectly collected.)

168) Samples from the same patient were received on 2 consecutive days. Test results are summarized below: How should the request for crossmatch be handled? a. crossmatch A, Rh-positive units with sample from day 1 b. crossmatch B, Rh-positive units with sample from day 2 c. crossmatch AB, Rh-positive units with both samples d. collect a new sample and repeat the tests

d (Lactate dehydrogenase [LD] catalyzes the interconversion of lactic and pyruvic acids. Electrophoretically, using agarose or cellulose acetate medium, LD can be separated into 5 isoenzymes, LD1-LD5. CK and lipase have 3 isoenzymes; AST has 2.)

168) The enzyme present in almost all tissues that may be separated by electrophoresis into 5 components is: a. lipase b. transaminase c. creatine kinase d. lactate dehydrogenase

a (Erythrocytes contain 150 times more LD activity than serum, mostly LD1 and LD2. Rupture of the RBC membranes as in hemolysis will elevate the serum level of the enzyme. LD5 is increased in liver disorders and drug toxicity if the liver is involved.)

169) A common cause of a falsely increased LD1 fraction of lactic dehydrogenase is: a. specimen hemolysis b. liver disease c. congestive heart failure d. drug toxicity

b (Definition of Bence Jones protein.)

169) Bence Jones proteins are: a. immunoglobulin catabolic fragments in the urine b. monoclonal light chains c. whole immunoglobulins in the urine d. Fab fragments of a monoclonal protein

c (A serological test to confirm the ABO on all RBC units and Rh on units labeled as Rh-negative must be performed prior to transfusion. Any errors in labeling must be reported to the collection facility.)

169) The following test results are noted for a unit of blood labeled group A, Rh-neg, anti-A 4+ anti-B 0 anti-D 3+ What should be done next? a. transfuse as a group A, Rh-neg b. transfuse as a group A, Rh-pos c. notify the collecting facility d. discard the unit

d (Diluting cells tor counting should not disturb them osmotically. Saline is the best choice. Water can lyse the cells, trichloroacetic acid will precipitate the sample, and acetic acid will form a clot with hyaluronic acid in the sample.)

169) To prepare the reagent used for mucin clot determination of synovial fluid, water is mixed with: a. hydrochloric acid b. sodium hydroxide c. trichloroacetic acid d. glacial acetic acid

c (Role of fructosamine.)

17) A patient with Type I, insulin-dependent diabetes mellitus has the following results: After reviewing these test results, the technologist concluded that the patient is in a: a. "steady state" of metabolic control b. state of flux, progressively worsening metabolic control c. improving state of metabolic control as indicated by fructosamine d. state of flux as indicted by the fasting glucose level

d (Sterile docking devices allow entry into donor units without affecting the expiration date of the product.)

17) A unit of Red Blood Cells expiring in 35 days is split into 5 small aliquots using a sterile pediatric quad set and a sterile connecting device. Each aliquot must be labeled as expiring in: a. 6 hours b. 12 hours c. 5 days d. 35 days

c (All Mueller-Hinton agar used for disk diffusion susceptibility testing should be poured to a depth of 4 mm. If the depth of the media is < 4 mm, this may be associated with excessively large zones and false-positive susceptibility results. Agar that is > 4 mm in depth may cause excessively small zone sizes.)

17) Susceptibility testing performed on quality control organisms using a new media lot number yielded zone sizes that were too large for all antibiotics tested. The testing was repeated using media from a previously used lot number, and all zone sizes were acceptable. Which of the following best explains the unacceptable zone sizes? a. the antibiotic disks were not stored with the proper desiccant b. the depth of the media was too thick c. the depth of the media was too thin d. the antibiotic disks were not properly applied to the media

b (Colors associated with urine are due to pigments. Melanin is black. Porphyrin is port wine [red]. Bilirubin is amber to brown.)

17) Urine that develops a port wine color after standing may contain: a. melanin b. porphyrins c. bilirubin d. urobilinogen

d (>20% of myeloblasts without other immature stages differentiates AML from CML and myeloid metaplasia; erythroleukemia requires at least 50% erythroid precursors in marrow.)

170) 50% - 90% myeloblasts in a peripheral blood smear is typical of which of the following? a. chronic myelocytic leukemia b. myelofibrosis with myeloid metaplasia c. erythroleukemia d. acute myelocytic leukemia

d (Findings differentiate between CMML vs CML, AML, and myelofibrosis.)

173) Which of the following is most closely associated with chronic myelononocytic leukemia? a. Philadelphia chromosome b. DIC c. micromegakaryocytes d. lysozymuria

c (These are classic gram stain, growth and biochemicals for Haemophilus influenzae.)

170) A 4-year-old is admitted with symptoms of meningitis, and a Gram stain of the cerebrospinal fluid reveals small, pleomorphic, gram-negative coccobacilli. After 24 hours incubation at 35 C, small, moist, gray colonies, which are oxidase variable, are found on the chocolate agar plate only. Which of the following biochemical data would be consistent with this isolate? a. CTA dextrose: positive / CTA maltose: positive / ONPG: negative b. sodium hippurate hydrolysis: positive / A disc: negative / CAMP test: positive c. X factor: no growth / V factor: no growth / XV factor: growth / horse blood: no hemolysis d. catalase: positive / esculin hydrolysis: positive / methyl red: positive / "umbrella" motility at room temperature

b (The 3 CK isoenzymes are CK1 or CKBB, CK2 or CKMB, CK3 or CKMM. CKMB is primarily located in myocardial tissue. Damage to the myocardial will cause an elevation of the CKMB level.)

170) The presence of which of the following isoenzymes indicates acute myocardial damage? a. CK-MM b. CK-MB c. CK-BB d. none

c (Samples must be labeled with 2 independent patient identifiers and the date of collection. This information should be identical to that on the patients identification band and request. There must be a mechanism to identify the phlebotomist, but initialing the sample tubes is not required.)

170) What information is essential on patient blood sample labels drawn for compatibility testing? a. biohazard sticker for AIDS patients b. patients room number c. unique patient medical # d. phlebotomist initials

c (Helicobacter pylori is known to cause gastritis and is a gram-negative, curved bacillus.)

171) A gram stain of a touch prep from a gastric biopsy shows gram-negative bacilli that are slender and curved. The most likely pathogen is: a. Burkholderia cepacia b. Corynebacterium urealyticum c. Helicobacter pylori d. Pasteurella multocida

b (Granulocytes must be compatible with recipient's plasma. Granulocyte products have an expiration of 24 hours.)

171) Granulocytes for transfusion should: a. be administered through a micro aggregate filter b. be ABO compatible with the recipients serum c. be infused within 72 hrs. of collection d. never be transfused to patients with a history of febrile transfusion reaction.

b (Creatine kinase [CK] is located in brain tissue and heart and skeletal muscle. Diseases involving the tissue site will increase the level of the enzyme activity. CK activity is not increased in hepatitis.)

171) In which of the following conditions would a normal level of creatine kinase be found? a. acute myocardial infarct b. hepatitis c. progressive muscular dystrophy d. intramuscular injection

b (By definition, a gram equivalent weight of an element or compound is the mass that will combine with or replace 1 mole of hydrogen.)

171) Which of the following is the formula for calculating the gram equivalent weight of a chemical? a. MW x oxidation number b. MW/oxidation number c. MW + oxidation number d. MW - oxidation number

b (Moles = grams / molecular weight.)

172) 80 grams of NaOH (MW=40) are how many moles? a. 1 b. 2 c. 3 d. 4

b (Haemophilus influenzae is recovered on chocolate agar. Francisella and Bordetella pertussis are fastidious and require special media for growth. Bacteroides is an anaerobe that will not grow aerobically.)

172) A CSF has been inoculated onto SBA and choc agar plates and into a tube of trypticase soy broth. All media were incubated in an atmosphere of 5% CO2. Which of the following organisms would usually be isolated by this procedure? a. Francisella tularensis b. Haemophilus influenzae c. Bordetella pertussis d. Bacteroides fragilis

c (Because neonates are immunologically immature, alloimmunization to red cell antigens is very rare during the neonatal period. No crossmatching is required if the initial antibody screen performed with either the baby's or mother's plasma is negative.)

172) A neonate will be transfused for the first time with group O Red Blood Cells. Which of the following is appropriate compatibility testing? a. crossmatch with mothers serum b. crossmatch with baby's serum c. no crossmatch is necessary if initial plasma screening is negative d. no screening or crossmatching is necessary for neonates

c (B cells make humoral antibodies.)

172) Humoral antibodies are produced by which cells? a. macrophages b. T lymphs c. B lymphs d. neutrophils

b (A positive DAT will interfere with weak D testing, causing both the patient and control to demonstrate positive results. Any positive result in the control tube invalidates any results.)

173) A group B, Rh-neg patient has a positive DAT. Which of the following situations would occur? a. all major crossmatches would be incompatible b. the weak D test and control would be positive c. the antibody screen test would be positive d. the forward and reverse ABO groupings would not agree

c (Electrolyte equivalents can be calculated from the equation: mg/dL x 10 = 10 mg/L. Because mg/mEq weight is the millimolar weight in mg/valence, mg/ L / mg/ mEq = mEq/L.)

173) A serum potassium (MW = 39) is 19.5 mg / 100mL. This value is equal to how many mEq/L? a. 3.9 b. 4.2 c. 5.0 d. 8.9

d (Pseudomonas aeruginosa grows at 42 C, but this temperature is inhibitory for other Pseudomonas species.)

173) If present, a characteristic that is helpful in separating Pseudomonas aeruginosa from other members of the Pseudomonas family is: a. positive test for cytochrome oxidase b. oxidative metabolism in the OF test c. production of fluorescein pigment d. growth at 42 C

d (IgA and IgE do not initiate the classical pathway of complement but can initiate the alternative pathway.)

173) Initiation of the activation mechanism of the alternative complement pathway differs from that of the classical pathway in that: a. antigen-antibody complexed containing IgM or IgG are required b. endotoxin alone cannot initiate activation c. C1 component of complement is involved d. antigen-antibody complexed containing IgA or IgE may initiate activation

a (After an acute myocardial infarction [AMI], CK activity increases 4-6 hours after the symptoms, peaks at 12-24 hours and returns to normal within 48-72 hours. AST increases 6-8 hours after the infarction. Elevated levels of LD are noted 12-24 hours after the symptoms. ALT activity does not increase with a AMI.)

173) When myocardial infarction occurs, the first enzyme to become elevated is: a. CK b. LD c. AST d. ALT

c (Although, CK-MB activity is more specific for the myocardium, CK-MM is present in both the skeletal and heart muscles. An increase of the isoenzyme activity may occur after a AMI. Only one peak would be present for a brain tumor and muscular dystrophy; no peaks would be present for hepatitis since the liver is not a tissue source of CK.)

174) A scanning of a CK isoenzyme fractionation revealed 2 peaks: A slow cathodic peak (CK-MM) and an intermediate peak (CK-MB). A possible interpretation for this pattern is: a. brain tumor b. muscular dystrophy c. myocardial infarction d. viral hepatitis

c (Definition of 'hiatus' as opening or break demonstrates the absence of intermediate maturing cells)

174) The absence of intermediate maturing cells between the blast and mature neutrophil commonly seen in acute myelocytic leukemia and myelodysplastic syndrome is called: a. subleukemia b. aleukemic leukemia c. leukemic hiatus d. leukemoid reaction

c (Patients with multiple myeloma demonstrate rouleaux formation, which can cause the appearance of agglutination. If the cells are washed to remove residual plasma, and tests repeated, an accurate red cell typing is obtained. By performing a saline replacement with the reverse typing, true agglutination will remain when the cell buttons of the reverse cells are resuspended in saline.)

174) The following reaction were obtained: cells tested: anti-A = 4+| anti-B = 3+| anti-A,B = 4+ serum test: A1 cells = 2+ | B cells = 4+ The technologist washed the patients cells with saline and repeated the forward typing. A saline replacement technique was used with the reverse typing. the following results were obtained: cells tested: anti-A = 4+ | anti-B = 0 | anti-A,B = 4+ serum test: A1 cells = 0 | B cells = 4+ the results are consistent with: a. acquired immunodeficiency disease b. Bruton agammaglobulinemia c. multiple myeloma d. acquired "B" antigen

c (Classical complement pathway: C1 is activated by binding 2 Fcs of immunoglobulin, then C4 is split to C4a and C4b; C4b binds, and next C2 is bound and cleaved; this forms C3 convertase, so C3 binds and is cleaved; C3b stays bound; C3a floats away; C5 next is split and C5b binds and C5a floats away. C6, C7, C8, and C9 bind sequentially and cause a hole in the membrane.)

174) Which of the following is cleaved as a result of activation of the classical complement pathway? a. properdin factor B b. C1q c. C4 d. C3b

a (Moles = grams/molecular weight.)

174) Which of the following is the formula for calculating the number of moles of a chemical? a. g/GMW b. g x GMW c. GMW/g d. (g x 100)/GMW

a (Molality is the number of moles of solute per 1 kg of solvent.)

175) A 1 molal solution is equivalent to: a. a solution containing 1 mole of solute per kg of solvent b. 1,000 mL of solution containing 1 mole of solute c. a solution containing 1 g equivalent weight of solute in 1 L of solution d. a 1 L solution containing 2 moles of solute

c (Classical complement pathway [see answer 174]; alternative pathway: C3 + Factor B --> iC3B + Factor D and Mg++ --> C3Bb or C3bBb; this is stabilized by properdin to make C3bBbC3bP, which is a C5 convertase, and C5 is split to C5a and C5b, and next C6, C7, C8 and C9 are activated and form a hole in the cell.)

175) The component associated only with the alternative pathway of complement activation is: a. C4 b. C1q c. properdin factor B d. C3a

d (ABO immunoglobulins develop at approximately 3 months of age, attain adult levels by age 10, and may, but not always, decline in titer in the elderly)

175) What is the most likely cause of the following ABO discrepancy? Cells: anti-A = 0 | anti-B = 0 serum: A1 cells = 0 | B cells= 0 a. recent transfusion with group O blood b. antigen depression due to leukemia c. false-neg cell typing due to rouleaux d. obtained from the heel stick of a 2 month old baby

d (Classic discrimination between CML vs other hematological conditions: sideroblastic anemia, AML, myelofibrosis.)

175) Which of the following is most closely associated with chronic myelogenous leukemia? a. ringed sideroblasts b. DIC c. micromegakaryocytes d. Philadelphia chromosome

a (Campylobacter jejuni will grow at 37 C, but prefers 42 C. It is oxidase and catalase positive and motile.)

175) Which of the following results is typical of Campylobacter jejuni? a. optimal growth at 42 C b. oxidase negative c. catalase negative d. nonmotile

a (Campylobacter is microaerophilic, and requires a decreased oxygen and increased carbon dioxide atmosphere for growth.)

176) Optimum growth of Campylobacter jejuni is obtained on suitable media incubated at 42 C in an atmosphere containing: a. 6% O2, 10%-15% CO2, 85%-90% nitrogen b. 10% H2, 5% CO2, 85% nitrogen c. 10% H2, 10% CO2, 80% nitrogen d. 25% O2, 5% CO2, 70% nitrogen

a (Molarity is a number that expresses the number of moles of substance in 1 L of solution.)

176) Which of the following is the formula for calculating the molarity of a solution? a. number of moles of solute/L of solution b. number of moles of solute x 100 c. 1 GEW of solute x 10 d. 1 GEW of solute/L of solution

b (Acquired B occurs in group A individuals and is due to deacetylation of the A antigen by bacterial enzymes. Detection of acquired B is dependent upon the source of anti-B used.)

176) Which of the following patient data best reflects the discrepancy seen when a person's red cells demonstrate the acquired-B phenotype? a. A b. B c. C d. D

c (Increased levels of aldolase and CK are seen with skeletal muscle disease. The magnitude of the elevation is dependent on the type of skeletal muscle disease.)

177) A 10 yr. old child was admitted to pediatrics with an initial diagnosis of skeletal muscle disease. The best confirmatory tests would be: a. creatine kinase and isocitrate dehydrogenase b. gamma-glutamyl transferase and alkaline phosphatase c. aldolase and creatine kinase d. lactate dehydrogenase and malate dehydrogenase

d (Acute leukemia is characterized by a maturation defect, whereby immature hematopoietic progenitors cannot overcome a block in differentiation, also known as the leukemic hiatus.)

177) A block in the differentiation or maturation of, and an accretion of immature hematopoietic progenitors is a hallmark of: a. chronic lymphocytic leukemia b. myeloproliferative diseases c. polycythemia vera d. acute myelogenous leukemia

b (The tissue damage resulting from type III hypersensitivity is caused by the deposition of immune complexes, which recruit neutrophils to the tissues. The neutrophils release their lysosomal enzymes, resulting in inflammation and damage to the surrounding tissues.)

209) Which of the following is an important cellular mediator of immune complex tissue injury? a. mast cell b. neutrophil c. basophil d. eosinophil

a (Tn is caused from a somatic mutation and the phenomenon is persistent. Resolution of the red cell typing can be performed with enzyme-treated patient cells, since Tn is denatured by enzymes. Although the reactivity with anti-A may be weak, testing with anti-A1 lectin gives strong reactivity, unlike subgroups of A, which are weakly reacting with anti-A and nonreactive with A1 lectin.)

177) Which of the following is characteristic of Tn polyagglutinable red cells? a. if group O, they may appear to have acquired a group A antigen b. they show strong reactions when the cells are enzyme treated c. they react with Arachis hypogaea lectin d. the polyagglutination is a transient condition

a (Haemophilus influenzae is indigenous flora of the upper respiratory tract.)

178) Haemophilus influenzae is most likely considered normal indigenous flora in the: a. oropharynx b. female genital tract c. large intestine d. small intestine

a (In the immunoinhibition technique for CK-MB determination, antibodies are directed against the M and B units of the enzymes. Anti-M inhibits all M activity but not B activity. CK activity is measured before and after inhibition. The activity remaining after inhibition is a result of the B subunit for BE and MB activity.)

178) In the immunoinhibition phase of the CK-MB procedure: a. M subunit is inactivated b. B subunit is inactivated c. MB is inactivated d. BB is inactivated

c (Mixed-field reactivity is a characteristic of the A3 subgroup. Transfusion history would be important to be sure it is not 2 cell populations.)

178) Mixed field agglutination encountered in ABO grouping with no history of transfusion would most likely be due to: a. Bombay phenotype (Oh) b. T activation c. A3 red cells d. positive IAT

b (Discriminates between conditions due to granulocytic precursor, which would exclude lymphocytic leukemia.)

179) All of the following conditions are myeloproliferative disorders except: a. myelocytic leukemia b. lymphocytic leukemia c. polycythemia vera d. idiopathic thrombocythemia

d (The beta-lactamase enzyme produced by Haemophilus influenzae inactivates the antibiotics that have a beta-lactam ring in their structure, such as penicillins and cephalosporins.)

179) Haemophilus influenzae becomes resistant to ampicillin when the organism produces a: a. capsule of polysaccharide material b. affinity for the beta-lactam ring of the ampicillin c. porphobilinogen d. beta-lactamase enzyme

c (Of the 3 CK isoenzymes, CK-MB is located in the myocardial. The fraction is elevated with an acute myocardial infarction [AMI]. CK-MM is elevated in acute muscular stress following strenuous exercise. CK-BB is increased in brain injury.)

179) The presence of increased CK-MB activity on a CK electrophoresis pattern is most likely found in a patient suffering from: a. acute muscular stress following strenuous exercise b. malignant liver disease c. myocardial infarction d. severe head injury

b (Polyagglutination is a property of the cells. Most adult plasma agglutinate the cells due to naturally occurring antibodies directed towards the crypt antigens.)

179) Which of the following is a characteristic of polyagglutinable red cells? a. can be classified by reactivity with Ulex europaeus b. are agglutinated by most adult sera c. are always an acquired condition d. autocontrol is always positive

b (Coagulase-negative staphylococci are commonly associated with contaminated blood cultures; however, they are also increasing as a cause of true bacteremia. Significant bacteremia in a patient with endocarditis is usually continuous and low grade. In most cases, all blood cultures drawn will yield positive results. The facts that only 1 bottle of 1 set was positive, and that the bottle did not become positive until day 5 of incubation, indicate that this isolate is most likely a contaminant.)

18) 3 sets of blood cultures were obtained from an adult patient with fever and suspected endocarditis. The aerobic bottle of one set had growth of Staphylococcus epidermidis at 5 days of incubation. This indicates that: a. there was a low grade bacteria infection b. the organism is most likely a contaminant c. the patient has a line infection d. the blood culture bottles are defective

a (Small percentage of normal healthy individuals and individuals with disorders not classified as autoimmune may have positive ANA. But positive ANA occurs most commonly in patients with systemic rheumatic diseases.)

18) A consistently and repeatedly negative IFA-ANA is: a. strong evidence against untreated SLE b. associated with active SLE c. characteristic of SLE with renal involvement d. associated with lupus inhibitor

c (Chelex 100 resin is often used to extract DNA from forensic samples, but may also be used for clinical samples.)

18) Chelex 100 resin is used to: a. purify mRNA from total RNA b. extract total RNA from cells c. extract DNA from cells d. remove unincorporated primers from PCR reactions prior to gel analysis of PCR products

b (Specificity is defined as negativity in the absence of disease.)

18) Diagnostic specificity is defined as the percentage of individuals: a. with a given disease who have a positive result by a given test b. without a given disease who have a negative result by a give test. c. with a given disease who have a negative result by a given test d. without a given disease who have a positive result by a given test

b (Erythropoietin action.)

18) Erythropoietin acts as: a. shorten the replication time of the granulocytes b. stimulate RNA synthesis of erythroid cells c. increase colony-stimulating factors produced by the B-lymphs d. decrease the release of marrow reticulocytes

a (Interpretation of glycated hemoglobin.)

18) Total glycosylated hemoglobin levels in a hemolysate reflect the: a. average blood glucose levels of the past 2-3 months b. average blood glucose levels for the past week c. blood glucose level at the time the sample is drawn d. hemoglobin A1c level at the time the sample is drawn

c (If storage devices do not have automated temperature recording, temperature must he manually monitored every 4 hours.)

18) When platelets are stored on a rotator set on an open bench top, the ambient air temps. must be recorded: a. once a day b. twice a day c. every 4 hours d. every hour

d (Haemophilus parainfluenzae requires NAD for growth but not hemin. This distinguishes it from H influenzae. H haemolyticus is hemolytic, and H ducreyi does not cause epiglotittis.)

180) An isolate on chocolate agar from a patient with epiglottitis was suggestive of Haemophilus species. Additional testing showed that the isolate required NAD for growth and was nonhemolytic. The organism is most likely Haemophilus: a. haemolyticus b. ducreyi c. influenzae d. parainfluenzae

b (False positives for fecal occult blood occur when patient diet includes food that produces peroxidase. Horseradish, broccoli, radishes, melons, and other foods can cause a false positive. Patients that take aspirin may have some occult bleeding that is not associated with colorectal cancer, but acetaminophen is not a cause of bleeding. Ascorbic acid at high doses can interfere with the test and cause a false-negative.)

180) False-positive results can occur for fecal occult blood due to the ingestion of: a. ascorbic acid b. horseradish c. acetaminophen d. blueberries

d (The LD isoenzymes as a percentage of total LD activity are LD1: 14%-26%, LD2: 29%-39%, LD3: 20%-26%, LD4: 8%-16% and LD5: 6%-16%. The scan indicates the fraction LD5 is increased. Elevated levels of LD5 are seen with viral hepatitis.)

180) Refer to the following illustration: Which of the following is the most likely interpretation of the LD isoenzyme scan illustrated above? a. myocardial infarction b. megaloblastic anemia c. acute pancreatitis d. viral hepatitis

d (Recognizes red cell increase secondary to stimulation of excessive erythropoietin produced in kidney.)

180) The following results were obtained on a 55-year-old man complaining of headaches and blurred vision: These results are consistent with: a. neutrophilic leukemoid reaction b. polycythemia vera c. chronic myelocytic leukemia d. leukoerythroblastosis in myelofibrosis

d (Recognizes red cell increase secondary to stimulation of excessive erythropoietin produced in kidney.)

181) A patient has a tumor that concentrates erythropoietin. He is most likely to have which of the following types of polycythemia? a. polycythemia vera b. polycythemia, secondary to hypoxia c. benign familial polycythemia d. polycythemia associated with renal disease

d (C1 x V1 = C2 x V2. Therefore [0.25] x [x] = [100] x [0.05]. Solve for x.)

181) How many mL of 0.25 N NaOH are needed to make 100 mL of a 0.05% solution of NaOH? a. 5 mL b. 10 mL c. 15 mL d. 20 mL

b (The LD1 and LD2 fractions are increased in hemolytic anemia due to the intramedullary hemolysis. LD5 is increased with hepatic damage. LD3 may be increased with acute pancreatitis. The LD isoenzyme pattern in renal disease is very similar to a normal pattern except for the higher absolute values.)

181) Increased serum lactic dehydrogenase activity due to elevation of fast fraction (1 and 2) on electrophoretic separation is caused by: a. nephrotic syndrome b. hemolytic anemia c. pancreatitis d. hepatic damage

a (The principle of occult blood testing is based on the oxidation of guaiac. This occurs in the presence of hydrogen peroxide [the reagent] and the enzyme peroxidase. Hemoglobin has a pseudoperoxidse activity that drives the reaction, making oxidized guaiac, which is blue in color.)

181) The chromogen for the fecal occult blood test is: a. gum guaiac b. NADH c. o-toluidine d. p-aminocinnamaldehyde

c (Bone marrow is considered the most sensitive specimen for the recovery of Brucella.)

181) Which of the following specimens is considered to be the most sensitive for the recovery of Brucella in cases of chronic infection? a. blood b. urine c. bone marrow d. lymph node

a (A small amount of fluid fills the cavity between the cavity wall [the parietal membrane] and the organ [visceral membrane]. An increase in the fluid can be due to infection, inflammation, cancer, and defects in hydrostatic and colloidal pressure. An accumulation of fluid is called an effusion.)

182) A buildup of fluid in a body cavity is called: a. an effusion b. a transudate c. an exudate d. metastasis

a (Elevation of the levels of CK and AST is seen in muscle damage due to the crush injury to the thigh. AST levels can increase up to 4-8 times the upper limit of normal. Cerebrovascular accident and pulmonary infarction have increased CK-BB levels. In acute hepatitis, the AST level may be 100 times the upper limit of normal.)

182) A serum sample drawn in the emergency room from a 42-year-old man yielded the following laboratory results: Which of the following conditions might account for these values? a. crush injury to the thigh b. cerebrovascular accident c. pulmonary infarction d. early acute hepatitis

d (Unexpected reactivity with reverse cells should include a test with screen cells at immediate spin to determine if alloantibodies are present. Resolution of the ABO discrepancy can be performed with group B cells that lack the corresponding antigen for the identified alloantibody.)

182) The following results were obtained on a patient's blood sample during routine ABO and Rh testing: Select the course of action to resolve this problem: a. draw a new blood sample from the patient and repeat all test procedures b. test the patient's serum with A2 cells and the patient's red cells with anti-Al lectin c. repeat the ABO antigen grouping using 3x washed saline-suspended cells d. perform antibody screening procedure at immediate spin using group 0 cells

b (Although monoclonal anti-D react with most D+ red blood cells, cells with fewer antigen sites requires testing after the antiglobulin test. The test is referred to as a test for weak D.)

184) The test for weak D is performed by incubating patients red cells with: a. several different dilutions of anti-D serum b. anti-D serum followed by washing and antiglobulin serum c. anti-Du serum d. antiglobulin serum

b (Chronic hepatitis is a chronic inflammation of the hepatocytes that persists for at least 6 months. The serum enzyme levels may be variable depending on the condition. ALT, AST and ALP may be increased by 2 times the upper limit of normal. GGT is slightly increased.)

184) This is most consistent with: a. acute hepatitis b. chronic hepatitis c. obstructive jaundice d. liver hemangioma

b (Pasteurella multocida does not grow on MacConkey agar, and is associated with wounds resulting from dog and cat bites. Vibrio cholerae is motile and Pseudomonas and Aeromonas grow on MacConkey.)

185) A gram negative bacillus with bipolar staining was isolated from a wound infection caused by a bite from a pet cat. The following characteristic reactions were seen: oxidase - positive glucose OF - fermentative catalase - positive motility - negative MacConkey agar - no growth Which of the following is the most likely organism? a. Pseudomonas aeruginosa b. Pasteurella multocida c. Aeromonas hydrophila d. Vibrio cholerae

c (Hemosiderin is an indication that RBCs have been processed and degraded. The presence of hemosiderin, then, indicates bleeding, which limits the answers to c and d. The degradation of RBCs to hemosiderin takes time, so that a traumatic tap would not show hemosiderin, but a previous hemorrhage would.)

195) The finding of hemosiderin laden macrophage in a CSF sample indicates: a. bacterial infection b. viral infection c. previous hemorrhage d. traumatic tap

d (Smear findings demonstrate drug impact: megaloblastic changes due to interference with DNA synthesis [oval macrocytes] as well as other toxic nuclear effects ([Howell-Jolly, hypersegmentation].)

185) A patient has been treated for polycythemia vera for several yrs. His blood smear now shows: Oval macrocytes Howell-Jolly bodies Hypersegmented neutrophil Large, agranular platelets the most probable cause of this blood picture is: a. iron deficiency b. alcoholism c. dietary B12 deficiency d. chemotherapy

a (Amniotic fluid can be collected to test for birth defects such as trisomy 21 [Down syndrome], spina bifida [increased AFP and acetylcholinesterase], hemolytic disease of the newborn [bilirubin], and fetal lung maturity [FLM]. FLM can be determined by increased lamellar body production in the amniotic fluid. Lamellar bodies are phospholipids produced by the maturing pneumocytes. They are approximately the size of small platelets, and can be counted by instruments that are used to count platelets.)

185) Amniotic fluid is tested for the concentration of lamellar bodies. This test determines: a. fetal lung maturity (FLM) b. HDFN c. alpha-fetoprotein (AFP) d. trisomy 21

b (The heat activation method of ALP isoenzyme separation involves heating an aliquot of the serum sample at 56 C for 10 minutes. An untreated aliquot of the sample along with the heated one are assayed for ALP activity.)

185) What specimen preparation is commonly used to perform the alkaline phosphatase isoenzyme determinations? a. serum is divided into 2 aliquots, one is frozen and the other is refrigerated b. serum is divided into 2 aliquots, one is heated at 56 C and the other is unheated c. no preparation is necessary since the assay used EDTA plasma d. protein-free filtrate is prepared first

c (Accuracy is the comparison of a result with the true value.)

185) Which of the following parameters of a diagnostic test will vary with the prevalence of a given disease in a population? a. precision b. sensitivity c. accuracy d. specificity

b (Amniocentesis is an invasive procedure, and should not be used as a screen. In women with a high MSAFP, amniocentesis is used to collect fluid to detect levels of AFP and acetylcholinesterase. High levels are predictive of neural tube disorders, such as spina bifida and anencephaly. Neural tube disorders are linked to a low folic acid level in the mother in early pregnancy. c is a distractor in that, while Rh- women may have an Rh+ fetus with hemolytic disease, Rh+ women do not have that complication.)

186) Amniocentesis should be performed to: a. screen for Down syndrome b. to confirm a high maternal serum alpha-fetoprotein c. to test bilirubin levels for an Rh pos. mother d. test folic acid levels in fetal blood

d (Differentiates morphologic features of IM.)

186) In infectious mono, lymphocytes tend to be: a. small with little cytoplasm b. normal c. decreased in number d. enlarged and indented by surrounding structures

b (The use of manufacturer inserts is not acceptable in place of a procedure manual.)

186) Package inserts may be used: a. instead of a typed procedure b. as a reference in a procedure c. at the bench but not in the procedure manual d. if initialed and dated by the lab director

d (The Regan isoenzyme is an abnormal ALP isoenzyme. The carcinoplacental ALP has properties similar to the placental enzyme, in that it is also heat stable [65 C, 30 min]. It has been detected in lung, breast, ovarian and colon cancer.)

186) Regan isoenzyme has the same properties as alkaline phosphatase that originates in the: a. skeleton b. kidney c. intestine d. placenta

d (Some subgroups of A are only recognized because of their lack of anti-A in the reverse typing. Often, the donors are confirmed as subgroups of A by an adsorption-elution technique.)

186) The following results were obtained when testing a sample from a 20 year old first time blood donor Forward: anti-A = 0 | anti-B = 0 Reverse: A1 cells = 0 | B cells = 3+ What is the most likely cause of this ABO discrepancy? a. loss of antigen due to disease b. acquired B C. phenotype Oh (Bombay) d. weak subgroup of A

c (The mom does not have the D gene. The father would have to have inherited one gene that produces D and another gene that does not produce D. The mom and dad both passed on genes that do not produce D.)

187) A mother is Rh neg. and the father is Rh pos. their baby is Rh neg. It may be concluded that: a. the father is homozygous for D b. the mother is heterozygous for D c. the father is heterozygous for D d. at least 1 of the 3 Rh typings must be incorrect

d (Cystic fibrosis is a caused by a defective ion channel, which causes an accumulation of chloride in the sweat.)

187) A sweat chloride >60 mEq/L (60 mmol/L) is indicative of: a. multiple sclerosis b. muscular dystrophy c. respiratory distress syndrome d. cystic fibrosis

d (Discriminates between reactive cell line and more homogenous malignant cell line.)

187) In comparison to malignant lymphoma cells, reactive lymphs: a. have a denser nuclear chromatin b. are known to be T cells c. have more cytoplasm and more mitochondria d. are morphologically more variable throughout the smear

b (The major serum ALP isoenzymes are located in the liver, bone, intestine and placenta. Placenta ALP is most heat stable followed by the intestinal liver and bone fractions in decreasing order of stability.)

187) The most heat labile fraction of alkaline phosphatase is obtained from: a. liver b. bone c. intestine d. placenta

c (Enterobacteriaceae, such as E coli, Serratia and Enterobacter, are oxidase negative. The only selection that is oxidase positive is Aeromonas. It is associated with wounds contaminated with water.)

187) While swimming in a lake near his home, a young boy cut his foot, and an infection developed. The culture grew a nonfastidious gram-negative, oxidase positive, beta-hemolytic, motile bacilli that produced deoxyribonuclease. The most likely identification is: a. Enterobacter cloacae b. Serratia marcescens c. Aeromonas hydrophila d. Escherichia coli

a (Some blood group antibodies, in the presence of their corresponding antigen and complement, activate the complement cascade and demonstrate in-vitro hemolysis.)

188) Some blood group antibodies characteristically hemolyze appropriate red cells in the presence of: a. complement b. anticoagulants c. preservatives d. penicillin

a (Chido antibodies are considered clinically insignificant)

213) Transfusion of Ch+ (Chido-positive) red cells to a patient with anti-Ch has been reported to cause: a. no clinically significant red cell destruction b. clinically significant immune red cell destruction c. decreased 51Cr red cell survivals d. febrile transfusion reactions

a (All of the answers are mutations. Delta-F508 is the deletion of phenylalanine at position 508 of the CFTR protein. Trisomy 21 is a third copy of chromosome 21, associated with Down syndrome. The Philadelphia chromosome is a gene translocation associated with acute myelocytic leukemia. Fragile X is a form of mental retardation caused by an increased number of nucleotide repeats.)

188) The most common genetic defect associated with cystic fibrosis is called: a. delta-F508 b. trisomy 21 c. Philadelphia chromosome d. fragile X

d (Isoenzymes are multiple forms of an enzyme that possess the ability to catalyze a reaction, but differ in structure. For enzymes located in many tissue sites, an increased total enzyme activity cannot be associated with a specific clinical disorder. However, since the isoenzyme fractions are located in various tissue sources, measurement of the different fractions are considered a more specific indicator of various disorders than total levels.)

189) Isoenzyme assays are performed to improve: a. precision b. accuracy c. sensitivity d. specificity

a (Campylobacter coli/jejuni require a microaerophilic atmosphere for optimal recovery. The use of selective media is recommended for recovery from fecal specimens. Selective media for Campylobacter contains antibiotics to inhibit the growth of enteric gram-negative flora. Unlike other enteric pathogens, C coli/jejuni grow well at 42 C.)

19) In order to isolate Campylobacter coli/jejuni the fecal specimen should be: a. inoculated onto selective plating media and incubated in reduced oxygen with added CO2 at 42 C b. stored in tryptic soy broth before plating to ensure growth of the organism c. inoculated onto selective plating media and incubated at both 35 C and at room temp d. incubated at 35 C for 2 hrs. in Cary-Blair media before inoculating onto selective plating media

d (SYBR Green is the most sensitive of the listed dyes.)

19) The fluorescent dye that exhibits the greatest sensitivity for quantization of DNA is: a. ethiduim bromide b. Hoechst 33258 c. propidium iodide d. SYBR green

b (The 2 pigments are porphyrin and urochrome. Urochrome is "urine color," which is yellow. Porphyrin is red.)

19) Urine from a 50 yr. old man was noted to turn dark red on standing. This change is caused by: a. glucose b. porphyrins c. urochrome d. creatinine

d (Hgb A1C structure.)

19) Which of the following hemoglobins has glucose-6-phosphate on the amino-terminal valine of the beta chain? a. S b. C c. A2 d. A1c

b (Fresh Frozen Plasma is stored at -18 C or below for 12 months.)

19) Which of the following is the correct storage temp for the component listed: a. Cryo AHF = 4 C b. FFP = -20 C c. Frozen RBC = -40 C d. Platelets = 37 C

b (C1 esterase inhibitor is associated with hereditary angioedema.)

190) Hereditary angioedema is characterized by: a. decreased activity of C3 b. decreased activity of C1 esterase inhibitor c. increased activity of C1 esterase inhibitor d. increased activity of C2

d (Media for isolation of Legionella should be incubated at 35 - 37 C for at least 7 days.)

190) Serum samples collected from a patient with pneumonia demonstrate a rising antibody titer to Legionella. A bronchoalveolar lavage (BAL) specimen from this patient had a positive antigen test for legionella but no organisms were recovered on buffered charcoal yeast extract medium after 2 days of incubation. The best explanation is that the: a. antibody titer represents an earlier infection b. positive antigen test is a false positive c. specimen was cultured on the wrong media d. culture was not incubated long enough

c (Safety guidelines indicate that before attempting to solve any trouble shooting steps for any equipment it is necessary to turn the power off for the instrument. This step is necessary to prevent any possible electric shock.)

190) The first step to be taken when attempting to repair a piece of electrical equipment is: a. check all the electronic connections b. reset all the printed circuit boards c. turn the instrument off d. replace all the fuses

b (Presence of agglutination at AHG phase with both screening cells and autocontrol is indicative of warm autoantibody.)

190) The following results were obtained in pre-transfusion testing: screen cell 1: @37 C = 0 | @IAT = 3+ screen cell 2: @37 C = 0 | @IAT = 3+ autocontrol: @37 C = 0 | @IAT = 3+ The most probable cause of these results are: a. rouleaux b. a warm autoantibody c. a cold autoantibody d. multiple alloantibodies

a (The holoenzyme is the active system formed by a protein portion called the apoenzyme and a cofactor which can be an activator if inorganic and a coenzyme if organic.)

190) The protein portion of an enzyme complex is called the: a. apoenzyme b. coenzyme c. holoenzyme d. proenzyme

d (A traumatic tap is a collection of a body fluid that has blood contamination. The first tube will have most of the contamination, so that subsequent tubes will show fewer blood cells. Cell counts should routinely be performed on the last tube, to minimize the amount of cellular contamination due to the tap.)

191) A CSF was collected from a 5 yr. old with a fever and 3 tubes were transported to the lab. Tube 1 had 50,000 RBC/mL and 48 WBC/mL. Tube 3 had 10 RBC/mL and 0 WBC/mL. What is the most likely explanation for the discrepancy? a. tube 3 was QNS b. bacterial meningitis c. subarachnoid hemorrhage d. traumatic tap

a (Presence of agglutination at AHG phase with screening cells and 2 out of 6 donor units indicates antibody in patient serum to antigen[s] on screening cells and donor cells. The presence of an autoantibody would most likely react with all cells, including the autologous control or DAT.)

191) A patient is typed as group O, Rh pos. and crossmatched with 6 units of blood. At the IAT phase of testing, both antibody screening cells and 2 crossmatched units are incompatible. What is the most likely cause of the incompatibility? a. recipient alloantibody b. recipient autoantibody c. donors have pos. DAT's d. rouleaux

b (Definition of atomic absorption.)

191) The methodology based on the amount of energy absorbed by a substance as a function of its concentration and using a specific source of the same material as the substance analyzed is: a. flame emission photometry b. atomic absorption spectrophotometry c. emission spectrography d. x-ray fluorescence spectrometry

a (Both Acinetobacter and Moraxella display resistance to penicillin, and some species grow on MacConkey agar. Acinetobacter are oxidase negative, and Moraxella are oxidase positive.)

191) Which characteristic best differentiates Acinetobacter species from Moraxella species? a. production of indophenol oxidase b. growth on MacConkey agar c. motility d. susceptibility to penicillin

c (Approximately 20%-30% of the seminal fluid is prostatic fluid. The composition of the prostatic fluid is acid phosphatase, citric acid, and proteolytic enzymes. The activity of prostatic acid phosphatase may be measured in seminal fluid for medicolegal cases involving rape.)

191) Which of the following chemical determinations may be of help in establishing the presence of seminal fluid? a. LD b. ICD (isocitrate dehydrogenase) c. acid phosphatase d. alkaline phosphatase

d (C3 deficiency is the worst deficiency to have; it causes a lifelong history of life-threatening infections.)

191) Which of the following has been associated with patients who have homozygous C3 deficiency? a. undetectable hemolytic complement activity in the serum b. SLE c. no detectable disease d. a lifelong history of life-threatening infections

c (Characteristics of ALL; onset highest 1-5 years of age with peak at 2-3 years of age; presents with lethargy, fever, bone pain, with poor prognosis under 1 year of age.)

191) Which of the following is true of acute lymphoblastic leukemia? a. occurs most commonly in children 1 -2 yrs. old b. patient is asymptomatic c. massive accumulation of primitive lymphoid-appearing cells in bone marrow occurs d. children under 1 yr. of age have a good prognosis

a (Recognizes ALL as possible progression for 1/3 of CML cases.)

192) A 50-year-old woman who has been receiving busulfan for three years for chronic myelogenous leukemia becomes anemic. Laboratory tests reveal: - Thrombocytopenia - Many peroxidase-negative blast cells in the peripheral blood - Bone marrow hypercellular in blast transformation - Markedly increased bone marrow TdT Which of the following complications is this patient most likely to have? a. acute lymphocytic leukemia b. acute myelocytic leukemia c. acute myelomonocytic leukemia d. busulfan toxicity

a (Growth at 42 C and pyocyanin production are classic tests for the identification of Pseudomonas aeruginosa. Gelatin hydrolysis separates Pseudomonas putida [negative] from Pseudomonas fluorescence [positive].)

192) An organism has been identified as a member of the fluorescent group of Pseudomonas. Which of the following sets of tests should be used to determine the species of the organism? a. growth at 42 C, pyocyanin production, gelatinase production b. pyocyanin production, gelatinase production, OF glucose c. growth at 37 C, pyocyanin production, OF glucose d. gelatinase production, growth at 52 C, H2S

c (C1, C4, C2 deficiencies are associated with a lupus erythematosus-like syndrome.)

192) Hereditary deficiency of early complement components is associated with: a. pneumococcal septicemia b. small bowel obstruction c. lupus erythematosus like syndrome d. gonococcemia

b (Reaction with anti-IgG in the DAT and with both screening cells and autocontrol at the AHG phase is indicative of a warm autoantibody.)

192) Refer to the following data: - hemoglobin: 7.4 g/dL (74 g/L) - reticulocyte count: 22% Which clinical condition is consistent with the lab results shown above? a. cold hemagglutinin disease b. warm autoimmune hemolytic anemia c. penicillin-induced hemolytic anemia d. delayed hemolytic transfusion reaction

a (Francisella tularensis is the causative agent of tularemia. It has a specific growth requirement for cysteine.)

193) Appropriate culture requirements for a specimen from a patient suspected of having tularemia include: a. a media with cysteine such as BYCA b. colistin nalidixic acid agar c. Mueller-Hinton agar with 5% sheep blood agar d. Regan-Lowe media

d (C5, C6, C7, C8 deficiencies associated with a worse Neisseria infection if the patient is exposed due to less pathogen clearance.)

193) Hereditary deficiency of late complement components can be associated with which of the following conditions? a. pneumococcal septicemia b. small bowel obstruction c. SLE d. a systemic gonococcal infection if exposed

d (Visual wavelength calibration.)

193) One means of checking a spectrophotometer wavelength calibration is the visible range is by using a: a. quartz filter b. diffraction grating c. quartz prism d. didymium filter

b (CSF looks like water. If it is yellow or pink it is called xanthochromic. This indicates a previous bleed into the CNS. Opalescence is a haziness, usually due to the presence of lipids.)

193) The appearance of normal CSF is: a. pale yellow and clear b. colorless and clear c. opalescent d. xanthochromic

a (Recognizes ALL as most common malignancy of children.)

193) The most common form of childhood leukemia: a. acute lymphocytic b. acute granulocytic c. acute monocytic d. chronic granulocytic

c (C2 deficiency is associated with a lupus like syndrome.)

194) For several months a 31-year-old woman has had migratory polyarthritis and a skin rash. Upon admission to the hospital, the following laboratory data were obtained: This patient's test results are consistent with: a. dermatomyositis b. C1INH deficiency c. systemic lupus erythematosus d. mixed connective tissue disease

b (Spectrophotometer wavelength.)

194) In spectrophotometry, the device that allows for a narrow band of wavelengths is the: a. hollow cathode lamp b. monchromator c. refractometer d. photodetector

c (Morphology of a myelocyte.)

235) The large cell indicated by the arrow in image below is a: a. myeloblast b. promyelocyte c. myelocyte d. metamyelocyte

d (The major crossmatch tests the recipient's plasma with donor's cells. This would detect any antibody in the recipient that would react with antigens on the donor's RBCs. If a patient were mistyped as a group O rather than group A, then group O cells would be selected for crossmatch and no incompatibility would be found.)

195) The major crossmatch will detect a (an): a. group A patient mistyped as a group O b. unexpected red cell antibody in the donor unit c. Rh-neg donor unit mislabeled as Rh-pos d. recipient antibody directed against antigens on the donor red cells

a (The patient has a negative antibody screen, but one unit is found to be incompatible. The antibody is most likely directed towards a low-incidence antigen.)

196) A 42-year-old female is undergoing surgery tomorrow and her physician requests that 4 units of Red Blood Cells be crossmatched. The following results were obtained: What is the most likely cause of the incompatibility of donor 1? a. single alloantibody b. multiple alloantibodies c. Rh incompatibilities d. donor 1 has a positive DAT

d (Buffered charcoal yeast extract agar is recommended for culture of specimens for Legionella.)

196) A laboratory aid receives a bronchoscopy sample with the request to culture for Legionella. The assistant asks a microbiologist for direction of plating protocol. The correct response from the microbiologist is: a. culture on thiosulfate citrate bile salt media b. incubate the culture media anaerobically c. reject the specimen and request a sputum sample d. culture on buffered charcoal yeast extract agar with antibiotics

a (Normal CSF glucose is approximately 60% of plasma glucose. Glucose is decreased in bacterial meningitis, and lactate is produced. Normal protein in CSF is 15-45 mg/dL. Protein is slightly elevated in bacterial meningitis.)

196) Which CSF results are most consistent with bacterial meningitis? a. sample A b. sample B c. sample C d. sample D

d (Characteristics features of hairy cell leukemia include all answers listed as well as decreased resistance to infection.)

196) Which of the following is not a characteristic usually associated with hairy cell leukemia? a. pancytopenia b. mononuclear cells with ruffled edges c. splenomegaly d. increased resistance to infection

d (Thiosulfate citrate bile salt agar is a selective media for Vibrio, and it also differentiates sucrose-fermenting species, such as V cholerae and V alginolyticus)

197) A community hospital microbiology lab is processing significant numbers of stool cultures because of an outbreak of diarrhea following heavy rains and flooding in the country. A media that should be incorporated in the plating protocol is: a. colistin nalidixic acid for Listeria b. MacConkey agar with sorbitol for Campylobacter c. mannitol salt agar for Enterococcus species d. thiosulfate citrate bile salts sucrose for Vibrio species

d (Recognition that Gaucher cell is from lipid storage disease, not plasma cell variant.)

197) Morphologic variants of plasma cells do not include: a. flame cells b. morula cells c. grape cells d. Gaucher Cells

a (In hepatic encephalopathy, ammonia levels in the plasma build up, and ammonia can be found in the spinal fluid. Ammonia is volatile, and not easily measured. Glutamine is a byproduct of ammonia, and is stable, making it a better choice.)

197) Which of the following is the best indicator of Reye syndrome for CSF (hepatic encephalopathy): a. glutamine b. ammonia c. ALT d. bilirubin

d (Since crossmatching is a test between the patient's plasma and donor's cells, any incompatibility is due to the donor's red cells. If a patient is negative for clinically significant antibodies to common antigens, an incompatible unit by the antiglobulin test is due to either a positive DAT on the donors red cells or the patient has an antibody to a low-incidence antigen that the donor's cells possess.)

197) Which of the following would most likely be responsible for an incompatible antiglobulin crossmatch? a recipients red cells possess a low frequency antigen b. anti-K antibody in donor serum c. recipients red cells are polyagglutinable d. donor red cells have a positive DAT

b (If a patient is negative for clinically significant antibodies, and a single crossmatch is incompatible, the incompatibility is either due to donor cells with a positive DAT or the patient has an antibody to a low-incidence antigen that the donor's cells possess.)

198) A reason why a patient's crossmatch may be incompatible while the antibody screen is negative is: a. the patient has an antibody against a high incidence antigen b. the incompatible donor unit has a positive DAT c. cold agglutinins are interfering in the crossmatch d. the patient's serum contains warm autoantibody

a (Immediate hypersensitivity is involved in bronchial asthma.)

198) Which immunologic mechanism is usually involved in bronchial asthma? a. immediate hypersensitivity b. antibody mediated cytotoxicity c. immune complex d. delayed hypersensitivity

c (It a patient is negative tor clinically significant antibodies, and a single crossmatch is incompatible, the incompatibility is either due to donor cells with a positive DAT or the patient has an antibody to a low-incidence antigen that the donor's cells possess.)

199) A blood specimen types as A, Rh-pos with a negative antibody screen. 6 units of group A, Rh-pos RBCs were crossmatched and 1 unit was incompatible in the antiglobulin phase. The same result was obtained when the test was repeated. Which should be done first? a. repeat the ABO grouping on the incompatible unit using a more sensitive technique b. test a panel of red cells that possesses low incidence antigens c. perform a DAT on the donor unit d. obtain a new specimen and repeat the crossmatch

d (Antihistamines block histamine binding to histamine receptors.)

199) Antihistamines like Benadryl: a. depress IgE production b. block antigen binding to surface IgE c. bind histamine d. block H1 histamine receptors

d (Both organisms are gram-negative bacilli and grow on MacConkey agar. Neither ferments glucose. Stenotrophomonas is oxidase negative, while most other nonfermenters are oxidase positive.)

199) Differentiating tests that will separate Burkholderia from Stenotrophomonas include: a. Gram stain reaction b. growth on MacConkey c. glucose fermentation d. oxidase

d (To obtain the maximum absorbance of the measured chromogen.)

202) A technologist is asked to write a procedure to measure the Evan blue concentration on a spectrophotometer. The technologist is given 4 standard solutions of Evan blue: The first step is to: a. calculate the slope of the calibration curve b. determine the absorbance of the 4 standards c. find the wavelength of the greatest % transmittance for Evan blue d. find the wavelength of the greatest absorbance for Evan blue

a (Consider each answer, and determine if it is are Unique to a particular site: a PSA is used to screen for prostate cancer, because it is high in secretions from the prostate; therefore, semen is high in PSA. PSA can be used forensically to identify a fluid as semen. b Alkaline phosphatase is found predominantly in bone and liver. On the other hand, acid phosphatase, not a choice, is high in prostate secretions, and semen. c Fructose is the major carbohydrate found in semen, but it is not unique to semen. d Hyaluronic acid is associated with synovial fluid.)

199) Which marker can be used to identify a body fluid as semen? a. PSA b. alkaline phosphatase c. fructose d. hyaluronic acid

d (When 0.001 mL of urine is plated, the growth of one colony is equivalent to 1000 CPU/mL. Thus 70 colonies is 70,000 CPU/mL.)

2) A sheep blood agar plate inoculated with 0.001 mL of urine grows 70 colonies of Staphylococcus aureus. How many colony forming units per mL of urine should be reported: a. 70 b. 700 c. 7000 d. 70000

d (The G2 phase of the cell cycle follows the S phase, where DNA content goes from 2N to 4N or from diploid to tetraploid.)

2) During the G2 phase of the cell cycle, the DNA content of a cell is: a. haploid b. diploid c. triploid d. tetraploid

a (Urobilinogen is degraded by light.)

2) False results in the urobilinogen testing may occur if the urine specimen is: a. exposed to light b. adjusted to a neutral pH c. cooled to room temp d. collected in a nonsterile container

c (Body fluid glucose reference ranges.)

2) The following results are from a 21-year-old patient with a back injury who appears otherwise healthy: - whole blood glucose = 77 mg/dL (4.2 mmol/L} - serum glucose = 88 mg/dL (4.8 mmol/L) - CSF glucose = 56 mg/dL (3.1 mmol/L) The best interpretation of these results is that: a. the whole blood and serum values are expected but the CSF value is elevated b. the whole blood glucose value should be higher than the serum value c. all values are consistent with a normal healthy individual d. the serum and whole blood values should be identical

b (The Joint Commission requires 2 patient identifiers when providing care, treatment, or service.)

2) Which action by the phlebotomist will comply with the College of American Pathologists (CAP) Patient Safety Goal "to improve patient and sample identification at the time of specimen collection" and The Joint Commission Patient Safety Goal to "improve the accuracy of patient identification"? a. match the name and room number on the patient's ID bracelet to the name and room number on the preprinted collection label b. match the name and medical record number on the patient's ID bracelet to the name and medical record number on the preprinted collection label c. verify patient information by stating the patient's name when approaching the patient d. label the collection tubes prior to the blood draw at the patient's bedside

c (Ax cells react more strongly with anti-A,B than with anti-A. If anti-A is nonreactive, Ax cells may be detected with anti-A,B.)

235) The purpose of testing with anti-A,B is to detect: a. anti-A1 b. anti-A2 c. subgroups of A d. subgroups of B

d (Blood may be returned to the blood bank after issue provided that 1] the container has not been entered, 2] at least 1 sealed segment is attached to the container, 3] visual inspection of the unit is satisfactory and documented, and 4] the unit has been maintained at the appropriate storage or transport temperature. Studies have shown that refrigerated components retain an acceptable temperature of <10 C for up to 30 minutes after removal from the refrigerator.)

20) A unit of RBCs is issued at 9 am and at 9:10 am it is returned to the blood bank. the container has NOT been entered but the unit has NOT been refrigerated during this time span. the best course of action for the tech is to: a. culture the unit for bacteria b. discard the unit if not used within 24 hrs. c. store the unit at room temp d. record the return and place the unit back into inventory

d (Sputum specimen quality is assessed to determine if the specimen is representative of the site of infection. The presence of white blood cells is an indicator of infection, and presence of squamous epithelial cells is an indicator of oropharyngeal contamination. In this specimen, >25 epithelial cells per low power field is an indicator of poor specimen quality, and the bacteria present are representative of oropharyngeal flora.)

20) An expectorated sputum is sent to the laboratory for culture from a patient with respiratory distress. The Gram stain of the specimen shows many squamous epithelial cells (>25/Ipf) and neutrophils. The microscopic appearance of the organisms present include: - moderate gram-positive cocci in chains and diplococci - moderate gram-negative diplococci - moderate palisading gram-positive bacilli all in moderate amounts This Gram stain is most indicative of: a. a pneumococcal pneumonia b. an anaerobic infection c. an Haemophilus pneumonia d. oropharyngeal flora

d (National Labor Relations Act [NLRA] includes the right to form or join unions, freedom to bargain collectively with the employer and the right to engage in group activity.)

20) Employees are guaranteed the right to engage in self-organization and collection bargaining through representatives of their choice, or to refrain from these activities by which of the following? a. Civil Rights Act b. Freedom of Information Act c. Clinical Laboratory Improvements Act d. National Labor Relations Act

c (Based on the biochemicals listed the most likely identification of the organism is Neisseria gonorrhoeae. N gonorrhoeae is most commonly isolated from the genitourinary tract.)

205) The following results were obtained from a culture of unknown origin: Gram Stain: gram-negative diplococci indophenol oxidase - positive glucose - positive maltose - negative sucrose - negative The most likely source of the specimen would be the: a. respiratory tract b. blood c. genitourinary tract d. cerebrospinal fluid

b (a is false, you don't use glass tubes in urinalysis. b is true-clarity or haziness is due to solids and cells in the solution. These settle to the bottom of the tube; therefore, the sample should be well mixed. It is not necessary to add anything to view clarity. Furthermore, adding SSA will precipitate urines with positive protein. Allowing the specimen to cool to room temperature may cause amorphous crystals to form, so d is false.)

20) The clarity of a urine sample should be determined: a. using glass tubes only; never plastic b. following thorough mixing of the specimen c. after addition of sulfosalicylic acid d. after the specimen cools to room temp

a (In a rubella infection a titer of 1:8 or greater indicates protective antibodies. An acute infection would indicate a rise in titer between 2 serum samples collected 2 or more weeks apart.)

20) The following results are from a rubella titer performed on acute and convalescent sera using a 2-fold serial dilution: After evaluating the above results, the best interpretation is: a. results are consistent with active infection with rubella b. variation in the acute serum titers invalidates these results c. test should be repeated by a different technologist d. patient was not infected with rubella

c (The absorptivity constant for DNA is 50 ug/mL.)

20) When quantifying the amount of genomic DNA in a sample by spectrophotometry, an OD 260 of 1.0 corresponds to what concentration of DNA? a. 10 ug/mL b. 20 ug/mL c. 50 ug/mL d. 100 ug/mL

c (Emergency release of blood cannot use previous records. Blood typing must be performed on the current sample. In this case, group O Rh-negative is the best choice since there is evidence the patient is Rh-negative.)

200) During emergency situations when there is no time to determine ABO group and Rh type on a current sample for transfusion, the patient is known to be A, Rh-neg. the technologist should: a. refuse to release any blood until the patient's sample has been typed b. release A Rh-neg RBC's c. release O Rh-neg RBC's d. release O Rh-pos RBC's

a (All of these stains can be used to view sperm; however, the eosin-nigrosan stain is differential between live and dead sperm. Living cells will stain bluish white, dead cells stain red.)

200) Which stain is used to measure sperm viability? a. eosin nigrosin b. Wright c. toluidine blue d. Papanicolaou

b (When group specific units of Red Blood Cells are not available, group compatible units are selected. Since the patient is AB, group A would be selected to conserve group O units for group O patients. Rh-negative patients should receive Rh-negative units of red blood cells.)

201) A 29 year old male is hemorrhaging severely. He is AB, Rh-neg. 6 units of blood are required STAT. Of the following types available in the blood bank, which would be most preferable for crossmatch? a. AB, Rh-pos b. A, Rh-neg c. A, Rh-pos d. O, Rh-neg

c (IgE is in too low a concentration for precipitation; therefore it cannot be detected by this method.)

201) A child has severe hay fever. A total IgE measurement was performed by the ouchterlony immunodiffusion method. No lines of precipitation appeared on the immunodiffusion plate. The most likely explanation is: a. IgE antibodies are not produced in children who have hay fever b. hay fever is mediated by the cellular system c. IgE is too low a concentration to be detected by this method d. IgA is the antibody commonly produced in people with hay fever

a (Principle of nephelometry.)

201) Nephelometers measure light: a. scattered at a right angle to the light path b. absorbed by suspended particles c. transmitted by now-particulate mixtures d. reflected back to the source form opaque suspensions

a (Identification of condition with morphologic alteration in neutrophils, representing giant fused lysosomes.)

201) Of the following, the disease most closely associated with cytoplasmic granule fusion is: a. Chediak-Higashi syndrome b. Pelger-Huet anomaly c. May-Hegglin anomaly d. Alder-Reilly anomaly

a (The total volume of the 9 large cells is 0.9 uL. Counting 10 of the large cells is the equivalent of 1 uL.)

201) The dimensions of a hemacytometer are: a. 3 x 3 x 0.1 mm b. 1 x 1 x 10 mm c. 3 x 10 x 1 mm d. 1 x 1 x 0.3 mm

d (Lipid analysis using overnight refrigeration involves incubating the sample at 4 C overnight. The chylomicrons, present as a thick homogenous cream layer, may be observed floating at the plasma surface.)

202) A 1 yr. old girl with a hyperlipoproteinemia and lipase deficiency has the following lipid profile: cholesterol = 300 mg/dL LDL increased HDL decreased triglycerides = 200 mg/dL chylomicrons present A serum specimen from this patient that was refrigerated overnight would most likely be: a. clear b. cloudy c. creamy layer over cloudy serum d. creamy layer over clear serum

a (This patient has an anti-A1, which eliminates A1B cells immediately. Rh-negative units should be conserved for Rh-negative patients when Rh-positive units are available. Selection of group B units provides compatible units quickly.)

202) A patient is group A2B, Rh-pos and has an antiglobulin reacting anti-A1 in his serum. He is in the operating room bleeding profusely and group A2B RBCs are not available. Which of the following blood types is first choice for crossmatching? a. B, Rh-pos b. B, Rh-neg c. A1B, Rh-pos d. O, Rh-neg

c (Nephelometric methods are based upon light scatter being proportional to the number of particles in suspension, such as antigen-antibody complexes, which are physically larger than uncomplexed molecules.)

205) The measurement of light scattered by particles in the sample is the principle of: a. spectrophotometry b. fluorometry c. nephelometry d. atomic absorption

d (Brucella causes undulant fever and is a cause of fever of unknown origin. It is slow growing, and is associated with laboratory-acquired infections. It is also a potential agent of bioterrorism. Suspected Brucella isolates should not be tested in automated or manual identification systems.)

202) The lab receives a blood culture from a veterinarian who has been ill for many weeks with fevers in the afternoon and evenings, arthritis, and fatigue. The blood culture is positive after 5 days, and the organism has the following characteristics: Gram stain: small, gram-negative, coccobacilli SBA: growth after 48 hrs. with small, smooth, raised colonies. What should the microbiologist do next? a. consider the growth contamination and perform another gram stain b. perform biochemical identification for HACEK organisms c. perform identification and susceptibility testing using an automated system d. take extra safety precautions for possible Brucella

d (Description of MH anomaly.)

202) Which of the following anomalies is an autosomal dominant disorder characterized by irregularly-sized inclusions in polymorphonuclear neutrophils, abnormal giant platelets and often thrombocytopenia? a. Pelger-Huet b. Chediak-Higashi c. Alder-Reilly d. May-Hegglin

a (Skin tests are used to determine whether the delayed type hypersensitivity response mediated by T cells is functioning properly. Another tests listed evaluate humoral antibody responses.)

202) Which test is used to evaluate the cellular immune system in a patient? a. skin test for commonly encountered antigens b. determination of isohemoagglutinin titer c. immunoelectrophoresis of serum d. measurement of anti-HbsAg after immunization

d (The strength of agglutination is dependent upon optimal antigen to antibody ratio. Excessive amount of antigen does not allow maximal uptake of antibody per red cell and therefore agglutination is negatively affected leading to weaker or negative results.)

203) A 10% red cell suspension in saline is used in a compatibility test. Which of the following would most likely occur? a. a false-positive result due to antigen excess b. a false-positive result due to the prozone phenomenon c. a false-negative result due to the prozone phenomenon d. a false-negative result due to antigen excess

b (Morphologic alteration of neutrophils; majority of nuclei are bilobed and rounded.)

203) Of the following, the disease most closely associated with granulocyte hyposegmentation is: a. May-Hegglin b. Pelger-Huet c. Chediak-Higashi d. Gaucher disease

b (While T cells help B cells in the process of antibody production, they are not capable of secreting immunoglobulins themselves. Immunoglobulins are produced only by B cells and plasma cells.)

203) T cells are incapable of: a. collaborating with B cells in antibody responses b. secretion of immunoglobulins c. secretion of cytokines d. producing positive skin tests

c (Neisseria gonorrhoeae requires an enhanced CO2 atmosphere for optimal growth.)

203) The primary isolation of Neisseria gonorrhoeae requires: a. anaerobic conditions b. starch media c. carbon dioxide d. blood agar

c (Wavelength accuracy is an essential performance parameter to be tested for the spectrophotometer. For the narrow- spectral bandwidth instruments, holmium oxide glass may be scanned over the range of 280-650 nm. The material shows very sharp absorbance peaks at the well-defined wavelengths. Prism and diffraction gratings are monochromatic devices. The didymium filter is used with broader bandwidth instruments.)

203) Which of the following is used to verify wavelength settings for narrow bandwidth spectrophometers? a. didymium filters b. prisms c. holmium oxide glass d. diffraction gratings

b (Of the possible types of meningitis listed, only Neisseria meningitidis is oxidase positive.)

204) Assuming the agent isolated from a patient's spinal fluid produces a positive oxidase test, the most likely diagnosis is: a. tuberculosis meningitis b. meningococcal meningitis c. viral meningitis d. pneumococcal meningitis

a (Sweat chloride is measured to diagnose cystic fibrosis. Sweating is induced by stimulating local sweat glands by driving pilocarpine into the skin, using a process called pilocarpine iontophoresis.)

204) Pilocarpine iontophoresis refers to the specific process of: a. inducing sweat b. separating proteins in CSF c. measuring ions in sweat d. measuring pilocarpine in CSF

c (Phagocytosis is mediated by macrophages and neutrophils, not by lymphocytes.)

204) T lymphs are incapable of functioning as: a. cytotoxic cells b. helper cells c. phagocytic cells d. regulatory cells

b (Morphologic alteration of neutrophils; majority of nuclei are bilobed, rounded and dumbbell-shaped.)

204) Which of the following cell types is characteristic of Pelger-Huet anomaly: a. band form b. pince-nez form c. normal neutrophil d. myelocyte

a (Fluorescence occurs when a molecule absorbs light at one wavelength and emits light of a longer wavelength. The detector in a fluorometer is usually at right angles to the incident light source. A xenon or mercury vapor lamp emits enough UV light to be useful as a light source in a fluorometer.)

204) Which of the following statements about fluorometry is true? a. a compound fluoresces when it absorbs light at one wavelength and emits light at a second wavelength b. the detector in a fluorometer is positioned at 180 degrees from the excitation source c. fluorometry is less sensitive that spectrophotometry d. an incandescent lamp is commonly used in a fluorometer

a (Determining the patients phenotype allows focusing identification procedures toward antibodies the patient can develop.)

205) A patient received 4 units of blood 2 years previously and now has multiple antibodies. He has not been transfused since that time. It would be most helpful to: a. phenotype his cells to determine which additional alloantibodies may be produced b. recommend the use of directed donors, which are more likely to be compatible c. use proteolytic enzymes to destroy the "in vitro" activity of some of the antibodies d. freeze the patient's serum to use for antigen typing of compatible units

c (Natural killer cells do not specifically bind to tumor antigens, as do T lymphocytes and antibodies, and can kill tumor cells without having had prior exposure to them.)

205) Nonspecific killing of tumor cells is carried out by: a. cytotoxic T cells b. helper T cells c. natural killer cells d. antibody and complement

a (Giant granulocyte inclusions due to fusion of lysosomes.)

205) Which of the following is associated with Chediak-Higashi syndrome? a. membrane defect of lysosomes b. Dohle bodies and giant platelets c. two-lobed neutrophils d. mucopolysaccharidosis

d (Neisseria gonorrhoeae requires an enhanced carbon dioxide atmosphere for optimal growth.)

206) An urethral swab obtained from a man with a urethral exudate was plated directly on chocolate agar and modified Thayer-Martin agar. A Gram stain was made. The gram stain showed gram-negative diplococci. The culture plates were incubated at 35 C, but had no growth at 48 hrs. The most likely failure for organism growth is that the: a. wrong media were used b. gram stain was misread c. organism only grows at room temp d. organism requires CO2 for growth

b (Light from the lamp or light source of a photometer is reduced to a specific wavelength by the monochromator. In spectrophotometers, a diffraction-grate is used as a monochromator.)

206) In a spectrophotometer, light of a specific wavelength is isolated from the light source by the: a. double beam b. monochromator c. aperture d. slit

c (Sweat conductivity testing is relatively easy to perform, and used as a screening method. A positive should be confirmed with a chloride measurement by coulometry or by ion-selective electrode. Pilocarpine iontophoresis is used to stimulate sweat production, not to measure it.)

206) Methods used as screening tests for cystic fibrosis include: a. coulometric Cl measurement b. Cl selective electrodes c. sweat conductivity d. pilocarpine iontophorisis

a (Immunoassays that employ monoclonal antibody reagents are commonly used to quantify circulating tumor antigens because they are highly sensitive and specific.)

206) Tumor markers found in the circulation are most frequently measured by: a. immunoassays b. thin-layer chromatography c. high-pressure liquid chromatography d. colorimetry

d (Morphologic alteration of neutrophils- differentiated from Chediak-Higashi; May-Hegglin and Pelger-Huet anomalies.)

206) Which of the following is associated with Alder-Reilly inclusions? a. membrane defect of lysosomes b. Dohle bodies and giant platelets c. chronic myelogenous leukemia d. mucopolysaccharidosis

b (The Neisseria species listed all ferment several carbohydrates, Moraxella catarrhalis is biochemically inert and does not ferment carbohydrates.)

207) A gram stain performed on a sputum specimen revealed gram-neg diplococci within PMN's. Oxidase testing is positive and carbohydrate degradation tests are inert. The organism is: a. Neisseria lactamica b. Moraxella catarrhalis c. Neisseria meningitidis d. Neisseria sicca

c (Mass spectrometers detect mass to charge ratios of ionized molecules.)

207) A mass spectrometer detects which property of ionized molecules? a. column retention time b. charge to mass ratio c. mass to charge ratio d. fluorescence

b (Antibodies are important in defense against bacterial infections. Patients with B cell deficiencies are unable to produce adequate amounts of antibodies, and therefore exhibit increased bacterial infections. Also, B cells are not phagocytic and do not produce complement.)

207) A patient with a B-cell deficiency will most likely exhibit: a. decreased phagocytosis b. increased bacterial infections c. decreased complement levels d. increased complement levels

a (Lewis antibodies may bind complement and fresh serum that contains anti-Lea may hemolyze Le[a+] red cells in vitro. Approximately 22% of the population is Le[a+].)

207) An antibody that causes in-vitro hemolysis and reacts with the red cells of 3 out of 10 crossmatched donor units is most likely: a. anti-Lea b. anti-s c. anti-k d. anti-E

b (Morphologic alteration of neutrophils- differentiated from Chediak-Higashi, CML and Alder-Reilly anomalies)

207) Which of the following is associated with May-Hegglin anomaly? a. membrane defect of lysosomes b. Dohle bodies and giant platelets c. chronic myelogenous leukemia d. mucopolysaccharides

c (Dark green amniotic fluid indicates the presence of meconium. This is the infant's first bowel movement, and indicates fetal distress.)

207) Which pair does not match with respect to amniotic fluid? a. colorless --- normal b. dark red/brown --- fetal death c. dark green --- HDFN d. blood-streaked --- traumatic tap

a (Morphologic alteration of neutrophils- primary feature is hyposegmentation.)

208) A differential was performed on an asymptomatic patient. The differential included 60% neutrophils, 55 of which had 2 lobes and 5 had 3 lobes. There were no other abnormalities. This is consistent with which of the following anomalies? a. Pelger-Huet b. May-Hegglin c. Alder-Reilly d. Chediak-Higashi

a (A fetus with hemolytic disease will have increased bilirubin in the amniotic fluid. Bilirubin absorbs light at 450 nm. The change in absorbance between the expected and observed value is plotted on a Liley graph, and used to assess the fetus. This is also called a Delta-OD 450.)

208) Amniotic fluid is evaluated using a Liley graph and change in absorbance at 450 nm. What is being evaluated and why? a. bilirubin, which increases in HDN b. AFP, which increases in spina bifida c. SCG, which increases in Down syndrome d. lamellar bodies, which increase with fetal lung maturity

c (Lipoprotein lipase hydrolyzes triglycerides and chylomicrons during normal lipid metabolism. A deficiency in lipoprotein lipase results in markedly increased serum chylomicrons and triglycerides.)

208) Chylomicrons are present in which of the following dyslipidemias? a. familial hypercholesterolemia b. hypertriglyceridemia c. deficiency in lipoprotein lipase activity d. familial hypoalphalipoproteinemia

b (One of the less commonly used methods for identification of Neisseria gonorrhoeae grown in culture is coagglutination.)

208) Coagglutination is associated with: a. Chlamydia trachomatis b. Neisseria gonorrhoeae c. Streptococcus pneumoniae d. Klebsiella pneumonia

a (Penicillin resistance in Streptococcus pyogenes and S agalactiae have not been described. Neisseria meningitidis is rarely resistant to penicillin, which can be beta-lactamase mediated in some isolates. The primary mechanism of penicillin resistance in N gonorrhoeae is beta-lactamase.)

209) Clinical resistance to penicillin correlates most frequently with beta-lactamase production in: a. Neisseria gonorrhoeae b. Neisseria meningitidis c. Streptococcus agalactiae d. Streptococcus pyogenes

b (Identification of cellular structure where accumulation of partially degraded mucopolysaccharides occurs.)

209) The cytoplasmic abnormality of the white blood cell of Alder-Reilly anomaly is found in the: a. endoplasmic reticulum b. lysosomes c. mitochondria d. ribosomes

d (Interference HgbA1C.)

21) In using ion-exchange chromatographic methods, falsely increased levels of Hgb A1c might be demonstrated in the presence of: a. iron deficiency anemia b. pernicious anemia c. thalassemias d. Hgb S

b (The 2 items listed that produce a white color are white blood cells and semen. It is more likely that a woman would have a UTI with many WBC than enough post coital sperm contamination to make the urine white.)

21) Milky urine from a 24 yr. old woman would most likely contain: a. spermatozoa b. many white blood cells c. RBC's d. bilirubin

a (Antigen detected is located on the Fc portion of the IgG molecule.)

21) Rheumatoid factors are immunoglobulins with specificity for allotypic determinations located on the: a. Fc fragment of IgG b. Fab fragment of IgG c. J chain of IgM d. secretory component of IgM

c (JCAHO sets standards by which healthcare is measured and accredits institutions and laboratories worldwide.)

21) Which of the following organizations was formed to encourage the voluntary attainment of uniformly high standards in institutional medical care? a. Centers for Disease Control b. Health Care Finance Administration c. The Joint Commission d. Federal Drug Administration

a (Due to the small size of viruses, they are not visible using light microscopy. Electron microscopy is used to visualize viruses and the internal structure of microorganisms.)

21) Which type of microscope would be most useful in examining viruses and the structure of microbial cells? a. electron b. phase-contrast c. dark-field d. bright-field

d (Since all Neisseria species are oxidase positive, they possess the enzyme to oxidize tetramethyl-phenylenediamine.)

210) All species of the genus Neisseria have the enzyme to oxidize: a. naphthylamine b. dimethylaminobenzaldehyde c. glucopyranoside d. tetramethyl-phenylenediamine

c (Elevated levels of chylomicrons in serum or plasma will result in a turbid specimen. The large size of the chylomicron will reflect the light, causing a turbid appearance.)

210) Turbidity in serum suggests elevation of: a. cholesterol b. total protein c. chylomicrons d. albumin

d (While B cells are involved in humeral immunity through the production of antibodies, T cells mediate their responses through the release of soluble proteins called cytokines.)

210) Which of the following mediators is released during T cell activation? a. immunoglobulins b. thymosin c. serotonin d. cytokines

b (Lewis antibodies are usually IgM and agglutinate saline suspended cells. Approximately 22% of the population is Le[a+], which would account for 3 out of 10 donor units being incompatible. Anti-P1 is also an antibody that may react at immediate spin, but 79% of the white population and 94% of the black population are P1 +. Anti-C and anti-Fya are IgG antibodies that react at the antiglobulin phase.)

212) In the process of identifying an antibody, the technologist observed 2+ reactions with 3 out of the 10 cells in a panel after the immediate spin phase. There was no reactivity after incubation at 37 C and after the anti-human globulin test phase. The antibody most likely is: a. anti-P1 b. anti-Lea c. anti-C d. anti-Fya

a (Samples to be analyzed by gas-liquid chromatography must be volatile upon heating. The vaporized sample then flows, with the inert carrier gas, through the column.)

212) To be analyzed by gas liquid chromatography a compound must: a. be volatile or made volatile b. not be volatile c. be water-soluble d. contain a nitrogen atom

c (The only acronym listed here that is not a tumor marker is ANA. This stands for anti-nuclear antibody.)

212) Tumor markers that can be measured on body fluids include all except: a. CEA b. CA 125 c. ANA d. CYFRA 21-1

c (By definition, an allograft is a graft between genetically unidentical individuals of the same species. An isograft is between genetically identical individuals, a xenograft is between individuals of different species, and an autograft is transplanted from 1 location to another in the same individual.)

212) Which of the following terms describes a graft between genetic unidentical individuals belonging to the same species? a. autograft b. isograft c. allograft d. xenograft

d (Increased TSH, decreased free T4 and total T4, symptoms of cold intolerance and hair loss are consistent with hypothyroidism.)

213) A 45 yr. old woman complains of fatigue, heat intolerance, and hair loss. Total and free T4 are abnormally low. If the TSH showed marked elevation, this would be consistent with: a. Graves disease b. an adenoma of the thyroid c. thyrotoxicosis d. primary hypothyroidism

d (Steric-exclusion chromatography is based upon smaller molecules being trapped by the porous column material, but larger molecules are carried along by the mobile phase.)

213) A true statement about column chromatography methods, including high-performance liquid and gas chromatography, is that it: a. all utilizes a flame ionization detector b. requires derivation of nonvolatile compounds c. can be used to separate gases , liquids, or soluble solids d. can be used for adsorption, partition, ion-exchange and steric-exclusion chromatography

c (Primary characteristic distinguishing May-Hegglin from other neutrophil and lipid storage anomalies.)

213) Of the following, the disease most closely associated with pale blue inclusions in granulocytes and giant platelets: a. Gauchers disease b. Alder-Reilly c. May-Hegglin d. Pelger-Huet

d (Semen should liquefy and pour in droplets after 30 - 60 minutes. The normal pH is 7.2 - 8.0. Within an hour after collection, more than 50% should be motile, and their motility grade should be 2.0 or greater.)

213) Which semen result is abnormal? a. sample pours in droplets after 60 minutes b. >50% are motile within 1 hour of collection c. pH 7.5 d. motility of 1.0

a (Effect of conventional chemotherapy on WBC count.)

214) An oncology patient has the following results: The most probable explanation is: a. chemotherapy b. cold antibody c. clotted specimen d. inadequate mixing

d (Of the choices provided, only fermentation of carbohydrates provides definitive identification of Neisseria gonorrhoeae.)

214) Definitive identification of Neisseria gonorrhoeae is made with the: a. gram stain b. oxidase test c. degradation of amino acid d. hydrolysis of carbohydrates

a (Lactate is increased in the CSF in conditions that cause hypoxia. The highest lactate values [>35 mg/dL] are found in patients with bacterial meningitis. Viral, tubercular and fungal meningitis have lactate levels above normal, but not as high as bacterial meningitis.)

214) Increased CSF lactate is found in: a. bacterial meningitis b. Reye encephalopathy c. spina bifida d. multiple sclerosis

c (T3 is more biologically active, 80% of T4 is converted into T3.)

214) The majority of thyroxine (T4) is converted into the more biologically active hormone: a. thyroglobulin b. thyroid-stimulating hormone c. triiodothyronine d. thyrotropin-releasing hormone

b (Congenital hypothyroidism presents with very low thyroid hormones and is best confirmed by serum TSH.)

215) A 2 yr. old child with a decreased serum T4 is described as being somewhat dwarfed, stocky, overweight, and having coarse features. Of the following, the most informative additional lab test would be the serum: a. thyroxine binding globulin b. thyroid-stimulating hormone c. triiodothyronine d. cholesterol

b (Neisseria gonorrhoeae is oxidase positive and ferments glucose but not maltose.)

215) A gram-negative diplococcus that grows on modified Thayer-Martin medium can be further confirmed as Neisseria gonorrhoeae if it is: a. oxidase positive, glucose positive, and maltose positive b. oxidase positive and glucose positive, maltose negative c. oxidase positive and maltose positive, glucose negative d. glucose positive, oxidase negative and maltose negative

c (Definition of absolute neutropenia.)

215) A leukocyte count and differential on a 40-year-old Caucasian man revealed: This data represents: a. absolute lymphocytosis b. relative neutrophilia c. absolute neutropenia d. leukopenia

d (Meningitis, hemorrhage, and neurologic diseases, such as MS, will increase CSF protein. Decreased protein can be found due to CSF leakage, recent puncture, and rapid CSF production.)

215) Decreased CSF protein can be found in: a. meningitis b. hemorrhage c. multiple sclerosis d. CSF leakage

c (Electrophoresis involves the migration of a charged molecule or particle in a liquid medium under the influence of an electric field. The 3 roles of the buffer are 1] to carry the applied current; 2] to establish the pH for the electrophoresis procedure and 3] to determine the electrical charge on the solute.)

215) In electrophoretic analysis, buffers: a. stabilize electrolytes b. maintain basic pH c. act as a carrier for ions d. produce an effect on protein configuration

a (Definition of relative lymphocytosis.)

216) Al leukocyte count and differential on a 40-year-old Caucasian man revealed: This represents: a. relative lymphocytosis b. absolute lymphocytosis c. relative neutrophilia d. leukopenia

b (Moraxella catarrhalis is an oxidase- positive gram-negative diplococcus that is usually beta-lactamase positive. It does not ferment carbohydrates.)

216) An organism previously thought to be nonpathogenic, Moraxella catarrhalis, is now known to be associated with opportunistic respiratory infection and nosocomial transmission. Characteristic identification criteria include: a. oxidase negative b. carbohydrate negative (asaccharolytic) c. beta-lactamase negative d. gram-negative bacilli

b (Reactivity at 37C and AHG indicate the presence of an IgG antibody. Anti-M, although usually IgM, may be partly or wholly IgG. Anti-M is ruled out on cell4. Anti-Leb is usually IgM and can be ruled out on cells 4 and 8. This leaves anti-E, anti-Fya and anti-K.)

216) Based on the results of the above panel, the most likely antibodies are: a. anti-M and anti-K b. anti-E, anti-Jka and anti-K c. anti-Jka and anti-M d. anti-E and anti-Leb

c (Cytotoxic T cells, which are capable of destroying targets such as tumor cells and virus-infected cells, bear the CD8 surface marker, while the other cell types listed are positive for the CD4 surface marker.)

216) T lymphs that possess the CD8 surface marker mediate which of the following T cell functions? a. delayed type hypersensitivity b. regulatory c. cytotoxic d. helper

b (Waterhouse-Friederichsen syndrome is a known complication of severe Neisseria meningitidis infection.)

217) An autopsy performed on an 8 yr. old child revealed Waterhouse-Friederichsen syndrome. Blood and throat cultures taken just prior to death were positive for which organism? a. Neisseria gonorrhoeae b. Neisseria meningitidis c. Haemophilus influenzae d. Klebsiella pneumoniae

c (Ion-selective electrodes [ISE] are a potentiometric method of analysis. ISEs respond to individual ions present in a sample.)

217) An ion-selective electrode (ISE) measures the: a. activity of one ion only b. concentration of one ion only c. activity of one ion much more than the other ions present d. activity of only H+ ions

b (Anaphylactic, or type I hypersensitivity, occurs very rapidly, usually within 30 minutes after antigen exposure. While transfusion reactions and serum sickness are also examples of immediate hypersensitivity, they generally do not occur as rapidly. Contact dermatitis is a delayed hypersensitivity reaction, manifesting between 24 and 72 hours after antigen exposure.)

218) The most rapid immediate hypersensitivity reaction is associated with: a. transfusion b. anaphylaxis c. contact dermatitis d. serum sickness

d (Neonates are screened using total T4.)

218) The screening test for congenital hypothyroidism is based upon: a. TSH level in the newborn b. thyroid-binding globulin level in the newborn c. iodine level in the newborn d. total thyroxine (T4) level in the newborn

a (Ion-selective electrodes portend their selectivity properties through the use of particular membranes, as the selected ions interact with the membranes used in ion-selective electrodes.)

218) The selectivity of an ion-selective electrode is determined by the: a. properties of the membrane used b. solution used to fill the electrode c. magnitude of the potential across the membrane d. internal reference electrode

b (To determine the g factor, use 0.0001 x 30 cm x [3,000]^2. Remember that the radius is half the diameter.)

219) A centrifuge head has a diameter of 60 cm and spins at 3,000 RPMs. What is the maximum achievable G force. G = 0.00001 x radius in cm x RPM squared a. 1.8 G b. 2,700 G c. 27,000 G d. 90,000 G

d (Media containing potassium tellurite is used for the recovery of Corynebacterium diphtheriae.)

219) A gram stain of organisms on Loeffler agar showed pleomorphic gram-positive bacilli. The organism should be subcultured to: a. blood b. chocolate c. MacConkey d. potassium tellurite

b (Calculation of absolute eosinophil count.)

219) What is the absolute eosinophil count? a. 170/mL (0.17 x 10^9/L) b. 510/mL (0.51 x 10^9/L) c. 2,550/mL (2.55 x 10^9/L) d. 4,760/mL (4.76 x 10^9/L)

c (Match the color to the pigment. Bile is green/brown. Porphyrins are red. Melanin is black. Blood cells are red or white.)

22) A brown/black urine would most likely contain: a. bile pigment b. porphyrins c. melanin d. blood cells

b (Daily disk diffusion quality control can be converted to weekly testing when 30 days of consecutive testing demonstrates no more than 3 antibiotic/organism combinations outside of the acceptable limits.)

22) After satisfactory performance of daily disk diffusion susceptibility quality control is documented, the frequency of quality control can be reduced to: a. twice a week b. every week c. every other week d. every month

a (Acetone in carbohydrate metabolism.)

22) An increase in serum acetone is indicative of a defect in the metabolism of: a. carbohydrates b. fats c. urea nitrogen d. uric acid

d (Red Blood Cells are stored at 1-6C.)

22) Optimum storage temperature for RBCs: a. -80 C b. -20 C c. -12 C d. 4 C

a (IgM RF is the species most commonly measured in clinical assays.)

22) Rheumatoid factor in a patient's serum may cause a false: a. positive test for the detection of IgM class antibodies b. negative test for the detection of IgM class antibodies c. positive test for the detection of IgG class antibodies d. negative test for the detection of IgG class antibodies

c (Accreditation is the approval of an institution or program based on a review by one or more independent examiners that specific requirements or predetermined standards are met.)

22) The process by which an agency or organization uses predetermined standards to evaluate and recognize a program of study in an institution is called: a. regulation b. licensure c. accreditation d. credentialing

a (Age, sex, physical complaint, with elevated TSH point to primary hypothyroidism.)

220) A 68 yr. old female patient tells her physician of being cold all the time and recent weight gain, with no change in diet. The doctor orders a TSH level, and the lab reports a value of 8.7 uU/mL (reference range = 0.5-5.0 uU/mL) this patient most likely has: a. primary hypothyroidism b. Graves disease c. a TSH secreting tumor d. primary hyperthyroidism

a (CD3 is a marker used to identify T lymphocytes. It is present on the surface of all mature T-cells, regardless of T-cell subset.)

220) An immunofluorescence test using reagent antibody directed against the CD3 surface marker would identify which of the following cell typed in a sample of human peripheral blood? a. all mature T lymphs b. T helper lymphs only c. cytotoxic T lymphs only d. T regulatory cell only

b (Other distractors do not apply to centrifugal force.)

220) In a centrifugal analyzer, centrifugal force is used for: a. add reagents to the rotor b. transfer liquids from the inner disc to the outer cuvette c. measure changes in optical density in the centrifugal force field d. counteract the tendency of precipitates to settle in the cuvette

a (Hemacytometer calculation .)

220) Which of the following is the formula for manual white cell count? a.(number of cells counted x dilution x 10)/number of squares counted b. (number of cells counted x dilution)/10 x number of squares counted c. number of cells counted x dilution d. number of cell counted x number of squares counted

a ( The direct antiglobulin test [DAT] is used to identify red blood cells that have been coated with antibody in vivo.)

220) Which of the following tests is most commonly used to detect antibodies attached to a patient's red blood cells in-vivo? a. DAT b. complement fixation c. IAT d. immunofluorescence

c (Bacillus cereus is the etiologic agent of 2 distinct types of food poisoning syndromes. Spores can survive cooking and germinate. Vegetative cells multiply and produce toxin.)

220) Which organism commonly causes food poisoning by consumption of foods containing excessive populations of organisms or preformed enterotoxin? a. Salmonella enteritidis b. Shigella sonnei c. Bacillus cereus d. Escherichia coli

a (Nocardia are capable of growing on Lowenstein-Jensen agar within 7 days and demonstrate branching, beaded gram-positive bacilli on Gram stain. Nocardia are partially acid fast, and are stained best with the modified acid-fast stain.)

221) An organism recovered from a sputum has the following characteristics: - Culture: growth at 7 days on Lowenstein-Jensen agar, incubated under aerobic CO2 at 35 C - Gram stain: delicate branching gram-positive bacilli - Acid-fast stain: branching, filamentous, "partially" acid-fast bacterium These results are consistent with which of the following genera? a. Nocardia b. Mycobacterium c. Actinomyces d. Streptomyces

b (In cold agglutinin syndrome, anti-I acts as a complement binding antibody with a high titer and high thermal amplitude. the complement cascade is activated and C3d remains on the red cell membrane of circulating cells.)

221) Anti-I may cause a positive direct antiglobulin test (DAT) because of: a. anti-I agglutinating the cells b. C3d bound to the red cells c. T-activation d. C3c remaining on the red cells after cleavage of C3b

b (hCG is the primary marker for early pregnancy.)

221) Which of the following is secreted by the placenta and used for the early determination of pregnancy? a. follicle stimulating hormone (FSH) b. Human chorionic gonadotropin (HCG) c. luteinizing hormone (LH) d. progesterone

c (The devices listed are electronic, but the only device that measures rpm is a tachometer. RPM must be consistent to produce adequate G force.)

221) Which of the following is the best guide to consistent centrifugation? a. potentiometer setting b. armature setting c. tachometer reading d. rheostat readings

b (Point-of-care testing devices use a small volume of sample and are therefore particularly useful in neonates, small babies, and those with increased risk from phlebotomy.)

222) A benefit of microassays, such as point of care methods, include: a. increased analytical reliability b. reduced sample volume c. increased diagnostic specificity d. reduced numbers of repeated tests

b (Hemacytometer calculation .)

222) If a WBC count is performed on a 1:100 dilution and the number of cells counted in eight squares is 50, the total WBC count is: a. 5,000/mL (5.0 x 10^9/L) b. 6,250/mL (6.25 x 10^9/L) c. 50,000/mL (50.0 x 10^9/L) d. 62,500/mL (62.5 x 10^9/L)

b (Corynebacterium and Listeria are catalase-positive and gram-positive bacilli. Listeria demonstrates "tumbling" motility that is best demonstrated following growth at 25 C. A few species of Corynebacterium species are motile when grown at 35 C.)

222) The best procedure to differentiate Listeria monocytogenes from Corynebacterium species is: a. catalase b. motility at 25 C c. motility at 35 C d. Gram stain

c (In order to determine if an isolate of Corynebacterium diphtheriae produces toxin, testing for the presence of diphtheria toxin must be performed using methods such as the Elek test or PCR.)

223) Establishing the pathogenicity of a microorganism isolated from a child's throat and identified as Corynebacterium diphtheria would depend upon: a. the morphological appearance as revealed by Gram stain b. the type of hemolysis on blood agar c. a positive toxigenicity test d. the appearance of growth of Tinsdale tellurite agar

d (After washing cells for the DAT or IAT procedure, the AHG should be added immediately and read. Delay can cause a weakened or negative result due to dissociation of the bound IgG in the prolonged time before reagent is added.)

223) In DAT and IAT tests, false-negative reactions may result if the: a. patient's blood specimen was contaminated with bacteria b. patient's blood specimen was collected into tubes containing silicon gel c. saline used for washing the serum/cell mixture has been stored in glass or metal containers d. addition of AHG is delayed for 40 minutes or more after washing the serum/cell mixture.

a (Dry cobalt-treated CaCl2 is blue.)

223) In the proper use of cobalt treated-anhydrous CaCl2, the desiccant should be: a. changed when it turns pink b. changed when it turns blue c. kept in the dark d. kept in the cold

b (Enzyme-multiplied immunoassay is a homogeneous enzyme immunoassay used to quantitate drugs, hormone, etc. Unlike the heterogeneous assays, the free-labeled reactant does not have to be separated from the bound-labeled reactant.)

224) Enzyme multiplied immunoassay technique differ from all other types of enzyme immunoassays in that: a. lysozyme is the only enzyme used to label the hapten molecule b. no separation of bound and free antigen is required c. inhibition of the enzyme label is accomplished with polyethylene glycol d. antibody absorption to polystyrene tubes precludes competition to labeled and unlabeled antigen

a (The change in absorbance of amniotic fluid at 450 nm is used to detect hemolytic disease of newborn [HDFN].)

224) In amniotic fluid, the procedure used to detect hemolytic disease of the newborn is: a. measurement of absorbance at 450 nm b. creatinine c. lecithin/sphingomyelin ratio d. estriol

c (In flow cytometry, a laser beam hits cells as they pass through the instrument in single file. The amount of light scatter is measured from each cell at 2 different angles, and is used to identify the cell type on the basis of size and granularity. Cells are also identified on their ability to emit fluorescence after they have been incubated with fluorescent-labeled monoclonal antibodies that bind to specific surface markers.)

224) In flow cytometry, labeled cells: a. scatter the light and absorb fluorescence b. absorb fluorescence and emit electronic impulses c. scatter the light and emit fluorescence d. absorb both fluorescence and light

d (Erysipelothrix rhusipathiae is the only gram-positive bacillus that produces hydrogen sulfide when inoculated into triple sugar iron agar.)

224) Which feature distinguishes Erysipelothrix rhusiopathiae from other clinically significant non-spore-forming, gram-positive, facultatively anaerobic bacilli? a. "tumbling" motility b. beta-hemolysis c. more pronounced motility at 25 C than 35 C d. H2S production

c (A decrease in the number of T helper [CD4+] cells and a decrease in the ratio of CD4+:CD8+ cells is a characteristic finding of HIV infection and AIDS.)

225) A marked decrease in the CD4 lymphocytes and decrease in the CD4/CD8 ratio: a. is diagnostic for bacterial septicemia b. may be seen in most hereditary immunodeficiency disorders c. is associated with a viral induced immunodeficiency d. is only seen in patients with advanced disseminated cancer

b (Serum hCG levels peak at 8-10 weeks.)

225) During a normal pregnancy, quantitative human chorionic gonadotropin levels peak how many weeks after the last menstrual period? a. 2 - 4 b. 8 - 10 c. 14 - 16 d. 18 - 20

d (The toxin produced by Clostridium botulinum is a neurotoxin that is excreted by the organism in food materials, and is then ingested.)

236 The etiologic agent of botulism is: a. highly motile b. non-spore-forming c. Clostridium perfringens d. an exotoxin producer

d (Antiglobulin reagent is used to detect the presence of red cells which have been coated in-vivo with IgG and/or C3d. Antiglobulin reagent may be polyspecific [contains an anti-IgG and anti-C3d] or monospecific [anti-IgG or anti- C3d].)

225) In the direct antiglobulin test, the antiglobulin reagent is used to: a. mediate hemolysis of indicator red cells by providing complement b. precipitate anti-erythrocyte antibodies c. measure antibodies in a test serum by fixing complement d. detect preexisting antibodies on erythrocytes

d (Listeria may be confused with some streptococci because Listeria is beta- hemolytic and is capable of hydrolyzing esculin.)

225) Listeria can be confused with some streptococci because of its hemolysis and because it is: a. nonmotile b. catalase negative c. oxidase positive d. esculin positive

c (Enzymes are one of the several labels used in labeled immunoassays. The catalytic property of the enzyme is used to detect and quantitate the immunological reaction. Alkaline phosphatase and horseradish peroxidase are examples of enzymes used. The product of the enzymatic activity is monitored spectrophotometrically.)

225) Which of the following statements about immunoassays using enzyme labeled antibodies or antigens is correct? a. inactivation of the enzyme is required b. the enzyme label is less stable than an isotopic label c. quantitation of the label can be carried out with a spectrophotometer d. the enzyme label is not an enzyme found naturally in serum

c (WBC correction for nRBC.)

226) A blood smear shows 80 nucleated red cells per 100 leukocytes. The total leukocyte count is 18 x 10^3/mL (18 x 10^9/L). The true white cell count expressed in SI units is: a. 17.2 x 10^3/mL (17.2 x 10^9/L) b. 9.0 x 10^3/mL (9.0 x10^9/L) c. 10.0 x 10^3/mL (10.0 x 10^9/L) d. 13.4 x 10^3/mL (13.4 x 10^9/L)

c (AHG control cells are IgG-sensitized cells that react with the anti-IgG in the AHG reagent to demonstrate AHG was added and not neutralized by insufficient washing of the tests prior to its addition.)

226) AHG control cells: a. can be used as a positive control for anti-C3d reagents b. can be used only for the IAT c. are coated only with IgG antibody d. must be used to confirm all positive antiglobulin reactions

c (Bacillus anthracis are large, rectangular gram-positive bacilli that produce colonies with an irregular edge [often described as a "medusa-head" appearance] on blood agar. Colonies are nonhemolytic, catalase positive and nonmotile.)

226) Fluid from a cutaneous black lesion was submitted for routine bacterial culture. After 18 hours of incubation at 35 C there was no growth on MacConkey agar, but 3+ growth on sheep blood agar. The colonies were nonhemolytic, nonmotile, 4-5 mm in diameter and off-white with a ground glass appearance. Each colony had an irregular edge with comma-shaped outgrowths that stood up like "beaten egg whites" when gently lifted with an inoculating needle. A Gram stain of a typical colony showed large, gram-positive rectangular bacilli. The organism is most likely: a. Clostridium perfringens b. Aeromonas hydrophila c. Bacillus anthracis d. Mycobacterium marinum

c (Cold agglutinin disease is associated with cold reactive antibodies that typically activate complement. Cells that do not undergo lysis due to complement activation have C3d attached to the red blood cells.)

227) A 56 year old female with cold agglutinin disease has a positive DAT. When the DAT is repeated using monospecific antiglobulin sera, which of the following is most likely to be detected? a. IgM b. IgG c. C3d d. C4a

d (Nocardia asteroides are partially acid fast, and do not hydrolyze the substrates casein, tyrosine, or xanthine.)

227) A branching gram-positive, partially acid-fact organism is isolated from a bronchial washing on a 63 yr. old woman receiving chemotherapy. The organism does not hydrolyze casein, tyrosine or xanthine. The most likely identification is: a. Actinomadura madurae b. Nocardia caviae c. Streptomyces somaliensis d. Nocardia asteroides

d (An elevation in the MCHC occurs in approximately 50% of individuals with hereditary spherocytosis. Elevation in the MCHC above the upper normal limit should prompt an investigation of the sample for autoantibodies that agglutinate RBC, or on older instruments when the hemoglobin concentration is artifactually elevated by lipemia.)

227) A mean cell hemoglobin concentration over 36 g/dL is frequently found in : a. hereditary spherocytosis b. lipemia c. active cold agglutinin disease d. all of the above

a (An elevated hematocrit decreases the amount of plasma in whole blood, and causes an effective increase in the amount of citrate added to that plasma. Recalcification incompletely overcomes the additional citrate, and prolongations in clotting time tests can be expected, unless the amount of citrate is adjusted for the abnormal hematocrit.)

228) An unexplained elevation of the prothrombin time in a 72 yr. old smoker who has been diagnosed with chronic pulmonary obstructive disease is most likely due to: a. an elevated hematocrit b. a decreased hematocrit c. vitamin K deficiency d. decreased thrombin activity

d (Anaerobic bacteria characteristically produce foul-smelling metabolic end products.)

228) The characteristic that is most commonly associated with the presence of strict anaerobic bacteria and can be taken as presumptive evidence of their presence in a clinical specimen is the: a. presence of a single bacterial species b. production of gas in a thioglycollate broth culture c. growth on a blood agar plate incubated in an anaerobic jar d. presence of a foul, putrid odor from tissue specimens and cultures

d (All adrenal steroid hormones are enzymatically derived from cholesterol.)

228) What common substrate is used in the biosynthesis of adrenal steroids, including androgens and estrogens? a. cortisol b. catecholamines c. progesterone d. cholesterol

d (The specimen Gram stain suggests the presence of Bacillus or Clostridium. Since no growth was observed aerobically, the specimen should be inoculated to media that are incubated anaerobically.)

229) Gram stain of a thigh wound showed many gram-positive spore-forming bacilli. The specimen was placed on brain heart infusion blood agar and incubated aerobically at 35 C for 3 days. At the end of that time, the plates showed no growth. The most likely explanation is that some of the specimen should have been incubated: a. on chocolate agar b. for 5 days c. under 5% CO2 d. anaerobically

a (CD2 and CD3 are T-cell markers, while surface immunoglobulin is a B-cell marker.)

236) A patients abnormal lymphocytes are positive for CD2 antigen, lack C3 receptors and are negative for surface immunoglobulins. This can be classified as a disorder of: a. T cells b. B cells c. monocytes d. natural killer cells

c (EDTA chelates calcium preventing blood to clot. This chelation of calcium also will stop the complement cascade. Calcium ions are necessary for C1 to attach to IgG on the red blood cells.)

229) Use of EDTA plasma prevents activation of the classical complement pathway by: a. causing rapid decay of complement components b. chelating Mg++ ions, which prevents the assembly of C6 c. chelating Ca++ ions, which prevents assembly of C1 d. preventing chemotaxis

c (Flow cytometry uses fluorescent-labeled monoclonal antibodies to identify cells of interest by binding to specific components within or on the surface of the cells.)

229) What is the immunologic method utilized in the flow cytometer? a. latex agglutination b. enzyme linked immunoassay c. immunofluorescence d. radioimmunoassay

d (Morphology of a monocyte, although the RBC inclusions [Pappenheimer bodies] look more interesting.)

229) Which of the following statements about this field is true? a. reticulocytes are demonstrable b. toxic granulation is present c. the cell in the center is a basophilic normoblast d. the large cell on the left is a monocyte

b (If the seal is broken during processing, components are considered to be prepared in an open system, rather than a dosed system. The expiration time for Red Blood Cells in an open system is 24 hours)

23) If the seal is entered on a unit of RBCs stored at 1 C to 6 C, what is the maximum allowable storage period in hours? a. 6 b. 24 c. 48 d. 72

b (The amount of antibiotic used in disk diffusion susceptibility testing is standardized and constant. Once the disk is placed on the inoculated plate and makes contact with the agar, the antibiotic in the disk begins to diffuse out. As it diffuses into the media, the concentration of antibiotic gets lower the further it diffuses from the disk.)

23) In disk diffusion susceptibility testing, as a antimicrobial agent diffuses away from the disk, the concentration antibiotic is: a. increased b. decreased c. unchanged d. inoculum dependent

a (The only normal pigment/substance listed here is urochrome.)

23) The yellow color of urine is primarily due to: a. urochrome pigment b. methemoglobin c. bilirubin d. homogenistic acid

c (Methyldopa is frequently listed as the prototype for drug-independent antibody mechanism where autoantibody is present on the red cells and may also be present in the plasma.)

230) Which of the following medications is most likely to cause production of autoantibodies? a. penicillin b. cephalothin c. methyldopa d. tetracycline

c (Most infections involving anaerobes are polymicrobic and can include obligate aerobes, facultative anaerobes, microaerophilic bacteria in addition to anaerobic bacteria.)

231) Anaerobic infections differ from aerobic infections in which of the following? a. they usually respond favorably with aminoglycoside therapy b. they usually arise from exogenous sources c. they are usually polymicrobic d. Gram stains of specimens are less helpful in diagnosis

a (A deficiency of vitamin A leads to night blindness, and if prolonged total blindness.)

231) Night blindness is associated with deficiency of which of the following vitamins? a. A b. C c. niacin d. thiamine

b (Informative communication shares knowledge about laboratory processes and policies.)

233) A major laboratory policy change that will affect a significant portion of the laboratory personnel is going to take place. In order to minimize the staff's resistance to this change, the supervisor should: a. announce the policy change the day before it will become effective b. discuss the policy change in detail with all personnel concerned, well in advance of implementation c. announce only the positive aspects of the policy change in advance d. discuss only the positive aspects of the policy with those concerned

c (A negative reaction after the addition of check cells indicates AHG serum was not present. Inadequate washing of red cells may leave residual patient serum behind, which can neutralize AHG serum.)

233) Crossmatch results at the antiglobulin phase were negative. When 1 drop of check cells was added, no agglutination was seen. The most likely explanation is that the: a. red cells were over washed b. centrifuge speed was to high c. residual patient serum inactivated the AHG reagent d. laboratorian did not add enough check cells

d (Morphological description of a monocyte.)

233) This cell is a: Size: 12 to 16 um Nucleus: oval, notched, folded over to horseshoe shape Chromatin: fine lacy, stains light purple-pink Nucleoli: none present Cytoplasm: abundant, slate gray, with many fine lilac-colored granules a. promyelocyte b. lymphocyte c. neutrophil d. monocyte

a (Anaerobes normally inhabit skin and mucous membranes as part of the normal flora. Distractors b, c, and d are virtually always unacceptable for anaerobic culture, because they normally contain anaerobic organisms. It is difficult to interpret culture results from these specimens and distinguish between pathogens and normal flora.)

234) Acceptable specimens for the culture of anaerobic bacteria that cause disease include: a. abscesses b. gingival swabs c. skin swabs d. vaginal swabs

b (Morphology of a basophil.)

234) The large cell in the center of the image would be best described as a(n): a. neutrophil b. basophil c. eosinophil d. myelocyte

c (The employee moved into a participating situation.)

234) When employees are going to be responsible for implementing a change in procedure or policy, the manager should: a. make the decision and direct the employees to implement it b. solicit the employee input but do what he/she thinks should be done c. involve the employees in the decision-making process from the very beginning d. involve only those employees in the decision-making process who would benefit from the change

d (Weak antibodies may be missed if there are excess RBC antigens as there may be too few antibodies to bind to red cell antigens.)

234) Which of the following might cause a false-negative IAT? a. over-reading b. IgG coated screen cells c. addition of an extra drop of serum d. too heavy a cell suspension

d (A deficiency of niacin may be seen with chronic alcoholism, and is known as pellagra.)

235) Pellagra is associated with deficiency of which of the following vitamins? a. A b. B1 c. thiamine d. niacin

d (Propionibacterium acnes is part of the normal flora of the skin, so it is frequently isolated from improperly collected blood cultures.)

235) Propionibacterium acnes is most often associated with: a. normal oral flora b. post-antibiotic diarrhea c. tooth decay d. blood culture contamination

c (A solution of 8% bovine albumin can be prepared by diluting the more concentrated solution with normal saline. The formula to be used is: [volume 1 x concentration 1] = [volume 2 x concentration 2]. A solution of 6%-8% albumin is used with some anti-D reagents as a control for spontaneous agglutination.)

236) What is the most appropriate diluent for preparing a solution of 8% bovine albumin for red cell control reagent? a. deionized water b. distilled water c. normal saline d. Alsever solution

a (Corynebacterium species and Propionibacterium species do not produce spores, and Bacillus is an aerobic organism.)

237) A strict anaerobe that produces terminal spores is: a. Clostridium tetanus b. Corynebacterium diphtheriae c. Bacillus anthracis d. Propionibacterium acnes

d (Vanillylmandelic acid [VMA] is the major metabolite of epinephrine and norepinephrine. VMA is measured in a 24-hour urine.)

237) The urinary excretion product measured as an indicator of epinephrine production is: a. dopamine b. dihydroxyphenylalanine c. homovanillic acid d. vanillylmandelic acid

b (Rh antibodies show enhanced reactivity with enzyme pretreated cells. Treatment of red cells with enzymes weakens reactivity with antibodies in the MNS and Duffy systems.)

237) Which of the following antigens gives enhanced reactions with its corresponding antibody following treatment of the red cells with proteolytic enzymes? a. Fya b. E c. S d. M

b (Actinomyces and Eubacterium are anaerobic gram-positive bacilli that do not form spores; Bacillus is not an anaerobic organism. Clostridium perfringens is a spore-forming anaerobe organism.)

238) An anaerobic, spore-forming, nonmotile, gram-positive bacillus isolated from a foot wound is most likely: a. Actinomyces israelii b. Clostridium perfringens c. Bacillus subtilis d. Eubacterium lentum

b (Patients may have antibodies to components of reagents. Washing the patient's cells prior to testing to remove their plasma from the cell suspension will resolve the reactivity with anti-B.)

238) In a prenatal workup, the following results were obtained: ABO discrepancy was thought to be due to an antibody directed against a component of the typing sera. Which test would resolve this discrepancy? a. A1 lectin b. wash patient's RBCs and repeat testing c. anti-A,B and extend incubation of the reverse group d. repeat reverse group using A2 cells

a (Enzyme treatment would allow for differentiation of the remaining antibodies after rule outs. The Fya antigen would be denatured, allowing determination of whether anti-Jka and -K are present, and to confirm anti-E.)

239) Based on the results of the above panel, which technique would be most helpful in determining antibody specificity? a. proteolytic enzyme treatment b. urine neutralization c. autoadsorption d. saliva inhibition

c (Problem solving includes 3 steps: define problem, write down the facts, examine solution [consider options].)

239) The ability to make good decisions often depends on the use of a logical sequence of steps that include: a. defining problem, considering options, implementing decisions b. obtaining facts, considering alternatives, reviewing results c. defining problem, obtaining facts, considering options d. obtaining facts, defining problem, implementing decision

d (Products of glycolysis.)

24) Blood samples were collected at the beginning of an exercise class and after thirty mins. of aerobic activity. Which of the following would be most consistent with the post exercise sample? a. normal lactic acid, low pyruvate b. low lactic acid, elevated pyruvate c. elevated lactic acid, low pyruvate d. elevate lactic acid, elevated pyruvate

a (Hydrogen bonds hold the 2 strands of DNA together in a double-stranded DNA molecule.)

24) Denaturation of DNA during a PCR reaction refers to breaking: a. hydrogen bonds between nitrogenous bases in base-paired nucleotides b. phosphodiester bonds between nitrogenous bases in base-paired nucleotides c. covalent bonds between nitrogenous bases in base-paired nucleotides d. peptide bonds between nitrogenous bases in base-paired nucleotides

d (The 2 most common causes of failure of the Gas Pak system is a defective gasket in the jar lid that allows escape of gas from inside the jar and inactivated catalyst pellets.)

24) The most common cause for failure of a Gas Pak anaerobic jar to establish an adequate environment for anaerobic incubation is: a. the failure of the oxidation reduction potential indicator system due to deterioration of methylene blue b. the failure of the packet to generate adequate H2 and/or CO2 c. condensation of water on the inner surface of the jar d. catalysts that have become inactivated after repeated use

c (ACTH stimulation tests, using synthetic ACTH, will differentiate primary from secondary adrenal insufficiency. Synthetic ACTH will not cause the adrenal gland to respond in primary insufficiency.)

240) A diagnosis of primary adrenal insufficiency requires demonstration of: a. decreased urinary 17-keto and 17-hydroxysteroids b. decreased cortisol production c. impaired response to ACTH stimulation d. increased urinary cortisol excretion after metyrapone

a (Association of pseudogout with calcium pyrophosphate crystals.)

240) In synovial fluid, the most characteristic finding in pseudogout is: a. calcium pyrophosphate dihydrate crystals b. cartilage debris c. monosodium urate crystals d. hemosiderin-laden macrophages

c (Soluble forms of some blood group antigens can be prepared from other sources and used to inhibit reactivity of the corresponding antibody, such as the HTLA antibodies anti-Ch and anti-Rg. Most HTLA antibodies, although weakly reactive in undiluted serum, will continue to react weakly at higher dilutions.)

240) Of the following, the most useful technique in the identification and classification of high-titer, low-avidity antibodies is/are: a. reagent red cell panels b. adsorption and elution c. titration and inhibition d. cold autoadsorption

a (The reverse CAMP test, production of lecithinase, and demonstration of double zone hemolysis are tests and characteristics used for the identification of Clostridium perfingens,)

240) The reverse CAMP test, lecithinase production, double zone hemolysis, and Gram stain morphology are all useful criteria in the identification of: a. Clostridium perfringens b. Streptococcus agalactiae c. Propionibacterium acnes d. Bacillus anthracis

d (For neutralization studies to be valid, the saline dilutional control must be reactive. Since neutralization studies involve adding a substance to the patient's plasma, nonreactivity in test tubes may be due to simple dilution. The saline control acts as the dilutional control and must be reactive. When the saline control is reactive, then if the tube with the substance is nonreactive, the interpretation that neutralization has occurred is made. If it is reactive, neutralization did not occur.)

241) To confirm a serum antibody specificity identified as anti-P1, a neutralization study was performed and the following results obtained: serum + P1 substance = NEGATIVE serum + saline = NEGATIVE What conclusion can be made from these results. a. anti-P1 is confirmed b. anti-P1 is ruled out c. a second antibody is suspected due to the results of the negative control d. anti-P1 cannot be confirmed due to the results of the negative control

b (Bacillus cereus is not an anaerobic organism; Eubacterium and Bidobacterium are anaerobic gram-positive bacilli that do not form spores. Clostridium septicum forms subterminal spores.)

243) An anaerobic gram-positive bacilli with subterminal spores was isolated from a peritoneal abscess. The most likely identification of this organism is: a. Bacillus cereus b. Clostridium septicum c. Eubacterium lentum d. Bifidobacterium dentium

b (Anti-Leb is confirmed because the tubes with Lewis substance are negative. Nonreactivity of the serum with Le[b+] cells indicates the anti-Leb in the serum was neutralized by the Lewis substance. The test is valid since the patient's serum with saline rather than substance added is still able to react with the Le[b+] cells.)

243) To confirm the specificity of anti-Leb, an inhibition study using Lewis substance was performed with the following results: What conclusion can be made from these results? a. a second antibody is suspected due to the positive control b. anti-Leb is confirmed because the tubes with Lewis substance are negative c. anti-Leb is not confirmed because the tubes with Lewis substance are negative d. anti-Leb cannot be confirmed because the saline positive is control

a (Botulism infection is confirmed by reference laboratories such as the CDC by demonstrating toxin in serum, feces, gastric contents or vomitus.)

244) The most meaningful laboratory procedure in confirming the diagnosis of clinical botulism is: a. demonstration of toxin in the patient's serum b. recovery of Clostridium botulinum from suspected food c. recovery of Clostridium botulinum from the patient's stool d. Gram stain of suspected food for gram-positive, sporulating bacilli

c (Selective media are needed to isolate Clostridium difficile from stool and CCFA is also differential-fermentation of lactose produces classic colony morphology for this organism.)

245) A stool sample is sent to the laboratory for culture to rule out Clostridium difficile. What media should the microbiologist use and what is the appearance of the organisms on this media? a. BBE: colonies turn black b. Brucella agar: red pigmented colonies c. CCFA: yellow, ground glass colonies d. CNA: double zone hemolytic colonies

c (The Gram stain and double zone of hemolysis are characteristics of Clostridium perfingens. C perfingens is lecithinase positive on the egg yolk agar test for lecithinase and lipase.)

246) A Gram stain of a necrotic wound specimen showed large gram-positive bacilli. There was 3+ growth on anaerobic media only, with colonies producing a double zone of hemolysis. To identify the organism, the microbiologist should: a. determine if the organism ferments glucose b. perform the oxidase test c. set up egg yolk agar plate d. test for bile tolerance

d (Angiotensin II directly stimulates the adrenal cortex to release aldosterone.)

246) Aldosterone is released by the adrenal cortex upon stimulation by: a. renin b. angiotensinogen c. angiotensin I d. angiotensin II

a (In the solid phase technology, the antibody screening cells are bound to the surface of the well. Antibody specific for antigen on the red blood cells attaches, resulting in a diffuse pattern of red blood cells in the well. A negative reaction would have manifested as a pellet of red blood cells in the bottom of the well.)

246) An antibody screen performed using solid phase technology revealed a diffuse layer of RBCs on the bottom of the well. these results indicate: a. a positive reaction b. a negative reaction c. serum was not added d. red cells have a positive DAT

a (Personal communication should be planned: P = plan your communication, L = listen to others, A = avoid assumptions / ensure communication is understood, N = network.)

247) Communication is enhanced by: a. a planned strategy that includes listening skills and ensuring an understanding with questions b. relying on email, memos and voice mail to communicate new info c. formal, hierarchical patterns instead of informal networking patterns d. assumptions if there are questions about the intent of the message

b ( The K antigen is integral to the red cell membrane and would not change in a patient. Errors in typing or patient identification may be detected when discrepancies are found when comparing historical records.)

247) On Monday a patient's K antigen typing result was positive. 2 days later the patient's K typing was negative. The patient was transfused with 2 units of FFP. The tech might conclude that the: a. transfusion of FFP affected the K typing b. wrong patient was drawn c. decreased serum bilirubin d. agglutination with normal adult ABO compatible sera

b (A daily "line-up" can be conducted at a time of day that is convenient to the department. Providing a question for discussion is an effective way to foster conversation.)

248) In its guidelines, CAP stresses that communication must be effective and efficient. An effective component to enhance a multifaceted communication plan in the clinical laboratory is: a. posting department goals on communication bulletin boards b. daily meetings with a question to foster conversation and focus the discussion c. written email to communicate all new changes in processes and procedures d. posting errors on communication bulletin boards to prevent repeat errors

d (vWF is a constituent of platelet alpha granules, and is synthesized in the megakaryocyte as it develops.)

262) vWF antigen can be found in which of the following? a. myeloblast b. monoblast c. lymphoblast d. megakaryoblast

a (Most testicular tumors are germ cell tumors which are characterized by elevated serum levels of alpha-fetoprotein [AFP]. Measurement of serum AFP is used in the diagnosis, therapy and follow-up of testicular cancer. The carcinoembryonic antigen is a marker for colon cancer. The serum levels of testosterone and prolactin are not increased in testicular cancer.)

249) Detection of which of the following substances is most useful to monitor the course of a patient with testicular cancer? a. alpha-fetoprotein b. carcinoembryonic antigen c. prolactin d. testosterone

c (Rouleaux will readily disperse in saline whereas true agglutination will remain after saline replacement.)

249) While performing an antibody screen, a test reaction is suspected to be rouleaux. A saline replacement test is performed and the reaction remains. What is the best interpretation: a. original reaction of rouleaux is confirmed b. replacement test is invalid and should be repeated c. original reaction was due to true agglutination d. antibody screen is negative

a (Cryoprecipitate must be transfused within 4 hours of pooling.)

25) Cryo AHF must be transfused within what period of time following thawing and pooling? a. 4 hrs. b. 8 hrs. c. 12 hrs. d. 24 hrs.

b (Annealing temperature determines the amount of mismatch tolerated between the primers and the target.)

25) What is the most critical step in determining the specificity of a PCR reaction? a. denaturation temp b. annealing temp c. extension temp d. number of cycles in the PCR reaction

a (Alpha-fetoprotein [AFP] is an oncofetal glycoprotein marker for hepatocellular carcinoma. Elevated levels of AFP [<200 mg\L] are seen in hepatitis and cirrhosis. However, in hepatocellular carcinoma, the levels can be greater than 1,000 mg\L.)

250) Increased concentrations of alpha-fetoprotein in adults are most characteristically associated with: a. hepatocellular carcinoma b. alcoholic cirrhosis c. chronic active hepatitis d. multiple myeloma

a (Myeloperoxidase as a marker of primary granules in granulocytes.)

250) Which of the following stains in closely associated with the lysosomal enzyme in primary granules? a. peroxidase b. Sudan black B c. periodic acid-Schiff d. Prussian blue

d ( The carcinoembryonic antigen [CEA] is a marker for colon, gastrointestinal and lung cancer. Elevated serum levels of CEA are primarily seen with colon cancer. Although the levels may be increased in individuals with benign conditions, the level of CEA elevation is greater for colon cancer.)

251) Carcinoembryonic antigen is most likely to be produced in a malignancy involving the: a. brain b. testes c. bone d. colon

b (Two reagents used for removing IgG from red blood cells are chloroquine diphosphate [CDP] and EDTA glycine acid [EGA]. Using either of these procedures is useful to reduce a patient's DAT and allow phenotyping with IAT reactive antisera.)

251) Which of the following is useful for removing IgG from RBCs with a positive DAT to perform a phenotype? a. bromelin b. chloroquine c. LISS d. DTT

a (Anti-Fya would not react with enzyme pretreated cells; a select cell panel would allow for individual reactivity of the remaining 2 antibodies. Thiol reagents would be used to disperse agglutination of IgM antibodies; the antibodies in question are IgG.)

252) A patient's serum contains a mixture of antibodies. One of the antibodies is identified as anti-D. Anti-Jka, Fya, and possibly another antibody are present. What technique may be helpful to identify the other antibody? a. enzyme panel: select cell panel b. thiol reagents c. lowering the pH and increasing the incubation time d. using albumin as an enhancement media in combination with selective adsorption

a (LIS systems use barcode labels for positive sample identification for many of the processes in the lab. Each LIS vendor will specify which of several barcode label formats are compatible with their system.)

252) CODE 128, ISBT 128, CODE 39 and Interleaved 2 and 5 symbologies are used by laboratory information systems to create which of the following? a. barcode labels b. worklists c. instrument download files d. patient reports

a (tPSA along with DRE is the recommended screen for prostate cancer in males over 50 years of age.)

252) Which of the following is useful in the detection and management of carcinoma of the prostate? a. total prostate-specific antigen b. prostatic acid phosphatase c. human chorionic gonadotropin d. alpha-fetoprotein

a (Health Level 7 [HL-7] is the organization that specifies requirements for electronic data transmission formatting. It is considered to be the industry standard.)

253) A standard electronic file format recommended for transmitting data from the laboratory information system to an electronic medical record is: a. Health Level 7 b. ISBT 128 c. FTP d. SNOMED

d (Markers are good for monitoring therapy and detecting recurrence of tumors.)

253) Which of the following statements most correctly describes the utility of clinical laboratory assays for tumor markers? a. tumor markers are useful to screen asymptomatic patients for tumors b. tumor markers are highly specific c. tumor markers indicate the likelihood of an individual developing a tumor d. tumor markers are useful in tracking the efficacy of treatment

d (Patient results can be automatically released to the patient record as long as they meet selected criteria defined and validated by the users. Auto-verification does not apply to manually entered results and results cannot be released unless QC has been performed and was within limits prior to releasing.)

254) Auto-verification of test results requires all of the following to be established by the lab except: a. patient results entered in the LIS via an instrument interface b. patient results evaluated based on validated rules defined in the LIS c. successful quality control testing obtained prior to releasing patient results d. review of results by a qualified technologist or technician

d (CA 125 is elevated in 80% of epithelial cell ovarian cancer.)

254) Cancer antigen 125 is a tumor marker associated with: a. breast cancer b. colon cancer c. lung cancer d. ovarian and endometrial carcinoma

d (Since the auto control is positive after the AHG phase and no reactivity was detected at immediate spin, the serology is most consistent with a warm autoantibody. An adsorption with autologous cells to remove the antibody to used the adsorbed plasma for alloantibody detection is the next step.)

255) A 26-year-old female is admitted with anemia of undetermined origin. Blood samples are received with a crossmatch request for 6 units of Red Blood Cells, The patient is group A, Rh-negative and has no history of transfusion or pregnancy. The following results were obtained in pretransfusion testing: The best way to find compatible blood is to: a. do an antibody identification panel b. use the saline replacement technique c. use the pre-warm technique d. perform a warm autoadsorption

c (PSA can be elevated due to EPH.)

255) In addition to carcinoma of the prostate, elevated prostate-specific antigen can occur due to: a. aspirin therapy b. exogenous steroid use c. benign prostatic hyperplasia d. stain therapy (cholesterol lowering drug)

c (The CAP requires documentation that calculated values that generate a patient report are reviewed every 2 years, or when a system change is made that may affect the calculations. This requirement applies to values calculated by the LIS or middleware.)

255) Validation of calculated test results performed by a laboratory information system must be performed: a. every 6 months b. annually c. biannually d. only upon initial LIS installation

a (In a flow cytometer, forward scatter provides an estimate of cell volume or size.)

256) In flow cytometric analysis, low angle or forward scatter of a laser light beam provides information that pertains to a cell's: a. volume b. viability c. granularity d. lineage

d (The Host Query mode of an instrument is a two-way communication process whereby the instrument reads the specimen barcode and queries the LIS for the orders. The tests to be run are sent to the instrument [i.e. bidirectional communication]. Once results are obtained, they are sent back to the LIS.)

256) The hematology lab is evaluating new instruments for purchase. The supervisor wants to ensure that the instrument they select has directional interface capabilities. The instrument specification necessary to meet this requirement is: a. 9,600 baud rate b. on-board test selection menu c. HL-7 file format d. host query mode

b (Several Clostridium species are obligate anaerobes, and will only grow in an anaerobic environment. The growth of one of these Clostridium species would confirm that an anaerobic environment has been achieved in the anaerobic jar.)

257) A control strain of Clostridium should be used in an anaerobe jar to assure: a. that plate media is working b. that an anaerobic environment is achieved c. that the jar is filled with a sufficient number of plates d. that the indicator strip is checked

c (A steady-state therapeutic drug level is achieved between 4 and 7 doses. Many variables affect when steady state is achieved.)

257) A drug has a half-life of 6 hrs. If a dose is given every 6 hrs., a steady state drug level would usually be achieved in: a. 3 - 5 hrs. b. 10 - 12 hrs. c. 24 - 42 hrs. d. 48 - 50 hrs.

c (Test definitions, as well as all types of stored master files, are contained in the database.)

257) The Chem. Dept. has requested that a new test be defined in the LIS to run on the existing analyzer. The new test set up is completed by the LIS coordinator. A few days later, the accessioning department receives a request for the new test but an error is displayed when they try to place the order. All other tests can be successfully ordered. The most likely cause of the error is the: a. instrument interface for the Chem. analyzer is down b. test was not defined on the Chem. worklist c. database did not properly update with the new test information d. ADT interface with the hospital system is down

d (An elution is the process of removal of antibody from red blood cells. The product of the elution method is an eluate. The eluate contains the antibody and can be used in antibody identification methods.)

258) The process of separation of antibody from its antigen is known as: a. diffusion b. adsorption c. neutralization d. elution

a (Validation of the LIS is required by various accrediting agencies in order to prove that the system is performing as expected. This must be done at implementation and over time as changes are made to the LIS.)

258) The process of testing and documenting changes made to a laboratory information system is known as: a. validation b. quality engineering c. customization d. hazard analysis

c (System back-up procedures are mandated by accrediting agencies and include creating an exact copy of sensitive LIS data at regularly defined intervals and storage of the backed-up data at a separate location.)

259) Performance of laboratory information system back-up procedures includes all of the following except: a. creating an exact copy of LIS data b. off-site storage of the data media c. shutting down the LIS and bringing it back up d. completion at regularly defined intervals

a (Macroscopic characteristics and microscopic morphology are observations used for the routine identification of molds.)

259) The major features by which molds are routinely identified are: a. macroscopic characteristics and microscopic morphology b. biochemical reactions and microscopic morphology c. macroscopic characteristics and selective media d. specialized sexual reproductive structures

a (PCR reactions in which multiple loci are amplified in the same reaction by multiple sets of primers are known as multiplex PCR.)

26) A PCR reaction in which 4 different sets of primers are used to simultaneously amplify 4 distinct loci in the same reaction tube is known as a: a. multiplex PCR reaction b. heteroplex PCR reaction c. polyplex PCR reaction d. quadraplex PCR reaction

d (Whole Blood-derived platelets are stored at 20-24 C with continuous gentle agitation. Platelets prepared by the PRP method may be stored for up to 5 days.)

26) Platelets prepared in a polyolefin type container, stored at 22-24 C in 50 mL of plasma and gently agitated can be used for up to: a. 24 hrs. b. 48 hrs. c. 3 days d. 5 days

d (Urine is an appropriate specimen for the detection of renal tuberculosis. Since feces contain anaerobic organisms as part of the indigenous flora, it is an unacceptable specimen for anaerobic culture. Foley catheter tips are also not acceptable for culture, because they are contaminated with colonizing organisms. Gram stain smears of rectal swabs for N gonorrhoeae should also not be performed, since the presence of organisms with similar morphologies may lead to over interpretation of smears.)

26) Which of the following specimen requests is acceptable? a. feces submitted for anaerobic culture b. Foley catheter tip submitted for aerobic culture c. rectal swab submitted for direct smear for gonococci d. urine for culture of acid-fast bacilli

c (NAPA is the active metabolite of procainamide.)

260) The drug procainamide is prescribed to treat cardiac arrhythmia. What biologically active liver metabolite of procainamide is often measured simultaneously? a. phenobarbital b. quinidine c. n-acetyl procainamide d. lidocaine

d (The primary metabolite of cocaine is benzoylecgonine, which is produced by the liver and eliminated in the urine. Benzoylecgonine is detected in drugs of abuse screens for cocaine.)

261) Cocaine is metabolized to: a. carbamazepine b. codeine c. hydrocodone d. benzolylecgonine

b (Each unit of Whole Blood or RBCs will increase the hematocrit by 3%-5%, so 2 units of RBCs will increase the hematocrit by twice as much as 1 unit of Whole Blood.)

261) How would the hematocrit of a patient with chronic anemia be affected by the transfusion of a unit of Whole Blood containing 475 mL of blood versus 2 units of RBCs, each with a total volume of 250 mL? a. patient's hematocrit would be equally affected by the Whole Blood or the RBC's b. RBC's would provide twice the increment in hematocrit as the Whole Blood c. Whole Blood would provide twice the increment in hematocrit as the RBC's d. Whole Blood would provide a change in hematocrit slightly less than the RBC's

d (Histoplasma capsulatum is most frequently transmitted by inhalation.)

261) Many fungal infections are transmitted to man via inhalation of infectious structures. Which of the following is usual]y contracted in this manner? a. Sporothrix schenckii b. Trichophyton rubrum c. Malassezia furfur d. Histoplasma capsulatum

b (Calcofluor white binds to cellulose and chitin present in fungal cell walls.)

262) A smear of skin tissue reveals fluorescent septate hyphae. The smear was prepared using: a. acridine orange b. calcofluor white c. auramine rhodamine d. periodic acid-Schiff

c (For emergency transfusions, group O- RBC units should be used.)

262) After checking the inventory, it was noted that there were no units on the shelf marked "May Issue Uncrossmatched: For Emergency Only" Which of the following should be place on this shelf? a. 1 unit of each of the ABO blood groups b. units of group O, Rh-pos Whole Blood c. units of group O, Rh-neg RBC's d. any units that are expiring at midnight

d (11-nor-THC-COOH is the urinary metabolite of cannabinoids [marijuana and hashish].)

262) The metabolite 11-nor-tetrahydrocannabinol-9-COOH can be detected by immunoassay 3 -5 days after a single use of: a. methamphetamine b. cocaine c. benzodiazepine d. marijuana

c (Bone marrow and splenic macrophages contain hemosiderin; histiocyte is a collective term for macrophages.)

264) Which of the following cells contain hemosiderin? a. megakaryocyte b. osteoclast c. histiocyte d. mast cell

c (Lithium [carbonate] is used to treat manic depression or bipolar disorder.)

265) A carbonate salt used to control manic depressive disorders is: a. digoxin b. acetaminophen c. lithium d. phenytoin

a (Cryptococcus neoformans is a cause of meningitis in immunocompromised patients, and produces a polysaccharide capsule.)

265) An organism that demonstrates budding yeast cells with wide capsules in an India ink preparation of spinal fluid is probably: a. Cryptococcus neoformans b. Histoplasma capsulatum c. Blastomyces dermatitidis d. Candida albicans

b (Each unit of RBCs is expected to increase the hematocrit level by 3%-5%, so it would take 2 units to raise the level 6%.)

265) How many units of RBCs are required to raise the hematocrit of a 70 kg nonbleeding man from 24% to 30% a. 1 b. 2 c. 3 d. 4

a (Megakaryocytes are the largest cell in the bone marrow.)

265) Which of the following cells is the largest cell in the bone marrow? a. megakaryocyte b. histiocyte c. osteoblast d. mast cell

d (Phenytoin [trade name Dilantin M] is an anticonvulsant therapeutic drug used to treat seizure disorders.)

266) An antiepileptic used to control seizure disorders is: a. digoxin b. acetaminophen c. lithium d. phenytoin

b (Average life span of platelets in peripheral blood is 9.5 days.)

266) Normal platelets have a circulating life-span of approximately: a. 5 days b. 10 days c. 20 days d. 30 days

d (Candida albicans produces germ tubes.)

266) The formation of germ tubes presumptively identifies: a. Candida tropicalis b. Candida parapsilosis c. Candida glabrata d. Candida albicans

d (The action of the drug theophylline is bronchodilation and smooth muscle relaxation.)

267) A drug that relaxed the smooth muscles of the bronchial passages is: a. acetaminophen b. lithium c. phenytoin d. theophylline

a (Cryptococcus neoformans is a cause of meningitis in immunocompromised patients, and produces a polysaccharide capsule.)

267) An HIV-positive patient began to show signs of meningitis. A spinal fluid was collected and cultured for bacteria and fungus. A budding, encapsulated yeast was recovered. Which organism is consistent with this information? a. Cryptococcus neoformans b. Aspergillus fumigatus c. Microsporum audouinii d. Sporothrix schenckii

a (Digoxin at therapeutic serum levels [O.5-1.5 ng/mL] improves cardiac muscle contraction and rhythm.)

268) A cardiac glycoside that is used in the treatment of congenital heart failure and arrhythmias by increasing the force and velocity of myocardial contraction is: a. digoxin b. acetaminophen c. lithium d. phenytoin

d (Class I HLA antigens on platelets are a known cause for platelet refractoriness. Leukoreduction of blood products is used as a mechanism to reduce or prevent patients from developing antibodies.)

268) Transfusion of plateletpheresis products for HLA-compatible donors is the preferred treatment for: a. recently diagnosed cases of TTP with severe thrombocytopenia b. acute leukemia in relapse with neutropenia, thrombocytopenia and sepsis c. immune thrombocytopenic purpura d. severely thrombocytopenic patients, known to be refractory to random donor platelets

c (Cryptococcus may not stain well on Gram stain. Rapid identification is important when yeast is detected in CSF, the presence of capsules seen in an India ink preparation would be presumptive for C neoformans.)

269) Gram stain examination of a CSF specimen indicates the presence of yeast-like cells with gram-positive granular inclusions. Which of the following techniques should be used next to assist in the identification of this organism? a. 10% KOH b. lactophenol cotton blue c. India ink d. periodic acid-Schiff

b (Aspirin interferes with prostaglandin metabolism in the platelet; see answer #267. answer 267 - Aspirin interferes with prostaglandin metabolism in the platelet by inhibiting cyclooxygenase, which participates in the conversion of arachidonic acid to protein G2; protein G2 is necessary to produce thromboxane, which stimulates secretion from the platelet granules.)

269) Platelet activity is affected by: a. calcium b. aspirin c. hyperglycemia d. hypoglycemia

d (TPO [thyroid peroxidase] is the microsomal antigen of the thyroid epithelial cell. TPO antibodies are positive for about 90% of patients with chronic thyroiditis.)

27) High titers of antimicrosomal antibodies are most often found in: a. rheumatoid arthritis b. SLE c. chronic hepatitis d. thyroid disease

a (Excessive loss of bicarbonate.)

27) Severe diarrhea causes: a. metabolic acidosis b. metabolic alkalosis c. respiratory acidosis d. respiratory alkalosis

c (Both osmolality and specific gravity measure solutes in a solution.)

27) Urine osmolality is related to: a. pH b. filtration c. specific gravity d. volume

b (RBC membrane defects are common to PNH, HS, and HE.)

27) Which of the following characteristics are common to hereditary spherocytosis, hereditary elliptocytosis, hereditary stomatocytosis, and paroxysmal nocturnal hemoglobinuria? a. autosomal dominant inheritance b. red cell membrane defect c. positive DAT d. measured platelet count

c (Materials collected from sites not harboring indigenous flora [sterile body fluids, abscess exudate and tissue] should be cultured for anaerobic bacteria. However, since anaerobes normally inhabit the skin and mucus membranes as part of the indigenous flora, specimens such as urine, sputum, and vaginal, eye and ear swabs are not acceptable for culture.)

27) Which of the following groups of specimens would be acceptable for anaerobic culture? a. vaginal, eye b. ear, leg tissue c. pleural fluid, brain abscess d. urine, sputum

d (FFP should be ABO compatible with the recipient's RBCs. Avoid FFP with antibodies to A or B antigens the patient may have. Group A plasma has anti-B, and should only be transfused to A or O recipients.)

270) Which of the following is consistent with standard blood bank procedure governing the infusion of FFP? a. only blood group specific plasma may be administered b. group O may be administered to recipients of all blood groups c. group AB may be administered to AB recipients only d. group A may be administered to both A and O recipients

d (FFP should be ABO compatible with the recipient's RBCs. Avoid FFP with ABO antibodies to A or B antigens the patient may have.)

271) A patient who is group AB, Rh-neg needs 2 units of FFP. Which of the following units of plasma would be most acceptable for transfusion? a. group O, Rh-neg b. group A, Rh-neg c. group B, Rh-pos d. group AB, Rh-pos

d (Conventional tests, such as carbohydrate assimilation, must be performed for definitive identification of the yeast in this CSF specimen, especially in light of the fact that the yeast is both cryptococcal antigen and germ tube negative.)

272) A yeast isolate from a CSF specimen produced the following results - India ink: no encapsulated yeast cells - cryptococcal antigen: negative - urea: negative - germ tube: negative What should the technologist do next to identify this organism? a. inoculate bird seed agar b. ascospore stain c. cycloheximide susceptibility d. carbohydrate assimilation

a (Cycloheximide is known to inhibit the growth of some fungal pathogens, including Cryptococcus neoformans.)

273) The recovery of some Cryptococcus species may be compromised if the isolation media contains: a. cycloheximide b. gentamicin c. chloramphenicol d. penicillin

c (Measurement of whole blood lead is the recommended test for children. In adults higher lead levels are significant; therefore other methods, such as erythrocyte protoporphyrin and delta- aminolevulinic acid, are acceptable for adults.)

274) Testing for the diagnosis of lead poisoning should include: a. erythrocyte protoporphyrin b. urine delta-aminolevulinic acid c. whole blood lead d. zinc protoporphyrin

b (Anaphylactic transfusion reactions are distinguished from other types of reactions by 1] the absence of fever, and 2] the reactions are sudden in onset after infusion of only a few mL of blood. Since the reaction is due to anti-IgA, washing the donor red blood cells to remove all plasma protein is indicated. Alternatively, blood products from IgA-deficient donors may be used.)

275) A patient received about 15 mL of compatible blood and developed severe shock, but no fever. If the patient needs another transfusion, what kind of RBC component should be given? a. RBCs b. RBCs, washed c. RBCs, Irradiated d. RBCs, leukocyte reduced

c (Preanalytical interferences with blood gases.)

275) Blood received in the lab for blood gas analysis must meet which of the following requirements? a. on ice, thin fibrin strands only, no air bubbles b. on ice, no clots, fewer than 4 air bubbles c. on ice, no clots, no air bubbles d. room temp, no clots, no air bubbles

a (FFP should be ABO compatible with the recipient's RBCs. Avoid FFP with ABO antibodies to A or B antigens the patient may have. Rh type is not significant.)

276) FFP from a group A, Rh-pos donor may be safely transfused to a patient who is group: a. A, Rh-neg b. B, Rh-neg c. AB, Rh-pos d. AB, Rh-neg

c (FFP should be ABO compatible with the recipient's RBCs. If patient's type has not been determined [currently], plasma lacking anti-A and anti-B should be given.)

277) A patient admitted to the trauma unit requires the emergency release of FFP. His blood donor card states that he is group AB, Rh-pos. Which of the following blood groups of FFP should be issued? a. A b. B c. AB d. O

c (FFP contains all factors, including cryoprecipitate. It does not have a higher risk of transmitting hepatitis than Whole Blood. It must be transfused within 24 hours of thawing and must be ABO compatible.)

278) Fresh Frozen Plasma: a. contains all labile coagulative factors except cryo AHF b. has a higher risk of transmitting hepatitis than does Whole Blood c. should be transfused within 24 hours of thawing d. need not be ABO compatible

c (Cryptococcal latex antigen test has been proven to be significantly more sensitive than staining methods.)

278) The most sensitive test for the initial diagnosis of cryptococcal disease is: a. India ink b. Gram stain c. cryptococcal antigen d. Giemsa stain

c (Each unit of platelets should increase the count 5,000-10,000 platelets/uL [5,000- 10,000/L]. Platelet antibodies can diminish this expected increment.)

279) 10 units of group A platelets were transfused to a group AB patient. The pretransfusion platelet count was 12 x 10^3/uL and the posttransfusion count was 18 x 10^3/uL. From this information the laboratorian would most likely conclude that the patient: a. needs group AB platelets to be effective b. clinical data does not suggest a need for platelets c. has developed antibodies to the transfused platelets d. should receive irradiated platelets

a (Polycythemia Vera is characterized by increased WBC, RBC, and platelet counts.)

279) In polycythemia vera, the platelet count is: a. elevated b. normal c. decreased d. variable

c (Candida albicans [positive] and C tropicalis [negative] can be used as control organisms for quality control of the germ tube test.)

279) Which of the following statements concerning the germ tube test is true? a. using a heavy inoculum enhances the rapid production of germ tubes b. germ tubes should be read after 2 hours incubation at 25 C c. Candida albicans and Candida tropicalis can be used as positive and negative controls, respectively d. serum will be stable for 1 year if stored at 4 prior to use

b (Campylobacter continues to be the most common enteric pathogen isolated from patients with diarrhea. Routinely fecal specimens should be cultured for Salmonella, Shigella and Campylobacter. Fecal specimens are not routinely cultured for enterotoxigenic E coli or C botulinum. E hartmanni is a nonpathogenic parasite and does not cause diarrhea.)

28) A liquid fecal specimen from a 3 month old infant is submitted for microbiological exam. In addition to culture on routine media for Salmonella and Shigella, this specimen should be routinely: a. examined for the presence of Entamoeba hartmanni b. examined for the presence of Campylobacter sp. c. screened for the detection of enterotoxigenic Escherichia coli d. placed in thioglycollate broth to detect Clostridium botulinum

d ("GEN 41096: Does the paper or electronic report include the following elements? 1. Name and address of testing laboratory[see note below] 2. Patient name and identification number, or unique patient identifier 3. Name of physician of record, or legally authorized person ordering test, as appropriate 4. Date and time of specimen collection, when appropriate 5. Date of release of report [if not on the report, this information should be readily accessible] 6. Time of release of report, if applicable [if not on the report, this information should be readily accessible] 7. Specimen source, when applicable 8. Test result[s] [and units of measurement, when applicable] 9. Reference intervals, as applicable [see note below] 10. Conditions of specimen that may limit adequacy of testing.")

28) A paper or electronic report of lab results must include: a. the name of the person who collected the specimen b. the test price c. a pathologist's signature d. the name and address of the testing laboratory

b (Specific gravity gives the concentration for the sample relative to water. It does not give specific information about H+ or Na+ ions.)

28) Urine specific gravity is an index of the ability of the kidney to: a. filter the plasma b. concentrate the urine c. alter the hydrogen ion concentration d. reabsorb sodium ions

d (Sodium fluoride exerts its preservative action by inhibiting the enzyme systems involved in glycolysis.)

280) ln a specimen collected for plasma glucose analysis, sodium fluoride: a. serves as a coenzyme of hexokinase b. prevents reactivity of non-glucose reducing substances c. precipitates proteins d. inhibits glycolysis

b (A high-fat diet increases the serum concentrations of triglycerides. Fasting overnight for 10-14 hours is the optimal time for fasting around which to standardize blood collections, including lipids.)

281) As part of a hyperlipidemia screening program, the following results were obtained on a 25 yr. old woman 6 hrs. after eating: triglycerides = 260 mg/dL cholesterol = 120 mg/dL Which of the following is the best interpretation of these results? a. both results are normal and not affected by the recent meal b. cholesterol is normal, but triglycerides are elevated, which may be attributed to the recent meal c. both results are elevated, indicating a metabolic problem in addition to the nonfasting state d. both results are below normal despite the recent meal, indicating a metabolic problem

d (The arthroconidia of Coccidioides immitis are highly infectious; cultures must be handled with care to minimize aerosols.)

281) Lab workers should always work under a biological safety hood when working with cultures of: a. Streptococcus pyogenes b. Staphylococcus aureus c. Candida albicans d. Coccidioides immitis

c (mucous membrane hemorrhage is typical of platelet disorders; remaining choices are typical of coagulation factor disorders.)

282) Which of the following is characteristic of platelet disorders? a. deep muscle hemorrhage b. retroperitoneal hemorrhage c. mucous membrane hemorrhages d. severely prolonged clotting times

a (The presence or exposure of excess gas [oxygen] in the syringe used to collect blood gas specimens will cause diffusion of carbon dioxide out of the specimen, oxygen into the specimen, and an increase in pH.)

283) Arterial blood that is collected in a heparinized syringe but exposed to room air would be most consistent with the changes in which of the following specimens? a. specimen A b. specimen B c. specimen C d. specimen D

a (Giant platelets, abnormal bleeding time, normal aggregation with ADP, decreased platelet count are characteristic of Bernard-Soulier.)

283) Which of the following is characteristic of Bernard - Soulier syndrome? a. giant platelets b. normal bleeding time c. abnormal aggregation with ADP d. increased platelet count

d (Previously immunized patients may have an undetectable level of antibody. Transfusion of antigen-positive donor red cells may cause an anamnestic response and result in a delayed hemolytic transfusion reaction. Symptoms may be mild, and present only as jaundice and unexplained anemia.)

284) An unexplained fall in hemoglobin and mild jaundice in a patient transfused with RBCs 1 week previously would most likely indicate: a. paroxysmal nocturnal hemoglobinuria b. posttransfusion hepatitis infection c. presence of HLA antibodies d. delayed hemolytic transfusion reaction

a (Dimorphism differentiates Sporothrix from the other fungi listed.)

285) Which of the following is a dimorphic fungus? a. Sporothrix schenckii b. Candida albicans c. Cryptococcus neoformans d. Aspergillus fumigatus

a (Manual chamber counts do not have a high degree of precision or reproducibility, and require a significant amount of time.)

287) The chamber counting method of platelet enumeration: a. allows direct visualization of the particles being counted b. has a high degree of precision c. has a high degree of reproducibility d. is the method of choice for the performance of 50 - 60 counts per day

a (Platelet count matches estimate well [average of 17.8 platelets per oil immersion field x 20,000].)

289) A phase-platelet count was performed and the total platelet count was 356 x 10^3/mL (356 x 10^9/L). 10 fields on the stained blood smear were examined for platelets and the results per field were: 16, 18, 15, 20,19, 17, 19, 18, 20, 16 The next step would be to: a. report the phase-platelet count since it correlated well with the slide b. repeat the phase-platelet count on a recollected specimen and check for clumping c. check ten additional fields on the blood smear d. repeat the platelet count using a different method

a (ABO antibodies activate complement and may cause intravascular hemolysis. The antibodies are naturally occurring against A and B antigens that the recipient lacks. Rh and Duffy antibodies may also cause hemolytic transfusion reactions, but the antibodies are the results of alloimmunization and not naturally present in recipients who lack the antigen. The incidence of septic transfusion reactions from bacterial contamination of Red Blood Cells is rare, about 1:500,000.)

289) After receiving a unit of RBCs, a patient immediately developed flushing, nervousness, fever spike of 102*F, shaking, chills and back pain. The plasma hemoglobin was elevated and there was hemoglobinuria. Laboratory investigation of this adverse reaction would most likely show: a. an error in ABO grouping b. an error in Rh typing c. presence of anti-Fya in patient's serum d. presence of gram-neg bacteria in blood bag

c (Enriched media such as chocolate agar has no inhibitory effects on bacterial growth and contains additional nutrients that support the growth of fastidious organisms such as H influenzae and Neisseria.)

29) Cerebrospinal fluid from a febrile 25 yr. old man with possible meningitis is rushed to the lab for a stat gram stain and culture. While performing the gram stain, the tech accidently spills most of the specimen. The smear shows many neutrophils and no microorganisms. Since there is only enough CSH to inoculate one plate, the tech should use a: a. blood agar plate b. chopped meat glucose c. chocolate agar plate d. Thayer-Martin plate

c (Cryoprecipitate has a shelf life of 12 months in the frozen state.)

29) Cryo AHF, if maintained in the frozen state at -18 C or below, has a shelf life of: a. 42 days b. 6 months c. 12 months d. 36 months

a (Blood gas instrumentation.)

29) Factors that contribute to a PCO2 electrode requiring 60-120 seconds to reach equilibrium include the: a. diffusion characteristics of the membrane b. actual blood PO2 c. type of calibrating standard d. potential of the polarizing mercury cell

b (Chronic Active Hepatitis [CAB] has at least 2 subsets, the classic or type I, which is associated with a positive ANA test and positive smooth muscle antibodies. The condition is associated with an attack on the hepatocytes.)

29) In chronic active hepatitis, high titers of which of the following antibodies are seen? a. antimitochondrial b. anti-smooth muscle c. anti-DNA d. anti-parietal cell

a (Osmolality measures the number of particles in a solution. Salts will dissociate into ions, and each ion contributes to the osmolality.)

29) Osmolality is a measure of: a. dissolved particles, including ions b. undissociated molecules only c. total salt concentration d. molecule size

a (52 mg/dL = 0.52 mg/mL. Therefore 0.52 mg/mL x 1136 mL = 591 mg.)

290) A 24 hr. urine specimen (total volume = 1,136 mL) is submitted to the lab for quantitative urine protein. Calculate the amount of protein excreted per day, if the total protein is 52 mg/dL a. 591 mg b. 487 mg c. 220 mg d. 282 mg

d (Patients receiving >1 blood volume replacement often develop thrombocytopenia and require platelet transfusion.)

290) A trauma patient who has just received 10 units of blood may develop: a. anemia b. polycythemia c. leukocytosis d. thrombocytopenia

d (Platelet clumping is reduced by collecting sample in sodium citrate.)

290) An automated platelet count indicates platelet clumping, which is confirmed by examining the smear. The technician should: a. repeat the count on the same sample b. report the automated count c. perform a manual count d. recollect in sodium citrate

d (Platelet satellitism is reduced by collecting sample in sodium citrate.)

291) The automated platelet count on an EDTA specimen is 58 x 10^3/mL (58 x 10^9/L). The platelet estimate on the blood smear appears normal, but it was noted that the platelets were surrounding the neutrophils. The next step should be to: a. report the automated platelet count since it is more accurate than a platelet estimate b. warm the EDTA tube and repeat the automated platelet count c. rerun the original specimen since the platelet count and blood smear estimate do not match d. recollect a specimen for a platelet count using a different anticoagulant

a (Creatinine clearance = [urine creatinine x urine volume [mL/min]/serum creatinine] x 1.73 / total body surface.)

291) The following results were obtained: urine creatinine = 90 mg/dL serum creatinine = 0.90 mg/dL total body surface = 1.73 m total urine in 24 hrs. = 1500 mL Given the above data, the patient's creatinine clearance in mL/min is: a. 104 b. 124 c. 144 d. 150

a (Creatinine clearance = [urine creatinine x urine volume [mL/min]/serum creatinine] x 1.73 / total body surface.)

292) A 45 yr. old male of average height and weight was admitted to the hospital for renal function studies. He had the following lab results: urine creatinine = 120 mg/dL serum creatinine = 1.5 mg/dL total volume = 1800/mL Calculate the creatinine clearance for this patient in mL/min: a. 100 b. 144 c. 156 d. 225

a (Free hemoglobin released from destruction of transfused donor red cells will impart a distinct pink or red color in the posttransfusion sample plasma.)

292) The most appropriate laboratory test for early detection of acute posttransfusion hemolysis is: a. a visual inspection for free plasma hemoglobin b. plasma haptoglobin concentration c. examination for hematuria d. serum bilirubin concentration

d (The immediate steps required to investigate a transfusion reaction include a clerical check of records and labels, visual inspection of postreaction plasma for hemolysis, and direct antiglobulin test and repeat ABO typing on the postreaction sample. Additional investigation is performed when there is evidence of hemolysis, bacterial contamination, TRALI, or other serious adverse event.)

293) During initial investigation of a suspected hemolytic transfusion reaction, it was observed that the posttransfusion serum was yellow in color and the DAT was negative. Repeat ABO typing on the posttransfusion sample confirmed the pretransfusion results. What is the next step in this investigation? a. repeat compatibility testing on suspected units b. perform plasma hemoglobin and haptoglobin determinations c. use enhancement media to repeat the antibody screen d. no further serological testing is necessary

a (In septic transfusion reactions, patients experience fever >101 [38.3 C], shaking chills, and hypotension. In severe reactions, patients develop shock, renal failure, hemoglobinuria, and DIC.)

294) Which of the following transfusion reactions is characterized by high fever, shock, hemoglobinuria, DIC, and renal failure? a. bacterial contamination b. circulatory overload c. febrile d. anaphylactic

b (BT assesses platelet number and function; platelet count only assesses platelet number; PT and APTT do not assess platelet number or function.)

295) A patient has been taking aspirin regularly for arthritic pain. Which of the following tests is most likely to be abnormal in this patient? a. platelet count b. template bleeding time c. prothrombin time d. activated partial thromboplastin time

c (Clinical signs of a hemolytic transfusion reaction include fever and chills, and, in severe cases, DIC. Circulatory overload, allergic and anaphylactic reactions are not characterized by fever and DIC.)

295) Hemoglobinuria, hypotension and generalized bleeding are symptoms of which of the following transfusion reactions? a. allergic b. circulatory overload c. hemolytic d. anaphylactic

c (Zygomycetes grow rapidly and fill the dish with cotton candy-like growth.)

295) On day 3 of a fungal culture, grayish cottony growth is observed that is filling the container. The most likely mold isolated is a: a. dermatophyte b. dimorphic mold c. zygomycete d. dematiaceous mold

c (BT assesses both platelet number and function.)

296) A platelet count done by phase microscopy is 200 x 10^3/mL (200 x 10^9/L) (reference range 150-450 x 10^3/mL (150-450 x 10^9/L). A standardized template bleeding time on the same person is 15 minutes (reference range 4.5 +/- 1.5 minutes). This indicates that: a. the Duke method should have been used for the bleeding time b. the manual platelet count is in error c. abnormal platelet function should be suspected d. the results are as expected

a (Bilirubin is a marker for red cell hemolysis. Bilirubin peaks at 5-7 hours after transfusion and is back to pretransfusion levels at 24 hours if liver function is normal.)

296) When evaluating a suspected transfusion reaction, which of the following is the ideal sample collection time for a bilirubin determination? a. 6 hrs. posttransfusion b. 12 hrs. posttransfusion c. 24 hrs. posttransfusion d. 48 hrs. posttransfusion

d (Delayed hemolytic transfusion reactions may occur in recipients who are previously immunized but who do not have detectable antibody, if they receive blood with the corresponding antigen. When there is a history of clinically significant antibodies, donor red cells should be phenotyped and antigen-negative blood selected. A complete antiglobulin crossmatch must be performed.)

297) A patient's records show a previous anti-Jkb, but the current antibody screen is negative. What further testing should be done before transfusion? a. phenotype the patient's red cells for Jkb antigen b. perform a cell panel on the patient's serum c. crossmatch type specific units and release only compatible units for transfusion d. give Jkb neg crossmatch compatible blood

d (Because the following relationship is true, A = light stopped and T - light passed through, A and T are inversely related. They are also logarithmically related, because the absorption of light is a logarithmic function.)

297) Which of the following is the formula for calculating absorbance given the percent transmittance of a solution? a. 1 - log(%T) b. log(%T)/2 c. 2 x log(%T) d. 2 - log(%T)

b (If the direct antiglobulin test is positive in a transfusion reaction investigation, the antibody should be eluted from the red cells and identified.)

298) A posttransfusion blood sample from a patient experiencing chills and fever show distinct hemolysis. The DAT is positive (mixed field). What would be the most helpful to determine the cause of the reaction? a. auto control b. elution and antibody ID c. repeat antibody screen on the donor unit d. bacteriologic smear and culture

a (Calcium is required for platelet aggregation.)

298) Platelet aggregation is dependent in vitro on the presence of: a. calcium ions b. sodium citrate c. thrombin d. potassium

d (The Henderson-Hasselbach equation describes the derivation of pH as a measure of acidity [using the acid dissociation constant, pKa] in biological and chemical systems.)

298) Which of the following is the Henderson Hasselbach equation? a. pKa=pH+log([acid]/[salt]) b. pKa=pH+log([salt]/[acid]) c. pH=pKa+log([acid]/[salt]) d. pH=pKa+log([salt]/[acid])

c (Lack of expected rise in hemoglobin after transfusion may be a sign of a delayed hemolytic transfusion reaction. If the DAT is positive, an elution should be performed to remove and identify the antibody coating the transfused donor red cells. In this case, the antibody is not detectable in the antibody screen, so a routine cell panel on the serum would not be helpful. Since the transfusion occurred 3 weeks previously, donor samples are not available for testing.)

299) A patient is readmitted to the hospital with a hemoglobin level of 7 g/dL (70 g/L) 3 weeks after receiving 2 units of red cells. The initial serological tests are: Which test should be performed next? a. antibody identification panel on the patient's serum b. repeat the ABO type on the donor units c. perform an elution and identify the antibody in the eluate d. crossmatch the post reaction serum with the 3 donor units

b (Renal clearance tests are used to assess kidney function. Renal clearance of a substance is a rate measurement that expresses the volume of blood cleared of that substance [typically creatinine] per unit of time. The unit for the clearance is mL/ min. To calculate creatinine clearance, the following information is required: Serum concentration [S], urine concentration [U] [Note: the serum and urine concentration must be in the same units, for example, mg/L or mg/dL], and volume of urine excreted per minute [V] [volume of urine collected divided by the time period in minutes]. The following formula can then be used: clearance [uncorrected for body mass] = [U] x V/[S].)

299) The creatinine clearance is equal to: a. urinary creatinine (mg/L) / [volume of urine (mL/min) x plasma creatinine (mg/L)] b. [urinary creatinine (mg/L) x volume (mL/min) / plasma creatinine (mg/L)] c. urinary creatinine (mg/L) / [volume of urine (mL/hour) x plasma creatinine (mg/L)] d. [urinary creatinine (mg/L) x volume (mL/hour)] / plasma creatinine (mg/L)

b (Liquid medium is recommended to provide more rapid recovery of AFB than solid medium. Isolation rates of mycobacteria is also higher with liquid medium.)

316) An AFB broth culture is positive for acid-fast bacilli at 1 week while the agar slant shows no growth. The most likely explanation for this is: a. the organism is a contaminant b. AFB grow more rapidly in liquid media c. PANTA was added to the broth d. the agar slant was incubated in 5% CO2

b (It is common practice to share samples between the microbiology department and urinalysis. Ideally, the culture is set up first to prevent contamination. If that is not feasible time wise, the sample should be aliquotted using aseptic technique, and refrigerated until it can be cultured.)

3) A clean catch urine is submitted to laboratory for routine urinalysis and culture. The routine urinalysis is done first, and 3 hrs. later, the specimen is sent to the microbiology department for culture. The specimen should: a. be centrifuged, and the supernatant culture b. be rejected due to the time delay c. not be cultured if no bacteria are seen d. be processed for culture only if the nitrate is positive

c (The focus must center on a root cause analysis of such errors to prevent them from occurring in the clinical setting.)

3) A swab and requisition are received in the microbiology laboratory with the following information: The laboratory receiving clerk notes a discrepancy between the patient identifiers on the requisition and the specimen label, and questions the emergency department nurse MB. This is the fourth specimen this week that has had a specimen discrepancy. What action is the most appropriate to prevent this type of error in the future? a. correct the error in the laboratory when the nurse communicates the appropriate patient name b. ask nurse MB to correct discrepancy and read the procedure for specimen labeling c. document error and communicate to laboratory supervisor so that the risk manager conducts a root cause analysis d. discard the specimen and call the emergency department for a recollect

a (Heteronuclear RNA is made up of newly transcribed mRNA that still contains intervening sequences.)

3) Intervening sequences are found in: a. heteronuclear RNA b. mature mRNA c. ribosomal RNA d. transfer RNA

a (The MIC is a basic laboratory measurement of the activity of an antibiotic against an organism. It is the lowest concentration of antibiotic that inhibits visible growth of the organism. It does not represent the concentration of antibiotic that is lethal to the organism.)

3) The lowest concentration of antibiotic that inhibits growth of a test organism is the : a. minimum inhibitory concentration b. serum inhibitory concentration c. minimum bactericidal titer d. minimum inhibitory titer

a (2/3 iron in body bound to Hgb.)

3) The majority of the iron in an adult is found as a constituent of: a. hemoglobin b. hemosiderin c. myoglobin d. transferrin

a (GTT diet preparation.)

3) The preparation of a patient for standard glucose tolerance testing should include: a. a high carbohydrate diet for 3 days b. a low carbohydrate diet for 3 days c. fasting for 48 hrs. prior to testing d. bed rest for 3 days

c (Sm is characteristic of a speckled pattern.)

3) Which of the ANA patterns shown above would be associated with high titers of antibodies to the Sm antigen? a. diagram A b. diagram B c. diagram C d. diagram D

c (Analyzers fitted with ion-selective electrodes usually contain potassium electrodes with liquid ion-exchange membranes that incorporate valinomycin. Valinomycin is a neutral carrier [ionophore] that binds potassium in the center of a ring of oxygen atoms.)

327) Valinomycin enhances the selectivity of the electrode used to quantitate: a. sodium b. chloride c. potassium d. calcium

a (CNA agar is a selective medium commonly used in the isolation of gram-positive aerobic and anaerobic organisms. Since the Gram stain indicates a mixture of gram-positive and gram-negative organisms, use of CNA will aid in the recovery of the gram-positive cocci in culture.)

30) A diabetic foot swab from a 82-year-old woman with recurrent infections is submitted for culture. The Gram stain reveals: - many neutrophils, no squamous epithelial cells - many gram-negative bacilli - many gram-positive cocci in chains The physician requests that all pathogens be worked up. In addition to the sheep blood and MacConkey agar plates routinely used for wound cultures, the technologist might also process a(n): a. CNA agar plate b. chocolate agar plate c. XLD agar plate d. chopped meat glucose

d (Chronic blood loss frequently results in iron deficiency anemia.)

30) Which of the following is most closely associated with iron deficiency anemia? a. iron overload in tissue b. target cells c. basophilic stippling d. chronic blood loss

b (Delayed hemolytic transfusion reactions are associated with extravascular hemolysis, rather than intravascular. Alloantibody coats the transfused antigen-positive donor cells in the recipient's circulation, producing a mixed-field positive reaction in the DAT.)

300) In a delayed hemolytic transfusion reaction, the DAT is typically: a. negative b. mixed-field positive c. positive due to complement d. negative when the antibody screen is negative

c (Red Blood Cells, Leukocyte-Reduced should be chosen, because febrile nonhemolytic transfusion reactions are either due to chemokines released from leukocytes in nonleukoreduced blood components or to patient antibodies directed towards donor HLA antigens on the leukocytes.)

303) A patient multiply transfused with RBCs developed a headache, nausea, fever and chills during his last transfusion. What component is most appropriate to prevent this reaction in the future? a. RBCs b. RBCs, irradiated c. RBCs, leukocyte reduced d. RBCs selected as CMV-reduced-risk

a (Leukocyte antibodies are a primary cause of febrile transfusion reactions. Leukocyte-reduced blood components reduce the risk of febrile nonhemolytic reactions.)

305) Leukocyte-poor RBCs would most likely be indicated for patients with a history of : a. febrile transfusion reaction b. iron deficiency anemia c. hemophilia A d. von Willebrand disease

d (Many species of mycobacteria are slow growing, and may take up to 12 weeks for growth.)

306) A first morning sputum is received for culture of mycobacteria. It is digested and concentrated by the N-acetyl-L-cysteine alkali method. Two Lowenstein-Jensen slants are incubated in the dark at 35 C with 5%-10% CO2. The smears reveal acid-fast bacilli, and after 7 days no growth appears on the slants. The best explanation is: a. improper specimen submitted b. incorrect concentration procedure c. exposure to CO2 prevents growth d. cultures held for insufficient length of time

c (Given the values of bicarbonate, carbonic acid, and the pK, the pH can be easily calculated using the Henderson- Hasselbach equation. The Henderson- Hasselbach equation describes the derivation of pH as a measure of acidity [using the acid dissociation constant, pKa] in biological and chemical systems.)

306) Normally the bicarbonate concentration is about 24 mEq/L and the carbonic acid concentration is about 1.2; pK=6.1, log 20 = 1.3. Using the equation pH = pK + log[salt]/[acid]. calculate the pH: a. 7.28 b. 7.38 c. 7.40 d. 7.42

a (Primary drugs for the treatment of Mycobacterium tuberculosis include isoniazid, rifampin, pyrazinamide, and ethambutol.)

329) Which of the following is considered a primary drug for the treatment of Mycobacterium tuberculosis? a. rifampin b. kanamycin c. rifabutin d. ethionamide

c (Vitamin K is present in green vegetables, fish, liver and tobacco and synthesized by bacteria in the intestine. Naturally occurring vitamin K is fat soluble. Warfarin is the most popular vitamin K antagonist. Heparin inhibits Factor Xa and thrombin.)

306) Which of the following statements concerning Vitamin K is NOT true? a. there are 2 sources of Vit. K: vegetables and bacterial b. Vit. K converts precursor molecules into functional coagulation factors c. heparin inhibits the action of Vit. K d. Vit. K is fat soluble

d (Specimens contaminated by normal flora must be decontaminated prior to inoculation of media; NaOH is the most common decontaminant.)

307) A first morning sputum specimen is received for acid-fast culture. The specimen is centrifuged, and the sediment is inoculated on 2 Lowenstein-Jensen slants, which are incubated at 35 C in 5%-10% CO2. After 1 week, the slants show abundant growth over the entire surface. Stains reveal gram-negative bacilli. To avoid this problem: a. utilize a medium that inhibits bacterial growth b. add sodium hypochlorite to the sediment before inoculation c. incubate the tubes at room temperature to retard bacterial growth d. decontaminate the specimen with sodium hydroxide

a (Sabouraud dextrose is fungal media; mycobacterial media should have been inoculated.)

308) A first morning sputum is received for culture of acid-fast bacilli. It is digested and concentrated by the N-acetyl-L-cysteine alkali method. Two Sabouraud dextrose slants are incubated in the dark at 35 C with 5%-10% CO2. The smears reveal acid-fast bacilli, but the slants show no growth after 8 weeks. The explanation is: a. improper media used b. incorrect concentration procedure used c. improper specimen submitted d. exposure to C02 prevents growth

d (Febrile nonhemolytic transfusion reactions are defined as fever of 1C or greater [over baseline temperature] during or after transfusion, with no other reason for the elevation than transfusion, and no evidence of hemolysis in the transfusion reaction investigation. Allergic reactions, citrate toxicity, and circulatory overload are not characterized by fever.)

308) Fever and chills are symptoms of which of the following transfusion reactions? a. citrate toxicity b. circulatory overload c. allergic d. febrile

c (The extrinsic pathway is initiated by the release of tissue thromboplastin that has been expressed after damage to a vessel Factor VII forms a complex with tissue thromboplastin and calcium. Factors II and VII are found in the common pathway, and Factor VIII is in the intrinsic pathway.)

308) Which of the following factors is used only in the extrinsic coagulation pathway? a. II b. V c. VII d. VIII

d (Factor XII is a contact factor, which is activated to Factor XIla. It is responsible for the activation of Factor XI to Factor XIa. Fibrinogen is converted to fibrin by the action of thrombin.)

309) Hageman Factor (XII) is involved in each of the following reactions except: a. activation of C1 to C1 esterase b. activation of plasminogen c. activation of Factor XI d. transformation of fibrinogen to fibrin

b (Allergic reactions are a type 1 immediate hypersensitivity reaction to an allergen in plasma. Most are mild reactions shown by urticaria [hives, swollen red wheals] which may cause itching.)

309) Hives and itching are symptoms of which of the following transfusion reactions? a. febrile b. allergic c. circulatory overload d. bacterial

a (The electrolyte exclusion effect applies only to indirect methods and is caused by the solvent displacing effect of high concentrations of lipid and protein in the sample resulting in falsely decreased values.)

309) Which of the following methods is susceptible to the solvent displacing effect that results in falsely decreased electrolyte values? a. indirect ion selective electrodes b. direct ion selective electrodes c. spectrophotometric d. fluorescence

d (Nucleolar pattern is characteristic of staining of the nucleolus seen as 1 or 2 large dots within each nucleus and is produced most frequently in the presence of antibody to nucleolar RNA.)

31) Anti-RNA antibodies are often present in individuals having an anti-nuclear antibody immunofluorescent pattern that is: a. speckled b. rim c. diffuse d. nucleolar

a (HIPAA protects health insurance coverage for workers and their families when they change or lose their jobs, and also addresses the security and privacy of health data.)

31) HIPAA is a federal law that requires: a. confidentiality of patient's health care information between 2 organizations b. reporting of errors in laboratory results c. access to patient records when there is a lawsuit d. unannounced inspections by accreditation agencies

c (An easy to make control for urinalysis is 6.5% NaCl. This has a specific gravity of 1.022, so c is correct. b is false, since the specific gravity of water is 1.000. d refers to osmolality, which is not measured by refractometry.)

31) To prepare a solution appropriate for quality control of the refractometer, a technician should use: a. urea with a specific gravity of 1.040 b. water with a specific gravity of 1.005 c. sodium chloride with a specific gravity of 1.022 d. calcium chloride with a specific gravity of 40

a (Chocolate agar and chocolate agar-based selective media [Martin Lewis] are routinely used for the recovery of Neisseria gonorrhoeae from genital specimens. Sputum and urine specimens are routinely processed using a general purpose media [blood agar] and a selective agar [EMB or MacConkey]. In addition chocolate agar is routinely included to enhance recovery of fastidious organisms such as H influenzae. CSF is routinely processed using blood and chocolate agars.)

31) Which of the following is the most appropriate specimen source and primary media battery? a. endocervical - chocolate, Martin Lewis b. sputum - sheep blood, Thayer-Martin, KV-laked blood c. CSF - Columbia CNA, MacConkey d. urine - sheep blood, chocolate, Columbia CNA

a (Prothrombin is produced by the liver cells dependent on Vitamin K as a coenzyme for its functionality. It is released into the blood as a zymogen as a precursor for its active form thrombin.)

310) Prothrombin is: a. a protein formed by the liver in the presence of Vitamin K b. an enzyme that converts fibrinogen into fibrin threads c. the end product of the reaction between fibrinogen and thrombin d. a protein released by platelets during coagulation

b (Lowenstein-Jensen is an egg-based medium, 7811 is an agar-based medium, and Lowenstein-Jensen Gruft is a selective egg-based medium.)

310) Which of the following combinations of media provides an egg base, agar base, and a selective egg or agar base media? a. Lowenstein-Jensen, American Thoracic Society (ATS), Middlebrook 7H11 b. Lowenstein-Jensen, Middlebrook 7H11, Lowenstein-Jensen Gruft c. Middlebrook 7H10, Petragnani, Lowenstein-Jensen d. Middlebrook 7H10, Middlebrook 7H11, Mitchison 7H11

d (Febrile nonhemolytic transfusion reactions occur in about 1% of transfusions, making it one of the most common types of reaction. Neither transfusion-associated circulatory overload [TACO] or anaphylactic transfusion reactions are characterized by fever. Bacterially contaminated Red Blood Cells are rare, and rapidly produce severe symptoms upon transfusion.)

311) A 65 yr. old woman experienced shaking, chill, and a fever of 102 F (38.9 C) approximately 40 minutes following the transfusion of a second unit of RBCs. The most likely explanation for the patient's symptoms is: a. transfusion of bacterial contaminated blood b. congestive heart failure c. anaphylactic transfusion reaction d. febrile transfusion reaction

c (In practice, a positive screening result for barbiturates obtained by immunoassay is confirmed by gas chromatography/mass spectrometry analysis of the urine specimen.)

311) To detect barbiturate abuse when analyzing urine specimens, immunoassay is the method of choice for screening. The method of choice for confirmation is: a. nephelometry b. thin layer chromatography c. gas chromatography/mass spectrometry d. ultraviolet absorption spectroscopy

b (Factor V is called labile factor, because its activity diminishes quickly at room temperature. Factor II: prothrombin, VII: stable factors, X: Stuart-Prower.)

311) Which of the following coagulation factors is considered to be labile? a. II b. V c. VII d. X

a (The most widely used digestion decontamination method is the N-acetyl-L-cysteine-2% NaOH method.)

311) Which of the following reagents should be used as a mucolytic, alkaline reagent for digestion and decontamination of a sputum for mycobacterial culture? a. N-acetyl-L-cystine and NaOH b. NaOH alone c. zephiran-trisodium phosphate d. oxalic acid

d (Febrile nonhemolytic transfusion reactions are caused by leukoagglutinins in the patient or cytokines released from donor leukocytes during storage. Since these reactions are not caused by red cell antibodies, transfusion investigation studies show no hemolysis or abnormal test results.)

312) A sickle cell patient who has been multiply transfused experiences fever and chills after receiving a unit of RBCs. Transfusion investigation studies show: DAT: neg Plasma hemolysis: none The patient is most likely reacting to: a. IgA b. plasma protein c. red cells d. white cells or cytokines

d (N-acetyl-L-cysteine [NALC] is a mucolytic agent; the concentration of NALC may be increased to digest thick, mucoid specimens.)

312) The function of N-acetyl-L-cysteine in the reagent for acid-fast digestion-decontamination procedure is to: a. inhibit growth of normal respiratory flora b. inhibit growth of fungi c. neutralize the sodium hydroxide d. liquefy the mucus

a (A clot is degraded by plasmin in the fibrinolytic system. Plasminogen is the zymogen produced when Factor XIIa and kallikrein are produced by contact activation.)

312) The most potent plasminogen activator in the contact phase of coagulation is: a. kallikrein b. streptokinase c. HMWK d. fibrinogen

b (The counterstain acts as a quenching agent and reduces the background fluorescence of cellular debris.)

313) When staining acid-fast bacilli with Truant auramine-rhodamine stain, potassium permanganate is used as a: a. decolorizing agent b. quenching agent c. mordant d. dye

b (Patients with a deficiency of Factor XII tend to have thrombotic complications. They do not have bleeding problems most likely due to the lack of activation of fibrin lysis, also due to pathway activation of IX by VIIa/TF complex as well as the activation of Factor XI by thrombin.)

313) Which of the following factor deficiencies is associated with either no bleeding or only a minor bleeding tendency, even after trauma or surgery? a. Factor X b. Factor XII c. Factor XIII d. Factor V

a (Separation of Hgb S from Hgb D.)

314) Hemoglobin S can be separated from hemoglobin D by which of the following methods? a. agar gel electrophoresis at pH 5.9 b. thin layer chromatography c. alkali denaturation d. ammonium precipitation

d (If exposed to light, agar-based media such as 7H11 may release formaldehyde, which is toxic to mycobacteria.)

314) Middlebrook 7H10 and 7H11 media must be refrigerated in the dark, and incubated in the dark as well. If these conditions are not met, the media may prove toxic for mycobacteria because: a. carbon dioxide will be released b. growth factors will be broken down c. light destroys the ammonium sulfate d. formaldehyde may be produced

a (Patients with a deficiency of Factor XII tend to have thrombotic complications. They do not have bleeding problems most likely due to the lack of activation of fibrin lysis, also due to pathway activation of IX by VIIa/TF complex as well as the activation of Factor XI by thrombin.)

314) Which of the following is characteristic of Factor XII deficiency. a. negative bleeding history b. normal clotting times c. decreased risk of thrombosis d. epistaxis

d (Noncardiogenic pulmonary edema, dyspnea, hypotension, and hypoxemia occurring within 6 hours of transfusion are clinical symptoms of TRALI.)

315) Symptoms of dyspnea, cough, hypoxemia, and pulmonary edema within 6 hrs. of transfusion is most likely which type of reaction? a. anaphylactic b. hemolytic c. febrile d. TRALI

d (Factors V and VII are helpful in distinguishing between liver disease and Vitamin K deficiency. Factor VII is a Vitamin K dependent factor; however, Factor V is not and will not be decreased. Both factors will be decreased in liver disease.)

315) The 2 factors that differentiate liver disease from Vit. K deficiency are: a. II and VII b. IX and VII c. VIII and IX d. V and VII

d (The oxalic acid method is superior to alkali methods for processing specimens contaminated with Pseudomonas.)

315) The method used for processing specimens for mycobacterial culture contaminated with Pseudomonas is: a. N-acetyl-L-cystine and NaOH b. NaOH c. zephiran-trisodium phosphate d. oxalic acid

a (Anaphylactic transfusion reactions are severe reactions that occur after infusion of a small amount of donor blood. Symptoms are hypotension, shock, respiratory distress dyspnea, and substernal pain. Anaphylactic reactions are usually caused by anti-IgA.)

316) A patient with a coagulopathy was transfused with FP24 (FFP within 24 hrs. of collection). After infusion of 15 mL, the patient experienced hypotension, shock, chest pain and difficulty in breathing. The most likely cause of the reaction is: a. anti-IgA b. bacterial contamination c. intravascular hemolysis d. leukoagglutinins

a (Proteins are amphoteric substances; that is, they contain acidic and basic groups. Their overall [net] charge is highly positive at low pH values, zero at a particular higher pH, and negative at still more alkaline pH values. At a pH of 4.5, the positively charged proteins will migrate toward the cathode in an electrophoretic system.)

316) The buffer pH most effective at allowing amphoteric proteins to migrate toward the cathode in an electrophoretic system would be: a. 4.6 b. 7.5 c. 8.6 d. 9.5

b (Liver disease affects all Vitamin K dependent factors [II, VII, IX, and X] which will be decreased Factor VIII is an acute phase reactant that may be elevated in liver disease.)

316) Which of the following factors typically shows an increase in liver disease? a. Factor VII b. Factor VIII c. Factor IX d. Factor X

d (Specimens processed for mycobacterial culture must be processed in a biological safety cabinet to minimize aerosol transmission of the organism.)

317) A bronchial washing is processed for acid-fast bacilli. Which of the following precautions should be taken in order to prevent infection of laboratory personnel? a. add an equal amount of NALC to the specimen b. process all specimens under ultraviolet light c. centrifuge specimen only after the addition of preservative d. process all specimens in a biological safety hood

c (A prolonged APTT with a normal PT denotes a problem with the intrinsic pathway, so Factors VIII, IX, XI would be looked at, these deficiencies would correct in a mixing study. A characteristic of prekallikrein deficiency is the correction of the PTT when incubated for 10 minutes with kaolin, Celite*, silica, or ellagic acid.)

317) A patient has a normal prothrombin time and a prolonged activated partial thromboplastin time using a kaolin activator. The APTT corrects to normal when the incubation time is increased. These suggest that the patient has: a. hemophilia A b. Hageman Factor (XII) deficiency c. Fletcher Factor deficiency d. Factor V deficiency

c (Leukoreduction of blood products reduces the risk of febrile nonhemolytic transfusion reactions, which are caused by leukoagglutinins or cytokines from white cells.)

317) To prevent febrile transfusion reactions, which RBC product should be transfused? a. RBCs irradiated b. CMV-neg RBC's c. RBCs, leukocyte reduced d. IgA-deficient donor blood

d (One reason to quarantine blood components before transfusion is hemolysis of the red cells. Hemolysis of red cells is an indication of contamination or improper storage.)

318) During the issue of an autologous unit of Whole Blood, the supernatant plasma is observed to be dark red in color. What would be the best course of action? a. the unit may be issued only for autologous use b. remove the plasma and issue the unit as RBC's c. issue the unit only as washed RBC's d. quarantine the unit for further testing

d (Long-term antibiotic therapy disrupts normal flora, which provide a source of Vitamin K synthesis, This results in a Vitamin K deficiency.)

319) A 4-year-old boy presents with chronic ear infections and is on prophylactic antibiotics. He presents with bleeding. Factor assays reveal: Possible causes are: a. Factor II deficiency b. lupus anticoagulant c. hemophilia d. Vitamin K deficiency

c (Transfusion-associated circulatory overload [TACO] is hypervolemia manifested by coughing, cyanosis, and pulmonary edema.)

319) Coughing, cyanosis and difficult breathing are symptoms of which of the following transfusion reactions? a. febrile b. allergic c. circulatory overload d. hemolytic

a (Rnp/Sm extractable nuclear antigens show coarse nuclear speckles.)

32) Anti-extractable nuclear antigens are most likely associated with which of the following anti-nuclear antibody immunofluorescent pattern: a. speckled b. rim c. diffuse d. nucleolar

b (Mechanism of genetic abnormality in thalassemia reduces globin chain production.)

32) Evidence indicates that the genetic defect in thalassemia usually results in: a. the production of abnormal globin chains b. a quantitative deficiency in RNA resulting in decreased globin chain production c. a structural change in the heme portion of the hemoglobin d. an abnormality in the alpha- or beta- chain binding or affinity

a (The required temperature for storage of thawed plasma is 1-6C.)

32) In the liquid state, plasma must be stored at: a. 1 - 6 C b. 22 C c. 37 C d. 56 C

d (CIN agar is a selective and differential medium for the isolation and differentiation of Y enterocolitica, This medium contains sodium desoxycholate, crystal violet, cefsulodin, irgason [triclosan], and novobiocin as selective agents, and mannitol as the carbohydrate.)

32) Which of the following in the most appropriate organism and media combination? a. Legionella species - Regan Lowe b. Clostridium Difficile - phenylethyl alcohol c. Campylobacter species - charcoal yeast extract d. Yersinia enterocolitica - cefsulodin-irgasan-novobiocin

a (All daughters of hemophiliac men are carriers of the disease; all sons are normal. The gene for Factor VIII lies on the X chromosome.)

320) A hemophiliac male and a normal female can produce: a. a female carrier b. a male carrier c. a male hemophiliac d. a normal female

b (The pH electrode, a glass electrode, contains a specially designed thin piece of glass as a membrane. The glass membrane is made of silicon dioxide, added oxides and various metals. The membrane is selectively sensitive to hydrogen ions.)

320) Hydrogen ion concentration in blood is usually determined by means of which of the following electrodes? a. silver b. glass c. platinum d. platinum-lactate

c (Transfusion-associated circulatory overload [TACO] is hypervolemia caused by blood transfusion in susceptible patients. Hemolytic [antibody to red cell antigen], febrile NHTR [leukoagglutinins or cytokines], and allergic [reaction to allergens in plasma] are immunologic reactions.)

320) Which of the following is a nonimmunologic adverse effect of a transfusion? a. hemolytic reaction b. febrile nonhemolytic reaction c. congestive heart failure d. urticaria

a (Coulometry is an electrochemical technique used to measure the amount of electricity passing between 2 electrodes in an electrochemical cell. An application of coulometry is the titration of chloride with silver ions generated by electrolysis from a silver wire at the anode. Polarography is also an electrochemical technique, hut measures current. Mass spectroscopy and chromatography are separation techniques.)

321) An automated method for measuring chloride which generates silver ions in the reaction is: a. coulometry b. mass spectroscopy c. chromatography d. polarography

d (Transfusion-induced hypervolemia causing edema and congestive heart failure is a feature of transfusion-associated circulatory overload [TACO]. Hypervolemia is not a complication of a hemolytic, febrile, or anaphylactic transfusion reaction.)

321) Congestive heart failure, severe headache and/or peripheral edema occurring soon after transfusion is indicative of which type of transfusion reaction? a. hemolytic b. febrile c. anaphylactic d. circulatory overload

b (Hypervolemia due to transfusion in susceptible patients, such as cardiac, elderly, infants, or severely anemic, causes circulatory overload [TACO] and associated respiratory and cardiac problems)

322) A patient with severe anemia became cyanotic and developed tachycardia, hypertension, and difficulty breathing after receiving 3 units of blood. No fever or other symptoms were evident. this is most likely what type of reaction? a. febrile b. transfusion-associated circulatory overload (TACO) c. anaphylactic d. hemolytic

a (Coulometry is still used for chloride determinations in body fluids, such as sweat. However, chloride ion-selective electrodes [ISE] are commonly used today.)

322) Coulometry is often used to measure: a. chloride in sweat b. the pH in saliva c. bicarbonate in urine d. ammonia in plasma

a (Septic transfusion reactions due to contaminated blood products are manifested by high fever, chills, hypotension, shock, nausea, diarrhea, renal failure, and DIC. Symptoms usually appear rapidly. Transfusion reaction investigation shows no evidence of unexpected blood group antibodies. A Gram stain and blood culture of the donor unit may detect the presence of aerobic or anaerobic organisms.)

323) A patient became hypotensive and went into shock after receiving 50 mL of a unit of RBCs. She had a shaking chill and her temperature rose to 104.8 F (40.0 C). A transfusion reaction investigation was initiated but no abnormal results were seen. What additional testing should be performed? a. gram stain and culture of the donor unit b. lymphocytotoxicity tests for leukoagglutinins c. plasma IgA levels d. elution and antibody screen

a (The reference pH electrode is often constructed of Ag and AgCl.)

323) In a pH meter reference electrodes may include: a. silver-silver chloride b. quinhydrone c. hydroxide d. hydrogen

c (Hepatitis transmission is unlikely, but has a higher risk of transmission through blood transfusion than CMV [rare], syphilis [no transfusion-transmitted cases reported in >30 years], or HIV [1:2,300,000 units].)

324) The most frequent transfusion-associated disease complication of blood transfusion is: a. cytomegalovirus (CMV) b. syphilis c. hepatitis d. AIDS

b (pH, PCO2, and PO2 are directly measured by modern blood gas analyzers; other parameters are calculated.)

325) Most automated blood gas analyzers directly measure: a. pH, HCO3 and %O2 saturation b. pH, PCO2 and PO2 c. HCO3, PCO2, and PO2 d. pH, PO2 and % O2 saturation

b (Factor XIII activity is <5% in congenital or acquired disorders. In adults, bleeding is slow and delayed The PT, APTT, fibrinogen and platelets will be normal.)

325) Patient presents with bleeding 48 hours post tooth extraction. Results are as follows: Possible causes are a deficiency in: a. plasminogen b. Factor XIII c. alpha2 anti-plasmin d. Factor XII

c (Mycobacterium tuberculosis can remain viable but dormant in macrophages for many years.)

325) The disease-producing capacity of Mycobacterium tuberculosis depends primarily upon: a. production of exotoxin b. production of endotoxin c. capacity to withstand intracellular digestion by macrophages d. lack of susceptibility to the myeloperoxidase system

b (Irradiation inhibits proliferation of T cells and subsequent GVHD.)

325) The purpose of a low-dose irradiation of blood components is to: a. prevent posttransfusion purpura b. prevent graft-vs-host disease c. sterilize components d. prevent noncardiogenic pulmonary edema

b (The PCO2 electrode is a self-contained potentiometric cell. CO2 gas from the sample or calibration matrix diffuses through the selective membrane and dissolves in the internal electrolyte layer. Carbonic acid is formed and dissociates, shifting the pH of the bicarbonate solution in the internal layer. This shift is related to the carbon dioxide in the sample.)

326) Blood PCO2 may be measured by: a. direct colorimetric measurement of dissolved CO2 b. a self-contained potentiometric electrode c. measurement of CO2-saturated hemoglobin d. measurement of CO2 consumed at the cathode

a (Patients suspected of a lupus anticoagulant present with a prolonged APTT. Laboratory assays to detect lupus anticoagulants are the APTT, dilute Russell viper venom test, and Kaolin clotting time. In the presence of a LA, the mixing study doesn't correct.)

326) Plasma from a patient with lupus coagulation inhibitor can show: a. prolonged APTT and normal PT b. prolonged thrombin timer c. no change with platelet neutralization d. complete correction when incubated with normal plasma

b (Mycobacterium tuberculosis can remain viable but dormant in macrophages for many years.)

326) Which species of Mycobacterium includes a BCG strain used for vaccination against tuberculosis? a. tuberculosis b. bovis c. kansasii d. fortuitum/chelonae complex

b (Several commercial PCR tests are available that identify Mycobacterium tuberculosis from positive respiratory specimens.)

327) AFB smear positive respiratory specimens may be reliably identified as Mycobacterium tuberculosis the same day the smear was read by: a. cording seen on the AFB smear b. nucleic acid amplification c. QuantiFERON-TB test d. DNA probes

c (Gamma irradiation of blood products prevents donor lymphocytes from replicating after transfusion and causing transfusion associated graft-vs-host disease in susceptible patients.)

327) Irradiation of donor blood is done to prevent which of the following adverse effects of transfusion? a. febrile transfusion reaction b. CMV c. transfusion associated graft-vs-host disease d. transfusion related acute lung injury (TRALI)

a (Interpretation of metabolic acidosis.)

33) A 68 yr. old man arrives in the emergency room with a glucose level of 722 mg/dL (39.7 mmol/L) and serum acetone of 4+ undiluted. An arterial blood gas from this patient is likely to be: a. low pH b. high pH c. low PO2 d. high PO2

d (Columbia CNA agar is a selective medium used for the isolation of gram- positive organisms. The medium contains colistin and nalidixic acid, which inhibits gram-negative organisms. MacConkey agar is a selective and differential medium used for the isolation of gram-negative organisms. The medium contains bile and crystal violet, which inhibits gram-positive organisms.)

33) A Gram stain from a swab of a hand wound reveals: - moderate neutrophils - no squamous epithelial cells - moderate gram-positive cocci in chains - moderate large gram-negative bacilli Select the appropriate media that will selectively isolate each organism: a. KV-laked agar, Thayer-Martin b. sheep blood, MacConkey c. Columbia CNA, chocolate d. Columbia CNA, MacConkey

a (Most specific glucose method.)

332) The most specific method for the assay of glucose utilizes: a. hexokinase b. glucose oxidase c. glucose-6-phosphatase d. glucose dehydrogenase

b (Scotochromogens produce deep yellow or orange pigment when grown in either light or darkness.)

332) The mycobacteria that produce a deep yellow or orange pigment both in the dark and light are: a. photochromogens b. scotochromogens c. nonchromogens d. rapid growers

a (Photochromogens produce nonpigmented colonies only when grown in darkness, but pigmented colonies after exposure to light.)

333) Mycobacteria that produce pigment only after exposure to light are classified as: a. photochromogens b. scotochromogens c. rapid growers d. nonchromogens

c (Lowenstein-Jensen is an egg-based medium used to isolate AFB. The other media are not used for the isolation of AFB.)

335) The best medium for culture of Mycobacterium tuberculosis is: a. Bordet-Gengou agar b. Loeffler medium c. Lowenstein-Jensen medium d. cystine blood agar

d (The APTT is performed to monitor the effects of unfractionated heparin, to detect factor deficiencies, and the presence of inhibitors. The PT tests the extrinsic pathway and requires tissue thromboplastin. It is also used to monitor warfarin therapy.)

338) The APTT: a. tests the extrinsic coagulation pathway b. monitors Coumadin therapy c. requires tissue thromboplastin d. monitors heparin therapy

d (Other distractors are not bilirubin methods.)

338) The most widely used methods for bilirubin measurement are those based on the: a. Jaffe reaction b. Schales and Schales method c. 8-hydroxyquinoline reaction d. Jendrassik-Grof method

b (A mixing study, when corrected, indicates a factor deficiency. Lack of correction indicates an inhibitor. Some inhibitors may only become evident after the patient's plasma is allowed to interact with the normal plasma after incubation. In this case, correction occurred immediately, as well as after incubation, confirming a factor deficiency.)

339) Aliquots of plasma with a prolonged PT and prolonged APTT are mixed using various ratios of patient plasma and normal plasma. All samples are incubated at 37 C and are tested at 10-,30-, and 60-min. intervals. The PT and APTT results on all of the mixtures are corrected .These results would indicate the presence of: a. circulating anticoagulant b. factor deficiency c. contaminated reagent d. antibodies

d (The diazo method of Malloy and Evelyn involves bilirubin reacting with diazotized sulfanilic acid to form azobilirubin.)

339) In the Malloy and Evelyn method for the determination of bilirubin: the reagent that is reacted with bilirubin to form a purple azobilirubin is: a. dilute sulfuric acid b. diazonium sulfate c. sulfobromophthalein d. diazotized sulfanilic acid

c (MSDS is an OSHA required document that provides information such as physical data [melting point, boiling point, Hash point etc.], toxicity, health effects, first aid, reactivity, storage, disposal, protective equipment, and spill/leak procedures.)

34) A technician is asked to clean out the chemical reagent storeroom and discard any reagents not used in the past 5 years. How should the technician proceed? a. discard chemicals into biohazard containers where they will later be autoclaved b. pour reagents down the drain, followed by flushing of water c. consult MSDS sheets for proper disposal d. pack all chemicals for incineration

a (Problems with analysis of Gram staining generally result from errors including interpretation of the slide [smear prepared too thick], excessive heat fixing, and improper decolorization. Inadequate decolorization with acetone/alcohol results in a smear in which host cells [neutrophils and squamous cells], as well as bacteria, all appear blue.)

34) Upon review of a sputum Gram stain, the technician notes that the nuclei of all of the neutrophils present in the smear are staining dark blue. The best explanation for this finding is: a. the slide was inadequately decolorized with acetone/alcohol b. the sputum smear was prepared to thin c. the cellular components have stained as expected d. the iodine was omitted from the staining procedure

a (Protein C is a vitamin K dependent protein; hence warfarin therapy will lower levels of this protein.)

340) A patient is taking 10 mg per day of coumadin. The results of which of the following lab tests will be most impacted? a. protein C level b. antithrombin III level c. Factor V Leiden mutation d. Factor VIII level

a (In the Jendrassik-Grof method for bilirubin measurement, the addition of caffeine plus diazotized sulfanilic acid and the serum produces azobilirubin. Ascorbic acid, alkaline tartrate and dilute HCI are added to the reaction mixture. The blue- green azobilirubin is measured. Bilirubin glucuronide, urobilin and urobilinogen are intermediaries in bilirubin metabolism. They are not measured by this method.)

340) In the Jendrassik-Grof reaction method for the determination of serum bilirubin concentration quantitation is obtained by measuring the green color of: a. azobilirubin b. bilirubin glucuronide c. urobilin d. urobilinogen

d (Mycobacterium avium and M intracellulare have the same biochemical pattern, but can be differentiated by the use of DNA probes.)

341) Differentiation of Mycobacterium avium from Mycobacterium intracellulare can be accomplished by: a. nitrate reduction test b. Tween hydrolysis test c. resistance to 10 ug thiophene-2-carboxylic acid hydrazide (TCH) d. DNA probe

a (Rapidly growing mycobacteria such as Mycobacterium fortuitum may be <10% acid fast and may not stain with fluorochrome.)

342) Which one of the following species of Mycobacterium does not usually fluoresce on fluorochrome stain? a. Mycobacterium fortuitum b. Mycobacterium tuberculosis c. Mycobacterium ulcerans d. Mycobacterium bovis

a (Nicotinamide adenine dinucleotide is the coenzyme system for the LD assay. NADH is the reduced form and NAD is the oxidized form. The coenzymes serve as a substrate for dehydrogenases reactions. ATP/ADP and Cu/Cu++ are not coenzymes. Fe is an activator for enzymatic reactions.)

343) In the assay of lactate dehydrogenase , the reaction is dependent upon which of the following coenzyme systems? a. NAD/NADH b. ATP/ADP c. Fe++/Fe+++ d. Cu/Cu++

b (A prolonged thrombin time may be considered evident of diminished or abnormal fibrinogen; however, the presence of AT activity, such as heparin, must be ruled out.)

344) The best test to determine if a sample is contaminated with heparin is: a. fibrinogen b. thrombin time c. prothrombin time d. stypven time

c (In the continuous monitoring method of the measurement of LD activity, the decrease in absorbance at each time interval indicates that the product formed [substrate converted] is constant up to 6 minutes. After 6 minutes, the substrate concentration is limited, and there is a decrease in the rate of product formation as indicated by the lack of linearity.)

344) This illustration represents the change in absorbance at 340 nm over a period of 8 minutes in an assay for lactate dehydrogenase. True statements about this figure include: a. the reaction follows zero order kinetics between 5 and 8 minutes b. the reaction is proceeding from lactate to pyruvate c. nonlinearity after 6 minutes is due to substrate exhaustion d. the change in absorbance is due to reduction of NAD to NADH

b (If the initial APTT remains prolonged in a mix, a second assay should be performed. The lupus anticoagulant has multiple targets. The second test is the DRVVT, which triggers coagulation at Factor X.)

346) If a patient presents with a prolonged APTT that does not correct upon mixing, the next test performed should be: a. Factor II b. DRVVT c. Factor VIII d. platelet count

b (Antibiotics and antifungal agents are added to viral transport medium to inhibit the growth of bacteria and fungus.)

346) Virus transport medium containing penicillin, gentamicin and amphotericin is used to collect and transport specimens for virus culture because this medium: a. enables rapid viral growth during the transport time b. inhibits bacterial and fungal growth c. destroys nonpathogenic viruses d. inhibits complement-fixing antibodies

c (D-dimers are produced from crosslinked and stabilized fibrin clots. This clot is dissolved by plasma and d-dimers are released. Therefore, d-dimers suggest a breakdown of fibrin clots, and indicate that clots have been formed at the site of injury.)

347) Excess D-Dimer indicate that clots have been: a. converted to fibrin monomers b. released into circulation c. formed and are being excessively lysed d. stimulated to activate platelets

d (Alkaline phosphatase catalyzes the hydrolysis of para-nitrophenylphosphate, forming phosphate and free 4-nitrophenyl [4-npp] which, under alkaline conditions, has a very intense yellow color. IFCC recommended methods use 4-npp as the substrate.)

347) The International Federation for Clinical Chemistry (IFCC) recommends the use of methods such as the Bessey-Lowry-Brock method for determining alkaline phosphatase activity. The substrate used in this type of method is: a. monophosphate b. phenylphosphate c. disodium phenylphosphate d. para-nitrophenylphosphate

b (In tissue smears or biopsy specimens, the presence of virally infected cells is indicated by intranuclear or intracytoplasmic inclusions.)

347) Which of the following indicates the presence of a viral infection in tissue smears or biopsies? a. cytopathic effect b. intranuclear inclusions c. cell lysis d. mononuclear inflammatory cells

a (D-dimers are produced from crosslinked and stabilized fibrin clots. This clot is dissolved by plasma and d-dimers are released. Therefore, d-dimers suggest a breakdown of fibrin clots, and indicate that clots have been formed at the site of injury.)

348) D-Dimers are produced from: a. cross-linked and stabilized fibrin clot b. decreased fibrinogen and platelets c. plasminogen converting to plasmin d. generation of thrombin from endothelial cells

a (Respiratory syncytial virus [RSV] infected the ciliated respiratory epithelium of the upper respiratory tract. A nasopharyngeal swab or aspirate is the optimal specimen for RSV recovery.)

348) Respiratory syncytial virus is best isolated using a(n): a. nasopharyngeal aspirate b. cough plate c. expectorated sputum d. throat swab

a (Aldolase catalyzes the splitting of fructose-1,6-diphosphate to glyceraldehyde- 3'-phosphate and dihydroxyacetone phosphate, an important reaction in the glycolytic breakdown of glucose to lactate.)

349) Which of the following is a glycolytic enzyme that catalyzes the cleavage of fructose-1, 6-diphosphate to glyceraldehyde-3-phosphate and dihydroxyacetone phosphate? a. aldolase b. phosphofructokinase c. pyruvate kinase d. glucose-6-phosphate dehydrogenase

d (Levels of CO2 and pH in metabolic alkalosis.)

35) A person suspected of having metabolic alkalosis would have which of the following laboratory findings? a. CO2 content elevated and PCO2 elevated, pH decreased b. CO2 content decreased and pH elevated c. CO2 content decreased, PCO2 decreased and pH decreased d. CO2 content elevated and pH elevated

a (The ACT monitors high dose heparin therapy at clinics, bedside, cardiac catheterization or at a surgical suite. The Stypven time is a test using snake venom for testing for lupus. The reptilase time is used to distinguish between heparin contamination and a decreased fibrinogen level.)

350) A bedside test that can be used to monitor heparin activity is the: a. activated clotting time b. stypven time c. reptilase time d. partial thromboplastin time

d (Compensatory mechanism in respiratory acidosis.)

54) In respiratory acidosis, a compensatory mechanism is the increase in: a. respiration rate b. ammonia formation c. blood PCO2 d. plasma bicarbonate concentration

b (Human papilloma virus [HPV] infects epithelial tissues throughout the body, including skin, larynx, and anogenital tissue. Persistent infection with oncogenic types of HPV and integration of HPV DNA into the cellular genome is a pathway leading to HPV-induced neoplasia, such as cervical cancer.)

350) The genus of virus associated with anogenital warts, cervical dysplasia and neoplasia is: a. herpes simplex virus b. papillomavirus c. cytomegalovirus d. coxsackierus

b (There are several enzymatic methods for measuring serum triglyceride. The first step of the coupled reactions involves the hydrolysis of triglyceride by lipase to produce glycerol and fatty acids. Glycerol is a reactant in one of 2 enzymatic sequences for the final measurement of triglycerides.)

350) The substance that is measured to estimate the serum concentration of triglycerides by most methods is: a. phospholipids b. glycerol c. fatty acids d. pre-beta lipoprotein

b (Herpes simplex virus is the most common cause of fatal sporadic encephalitis in the United States.)

351) Encephalitis is most commonly associated with which of the following viruses? a. Epstein-Barr b. herpes simplex c. coxsackie B d. varicella zoster

b (Fibrin stabilizing factor is needed to polymerize a clot. Primary screening tests are normal in Factor XIII deficiency. A screening test is based on the solubility of a fibrin clot in SM urea.)

351) Which of the following lab findings is associated with Factor VIII deficiency? a. prolonged activated partial thromboplastin time b. clot solubility in a 5 molar urea solution c. prolonged thrombin time d. prolonged prothrombin time

b (High-volume HDL method.)

351) Which of the following methods for quantitation of high-density lipoprotein is most suited for clinical laboratory use? a. Gomori procedure b. homogeneous c. column chromatography d. agarose gel electrophoresis

d (Chemiluminescent labels are based on the emission of light produced during a chemical reaction. These labels are very useful because they provide very low levels of detection [2 x 10^20 mol/L] with little or no background interference.)

352) A chemiluminescent EIA: a. measures absorption of light b. is less sensitive than radioisotopic reactions c. is monitored by the use of a gamma counter d. is quantitated by the amount of light produced by the reactions

b (Adenovirus infections are common. It causes up to 5% of all respiratory infections, and the prevalence of infection is higher [up to 14%].)

352) Colds and other acute respiratory diseases are most often associated with: a. Epstein-Barr virus b. adenovirus c. coxsackie B d. reovirus

d (Heparin acts by inhibiting thrombin and Factor Xa. Anticoagulants, such as sodium citrate, act by binding calcium to prevent a blood sample from clotting.)

352) Heparin acts by: a. precipitating fibrinogen b. binding calcium c. activating plasma d. inhibiting thrombin

b ("+++" is an indicator that the WBC count exceeds the upper reportable limit.)

354) In an automated cell counter the WBC printed result is "+++" The next step is to: a. repeat after warming the sample to 37 C b. make an appropriate dilution of the sample c. recalibrate the machine from pooled samples d. request a new sample immediately

a (The osmolality of a solution does not depend on the kind of particles but only on the number of particles, therefore it is called a colligative property.)

354) Which of the following applies to cryoscopic osmometry? a. temperature at equilibrium is a function of the number of particles in solution b. temperature plateau for a solution is horizontal c. freezing point of a sample is absolute d. initial freezing of a sample produces an immediate solid state

c (Low platelet count values should be verified with a slide estimate.)

355) A specimen run on an automatic cell counter has a platelet count of 19 x 10^3. The first thing the tech should do is: a. report the count after the batch run is completed b. request a new specimen c. review the stained blood smear d. notify the lab manager

b (Coulter principle of particle counting.)

356) The electrical resistance method of cell counting requires: a. equal-sized particles b. a conductive liquid c. 2 internal electrodes for current d. 3 apertures for counting

c (Formulas for calculation of indices; falsely increased RBC should not affect Hgb.)

359) On an electronic particle counter, if the RBC is erroneously increased, how will other parameters be affected? a. increased MCHC b. increased Hgb c. decreased MCH d. increased MCV

b (G-6-PD deficiency.)

36) An enzyme deficiency associated with a moderate to severe hemolytic anemia after the patient is exposed to certain drugs and characterized by red cell inclusions formed by denatured hemoglobin is: a. LD deficiency b. G-6-PD deficiency c. pyruvate kinase deficiency d. hexokinase deficiency

b (Maintains in Fe++ form by way of reduced glutathione.)

4) The main function of the hexose monophosphate shunt in the erythrocyte is to: a. regulate the level of 2,3-DPG b. provide reduced glutathione to prevent oxidation of hemoglobin c. prevent the reduction of heme iron d. provide energy for membrane maintenance

a (The correct answer is a, high health hazard. The ratings range for both systems is 0-4 with 0 being no hazard and 4 being the most severe hazard. The other distractors are incorrect due to the hazard type or number rating.)

36) If the HMIS or NFPA 704 hazardous material identification system has a number 4 in the left blue quadrant, it represents a: a. high health hazard b. low health hazard c. high reactivity hazard d. low reactivity hazard

b (Irradiation inhibits proliferation of T lymphocytes.)

36) Irradiation of a unit of RBC's is done to prevent the replication of donor: a. granulocytes b. lymphocytes c. red cells d. platelets

d (Component levels in metabolic acidosis.)

36) Metabolic acidosis is described as a: a. increase in CO2 content and PCO2 with a decrease pH b. decrease in CO2 content with an increased pH c. increase in CO2 with an increased pH d. decrease in CO2 content and PCO2 with a decreased pH

b (Through the action of the enzyme beta-galactosidase, ONPG cleaves into galactose and o-nitrophenol [a yellow compound].)

36) The ONPG test allows organisms to be classified as a lactose fermenter by testing for which of the following? a. permease b. beta-galactosidase c. beta-lactamase d. phosphatase

c (Although few labs [if any] use a urinometer, all of them should have a refractometer. These use 1 drop of sample and compensate for temperature. A pitfall is that they are disproportionately affected by glucose and protein.)

36) Use of a refractometer over a urinometer is preferred due to the fact that the refractometer uses: a. large volume of urine and compensates for temp b. small volume of urine and compensates for glucose c. small volume of urine and compensates for temp d. small volume of urine and compensates for protein

d (Repeat of one out-of-range controls the first appropriate course of action.)

360) On setting up the electronic particle counter in the morning, one of the controls is slightly below the range for the MCV. Which of the following is indicated? a. call for service b. adjust the MCV up slightly c. shut down the instrument d. repeat the control

a (Relapsing fever is caused by Borrelia recurrentis and is transmitted by the human body louse. Relapsing fever is characterized by the acute onset of high fever lasting 3 - 7 days, interspersed with periods of no fever lasting days to weeks.)

360) Relapsing fever in humans is caused by: a. Borrelia recurrentis b. Brucella abortus c. Leptospira interrogans d. Spirillum minus

b (Chlamydophila psittaci, the agent of psittacosis, is transmitted to humans via inhalation of nasal secretions, as well as infected fecal or feather dust, of psittacine birds [parrot family].)

361) Psittacosis is transmissible to man via contact with: a. insects b. birds c. cattle d. dogs

d ("+++" indicates a WBC that is above the reportable range of the instrument; dilute the sample and rerun.)

361) The following results were obtained on an electronic particle counter: What step should be taken before recycling the sample? a. clean the apertures b. warm the specimen c. replace the lysing agent d. dilute the specimen

c (Mycoplasma are smaller than other bacteria and lack a cell wall. Instead, they possess a trilaminar cell membrane. Because they lack a cell wall, they cannot be stained with Gram stain.)

363) Mycoplasmas differ from other bacteria in that they: a. do not cause disease in humans b. cannot grow in artificial inanimate media c. lack cell walls d. are not serologically antigenic

b (Howell-Jolly bodies and WBC fragments may be mistaken for platelets; clotting may affect platelet numbers; decreased Hct would have no effect on a phase platelet count.)

363) Which of the following will not cause erroneous results when using a phase optical system for enumerating platelets? a. incipient clotting b. decreased hematocrit c. Howell-Jolly bodies d. leukocyte cytoplasmic fragments

b (Problems with diluent contamination, voltage variation and calibration errors are not common and are detected by daily quality control.)

364) The most common cause of error when using automated cell counters is: a. contamination of the element b. inadequate mixing of the sample prior to testing c. variation in voltage of the current supply d. a calibrating error

d (Appearance of RBC histogram.)

369) Which area in the automated cell counter histogram represents the distribution curve? a. A b. B c. C d. D

d (G-6-PD deficiency-hemolytic stimulus.)

37) Patients with A(-) type G-6-PD deficiency are least likely to have hemolytic episodes in which of the following situations? a. following the administration of oxidizing drugs b. following the ingestion of fava beans c. during infections d. spontaneously

c (FFP thawed in a water bath should be protected so that entry ports are not contaminated with water. One can may use a plastic overwrap or keep ports above the water level.)

37) Plastic bag overwraps are recommended when thawing FFP in a 37 C water bath. because they prevent: a. FFP bag from cracking b. water from slowly dialyzing across the bag membrane c. the entry ports from being contaminated with water d. the label from peeling

a (Definition of respiratory acidosis.)

37) Respiratory acidosis is described as a(n): a. increase in CO2 content and PCO2 with a decreased pH b. decrease in CO2 content with an increased pH c. increase in CO2 content with an increased pH d. decrease in CO2 content and PCO2 with a decreased pH

b (The correct answer is b, formaldehyde.)

37) Which chemical is a potential carcinogen? a. potassium chloride b. formaldehyde c. mercury d. picric acid

b (Appearance of WBC histogram.)

370) Which area in the automated cell counter histogram indicates the lymphocyte curve? a. A b. B c. C d. D

d (McCoy cells, in addition to buffalo green monkey kidney cells, are susceptible to infection with Chlamydia trachomatis and are used routinely for the recovery of C trachomatis in culture.)

371) A cell culture line used for the recovery of Chlamydia trachomatis from clinical specimens is: a. HeLa 229 b. Hep- 2 c. BHK-21 d. McCoy

d (Pollen grains are common artifacts in stool specimens submitted for ova and parasite examination. Their appearance is similar to protozoan cysts.)

373) Artifacts found in a stool specimen that can be confused with ova or cysts are: a. partially digested meat fibers b. degenerated cells from the gastrointestinal mucosa c. dried chemical crystals d. pollen grains

a (Photo-optical endpoint is a change in optical density. Nephelometry is an immunometric method for measuring proteins. Chromogenic methods employs a color-producing substance called a chromophore. Immunologic assays are based on antigen-antibody reactions.)

373) The photo-optical method of endpoint detection is described as: a. change in optical density as a result of a fibrin clot b. measurement of turbidity of antigen-antibody formation c. decreased motion of a mechanical ball d. color-producing chromophore

d (Too large of a drop will produce a thick smear; slowly pushing the blood will affect the distribution of the cells.)

377) When evaluating a smear for a retic count, the tech observes that the RBC's are overlapping throughout the entire slide. The most likely explanation is: a. grease on the slide prevented even spreading b. improper proportions of blood and stain were used c. the slide was dried to quickly d. the drop used for the slide preparation was too large

b (In polycythemia, the decrease in plasma volume relative to whole blood alters the 9 parts blood to 1 part anticoagulant ratio falsely prolonging results. Therefore, a tube with a reduced volume of anticoagulant is need when the Hct is > 55%.)

379) A citrated blood specimen for coagulation studies is to be collected from a polycythemic patient. the anticoagulant should be: a. the standard volume b. reduced in volume c. changed to EDTA d. changed to oxalate

a (G-6-PD deficiency-anti-malarial drugs.)

38) A patient has a congenital nonspherocytic hemolytic anemia. After exposure to anti-malarial drugs the patient experiences a severe hemolytic episode. This episode is characterized by red cell inclusions caused by hemoglobin denaturation. Which of the following conditions is consistent with these findings? a. G-6-PD deficiency b. thalassemia major c. pyruvate kinase deficiency d. paroxysmal nocturnal hemoglobinuria

d (The correct answer is d, be secured upright to the wall or other stable source. The other distractors are incorrect practices. Compressed gas tanks should be stored away from flammable materials, have safety covers on when pressure regulators are unattached and transported chained to a hand cart or dolly.)

38) Compressed gas cylinders should: a. be stored with flammable materials b. be transported by rolling or dragging c. have safety covers removed when pressure regulators are unattached d. be secured upright to the wall or other stable source

a (The reagent strip is not affected by contrast dye. The refractometer reads the darker solution as density. A densitometer is a chemistry instrument, not a urinalysis instrument. A urinometer is not generally used in the modern lab.)

38) The method of choice for performing a specific gravity measurement of urine following administration of x-ray contrast dyes is: a. reagent strip b. refractometer c. urinometer d. densitometer

a (The most common cause of ring- enhancing brain lesions in a patient with HIV/AIDS is Toxoplasma gondii.)

380) Multifocal brain lesion in AIDS patients is commonly caused by: a. Toxoplasma gondii b. Pneumocystis jiroveci c. Cryptosporidium parvum d. Giardia lamblia

a (High-quality blood smears can be made from the EDTA tube within 2-3 hours of collecting the specimen.)

381) The specimen of choice for preparation of blood films for manual differential leukocyte counts is whole blood collected in: a. EDTA b. oxalate c. citrate d. heparin

b (The trophozoite of Entamoeba histolytica ranges in size from 12 - 60 mm, which is significantly larger than Endolimax nana. The nucleus of E histolytica displays evenly distributed peripheral chromatin unlike E coli, which has coarse peripheral chromatin and I butschlii, which has none.)

382) A patient is suspected of having amebic dysentery. Upon microscopic examination of a fresh fecal specimen for ova and parasites, the following data were obtained: - a trophozoite of 25 mm - progressive, unidirectional crawl - evenly distributed peripheral chromatin - finely granular cytoplasm This information probably indicates: a. Entamoeba coli b. Entamoeba histolytica c. Endolimax nana d. lodamoeba butschlii

a (Instruments count particles within defined size limits. The upper limit is to separate large platelets from erythrocytes. Large platelets may be counted as erythrocytes.)

382) A platelet determination was performed on an automated instrument and a very low value was obtained. The platelets appeared adequate when estimated from the stained blood film. The best explanation for this discrepancy is: a. many platelets are abnormally large b. blood sample is hemolyzed c. white cell fragments are present in the blood' d. red cell fragments are present in the blood

d (Trophozoites of Entamoeba coli demonstrate slow and undefined motility.)

383) Trophozoites of the cyst shown above are likely to: a. contain red blood cells b. have clear, pointed pseudopodia c. contain few, if any, vacuoles d. have slow, undefined motility

d (Platelet clumping is expected if smears are made directly from the finger. Smears must be made promptly before any clotting begins. It is important to examine the edges for platelet clumping.)

383) When platelets concentrate at the edges and feathered end of a blood smear, it is usually due to: a. abnormal proteins b. inadequate mixing of blood and anticoagulant c. hemorrhage d. poorly made wedge smear

a (Diarrhea associated with Giardia lamblia is caused by the consumption of contaminated water and results in greasy, foul-smelling stools.)

388) A 24-year-old woman, who just returned from vacationing in Russia, became ill with steatorrheal diarrhea. The above organism was found in her stool. The patient most likely is suffering from: a. giardiasis b. amebiasis c. ascariasis d. balantidiasis

b (Patients with infection often have at least 100,000 bacteria/mL of urine in the bladder. However one third of young women with symptomatic cystitis have less than 100,000 bacteria/mL of urine. The Infectious Disease Society of America consensus definition of cystitis is greater than or equal to 1,000 CFU/mL of a uropathogen.)

4) Which of the following clean catch urine culture colony counts indicates the patient likely has a urinary tract infection? a. 10^1 CFU/mL b. 10^3 CFU/mL c. 10^5 CFU/mL d. no growth

a (Protozoan trophozoites are fragile and begin to disintegrate as soon as they are passed. Liquid stool specimens should be preserved within 30 minutes of passage in order to adequately preserve parasite morphology. If a liquid specimen cannot be properly preserved, another specimen should be collected)

389) A liquid stool specimen is collected at 10:00 PM and brought to the laboratory for culture and ova and parasite examination It is refrigerated until 10:10 AM the next day, when the physician requests that the technologist look for amoebic trophozoites. The best course of action would be to: a. request a fresh specimen b. perform a concentration on the original specimen c. perform a trichrome stain on the original specimen d. perform a saline wet mount on the original specimen

c (Differential media contain compounds, often carbohydrates, that provide a presumptive identification based on colony color or a precipitate around the colony. Examples include MacConkey, Hektoen and xyloselysine desoxycholate agar.)

39) A medium that aids in the presumptive identification of organisms based on their appearance on the medium is called: a. enriched b. selective c. differential d. specialized

d (The distribution of data around the mean is the standard deviation. Using a specific confidence interval of 95.5%, or 2 standard deviations, the results fall between +/- 0 .2 [1 SD] or +/- O .4 [2 SD].)

394) On Monday a patient's Hgb determination was 11.3 g/dL and on Tuesday it measured 11.8 g/dL. The standard deviation of the method used is +/- 0.2 g/dL. Which of the following can be concluded about the Hgb values given? a. one value probably resulted from lab error b. there is poor precision, daily quality control chars should be checked c. the second value is out of range and should be repeated d. there is no significant change in the patient's Hgb concentration

c (The most common sign of Enterobius vermicularis infection is intense perianal itching.)

395) Refer to the following image: The egg depicted above is most likely to be found in children suffering from: a. diarrhea b. constipation c. perianal itching d. stomach pain

d (The ova of Enterobius vermicularis cannot be demonstrated in a routine ova and parasite examination. The adult female Enterobius worm migrates out of the anus and lays her eggs in the perianal folds. A scotch tape preparation of the skin of the perianal folds is used to collect ova.)

396) Refer to the following illustration: The specimen of choice for finding the above parasite is: a. stool b. duodenal washing c. rectal swab d. scotch tape preparation

d (Of the organisms listed, only Paragonimus and Hymenolepis can be identified to the species level by the appearance of their ova in stool.)

399) Organisms that can be easily identified to the species level from the ova in fecal specimens include: a. Metagonimus yokogawai, Heterophyes heterophyes b. Taenia solium, Taenia saginata c. Necator americanus, Ancylostoma duodenale d. Paragonimus westermani, Hymenolepis nana

d (Preanalytical [i.e. pre-examination] variables include all steps in the process prior to the analytic phase of testing, starting with the physician's order. Examples include accuracy of transmission of physicians' orders, specimen transport and preparation, requisition accuracy, quality of phlebotomy services, specimen acceptability rates, etc. This list is neither all-inclusive nor exclusive. The variables chosen should be appropriate to the laboratory's scope of care.)

4) Preanalytical variables in laboratory testing include: a. result accuracy b. report delivery to the ordering physician c. test turnaround time d. specimen acceptability

b (Storage at 4 C is sufficient for samples analyzed up to a week after collection. For longer periods [months or years] -20 is preferable.)

4) Sera to be tested for IFA-ANA 6 days after drawing is best stored at: a. room temp b. 5 C +/- 2 C c. -70 C in a constant temp freezer d. -20 C in a frost free self-defrosting freezer

d (A positive test for HbsAg at any time is an indefinite deferral.)

4) Which of the following constitutes permanent rejection status of a donor? a. a tattoo 5 months ago b. recent close contact with a patient with viral hepatitis c. 2 units of blood transfused 4 months previously d. confirmed positive test for HbsAg 10 yrs. earlier

d (Bilirubin is degraded by light. The other analytes will not be affected.)

4) Which of the following urine results is most apt to be changed by prolonged exposure to light? a. pH b. protein c. ketones d. bilirubin

d (Blood gas sample conditions.)

40) A blood gas sample was sent to the lab on ice, and a bubble was present in the syringe. The blood had been exposed to room air for at least 30 mins. The following change in blood gases will occur: a. CO2 content increased/PCO2 decreased b. CO2 content and PO2 increased/pH increased c. CO2 content and PCO2 decreased/pH decreased d. PO2 increased/HCO3 decreased

a (Patients with pernicious anemia have fewer of all types of blood cells, but they are abnormally large.)

40) Peripheral blood smears from patients with untreated anemia are characterized by: a. pancytopenia and macrocytosis b. leukocytosis and elliptocytosis c. leukocytosis and ovalocytes d. pancytopenia and microcytosis

c (The correct answer is c, 1 in the blue quadrant. The ratings range for both systems is O-4 with 0 being no hazard and 4 being the most severe hazard. The left blue quadrant represents a health hazard and a toxic chemical causes a health hazard. The rating of 1 would be a low or slight health hazard. The other distractors are incorrect due to the hazard type or rating.)

40) The HMIS or NFPA 704 hazardous material identification system rating for a slightly toxic chemical would be: a. 1 in the yellow quadrant b. 4 in the blue quadrant c. 1 in the blue quadrant d. 4 in the yellow quadrant

b (The rhabditiform larvae of hookworm and Strongyloides stercoralis can be differentiated by the size of their genital primordium. Strongyloides has a prominent genital primordium and a small buccal cavity. Hookworm larvae have a longer buccal cavity, and the genital primordium is not evident. S. stercoralis larvae have a notched tail whereas hookworm larvae have a tapering tail.)

402) A stool specimen for ova and parasite examination contained numerous rhabditiform larvae Which factor does not aid in the identification of larvae? a. larva tail morphology b. type of water vegetation consumed c. length of the buccal cavity d. appearance of the genital primordium

d (The correct answer is d, corrosive, which causes immediate damage to human tissue, such as a burn. Distractors a and b are health hazards, but generally do not cause immediate tissue damage. For distractor c, ignitable chemicals are both flammable and combustible and only will cause tissue damage if accidental flame or explosion occurs.)

41) A chemical that causes immediate visible destruction or irreversible alterations of human tissue at the contact site is best classified as: a. carcinogenic b. toxic c. ignitable d. corrosive

b (Anemia differentiation-vitamin B12 deficiency results in pernicious anemia [pancytopenia, macrocytosis].)

41) Lab tests performed on a patient indicate macrocytosis, anemia, leukopenia and thrombocytopenia. Which of the following disorders is the patient most likely to have? a. anemia of chronic disorder b. vitamin B12 deficiency c. iron deficiency d. acute hemorrhage

b (Electrolyte/blood gas values in respiratory acidosis.)

41) These results are most compatible with: a. respiratory alkalosis b. respiratory acidosis c. metabolic alkalosis d. metabolic acidosis

b (Cryoprecipitate is the fraction of plasma proteins that precipitate when FFP is slowly thawed at 1-6 C.)

42) A blood component prepared by thawing FFP at refrigerator temp and removing the fluid portion is: a. Plasma Protein Fraction b. Cryo AHF c. Factor IX Complex d. FP24

c (Best test for renal tubular function.)

42) Select the test which evaluates renal tubular function: a. IVP b. creatinine clearance c. osmolarity d. microscopic urinalysis

a (Antigen-antibody binding is governed by the law of mass action: free reactants are in equilibrium with bound reactants.)

42) Which of the following describes an antigen-antibody reaction? a. the reaction is reversible b. the reaction is the same as a chemical reaction c. a lattice is formed at prozone d. a lattice is formed at postzone

c (Fluorescein isothiocyanate fluoresces a visible green color when excited by UV.)

43) The most common label in direct fluorescent antibody technique is: a. alkaline phosphatase b. horseradish peroxidase c. fluorescein isothiocyanate d. calcofluor white

d (The correct answer is d, does not require labeling. The other distractors are incorrect and are only required on the original containers or secondary containers, which are not used immediately or by a different person.)

43) When hazardous chemicals are transferred from the original appropriately labeled containers to a secondary container for immediate use by the person performing the transfer, it: a. must be labeled with an emergency response phone number b. must be labeled with the identity or contents of the hazardous chemical c. must be labeled with hazard warning related to the effect on involved target organs d. does not require labeling

c (First react the target with an unlabeled antibody, then follow with a fluorescent dye.)

44) A substrate is first exposed to a patient's serum, then after washing, anti-human immunoglobulin labeled with a fluorochrome is added. The procedure described is: a. fluorescent quenching b. direct fluorescence c. indirect fluorescence d. fluorescence inhibition

c (Reagent strips must he handled carefully to prevent them from picking up excess moisture. Heating or refrigeration is not appropriate. They work optimally at room temperature. They should be stored in a dark, tightly capped bottle, not exposed to light.)

44) Urine reagent strips should be stored in a a. refrigerator (4 - 7 C) b. incubator (37 C) c. cool dry area d. open jar exposed to air

b (To ensure the reproducibility of disk diffusion testing, the inoculum must be standardized. If the inoculum is too dense [too many organisms], zone sizes would be smaller than expected and appear falsely resistant.)

44) Which of the following factors would make an organism appear to be more resistant on a disk diffusion susceptibility test? a. too little agar in the plate b. too many organisms in the inoculum c. the presence of 0.5% NaCl in the medium d. a medium with a pH of 7.4

c (The correct answer is c, evacuate the area. The other distractors are secondary responses after evacuating the area.)

45) A technologist spilled 10 gallons of formaldehyde on the floor. After determining the chemical poses a significant health hazard. the first action step would be to: a. notify emergency assistance b. control the spill with appropriate absorbent material c. evacuate the area d. don appropriate personal protective equipment

d (Describes the overall interaction of an antibody with its antigen.)

45) Avidity may be defined as the: a. degree of hemolysis b. titer of an antigen c. dilution of an antibody d. strength of a reacting antibody

c (Cephalothin is a first-generation cephalosporin, cefotetan and cefoxitin are second-generation cephalosporins, and ceftriaxone is a third generation cephalosporin.)

45) First generation cephalosporins can be adequately represented by: a. cefotetan b. ceftriaxone c. cephalothin d. cefoxitin

b (Definition of osmolal gap.)

45) Osmolal gap is the difference between: a. the ideal and real osmolality values b. calculated and measured osmolality values c. plasma and water osmolality values d. molality and molarity at 4*C

a (Myelofibrosis is often accompanied by folate deficiency, which causes macrocytic anemia.)

45) The most likely cause of the macrocytosis that often accompanies anemia of myelofibrosis is: a. folic acid deficiency b. increased reticulocyte count c. inadequate B12 absorption d. pyridoxine deficiency

b (Penicillin inhibits penicillin binding proteins that are essential to peptidoglycan [cell wall] synthesis. Chloramphenicol inhibits protein synthesis, colistin increases cell membrane permeability, and sulfamethoxazole inhibits folate metabolism.)

46) An antibiotic that inhibits cell wall synthesis is: a. chloramphenicol b. penicillin c. sulfamethoxazole d. colistin

b (A rising titer of antibody during the progression and resolution of an illness is an indication of infection with an organism.)

46) In the interpretation of agglutination tests for febrile diseases, which of the following is of the greatest diagnostic importance? a. anamnestic reactions caused by heterologous antigens b. rise in titer of the patient's serum c. history of previous vaccination d. naturally occurring antibodies prevalent when the disease is endemic

c (Most important buffer pair in plasma.)

46) The most important buffer pair in plasma is the: a. phosphate/biphosphate b. hemoglobin/imidazole c. bicarbonate/carbonic acid d. sulfate/bisulfate

a (Whole blood-derived Platelets are prepared by a light spin to separate the Red Blood Cells from the platelet-rich plasma [PRP], followed by a heavy spin of the PRP to concentrate the platelets.)

46) Which of the following is proper procedure for preparation of platelets from Whole Blood: a. light spin followed by hard spin b. light spin followed by 2 hard spins c. 2 light spins d. hard spin followed by a light spin

d (VDRL antigen contains 0.9% cholesterol.)

47) Cholesterol is added to the antigen used in flocculation tests for syphilis to: a. destroy tissue impurities present in the alcoholic beef heart extract b. sensitize the sheep RBC's c. decreased specificity of the antigen d. increase sensitivity of the antigen

b (Components of anion gap.)

48) What battery of tests is most useful in evaluating an anion gap of 22 mEq/L (22 mmol/L)? a. Ca++, Mg++, PO-4, and pH b. BUN, creatinine, salicylate and methanol c. AST, ALT, LD and amylase d. glucose, CK, myoglobin, and cryoglobulin

d (Aminoglycoside antibiotics such as gentamicin are active against Pseudomonas and routinely tested and reported on these isolates. Penicillin, erythromycin, and clindamycin are not active against Pseudomonas.)

48) Which of the following antibiotics would routinely be tested and reported for isolates of Pseudomonas aeruginosa? a. penicillin b. erythromycin c. clindamycin d. gentamicin

d (Iron studies in anemia of chronic disease show decreased serum iron and transferrin saturation; TIBC may be decreased as well.)

48) Which of the following represents characteristic features of iron metabolism in patients with anemia of a chronic disorder? a. result A b. result B c. result C d. result D

c (Effect of K+ from platelets on serum K)

49) A patient with a myeloproliferative disorder has the following values: The serum ICE should be confirmed by: a. repeat testing of the original serum b. testing freshly drawn serum c. testing heparinized plasma d. atomic absorption spectrometry

b (Whole blood-derived [random donor] Platelets should contain at least 5.5 x 10^10 platelets, be stored with continuous agitation at 20-24 C, and have a pH of 6.2 or higher when tested at the end of the storage period.)

49) A unit of Whole Blood-derived (random donor) Platelets should contain at least: a. 1.0 x 10^10 platelets b. 5.5 x 10^10 platelets c. 5.5 x 10^11 platelets d. 90% of the platelets from the original unit of Whole Blood

a (Non-identity: each antigen forms an independent precipitin line with the corresponding antibody at an equivalence point.)

49) Which of the following describes an antigen antibody precipitation reaction of non-identity? a. precipitin lines cross, forming double spurs b. precipitin lines fuse, forming a single spur c. no precipitin line are formed d. precipitin lines fuse, forming a single arc

a (Most Campylobacter species grow best under lower oxygen tension in an atmosphere of 5% oxygen, 10% carbon dioxide and 85% nitrogen. E coli and Proteus mirabilis are facultative anaerobes and Pseudomonas aeruginosa is an aerobe.)

49) Which of the following must be incubated in a microaerophilic environment for optimal recovery of the organism? a. Campylobacter jejuni b. Escheria Coli c. Pseudomonas aeruginosa d. Proteus mirabilis

d (The protein pad must be held at a pH of 3 in order to see the effect that protein has on the double indicators. In alkaline urine, the pH of 3 may be neutralized, and the indicators change color.)

53) When employing the urine reagent strip method, a false-positive protein result may occur in the presence of : a. large amounts of glucose b. x-ray contrast media c. Bence Jones protein d. highly alkaline urine

c (Polycythemia, in burn patient.)

53) Which of the following types of polycythemia is a severely burned patient most likely to have? a. polycythemia vera b. polycythemia, secondary to hypoxia c. relative polycythemia associated with dehydration d. polycythemia associated with renal disease

b (Ketones are byproducts of fat metabolism. During low carbohydrate or starvation diets, ketones can be found in the urine.)

54) A 17 yr. old girl decided to go on a starvation diet. After 1 week of starving herself, what substance would most likely be found in her urine? a. protein b. ketones c. glucose d. blood

b (Ratio of CSF glucose to blood glucose)

5) A healthy person with a blood glucose of 80 mg/dL (4.4 mmol/L) would have a simultaneously determined cerebrospinal fluid glucose value of: a. 25 mg/dL (1.4 mmol/L) b. 50 mg/dL (2.3 mmol/L) c. 100 mg/dL (5.5 mmol/L) d. 150 mg/dL (8.3 mmol/L)

b (Chromosomes with clearly defined short and long arms are known as submetacentric chromosomes.)

5) A metaphase chromosome with primary constriction that gives the chromosome clearly defined short and long arms is considered: a. acrocentric b. submetacentric c. metacentric d. telocentric

a (Antigen target in the homogeneous pattern is DNA.)

5) Antibodies directed at native DNA are most frequently associated with which pattern of fluorescence in the IFA-ANA test? a. rim b. diffuse c. speckled d. centromere

a (If multiple controls are out of range and the instrument and reagents are verified, recalibration or calibration verification is required before subsequent control analysis.)

5) The first procedure to be followed if the blood gas instrument is out-of-control for all parameters is: a. recalibrate, then repeat control b. repeat control on the next shift c. replace electrodes, then repeat control d. report patient results after duplicate testing

a (The traditional gravity displacement steam sterilization cycle is 121 C for 15 minutes at 15 pounds per square inch. Ethylene oxide is an alternative sterilization method.)

5) The steam autoclave method of sterilization: a. uses 15 lbs. of pressure for 15 minutes b. utilizes dry heat for 20 minutes c. produces a minimum temp of 100 C d. requires a source of ethylene oxide

c (Evaluate each statement. a is incorrect because these cells don't agglutinate. b is partially correct, but urobilinogen decreases in light. c is true, bacterial overgrowth does lead to an alkaline urine. d is false, ketones are produced by fat metabolism in the patient.)

5) Urine samples should be examined within 1 hr. of voiding because: a. RBC's, leukocytes and casts agglutinate on standing for several hrs. at room temp b. urobilinogen increases and bilirubin decreases after prolonged exposure to light c. bacterial contamination will cause alkalinization of the urine d. ketones will increase due to bacterial and cellular metabolism

a (The student should memorize the chemical reactions for each of the dipstick biochemicals. Diazo reagent is used for bilirubin.)

50) A reagent strip area impregnated with stabilized, diazotized 2,4-sichloroaniline will yield a positive reactions with: a. bilirubin b. hemoglobin c. ketones d. urobilinogen

c (Apheresis [single donor] Platelets should contain at least 3.0 x 10^11 platelets, be stored with continuous agitation at 20-24 C, and have a pH of 6.2 or higher when tested at the end of the storage period.)

50) Platelets prepared by apheresis should contain at least: a. 1 x 10^10 platelets b. 3 x 10^10 platelets c. 3 x 10^11 platelets d. 5 x 10^11 platelets

d (Thalassemia cause.)

50) Thalassemias are characterized by: a. structural abnormalities in the hemoglobin molecule b. absence of iron in hemoglobin c. decreased rate of heme synthesis d. decreased rate of globin synthesis

c (ELISA is a solid phase immunoassay that uses anti-immunoglobulins that are labeled with an enzyme that can be detected by the appearance of color on the addition of a substrate.)

50) Which test has the greatest sensitivity for antigen detection? a. precipitation b. agglutination c. ELISA d. complement fixation

d (Precipitation does not occur due to lack of free antibody.)

51) Excess antigen in precipitation gel reactions will: a. have no effect on the precipitate reaction b. not dissolve precipitate after formation c. enhance the precipitate reaction d. dissolve the precipitate after formation

c (The correct answer is c, designate a laboratory chemical hygiene officer.)

51) One of the elements of a written laboratory chemical hygiene plan is to: a. require employees who handle chemicals to have annual medical evaluations b. prohibit use of carcinogens c. designate a laboratory chemical hygiene officer d. perform chemical monitoring every 6 months of OSHA regulated substances

d (Anion gap, lactic acidosis.)

51) Serum anion gap is increased in patients with: a. renal tubular acidosis b. diabetes alkalosis c. metabolic acidosis due to diarrhea d. lactic acidosis

d (TCBS is a highly selective and differential medium for the recovery of most Vibrio species including V parahaemolyticus. Hektoen and Salmonella-Shigella agars are selective and differential for the isolation and differentiation of enteric pathogens such as Salmonella and Shigella. EMB is a selective and differential medium for gram negative enteric bacilli.)

51) Vibrio parahaemolyticus can be isolated best from feces on: a. eosin methylene blue agar b. Hektoen Shigella agar c. Salmonella Shigella agar d. thiosulfate citrate bile salts agar

c (Calculation of anion gap.)

52) The anion gap is useful for quality control of laboratory results for: a. amino acids and proteins b. blood gas analyzers c. Na, K, Cl, and CO2 d. Ca, Ph, Mg

d (The correct answer is d, hazardous chemicals.)

52) The purpose of the OSHA hazard Communication , General Industry Standard, 29 CFR, Subpart Z, 1910.1200, is to require employees to establish a program ensuring personnel are provide with information regarding the workplace dangers of: a. bloodborne pathogens b. environmental hazards c. general safety hazards d. hazardous chemical

a (The apheresis process is to remove whole blood, the desired component removed, and the remaining portion of blood returned to the donor/patient.)

53) A father donating platelets for his son is connected to a continuous flow machine, which uses the principle of centrifugation to separate platelets from whole blood. As the platelets are harvested, all other remaining elements are returned to the donor. This method of platelet collection is known as: a. apheresis b. autologous c. homologous d. fractionation

c (As many as 20%-40% of H influenzae produce beta-lactamases. Detection of these enzymes should be performed on any isolate considered to be a pathogen using the chromogenic cephalosporin [nitrocefin] test.)

53) Tests for beta-lactamase production in Haemophilus influenzae: a. are not commercially available b. include tests that measure a change to an alkaline pH c. should be performed on all blood and CSF isolates d. are not valid for any other bacterial species

b (Precipitation takes place when antibodies and soluble antigens are mixed.)

53) The visible serological reaction between soluble antigen and its specific antibody is: a. sensitization b. precipitation c. agglutination d. opsonization

a (Healthcare workers must be knowledgeable of chemical safety signage as they may be using chemicals as preservatives in specimens for transport. The flammable sign is from the Department of Transportation Hazardous Materials Warning Signs.)

53) This symbol indicates which of the following hazards? a. flammable b. electrical c. radiation d. biohazard

c (Buffered charcoal yeast extract medium is a specialized enrichment medium for the isolation of Legionella. The nutritive base includes yeast extract. Charcoal is added to the medium as a detoxifying agent.)

54) Media used to support growth of Legionella pneumophila should contain which of the following? a. X and V factors b. hemin and Vitamin K c. charcoal and yeast extract d. dextrose and laked blood

c (The proportion of antigen to antibody is optimal for maximal precipitation.)

54) The curve below was obtained by adding increasing amounts of a soluble antigen to fixed volumes of monospecific antiserum: The area on the curve for equivalence precipitate is: a. A b. B c. C d. D

b (Autologous donors have less stringent criteria than allogeneic donors. Donations must be collected at least 72 hours prior to surgery.)

54) To qualify as a donor for autologous transfusion a patient's hemoglobin should be at least: a. 8 g/dL (80 g/L) b. 11 g/dL (110 g/L) c. 13 g/dL (130 g/L) d. 15 g/dL (150 g/L)

b (The correct answer is b, use a mechanical device. Distractors a and c do not afford adequate protection and d leads to recapping injuries vs using a one-handed technique.)

54) When working with sharp equipment and objects use: a. double-glove technique with specimen handling gloves b. mechanical device c. paper towel or gauze as a barrier d. two-handed technique

c (Lead poisoning-basophilic stippling.)

54) Which of the following is most likely to be seen in lead poisoning? a. iron overload in tissue b. codocytes c. basophilic stippling d. ringed sideroblasts

a (Vomiting leads to dehydration and utilization of fat for energy. Fat metabolism produces ketones.)

55) A 2 yr. old child had a positive urine ketone. This would most likely be caused by: a. vomiting b. anemia c. hypoglycemia d. biliary tract obstruction

d (The correct answer is d, ensure proper balance is maintained. For distractor a, a disinfectant should be used. For distractor b, the cover should not be opened until the centrifuge comes to a complete stop. For distractor c, all specimen tubes need to be covered to prevent aerosol formation.)

55) For safe operation of a centrifuge: a. clean with soap/detergent when maintenance is performed or spills occur b. open the centrifuge cove when it is in the process of slowing down c. leave liquid specimen tubes uncovered during centrifugation d. ensure proper balance is maintained

a (Regan-Lowe agar is an enriched and selective medium for the isolation of B pertussis. Cephalexin is added to inhibit nasopharyngeal flora. It provides better isolation of B pertussis than Bordet-Gengou medium.)

55) The best medium for culture of Bordetella pertussis is: a. Regan-Lowe agar b. cystine blood agar c. Martin Lewis agar d. Ashdown agar

b (Excess of antigen results in soluble complexes.)

55) The curve below was obtained by adding increasing amounts of a soluble antigen to fixed volumes of monospecific antiserum: The area on the curve where no precipitate formed due to antigen excess is: a. A b. B c. C d. D

a (Only ABO and Rh is required with the patient's sample. Each autologous unit must be confirmed for ABO and Rh from an integrally attached segment.)

55) What is/are the minimum pretransfusion testing requirement(s) for autologous donations collected and transfused by the same facility? a. ABO and Rh typing only b. ABO/Rh type, antibody screen c. ABO/Rh type, antibody screen, crossmatch d. no pretransfusion testing is required for autologous donations

a (Effect of hemolysis on K.)

56) A potassium level of 6.8 mEq/L (6.8 mmol/L) is obtained. Before reporting the results the first step the technologist should take is to: a. check the serum for hemolysis b. rerun the test c. check the age of the patient d. do nothing, simply report out the result

b (Francisella tularensis is fastidious and not readily recovered in culture. Cysteine blood glucose agar is an enriched medium with beef heart infusion, peptones, glucose and rabbit blood. It also includes cystine, which is required by F tularensis for growth.)

56) The best medium for culture of Francisella tularensis is: a. Bordet-Gengou medium b. cystine glucose blood agar c. Loeffler medium d. charcoal selective medium

a (Prozone - suboptimal precipitation occurs in the region of antibody excess. Prozone effect - Occasionally, it is observed that when the concentration of antibody is high [i.e., lower dilutions], there is no agglutination and then, as the sample is diluted, agglutination occurs. The lack of agglutination at high concentrations of antibodies is called the prozone effect. Lack of agglutination in the prozone is due to antibody excess resulting in very small complexes that do not dump to form visible agglutination.")

56) The curve below was obtained by adding increasing amounts of a soluble antigen to fixed volumes of monospecific antiserum: The area on the curve for prozone is: a. A b. B c. C d. D

c (The correct answer is c, be filed for all incidents, including near miss incidents. For distractor a, employee's medical history is confidential For distractor b, all incidents require an incident report be filed. For distractor d, incidents reports need to be maintained for a defined period of time.)

57) Incident reports for occupational injury or illness should: a. include information on the employee's past medical history b. be filed only for incidents involving serious injury of illness c. be filed for all incidents including near miss incidents d. not be retained after review by a safety committee or officer

a (The correct answer is a and the other distractors are incorrect and used for other purposes as listed in the answers. The word immediate is the key to this answer.)

58) According to OSHA, what type of sign should be posted in an area where an immediate hazard exists and where special precautions are necessary? a. red, black and white "Danger" sign b. yellow, and black "Caution" sign c. green and white "Safety Instruction" sign d. orange and black "Biohazard " sign

a (Most common cause of low phosphate.)

70) A low concentration of serum phosphorus is commonly found in: a. patients who are receiving carbohydrate hyperalimentation b. chronic renal disease c. hypoparathyroidism d. patients with pituitary tumors

a (The mother has a 50% chance of passing on R1 and 50% chance of passing on r. The father will always pass on R1. Statistically, 50% of the children will be R1r and 50% of them will be R1R1.)

58) Given the most probable genotypes of the parents, which of the following statements best describes the most probable Rh genotypes of the 4 children? a. 2 are R1r, 2 are R1R1 b. 3 are R1r, 1 is rr c. 1 is R0r, 1 is R1r, 2 are R1R1 d. 1 is R0r', 1 is R1R1, 2 are R1r

a (Ascorbic acid inhibits reactions that use peroxidase.)

58) Microscopic analysis of a urine specimen yields a moderate amount of RBCs in spite of a negative result for occult blood using a reagent strip. The technologist should determine if this patient has taken: a. vitamin C b. a diuretic c. high blood pressure medication d. antibiotics

c (RBC morphology in lead poisoning.)

58) The characteristic morphologic feature in lead poisoning is: a. macrocytosis b. target cells c. basophilic stippling d. rouleaux formation

b (In the figure, B is the zone of antibody excess.)

58) The curve below was obtained by adding increasing amounts of a soluble antigen to fixed volumes of monospecific antiserum: The area in which the addition of more antibody would result in the formation of additional precipitate is: a. A b. B c. C d. D

a (Sheep blood agar is preferred because clear-cut patterns of hemolysis are obtained.)

58) When processing throat swabs for a group A streptococcus culture, the medium of choice is: a. sheep blood agar b. rabbit blood agar c. human blood agar d. horse blood agar

b (The correct answer is b, be grounded or double insulated. For distractor a, repairs should not be conducted while connected to facility wiring. For distractor c, safety checks should be performed initially, annually and whenever repairs are made. For distractor d, multiple outlet adapters are unsafe and should not be used.)

59) All laboratory instruments should: a. have repairs conducted while connected to facility wiring b. be grounded or double insulated c. have safety checks performed initially and then every 6 months d. be connected to multiple outlet adapters

c (For optimum detection of oxacillin-resistant S aureus, a suspension with a turbidity equivalent to a 0.5 McFarland standard should be inoculated into a cation adjusted Mueller-Hinton broth with 2% NaCl. Plates should be incubated at 35 +/- 2 C for 24 hours. Temperatures above 35 C may not detect oxacillin resistance.)

59) The ability to detect oxacillin resistant staphylococcus aureus may be enhanced by: a. shortening incubation of standard susceptibility plates b. incubation of susceptibility plates at 39 C - 41 C c. using Mueller-Hinton broth with 2% NaCl d. adjusting inoculum of 0.1 McFarland before inoculating susceptibility plates

c (Use of partial GTT information.)

6) A 25 yr. old man became nauseated and vomited 90 mins after receiving a standard 75 g carbohydrate dose for an oral glucose tolerance test. The best course of action is to: a. give the patient a glass of orange juice and continue the test b. start the test over immediately with a 50 g carbohydrate dose c. draw blood for glucose and discontinue the test d. place the patient in a recumbent position, reassure him and continue the test.

b (The Hct must be >38%. A donor may be 16 unless state law differs. Temperature must not exceed 99.5F/37.5C, blood pressure must be <180 mmHg systolic and <100 mmHg diastolic, pulse 50-100 unless an athlete [which can be lower]. Toxoids and vaccines from synthetic or killed sources have no deferral.)

6) Below are the results of the history obtained from a prospective female blood donor: - age: 16 - temperature: 99.0F (37.2C) - Hct: 36% - history: tetanus toxoid immunization 1 week previously How many of the above results excludes this donor from giving blood for a routine transfusion? a. none b. 1 c. 2 d. 3

b (Homogeneous pattern may indicate the presence of anti-DNA antibodies for both single- or double-stranded DNA.)

6) The tech observes apparent homogenous staining of the nucleus of interphase cells while performing an IFA-ANA, as well as staining of the chromosomes in mitotic cells. This result is: a. indicative of 2 antibodies, which should be separately reported after titration b. expected for anti-DNA antibodies c. inconsistent; the test should be reported with new reagent d. expected for anti-centromere antibodies

d (Development time line of production of Hgb beta chains.)

6) Which curve represents the production of beta polypeptide chains of hemoglobin? a. B b. C c. E d. D

d (The correct answer is d, touch a "contaminated" area phone with ungloved hands if hands are washed afterward. For distractor a, bench tops are to be cleaned after spills and at the end of each shift. For distractor b, personal protective equipment, such as a lab coat, is to be removed prior to leaving a "contaminated" area. For distractor c, cosmetics and lip balm are not to be applied in a "contaminated" area.)

60) If areas of the laboratory are designated as "clean " or "contaminated" it is appropriate for a technologist to: a. clean technical are bench tops after spills and on a weekly basis b. wear a lab coat in the break or lunch room c. apply lip balm in a contaminated area d. touch a contaminated area phone with ungloved hands if hands are washed afterwards

c (The use of swabs for collection of specimens for anaerobic culture is discouraged. Aspiration with a needle and syringe is recommended. Whenever possible cultures should be obtained before the administration of antibiotics to optimize organism recovery.)

60) The optimal wound specimen for cultures of anaerobic organisms should be: a. a swab of lesion obtained before administration of antibiotics b. a swab of lesion obtained after administration of antibiotics. c. a syringe filled with pus, obtained before administration of antibiotics d. a syringe filled with pus, after administration of antibiotics

b (Many anaerobic bacteria are commensal flora in the oropharynx. Anaerobic bacteria do not cause pharyngitis. The most common cause of pharyngitis is Streptococcus pyogenes. Other causes include Arcanobacterium haemolyticum, Corynebacterium diphtheriae, Neisseria gonorrhoeae and viruses.)

61) A 21 yr. old patient presents with pharyngitis. A throat swab is collected and submitted for anaerobic culture. This specimen should be: a. set up immediately b. rejected as unacceptable c. inoculated into thioglycollate broth d. sent to a reference lab

c (Identity arc indicates that the 2 antigens are identical in figure #1. Figure #2, partial identity, shares a determinant that is part of antigen #1.)

61) Which of the above figures demonstrates a reaction pattern showing 2 different antigenic molecular species? a. Figure #1 b. Figure #2 c. Figure #3 d. Figure #4

d (In infectious mononucleosis, the leukocyte is usually increased due to an absolute lymphocytosis. The platelet count is often mildly decreased.)

62) A 14-year-old boy is seen in the ER complaining of a sore throat swollen glands and fatigue. The CBC results are: What is the most likely diagnosis? a. acute lymphocytic leukemia b. chronic lymphocytic leukemia c. viral hepatitis d. infectious mononucleosis

a (This individual cannot have the k antigen on their cells. KOKO is rare and no Ken system antigens are detected on the red blood cells. Those individuals usually produce antibodies that are reactive with all normal cells. KK is the most probable genotype.)

62) An individual has been sensitized to the k antigen and has produced anti-k. What is her most probable Kell system genotype: a. KK b. Kk c. kk d. K0K0

a (3% SSA is used to confirm positive protein tests. 3% implies 3 g in 100 mL.)

62) To prepare the reagent used in confirmatory protein testing, a technician would: a. dissolve 3 g sulfosalicylic acid in 100 mL of water b. dissolve 5 g trichloroacetic acid in 100 mL of water. c. combine 3 mL of hydrochloric and 97 mL of water d. combine 5 mL of glacial acetic acid and 95 mL of water

c (The symbol in the figure represents a common labeling system for hazardous material identification, such as HMIS or NFPA 704 and the correct answer is c, chemical hazardous materials. All other distractors are incorrect. Distractors a and b do not use the NFPA hazard label at all, and there is more appropriate signage for radiation. If a chemical is radioactive, there would be a radioactive symbol in the bottom white area.)

62) What type of identification system does this symbol represent? a. transmission-based precautions b. physical environmental hazards c. chemical hazardous materials d. radiation hazards

b (Cyclohexamide, which inhibits protein synthesis, is the common agent used in Mycosel or mycobiotic agar to inhibit faster-growing saprophytic fungi. Penicillin and streptomycin do not inhibit fungi. Amphotericin B is not routinely used as an additive in fungal media.)

63) An antibiotic used to suppress or kill contaminating fungi in media is: a. penicillin b. cycloheximide c. streptomycin d. amphotericin B

d (The correct answer is d, approved safety can. For distractor a, flammable safety cabinets should not be vented to room air. For distractor b, flammable and combustible liquids that require refrigeration are only to be stored in an explosion-proof refrigerator. For distractor c, hazardous substances, particularly of that volume, are not to be stored in a fume hood.)

63) Flammable and combustible liquids in containers > 5 gallons should be stored in a(n) a. flammable safety cabinet vented to room air b. nonexplosive proof refrigerator c. fume hood d. approved safety can

a (Fy[a-b-] individuals are very rare with all populations other than the individual of African descent. 68% of African Americans are Fy[a-b-].)

63) Given the following typing results, what is the donors racial ethnicity: Le(a-b-) Fy(a-b-) Js(a+b+) a. African American b. Asian American c. Native American d. Caucasian

b (When the shape or size of the red blood cells prevents rouleaux formation a decreased or low ESR is expected. This is observed with sickle cells, acanthocytes, and spherocytes.)

63) Which of the following technical factors will cause a decreased erythrocyte sedimentation rate? a. gross hemolysis b. small fibrin clots in the sample c. increased room temp d. tilting of the tube

a (The baby is Rh-negative and lacks c, since there is no evidence of HDFN. Inheritance of no D and no c is denoted as r'. The baby must have inherited this gene from both parents, and is homozygous r'r'.)

64) A mother has the red cell phenotype D+C+E-c-e+ with anti-c (titer of 32 at AHG) in her serum. The father has the phenotype D+C+E-c+e+. The baby is Rh-negative and not affected with hemolytic disease of the newborn. What is the baby's most probable Rh genotype? a. r'r' b. r'r c. R1R1 d. R1r

b (The correct answer is b, pull the pin. The sequence of actions should be pull the pin, aim the hose, squeeze the lever, and sweep the flow.)

64) After receiving appropriate training, the first step in using a fire extinguisher is to: a. sweep the flow of the hose from side to side b. pull the pin c. squeeze the top handle or lever d. aim the hose at the bottom of the fire

d (Both hemoglobin and myoglobin will produce a red urine without RBCs. Hemoglobin will be precipitated by ammonium sulfate, but myoglobin will not. Myoglobin will remain in the supernate after centrifugation.)

64) Ammonium sulfate was added to red urine. The urine had a positive reaction for blood, but no RBCs were seen on microscopic exam. After centrifugation, the supernatant fluid is red. The abnormal color is caused by: a. pyriduim b. hemoglobin c. porphyrins d. myoglobin

c (There should be complete hemolysis in the streptolysin O control tube.)

64) The enzyme control tube in an ASO hemolytic assay exhibits no cell lysis. What is the most likely explanation for this? a. incorrect pH of buffer b. low ionic strength buffer c. oxidation of the enzyme d. reduction of the enzyme

b (Regulation of Ca++ and P043- metabolism.)

64) The regulation of calcium and phosphorous metabolism is accomplished by which of the following glands? a. thyroid b. parathyroid c. adrenal gland d. pituitary

d (Patients unable to synthesize normal amounts of hemoglobin show reduction in the MCH.)

64) Which of the RBC indices is a measure of the amount of hemoglobin in individual RBC's? a. MCHC b. MCV c. Hct d. MCH

a (Hyperparathyroidism.)

65) A patient has the following results: increased serum calcium decreased serum phosphate increased levels of parathyroid hormone: This patient most likely has: a. hyperparathyroidism b. hypoparathyroidism c. nephrosis d. steatorrhea

d (There are several sputum-screening systems for assessing the quality of respiratory specimens. In general, neutrophils are a positive indicator of quality, and squamous epithelial cells are a negative indicator of quality, suggesting oropharyngeal contamination. This specimen contains an abundance of squamous cells [> 10/low power field], and would be unacceptable for culture.)

65) A sputum specimen is received for culture and Gram stain. The Gram stained smear from this specimen is seen in the image (total magnification 100x): The technologist's best course of action would be to: a. inoculate appropriate media and incubate anaerobically b. inoculate appropriate media and incubate aerobically c. call the physician and notify him of this "life-threatening" situation d. call the patient care area and request a new specimen

a (RDW-CV is a new parameter available from automated instruments.)

65) The RDW-CV and RDW-SD performed by automated cells counters are calculations that provide: a. an index of the distribution of RBC volume b. a calculated mean RBC hemoglobin concentration c. a calculated MCH d. the mean RBC volume

c (Prozone-suboptimal precipitation occurs in the region of antibody excess.)

65) The following pattern of agglutination was observed in an antibody titration: This set of reactions most likely resulted from: a. faulty pipetting technique b. postzoning c. prozoning d. the presence of a high-titer, low-avidity antibody

a (Tetany and calcium.)

66) A hospitalized patient is experiencing increased neuromuscular irritability (tetany). Which of the following tests should be ordered immediately? a. calcium b. phosphate c. BUN d. glucose

d (ASO is an enzyme inhibition test. In the ASO test, a serum results in inhibition of the reagent's enzymatic ability to lyse human red blood cells.)

66) In a positive anti-streptolysin "O" enzyme inhibition test, the patient's: a. streptolysin "O" enzyme in the patient serum neutralizes the anti-streptolysin "O" reagent, resulting in no hemolysis b. RBC are hemolyzed by the streptolysin "O" enzyme in the reagent. c. anti-streptolysin "O" neutralizes the streptolysin "O" reagent, resulting in hemolysis d. anti-streptolysis "O" inhibits the reagent streptolysin "O" resulting in no hemolysis

b (The correct answer is b, bend at the knees and hips, lift with legs and buttocks. For distractors a and d, you should not bend at the waist. For distractor c, you should not lift with your back. For distractor d, you should not twist.)

66) In addition to keeping the load close to your body and tightening you abdominal muscles when lifting heavy boxes of supplies, it is important to bend at the: a. waist; lift with legs and buttocks b. knees and hips; lift with legs and buttocks c. knees and hips; lift with arms and back d. waist; twist your body when lifting

a (Tilting of the tube accelerates the fall of the red blood cells.)

66) The erythrocyte sedimentation rate can be falsely elevated by: a. tilting the tube b. refrigerated blood c. air bubbles in the column d. specimen being to old

b (It is not uncommon to find a urine specimen with positive protein. It is necessary to confirm positive protein if the urine pH is elevated, but this is not one of the choices. The choice of b will allow all results to be correct, but the choice of d would cause some of the results to be wrong.)

66) When performing a routine urinalysis, the technologist notes a 2+ protein result. He should: a. request another specimen b. confirm with the acid precipitation test c. test for Bence Jones protein d. report the result obtained without further testing

d (When reading a broth microdilution susceptibility test, growth in each well is determined by comparison with the growth control well and indicated by turbidity. The well with the lowest concentration of antibiotic displaying no growth is read as the minimum inhibitory concentration [MIC].)

67) Examine the broth microdilution susceptibility test shown above and determine the MIC for gentamicin. a. >64 mg/mL b. 32 mg/mL c. 16 mg/mL d. <2 mg/mL

c (Sulfosalicylic acid will precipitate protein. Ehrlich reagent is for urobilinogen. Diazo is for bilirubin, and copper reduction is for reducing substances.)

67) The confirmatory test for a positive protein result by the reagent strip method uses: a. Ehrlich reagent b. a diazo reaction c. sulfosalicylic acid d. a copper reduction tablet

c (The Xg blood group system is unique in that the gene encodes on the X chromosome. A negative mother would not have the Xg[a] to pass on. A positive father would, however, transmit the Xg[a] to all his daughters.)

67) The mating of an Xg(a+) man and a Xg(a-) women will only produce: a. Xg(a-)sons + Xg(a-) daughters b. Xg(a+)sons + Xg(a+)daughters c. Xg(a-) sons + Xg(a+) daughters d. Xg(a+) sons + Xg(a-) daughters

a (Tetany and magnesium.)

67) Which of the following is most likely to be ordered in addition to serum calcium to determine cause of tetany? a. magnesium b. phosphate c. sodium d. vitamin D

b (A positive glucose oxidase is specific for glucose. Glucose will also cause copper reduction [a positive Clinitest])

68) A urine specimen is analyzed for glucose by a glucose oxidase reagent strip and a copper reduction test. If both results are positive, which of the following interpretations is correct? a. galactose is present b. glucose is present c. lactose is not present d. sucrose is not present

d (All common Rh antigens are present on the red blood cells. R1 [DCe] and R2 [DcE] are frequent genotypes.)

68) Refer to the following data: Given the reactions above, which is the most probable genotype? a. R1R1 b. R1r' c. R0r" d. R1R2

b (Cell culture methods are best used during the acute phase of a viral infection, when the viral titer is high. Serological techniques, which detect antibody to the virus, would be better in detecting viral infection during the convalescence phase, when the patient is recovering and the number of viral particles bas decreased.)

68) Which of the following is most useful in establishing a diagnosis in the convalescence phase of a viral infection? a. slide culture b. serological techniques c. shell vial d. culture on McCoy media

d (The absolute concentration of each type of cell in a WBC differential is important for determining an increase or decrease.)

68) Which of the following is the formula for absolute cell count? a. number of cells counted /total count b. total count/number of cells counted c. 10 x total count d. % of cells counted x total count

a (R0R0 is the only correct choice here. R0 = D+C-E-c+e+.)

69) A patient's red cells type as follows: Which of the following genotype would be consistent with these results? a. R0R0 b. R1r c. R1R2 d. Rzr

c (Regulation of phosphate.)

69) Fasting serum phosphate concentration is controlled primarily by the: a. pancreas b. skeleton c. parathyroid glands d. small intestine

d (Serological tests are commonly used to detect antibodies in infections with viruses that are difficult to culture, such as rubella, HIV, and EBV. EIA is a common serological method because it is sensitive, specific, and can be automated.)

69) The best method to detect infections due to rubella, Epstein-Barr and HIV is: a. antigen detection by EIA b. cell culture c. antigen detection by western blot d. antibody detection by ELISA

b (Storage must inhibit bacterial growth but not kill the bacteria. Freezing and additives are not acceptable. The most commonly used method of preservation is refrigeration.)

7) A urine specimen comes to the laboratory 7 hrs. after it is obtained. It is acceptable for culture only if the specimen has been stored: a. at room temp b. at 4 - 7 C c. frozen d. with a preservative additive

d (Facultative anaerobes are organisms that can grow under both aerobic and anaerobic conditions.)

7) An aspirate of a deep wound was plated on blood agar plates and incubated aerobically and anaerobically. At 24 hrs. there was growth on both plates. This indicates that the organism is a: a. nonfermenter b. obligate anaerobe c. aerobe d. facultative anaerobe

b (Effect of glycolysis on glucose.)

7) Cerebrospinal fluid for glucose assay should be: a. refrigerated b. analyzed immediately c. heated to 56C d. stored at room temperature after centrifugation

c (One X chromosome is passed on to all sons by the mother; since she is a carrier of a mutant X allele, 50% of her sons will be affected.)

7) The 2 alleles for a sex-linked disease are X and x. The mutant allele is x. What is the percentage of male offspring that would be expected to be affected by the disease from parents who have the following genotypes. mother - Xx father - xY a. 0 b. 25 c. 50 d. 100

a (Peripheral pattern reacts with the antigenic determinants of doublestrandedness of DNA.)

7) The result of an anti-nuclear antibody test was a titer of 1:320 with a peripheral pattern. Which of the following sets of results best correlate with these results? a. anti-dsDNA titer 1:80 and a high titer of antibodies to Sm b. antimitochondrial antibody titer 1:160 and antibodies to RNP c. anti-Scl-70 and antibodies to single stranded DNA d. high titers of anti-SS-A and anti-SS-B

c (Development time line of production of Hgb gamma chains.)

7) Which curve represents the production of gamma polypeptide chains of hemoglobin? a. A b. B c. C d. D

b (Children with failure to thrive may have galactosuria. In this case, only a and b are reducing substances. Since the reagent strip for glucose is negative, this leaves galactose.)

70) An urinalysis performed on a 2 week old infant with diarrhea shows a negative reaction with the glucose oxidase reagent strip. A copper reduction tablet test should be performed to check the urine sample for the presence of: a. glucose b. galactose c. bilirubin d. ketones

d (The correct answer is d, electrical equipment. Distractors a and b would be class B fires, and distractor c would be class A fires.)

70) Class C fires involve: a. grease and oil b. xylene and alcohol c. paper, wood, and plastics d. electrical equipment

c (Recognize the laboratory findings in megaloblastic anemia.)

70) The lab tests performed on a patient indicate macrocytosis, anemia, leukopenia and thrombocytopenia. Which of the following disorders is the patient most likely to have? a. iron deficiency b. hereditary spherocytosis c. vitamin B12 deficiency d. acute hemorrhage

c (The Lewis antigens are developed by gene interaction. Both the Lewis and Secretor gene are required for red cells to type as Le[a-b+]. If a person has a Lewis gene, but not Secretor gene, then the cells type as Le[a+b-]. The Le[a-b-] phenotype is derived when the Lewis gene is absent and the Secretor gene may or may not be present. The Le[a+b-] phenotype occurs in 22% of the population, and Le[a-b-] occurs in 6%, so the most likely phenotype of a nonsecretor [se/se] is Le[a+b-].)

70) The red cells of a nonsecretor will most likely type as: a. Le(a-b-) b. Le(a+b+) c. Le(a+b-) d. Le(a-b+)

d (Deterioration of the antimicrobial agent in the disk will cause the zone sizes to be too small [falsely resistant]. Standardization of the inoculum turbidity to less than a 0.5 McFarland standard would result in an inoculum that is too light and resulting zone sizes that are too large. Incubation of the plates at 35 C and inoculating plates within 10 minutes of preparation would not have an adverse effect on zone sizes.)

70) When using a control stain of Staphylococcus aureus, the tech notice that the zone around the oxacillin disk is too small. Which of the following is the most likely explanation? a. inoculation of the plates 10 minutes after preparing the inoculum b. incubation of the Mueller-Hinton plates at 35 C c. use of a 0.25 McFarland standard to prepare inoculum d. use of outdated oxacillin disks

b ("Immunoassay" is a general term for an assay involving binding of an antibody to a specific antigen.)

70) immunoassays are based on the principle of: a. separation of bound and free analyte b. antibody recognition of homologous antigen c. protein binding to isotopes d. production of antibodies against drugs

a (Vaccines stimulate the host to produce antibodies against a specific antigen to prevent disease.)

71) A DPT vaccination is an example of: a. active humoral mediated immunity b. passive humoral mediated immunity c. cell mediated immunity d. immediate hypersensitivity

a (Recognize spherocytes on a blood smear.)

72) A RBC about 5 um in diameter that stains bright red and shows no central pallor is a: a. spherocyte b. leptocyte c. microcyte d. macrocyte

c (The correct answer is c, use eyewash station to flush eyes with water for 15 minutes. Distractors a and d are secondary, not the primary actions. Distractor b is incorrect and completely unsafe.)

72) A technologist splashed a corrosive chemical in his/her eyes. to prevent permanent injury, the first action should be to: a. bandage the eyes and seek immediate emergency medical assistance b. flush eyes with a chemical of opposite pH to neutralize the injury c. use the eyewash station to flush eyes with water for 15 mins. d. seek immediate emergency medical assistance

b (Lymphocytes and basophils are not phagocytic.)

72) Cells known to be actively phagocytic include: a. neutrophils, monocytes, basophils b. neutrophils, eosinophils, monocytes c. monocytes, lymphocytes, neutrophils d. lymphocytes, eosinophils, monocytes

a (Physiological feature of albumin.)

72) The primary function of serum albumin in the peripheral blood is to: a. maintain colloidal osmotic pressure b. increase antibody production c. increase fibrinogen formation d. maintain blood viscosity

d (Moraxella osloensis is a gram-negative coccobacillus that is often plump and occurs in pairs and demonstrates a morphology similar to Neisseria. The presence of this organism in endocervical specimens contaminated with vaginal secretions can lead to over interpretation of smears for N gonorrhoeae.)

72) Which of the following organisms may be mistaken for Neisseria gonorrhoeae in Gram stained smears of uterine cervix exudates? a. Lactobacillus species b. Streptococcus agalactiae c. Pseudomonas aeruginosa d. Moraxella osloensis

b (With the exception of the oxacillin disk screening test, disk diffusion is not recommended for testing S pneumoniae against beta-lactam agents. S pneumoniae does not produce beta-lactamase, so beta-lactamase testing would not be useful. The Schlichter test is not a method for determining an organism's susceptibility to given agent.)

73) A 73-year-old man diagnosed as having pneumococcal meningitis is not responding to his penicillin therapy. Which of the following tests should be performed on the isolate to best determine this organism's susceptibility to penicillin? a. beta-lactamase b. oxacillin disk diffusion c. penicillin disk diffusion d. Schlichter test

b (The correct answer is b, use emergency safety shower and flush body with water. Distractor a is secondary, not a primary first aid action step. Distractors c and d are incorrect and not appropriate first aid actions.)

73) A technologist spilled concentrated hydrochloric acid on his/her clothing and skin, affecting a large portion of the body. After removing involved clothing the next first aid treatment step would be to: a. seek immediate emergency medical assistance b. use emergency safety shower and flush body with water c. apply burn ointment to affected skin d. pour baking soda on the skin and bandage

a (The ocular lens has a magnification of 10x. This multiplied by the magnification of the objectives 10x [low power], 40x [high power], etc., equals the total magnification.)

73) In most compound light microscopes, the ocular lens has a magnification of: a. 10x b. 40x c. 50x d. 100x

c (Use knowledge of RBC indices to classify anemia.)

73) The following results were obtained on a patient's blood: Examination of a Wright-stained smear of the same sample would most likely show: a. macrocytic, normochromic erythrocytes b. microcytic, hypochromic erythrocytes c. normocytic, hypochromic erythrocytes d. normocytic, normochromic erythrocytes

a (The markers listed appear early during hepatitis B infection; HbsAg and HBeAg disappear prior to convalescence and recovery.)

73) The presence of HbsAg, anti-HBc and often HbeAg is characteristic of: a. early acute phase HBV hepatitis b. early convalescent phase HBV hepatitis c. recovery phase of acute HBV hepatitis d. past HBV infection

c (The baby appears to lack c since no HDFN was evident. The mom is most likely R1R1, so had to pass R1 onto the baby. The father must have passed on an Rh gene that also did not produce c, Given the choices, the father has to be R1r.)

74) A women types as Rh-pos. She has an anti-c titer of 32 at AHG. Her baby has a negative DAT and is not affected by HDFN. What is the fathers most likely Rh phenotype. a. rr b. r"r c. R1r d. R2r

d (The correct answer is d, lab coat. The other distractors are examples of engineering controls, not PPE.)

74) An example of personal protective equipment is: a. biological safety cabinet b. emergency safety shower c. eyewash station d. lab coat

a (Polychromatophilic red blood cells and basophilic stippling can be associated with accelerated heme synthesis.)

74) Evidence of active red cell regeneration may be indicated on a blood smear by: a. basophilic stippling, NRBC, and polychromasia b. hypochromia, macrocytes and NRBC's c. hypochromia, basophilic stippling and NRBC's d. Howell-Jolly bodies, Cabot rings and basophilic stippling

c (Adjusting the condenser of the diaphragm of the microscope also affects image resolution. Adjusting the main light source only changes the light intensity.)

74) The best way to lower the light intensity of the microscope is to: a. lower the condenser b. adjust the aperture diaphragm c. lower the rheostat d. raise the condenser

d (Basic principle of lab procedure timed urine.)

74) The first step in analyzing a 24 hr. urine specimen for quantitative urine protein is: a. subculture the urine for bacteria b. add the appropriate preservative c. screen for albumin using a dipstick d. measure the total volume

c (The correct answer is c, answering the telephone in a designated "clean" area. For distractor a, gloves do not need to be removed. For distractor b, it is essential to keep gloves on during specimen transportation, even through "clean" areas, such as the hallways. Care should be taken not to touch anything in the dean areas. For distractor d, gloves do not need to be changed after these patients unless known contamination has occurred, which is the same for any patient specimen under "standard precautions.")

75) Gloves worn in the lab for specimen processing must be removed and hands washed when: a. answering the telephone in the technical work area b. carrying a specimen outside the technical work area through "clean" area c. answering the telephone in a designated "clean" area d. after handling specimens from known isolation precaution patients

b (The diffracted light in phase microscopy enhances slight variations in the refractive indices of constituents with low refractive indices. Staining is not required to enhance low refractive index constituents when using phase microscopy.)

75) The advantage to using phase microscopy in urinalysis is to : a. provide higher magnification b. enhance constituents with a low refractive index c. allow constituents to stain more clearly d. provide a larger field of view

b (Hgb C crystals can be seen in patients with Hgb C disease, more often in individuals who have undergone splenectomy.)

75) The smear represented below displays: a. congenital ovalocytosis b. hemoglobin C disease c. poor RBC fixation d. delay in smear preparation

d (Certain antimicrobials, such as nitrofurantoin and norfloxacin, are used only or primarily to treat urinary tract infections. These agents should not be reported for pathogens recovered from other sites of infection.)

75) Which of the following antimicrobials would be inappropriate to report on an E coli isolated from a wound culture? a. gentamicin b. ampicillin c. cefazolin d. nitrofurantoin

c (the Fy[a-b-] phenotype occurs in 68% of the population of African descent, but is extremely rare in the other ethnic backgrounds. Lu[a-b-], Jk[a-b-] and K-k- are very rare in all ethnic backgrounds.)

75) Which of the following red cell typings are most commonly found in the African Americans: a. Lu(a-b-) b. Jk(a-b-) c. Fy(a-b-) d. K-k-

d (The scrub must use iodine, eg, PVP iodine complex. Donors who are sensitive to iodine can have the area cleaned with a preparation of 2% chlorhexidine and 70% isopropyl alcohol.)

8) Prior to blood donation, the intended venipuncture site must be cleaned with a scrub solution of: a. hypochlorite b. isopropyl alcohol c. 10% acetone d. PVP iodine complex

d (The frequency of compatible donors for this patient can be calculated by multiplying the percentage of the population that is e-C- x Fy[a-] x Jk[b-]. The blood supplier's immunohematology reference laboratory may have units in stock or can request blood from other IRLs through the American Rare Donor Program.)

76) 4 units of blood are needed for elective surgery. the patient serum contains anti-C, anti-e, anti-Fya and anti-Jkb. Which of the following would be the best source of donor blood: a. test all units in current stock b. test 100 group O Rh-neg c. test 100 group compatible donors d. rare donor file

a (The correct answer is a, working with caustic or toxic materials. For the other distractors, it is not required practice to use eye and face protection during these activities.)

76) Safety glasses, face shields or other eye and face protectors must be worn when: a. working with caustic or toxic materials b. present in technical work area c. viewing microbiology culture plates d. processing specimens using a splash barrier

b (Rouleaux and an increased sedimentation rate are caused by increased fibrinogen and/or with increased immunoglobulin.)

76) The presence of excessive rouleaux formation on a blood smear is often accompanied by an increased: a. reticulocyte count b. sedimentation rate c. hematocrit d. erythrocyte count

b (The image displays a negative D test result. There is no flattening of the zone of inhibition around the clindamycin disk adjacent to the erythromycin disk. Thus, there is no inducible clindamycin resistance, and the isolate is reported as clindamycin- susceptible, while the erythromycin is reported as resistant.)

77) A D test is performed on an isolate of Staphylococcus aureus to determine inducible clindamycin resistance: Based on the result seen in the image how should the erythromycin and clindamycin be reported? a. erythromycin: resistant; clindamycin: resistant b. erythromycin: resistant; clindamycin: susceptible c. erythromycin: susceptible; clindamycin: resistant d. erythromycin: susceptible; clindamycin: susceptible

b (Renal tubular epithelial cells lining the tubules absorb the urinary filtrate. In disorders producing f at in the filtrate, the fat is absorbed into the cells. When the cells slough from the tubules, they appear as oval fat bodies.)

77) Oval fat bodies are: a. squamous epithelial cells that contain lipids b. renal tubular epithelial cells that contain lipids c. free-floating fat droplets d. white blood cells with phagocytized lipids

b (The peripheral smear allows microscopic examination of the blood cells. The most characteristic finding in multiple myeloma is rouleaux formation of the red cells.)

77) The characteristic peripheral blood morphologic feature in multiple myeloma is: a. cytotoxic T cells b. rouleaux formation c. spherocytosis d. macrocytosis

a (The correct answer is a, secure long hair and jewelry. These items should be secured to avoid contamination with biohazards or physical injury. The other distractors are not safe or acceptable work practices. Food should not be stored in supply refrigerators at all, contact lenses should not be worn in laboratories and open weaved shoes are a spill hazard.)

77) To prevent injury, a safe lab work practice is to: a. secure long hair and jewelry b. store well-wrapped food in the supply fridge c. wear contact lenses for eye protection d. wear comfortable, rubber-bottomed ,open weaved shoes

b (Western blot has been the traditional confirmatory test for HIV antibody because it is very specific.)

77) When testing a patient for HIV antibody, which of the following is used to confirm a positive screening test? a. radioimmunoassay b. Western blot c. immunofluorescence d. ELISA

d (From the first 2 children it can be determined the mom has the haplotypes A2B12 and A23F18. The dad has the haplotypes Al B3 and A3B35. The expected B antigen in child #3 is B35.)

78) A family has been typed for HLA because 1 of the children needs a stem cell donor. Typing results are listed below: father: A1,3;B8,35 mother: A2,23;B12,18 child #1: A1,2;B8,12 child #2: A1,23;B8,18 child #3: A3,23;B18,? What is the expected B antigen in child #3? a. A1 b. A2 c. B12 d. B35

b (Osmosis occurs through the red blood cell membrane. In dilute urine, the cells absorb water and swell, lyse, and release hemoglobin.)

78) A microscopic exam of urine sediment reveals ghost cells. These RBCs are seen in urine with a: a. >2% glucose concentrations b. specific gravity <1.007 c. large amounts of ketone bodies d. neutral pH

a (The National Polycythemia Vera Study group criteria for the diagnosis of polycythemia Vera include increased red cell mass, increased platelet count, increased LAP score.)

78) In polycythemia vera, the hemoglobin, hematocrit, RBC count and red cell mass are: a. elevated b. normal c. decreased

c (Knowledgeable personnel separate waste into designated categories [i.e., chemical, routine, infectious, etc.] at the point of generation to reduce disposal costs and minimize employee exposure to hazardous materials.)

78) Safe handling and disposal of laboratory generated infectious waste require: a. disinfection of all waste b. thorough mixing of infectious and noninfectious waste c. separation of infectious and noninfectious waste d. incineration of all waste

c (Extended spectrum beta lactamases [ESBL] are inhibited by clavulanic acid. Confirmatory tests for the presence of ESBL are based on the enhanced activity of a beta-lactam antibiotic, usually cefotaxime or ceftazidime, when it is tested with clavulanic acid compared to the activity of the beta-lactam tested alone.)

78) Which of the following combinations is useful for confirming the presence of extended spectrum beta-lactamases of E coli? a. ampicillin + cefepime b. cefoxitin + penicillin c. ceftrazidime + clavulanic acid d. cefpodoxime + cefotaxime

d (White blood cells absorb water when they are in hypotonic [low specific gravity] urine, and swell. Granules in the WBCs then exhibit Brownian movement, producing the glittering effect in the cells.)

79) Glitter cells are a microscopic finding of: a. RBC in hypertonic urine b. RBC in hypotonic urine c. white blood cells in hypertonic urine d. white blood cells in hypotonic urine

a (A 10% solution of bleach is an effective and economical disinfectant, which inactivates HBV in 10 minutes and HIV in 2 minutes.)

79) Which of the following is the best choice for decontaminating bench tops contaminated by the AIDS virus? a. sodium hypochlorite bleach b. formalin c. a quaternary ammonium compound d. 100% alcohol

b (Repeating a QC measurement on a new sample of QC material may establish that the alert was caused by a deteriorated QC material rather than a method problem.)

8) Shown above is a Levy-Jennings quality control chart, which represents control values for 13 consecutive analyses for a particular serum constituent. If the 14th value is below the —2 SD limit, which of the following should be done? a. control should be repeated to see if it will fall within the established interval b. analysis system should be checked for a deteriorating component c. analysis system should be checked for a change in reagent lot number d. no action is needed

a (Anti-DNA antibodies are prototypic autoantibodies found in the sera of SLE patients.)

8) Systemic lupus erythematosus patients often have which of the following test results? a. high titers of DNA antibody b. decreased serum immunoglobulin levels c. high titers of anti-smooth muscle antibodies d. high titers of antimitochondrial antibody

d (Males are hemizygous for X-linked alleles because they have only one copy of the X chromosome.)

8) The term that best describes males regarding X-linked genes is: a. heterozygous b. homozygous c. haplozygous d. hemizygous

d (Unequivocal diagnosis of diabetes mellitus.)

8) Which of the following 2 hr. postprandial glucose values demonstrates unequivocal hyperglycemia diagnostic for diabetes mellitus? a. 160 mg/dL (8.8 mmol/L) b. 170 mg/dL (9.4 mmol/L) c. 180 mg/dL (9.9 mmol/L) d. 200 mg/dL (11.0 mmol/L)

d (Patients who have suffered severe burns to more than 15 % of their body generally show evidence of intravascular hemolysis. RBCs show changes including fragmentation, budding and microspherocytes formation.)

80) Many microspherocytes, schistocytes and budding off of spherocytes can be seen on peripheral blood smears of patients with: a. hereditary spherocytosis b. DIC c. acquired autoimmune hemolytic anemia d. extensive burns

c (The simplest method of disposing of needles is to dispose of the entire collection device into a container reserved for sharps.)

80) The safest method of disposing of hypodermic needles is: a. recap the needle with its protective sheath prior to discarding b. cut the needle with a special device before disposal c. discard the needle in an impermeable container without other handling immediately after use d. drop the needle in the waste basket immediately after use

c (Squamous epithelial cells line the female vagina and urethra, but only the distal part of the male urethra. In females, they may also indicate perianal contamination.)

80) What cell is most commonly associated with vaginal contamination? a. white b. transitional c. squamous d. glitter

b (The volume of blood collected is the single most important variable in the recovery of organisms in patients with bloodstream infections. Since many cases of adult bacteremia are of low magnitude, there is a direct relationship between the yield of blood culture [positivity] and volume of blood collected the collection of multiple blood culture sets from a single venipuncture is an unacceptable practice due to the potential for contamination. The practice of terminal subculture of blood culture bottles at 5 days is no longer recommended The use of chlorhexadine for skin antisepsis does not affect organism recovery, but aids in decreasing blood culture contamination.)

80) Which of the following is most important variable in the recovery of organisms in patients with bacteremia? a. subculture of all bottles at day 5 of incubation b. the volume of blood cultured c. use of chlorhexadine for skin antisepsis d. collection of multiple blood culture sets from a single venipuncture

b (Gonococcal urethritis in adult males is often diagnosed by the observation of gram-negative diplococci within or closely associated with neutrophils in smears prepared from urethral discharge. The Gram stain in males has a sensitivity of 90%-95% and a specificity of 95%-100% for diagnosing gonorrhea in symptomatic males.)

81) A 24-year-old man presents with pain on urination and urethral discharge. A Gram stain of the discharge is seen in the image: What is the most likely identification of this organism? a. Acinetobacter baumannii b. Neisseria gonorrhoeae c. Haemophilus ducreyi d. Escherichia coli

b (The reagent strip test for blood is positive for hemoglobin from lysed red blood cells, filtered hemoglobin from intravascular hemolysis, and myoglobin. With no RBCs present, the terminology is hemoglobinuria, indicating the presence of filtered hemoglobin.)

81) A reagent strip test for blood has been reported positive. Microscopic exam fails to yield RBCs. This patient's condition can be called: a. hematuria b. hemoglobinuria c. oliguria d. hemosiderinuria

d (Blood spills must be cleaned up and decontaminated by personnel using the proper PPR.)

81) Precautions for health care workers dealing with patients or patient specimens include: a. mouth pipetting when specimens lack a "Precaution" label b. reinserting needles into their original sheaths after drawing blood from a patient c. wearing a mask and disposable gown to draw blood d. prompt cleaning of blood spills with a disinfectant solution such as sodium hypochlorite

a (The RPR is the most widely used nontreponemal test to screen for syphilis because it is rapid and inexpensive. It uses unheated serum to test for antibodies to cardiolipin, a lipid released during tissue damage in syphilis patients, but also present in other diseases with tissue damage.)

81) The Rapid Plasma Reagin test: a. is useful in screening for syphilis b. is useful in diagnosing syphilis c. does not give false positives d. used heated plasma

a (RBC morphologic features of erythroleukemia include nuclear budding and fragmentation cytoplasmic vacuoles, Howell-Jolly bodies, ringed sideroblasts and megaloblastic changes.)

81) Which of the following is most closely associated with erythroleukemia? a. ringed sideroblasts, nuclear budding and Howell-Jolly bodies b. DIC c. micromegakaryocytes d. lysozymuria

c (The Gram stain demonstrates numerous neutrophils and small, pleomorphic gram- negative bacilli suggestive of Haemophilus. H influenzae is an important cause of lower respiratory tract infections in patient with pre-existing lung disease such as cystic fibrosis. Haemophilus are fastidious, and require the use of an enriched medium such as chocolate agar and incubation at 35 - 37 C in a moist environment supplemented with 5%-10% C02.)

82) A 10-year-old child with cystic fibrosis presents with cough and shortness of breath. Her sputum Gram stain is seen in the image: Based on the Gram stain what would be the best medium and incubation condition to optimize recovery of the organism seen? a. MacConkey agar incubated in CO2 b. Tinsdale agar incubated in ambient air c. chocolate agar incubated in CO2 d. CNA agar incubated in ambient air

a (Flocculation is the aggregation of fine particles to form small clumps. This reaction occurs in nontreponemal tests for syphilis when reagin antibody reacts with the fine cardiolipin antigen particles.)

82) Flocculation tests for syphilis detect the presence of: a. reagin antibody b. antigen c. hemolysin d. Forssmann antigen

b (RBCs absorb water when in dilute urine, and are also less preserved in alkaline urine. Therefore, as the cells swell in the alkaline urine, the cell membrane allows hemoglobin to leak from the cell, resulting in the empty cell membrane and the pale appearance.)

82) Ghost RBCs are seen in urine that is: a. acidic and dilute b. alkaline and dilute c. acidic and concentrated d. alkaline and concentrated

b (Hepatitis B infection is a global public health problem and is one of the most common infectious diseases in the world.)

82) Infection rate is highest for laboratory professionals exposed to blood and body fluids containing: a. hepatitis A b. hepatitis B c. CMV d. HIV

d (Rouleaux is the stacking of red cells like coins and is caused by increased amounts of immunoglobulins in the blood causing the RBCs to adhere to each other.)

82) The most characteristic peripheral blood smear finding in multiple myeloma is: a. plasmacytic satellitosis in the bone marrow b. many plasma cells in the peripheral blood c. many Mott cells in the peripheral blood d. rouleaux formation of the red cells

d (When a marker is in a child that the mother and alleged father do not have the alleged father cannot be the biological father of the child. This is a direct exclusion.)

83) In relationship testing a "Direct exclusion" is established when a genetic marker is: a. absent in child, present in mother and alleged father b. absent in child, present in mother and absent in alleged father c. present in child, absent in mother and present in father alleged father d. present in child, absent in the mother and alleged father

a (When 1 solid agglutinate is present, the reaction is classified as 4+.)

83) In the cold agglutinin test, the tubes containing the serum and erythrocytes are allowed to stand overnight in the refrigerator, and the results are read the next morning. If a disk of the erythrocytes floats up from the bottom of the tube with only the flick of a finger, this is read as a: a. 4+ reaction b. 2+ reaction c. 1+ reaction d. negative

b (In a hypotonic urine, WBCs will absorb water and swell. This results in the granules in the granulocytic WBCs to exhibit Brownian movement.)

83) The possibility of detecting glitter cells is associated with urine that is: a. acidic b. dilute c. alkaline d. concentrated

c (In patients with cold agglutinins, the automated cell counters show an erroneously elevated MCV and an erroneously decreased red blood cell count due to clumping of the red cells.)

83) The values below were obtained on an automated blood count system performed on a blood sample from a 25-year-old man: These results are most consistent with which of the following? a. megaloblastic anemia b. hereditary spherocytosis c. a high titer of cold agglutinins d. an elevated reticulocyte count

c (The 3 modes of HIV transmission are through intimate sexual contact, contact with blood, and perinatal. Needlestick injury falls under the second mode, and poses, on average, a 0.3% risk of transmission.)

83) Which of the following forms of exposure places a technologist at the highest risk for infection with HIV? a. aerosol inhalation (eg. AIDS patient sneezes) b. ingestion (eg. mouth pipetting) c. needlestick d. splash

b (Detection of group B Streptococcus [GBS] in the genital and gastrointestinal tracts of pregnant women can identify infants at risk for GBS infection. The CDC currently recommends the collection of vaginal and rectal swabs or a single swab inserted first into the vagina and then the rectum at 35 - 37 weeks gestation. The swab[s] should be inoculated into a selective broth medium such as LIM broth [Todd-Hewitt broth with colistin and nalidixic acid]. The use of vaginal/rectal swabs and selective broth medium greatly increases the recovery of GBS.)

84) A vaginal/rectal swab is collected from a pregnant patient to screen for group B streptococcus colonization. What is the best medium to use for specimen inoculation? a. blood agar b. LIM broth c. CNA agar d. thioglycollate broth

c (The child's genotype does not include E. The alleged father is homozygous for E. If he was the father the child would also have E. The father can be excluded from paternity.)

84) Relationship testing produces the following red cell phenotyping results: alleged father: ABO = B | Rh = D+C-c+E+e- mother: ABO = O | Rh = D+C+E-c-e+ child: ABO = O | Rh = D+C+E-c+e+ What conclusions may be made? a. there is no exclusion of paternity b. paternity may be excluded on the basis of ABO typing c. paternity may be excluded on the basis of Rh typing d. paternity may be excluded on the basis of both ABO and Rh typing

d (A 10% solution of bleach is an effective and economical disinfectant, which inactivates HBV in 10 minutes and HIV in 2 minutes.)

84) Which disinfectant inactivates HIV and HBV? a. alcohol b. iodine c. phenol d. sodium hypochlorite

a (Clue cells represent the attachment of the bacterium Gardnerella vaginalis to squamous epithelial cells. Gardnerella causes vaginal infections, and the cells lining the vagina are the squamous epithelial cells.)

85) Clue cells are a form of: a. squamous epithelial cells b. urothelial cell c. white blood cell d. renal tubular epithelial cell

b (Microbiological hazards are contained using a biological safety cabinet with the air exhausting through a HEPA filter.)

85) Filters generally used in biological safety cabinets to protect the laboratory worker from particulates and aerosols generated by microbiology manipulations are: a. fiberglass b. HEPA c. APTA d. charcoal

a (A cold agglutinin titer is read as the last dilution showing agglutination at 4 C.)

85) The following cold agglutinin titer results are observed: The best interpretation is: a. positive, 1:128 b. negative c. invalid because 37 C reading is negative d. repeat the 4 C readings

d (To avoid reactivation of complement, standard practice is to reheat the specimen for 10 minutes at 56 C if more than 4 hours has elapsed since the specimen was first inactivated.)

86) A VDRL serum sample is heat inactivated, then placed in the refrigerator for overnight storage. Before being tested, the serum must be: a. kept colder than 10 C b. allowed to equilibrate to room temp c. warmed to 37 C d. reheated to 56 C for 10 minutes

b (A very high WBC count causes turbidity in cyanmethemoglobin reagent-patient specimen that will result in falsely elevated hemoglobin values.)

86) A common source of interference in the cyanmethemoglobin method is: a. hemolysis b. very high WBC count c. cold agglutinins d. clumped platelets

c (Ax cells are more strongly reactive with anti-A,B than with anti-A and the plasma frequently has anti-Al present.)

86) A patient is typed with the following results: The most probable reason for these findings is that the patient is group: a. O; confusion due to faulty group O antiserum b. O; with an anti-A1 c. Ax; with an anti-A1 d. A1; with an anti-A

d (Oxacillin resistant staphylococci are resistant to all beta-lactam agents, beta-lactam/beta-lactamase inhibitor combinations and carbapenems. Results for these antibiotics should be reported as resistant or should not be reported.)

86) Susceptibility testing is performed on a Staphylococcus aureus isolate from a blood culture with the following results: - oxacillin: resistant - cefazolin: susceptible - clindamycin: susceptible - erythromycin: susceptible - trimethoprim/sulfamethoxazole: susceptible - vancomycin: susceptible What should the technologist do next? a. ceftriaxone should be reported instead of cefazolin b. clindamycin should be tested for inducible resistance prior to reporting c. the trimethoprim/sulfamethoxazole result should be removed since all S aureus are resistant d. the cefazolin result should be changed to resistant since the oxacillin result is resistant

a (A ratio and proportion procedure is most useful for calculations in which concentrations are not changed, as in this case. Set up a ratio of total area [18] to gamma globulin area [30] in units, and total protein content [6.5] to gamma globulin content [x] in g/dL. Solve for x.)

86) The following data was obtained from a cellulose acetate protein electrophoresis scan: albumin area = 75 units gamma globulin area = 30 units total area = 180 units total protein = 6.4 g/dL (65 g/L) The gamma globulin content in g/dL is: a. 1.1 g/dL (11 g/L) b. 2.7 g/dL (27 g/L) c. 3.8 g/dL (38 g/L) d. 4.9 g/dL (49 g/L)

b (Urothelial/transitional epithelial cells line the bladder, renal pelvis and ureters. These can be dislodged from the walls of the bladder during a catheterization procedure.)

86) Which of the following cells is most likely to be seen in the urine sediment following a catheterization procedure? a. squamous epithelial cell b. urothelial cell c. white blood cell d. renal tubular epithelial cell

d (Myoglobin is a product of muscle destruction as occurs with crush injuries. The reagent strip reaction for blood is positive with the presence of RBCs, hemoglobin, and myoglobin. Both hemoglobin and myoglobin are toxic to the renal tubules, resulting in decreased urine How, favoring cast formation and the sloughing of the damaged cells. Notice also that the specimen is clear.)

87) A patient admitted following an accident involving massive crush injuries has the following urinalysis results: The discrepancy between the large amount of blood and the absence of RBCs on microscopy is caused by: a. failure to mix the specimen before centrifuging b. mistaking RBCs for RTE cells c. contaminating oxidizing detergents in the container d. the presence of myoglobin in the urine specimen

c (The correct answer is c, discard directly into an appropriate sharps container, to avoid undo contact with the sharp. Distractor a is a prohibited practice. For distractors b and d, needle recapping or removal is not preferred, but if required, it must be done using mechanical devices or for re-capping only using a one-handed technique.)

87) Contaminated needles and syringes without safety self-sheathing devices should be: a. sheared by a needle cutter or bent b. re-capped using a two-handed technique c. discarded directly into an appropriate sharps container d. removed from the syringe/needle holder

c (The ABO blood group system was discovered by Karl Landsteiner.)

87) Human blood groups were discovered around 1900 by: a. Jules Bordet b. Louis Pasteur c. Karl Landsteiner d. PL Mollison

b (This is the definition of a hapten.)

87) Substances that are antigenic only when coupled to a protein carrier are: a. opsonins b. haptens c. adjuvants d. allergens

d (Bacillus stearothermophilus is commonly used as an indicator organism for the appropriate functioning of autoclaves. Unlike most Bacillus species, B stearothermophilus grows at 56 C.)

87) To quality control the autoclave, a vial of Bacillus stearothermophilus is autoclaved and should they be: a. inoculated to blood agar b. incubated at 37 C c. inoculated to chocolate agar d. incubated at 56 C

c (Mixed-field reactivity with anti-A and anti-A,B is a typical finding for A3 subgroups.)

88) Cells of the A3 subgroup will: a. react with Dolichos biflorus b. bE-with anti-A c. have a mixed field reaction with anti-A,B d. bE- with anti-H

c (Quality control zone sizes that are too small could indicate that the organism inoculum is too high, plates were poured too thick, or that the potency of the antibiotic disks is too low.)

88) In a quality control procedure on a new batch of Mueller-Hinton plates using a stock culture of staphylococcus aureus, all the disk zone sizes are too small. The most likely reason for this is that the: a. Mueller-Hinton plates were poured to thin b. potency of the antibiotic disks is too high c. bacterial suspension was not diluted to the proper concentration d. disks should have been set up on mannitol salt

a (The correct answer is a, bloodborne pathogens.)

88) Use of "standard" (universal) precautions minimizes exposure to: a. bloodborne pathogens b. chemical hazards c. radiation hazards d. environmental hazards

d (Some key findings for each condition are: a] glomerulonephritis - red blood cell casts, b] pyelonephritis - white blood cell casts, c] nephrotic syndrome - lipids and fatty casts, and d] cystitis [urinary tract infection] - white blood cells, bacteria, possible RBCs. This image has no casts, just WBCs, bacteria, and RBCs.)

88) What is the most likely diagnosis given this microscopic finding? a. glomerulonephritis b. pyelonephritis c. nephrotic syndrome d. cystitis

d (Hgb C crystals can be seen in patients with Hgb C disease, more often in individuals who have undergone splenectomy.)

88) With this blood picture, an additional test indicated is: a. alkali denaturation b. alkaline phosphatase stain c. peroxidase stain d. hemoglobin electrophoresis

b (Nontreponemal tests for syphilis, such as the RPR, detect antibody to cardiolipin antigen complexed with lecithin and cholesterol.)

89) A serological test for syphilis that depends upon the detection of cardiolipin-lecithin-cholesterol antigen is: a. FTA-ABS b. RPR c. MHA-TP d. TPI

d (The correct answer is d, thoroughly wash the wound with soap and water. The other distractors are secondary, not primary actions.)

89) After an accidental needle stick with a contaminated needle, the first action should be to: a. apply antiseptic ointment to the wound b. seek immediate medical assistance c. bandage the wound d. thoroughly wash the wound with soap and water

a (RBCs are smooth, circular, greenish objects. They can be confused with yeast and fat globules. To distinguish yeast from RBC, react an aliquot with water. RBCs will lyse, but yeast will not. Fat globules can be stained with Sudan III, turning them orange. RBCs should produce a positive blood result on a biochemical strip.)

89) Identify the formed element in this photomicrograph: a. RBC b. WBC c. epithelial cell d. yeast

c (the Kleihauer-Betke procedure is commonly used as a screening test to determine the amount of fetal blood that has mixed with maternal blood.)

89) The most appropriate screening test for detecting hemoglobin F is: a. osmotic fragility b. dithionite solubility c. Kleihauer-Betke d. heat instability test

d (Of the combinations listed, the use of E coli and Proteus mirabilis will produce a positive and negative result for indole, respectively. The remainder of the organisms are all positive for the test described.)

89) Which of the following combinations of organisms would be appropriate as controls to test the functions listed? a. beta-hemolysis - negative Escherichia coli and Streptococcus pyogenes b. catalase - negative Staphylococcus aureus and Staphylococcus epidermidis c. H2S production - negative Proteus mirabilis and Staphylococcus epidermidis d. indole - negative Escherichia coli and Proteus mirabilis

b (Antimicrobial resistance in Neisseria gonorrhoeae is widespread. The production of beta-lactamase [penicillinase] breaks open the beta lactam ring of penicillin, destroying its activity. Thus, N. gonorrhoeae that produce beta lactamase are resistant to penicillin.)

9) A penicillin resistant Neisseria gonorrhoeae produces: a. alpha-hemolysin b. beta-lactamase c. enterotoxin d. coagulase

b (Age effect on glucose.)

9) Serum levels that define hypoglycemia in pre-term or low birth weight infants are: a. the same as adults b. lower than adults c. the same as a normal full-term infant d. higher than a normal full-term infant

d (All mitochondria are passed on from a mother to her children, but not by a father to his children.)

9) The mode of inheritance of mitochondrial DNA is: a. dominant b. recessive c. codominant d. maternal

a (Development time line of production of Hgb epsilon chains.)

9) Which curve represents the production of epsilon polypeptide chains of hemoglobin? a. A b. B c. C d. D

b (The organism in this urine culture is a Staphylococcus species. Coagulase will differentiate S aureus from coagulase-negative staphylococci [CNS] and novobiocin susceptibility will differentiate S saprophyticus from other CNS. Saprophyticus is a common cause of urinary tract infections in young females.)

90) A urine Gram stain shows gram-positive cocci in clusters. The organism tested catalase positive. To speciate this organism from culture, the technician should perform a coagulase test and a/an: a. polymyxin B susceptibility b. novobiocin susceptibility c. oxidase d. beta-lactamase

d (Tamm-Horsfall protein is continuously excreted by the renal tubular cells. In conditions that cause urine stasis, the excreted protein aggregates into fibrils that mesh to form the matrix of casts.)

90) All casts typically contain: a. albumin b. globulin c. immunoglobulins G and M d. Tamm-Horsfall glycoprotein

d (Bombay phenotypes [Oh] lack H antigen on their red cells, and produce naturally occurring anti-H in their serum.)

90) Even in the absence of prior transfusion or pregnancy, individuals with the Bombay phenotype will always have naturally occurring: a. anti-Rh b. anti-K0 c. anti-U d. anti-H

b (The solubility test is a rapid test for Hgb S. This should not be used for screening newborns, needs to be corrected in severe anemia and is not specific for Hgb S as there are other hemoglobins that will sickle. The presence of Hgb S is confirmed by hemoglobin electrophoresis.)

90) The most appropriate screening test for hemoglobin S is: a. Kleihauer-Betke b. dithionite solubility c. osmotic fragility d. sucrose hemolysis

a (The correct answer is a, parenteral inoculation of blood. The other distractors are not likely transmission modes for bloodborne pathogens.)

90) What is the most likely mode of transmission for bloodborne pathogens in laboratory acquired infections? a. parenteral inoculation of blood b. contact with intact skin c. airborne transmission d. fecal-oral transmission

d (Hemoglobin concentration is calibrated using commercially available HiCN solutions of known content.)

91) Hematology standards include: a. stabilized RBC suspension b. latex particles c. stabilized avian RBC's d. certified cyanmethemoglobin solution

b (Hyaline casts have a low refractive index, and may not be visible under bright light. Urine microscopic analysis is first performed under reduced light, and the edges of the coverslip examined for the casts. Casts are larger than other sediment constituents and are pushed to the edges of the coverslip.)

91) Hyaline casts are usually found: a. in the center of the coverslip b. under subdued light c. under very bright light d. in the supernatant

c (Nutritionally deficient streptococci such as Abiotrophia do not grow on sheep blood agar without the addition of cysteine or proximity to S aureus colonies.)

91) The Gram stain from a blood culture shows gram-positive cocci in chains. No growth occurs on blood agar plates incubated both aerobically and anaerobically. Additional testing should be done to detect the presence of: a. Staphylococcus saprophyticus b. Aerococcus urinae c. Abiotrophia defectiva d. Streptococcus pneumoniae

a (Most examples of anti-Lua agglutinate saline suspended cells. Most examples of anti-Lub are IgG and reacts at 37 C. Anti-Lu3 usually reacts at the AHG phase as does anti- Luab.)

91) The antibody in the Lutheran system that is best detected at lower temps: a. anti-Lua b. anti-Lub c. anti-Lu3 d. anti-Luab

b (The VDRL test is the only serological test recommended for testing of spinal fluid because of the low incidence of false positives.)

91) The serological test for syphilis recommended for detecting antibody in cerebrospinal fluid is: a. nontreponemal antibody b. CSF-VDRL c. FTA-ABS d. MHA-TP

b (The correct answer is b, hepatitis B.)

91) Which infectious agent is considered to be the primary occupational health hazard regarding transmission of bloodborne pathogens? a. HIV b. HBV c. TB d. MRSA

d (Bile solubility testing of alpha-hemolytic streptococci differentiates S pneumoniae [soluble] from other alpha-hemolytic streptococci, such as viridans streptococci [insoluble].)

92) Viridans streptococci can be differentiated from Streptococcus pneumoniae by: a. alpha hemolysis b. morphology c. catalase reaction d. bile solubility

c (The correct answer is c, biological safety cabinet. Distractor a is used for pharmacy and other "clean room" functions. Distractor b does not provide adequate respiratory protection, and distractor d is used for chemical not biohazard control.)

92) When processing specimens for mycobacterial testing, what specific engineering control must be used? a. horizontal laminar flow hood b. barrier protection only c. biological safety cabinet d. fume hood

b (Coagulase is the biochemical test used to distinguish S aureus [positive] from coagulase-negative staphylococci [negative].)

93) A reliable test for distinguishing Staphylococcus aureus from other staphylococci is: a. oxidase b. coagulase c. catalase d. optochin susceptibility

b (Anti-Sda is an antibody to a high-prevalence antigen, which varies in strength from person to person. Most examples of anti-Sda characteristically present as small, mixed-field, retractile agglutinates that may have a shiny appearance when observed microscopically after the antiglobulin test.)

93) Anti-Sda is strongly suspected if: a. the patient has been previously transfused b. the agglutinates are mixed field and refractile c. the patient is group A or B d. only a small number of panel cells are reactive

c (The correct answer is c, must he provided by the employer free of charge. For distractor a, it is a series of 3, not a single injection. For distractor b, it is recommended, but not a mandatory vaccination. For distractor d, the vaccine is recommended for high-risk employees upon initial employment, not just when an exposure occurs.)

93) Hepatitis B vaccine is: a. administered as a single 1-time injection b. required for all healthcare employees c. must be provided by the employer free of charge d. recommended only when an exposure incident occurs

d (Broad casts indicate extreme stasis of urine How through the nephron. Stasis allows casts to form in the larger collecting ducts. Damage to the walls of the distal convoluted tubules also causes broader casts to form.)

93) Which of the following casts is most indicative of end stage renal disease? a. hemoglobin b. granular c. cellular d. waxy

b (The correct answer is b, absorb the spill with disposable absorbent material. For distractor a, immediately flooding the area with disinfectant may create aerosolization. For distractor c, it is unnecessary to evacuate the area for 30 minutes due to the size and nature of the spill. For distractor d, the area should be first cleaned with a disinfectant, not a detergent.)

94) When cleaning up a small (5 mL) blood spill on the counter-top, the first step after donning appropriate personal protective equipment is to: a. flood the area with an appropriate intermediate to high-level disinfectant b. absorb the spill with disposable absorbent material c. evacuate the area for 30 minutes d. clean the area with an aqueous detergent solution

c (Optochin susceptibility is used to differentiate S pneumoniae, which are susceptible from other alpha-hemolytic streptococci, which are resistant.)

95) In the optochin (ethylhydrocupreine hydrochloride) susceptibility test, if there is a zone of inhibition of 19-30 mm surrounding the disk following overnight incubation at 37 C, the colony most likely consists of: a. staphylococci b. streptococci c. pneumococci d. intestinal bacilli

a (The correct answer is a, sodium hypochlorite, which is a disinfectant. The other distractors are antiseptics, not disinfectants.)

95) The most effective disinfectant recommended for bloodborne pathogens is: a. sodium hypochlorite b. isopropyl alcohol c. chlorhexidine gluconate d. povidone-iodine

b (Centrally-placed nuclei are characteristic of spherical transitional cells.)

95) Which of the following aids in differentiating a spherical transitional cell from a round renal tubular cell? a. spherical transitional cell is larger b. eccentrically-placed nucleus in the renal tubular cell c. eccentrically-placed nucleus in the spherical transitional cell d. round renal tubular cell is larger

d (Pyelonephritis is an inflammation infection of the renal tubules. Therefore, white blood cell casts would indicate the location of the source of the inflammation/infection. Cystitis is an infection of the bladder.)

96) The urine microscopic constituents that best differentiate between cystitis and pyelonephritis are: a. WBCs b. bacteria c. RBCs d. WBC casts

b (Antibody to the surface antigen of hepatitis B virus [anti-HBs] is the major protective antibody in hepatitis B and provides evidence of immunity against this infection.)

96) What assay would confirm the immune status to hepatitis B virus: a. HbsAg b. anti-HBs c. IgM anti-HBcAg d. hepatitis V Ag

d (The low hemoglobin can result in a false-negative result.)

96) When using the turbidity method for detecting the presence of hemoglobin S, an incorrect interpretation may be made when there is a: a. concentration of <7 g/dL hemoglobin b. glucose concentration >150 mg/dL c. blood sugar > 2 hrs. old d. increased hemoglobin

b (Noninfectious sequelae associated with infection with Streptococcus pyogenes are glomerulonephritis and rheumatic fever.)

96) Which 2 diseases are usually preceded by infection with beta-hemolytic streptococci? a. rheumatic fever, undulant fever b. glomerulonephritis, rheumatic fever c. rheumatic fever, tularemia d. glomerulonephritis, undulant fever

d (Of all the methods listed, ELISA is the most sensitive and the only one that is used for detection of HbsAg in the clinical laboratory.)

97) The following procedure has been routinely used for detection of hepatitis B surface antigen because of its level of sensitivity: a. hemagglutination b. counter-immunoelectrophoresis c. radial imummunodiffusion d. ELISA

c (The correct answer is c, single use and disposable gloves. For distractor a, reusable utility gloves are not ideal or the best choice, but could be used if properly decontaminated after use and inspected for punctures or tears prior to reuse. Latex gloves should be avoided because of potential sensitization to the latex and the development of a latex allergy.)

97) When processing patient blood specimens and handling other potentially infectious material, the best choice of gloves is: a. reusable utility gloves b. latex gloves only c. single use and disposable gloves d. cut-resistant gloves

c (Group D streptococci and Enterococcus produce a positive bile esculin test; however, of these two, only Enterococcus grows in the presence of 6.5% NaCl.)

98) A gamma-hemolytic Streptococcus that blackens bile esculin agar but does not grow in 6.5% NaCl broth is most likely: a. group B Streptococcus b. Enterococcus c. group D Streptococcus d. Streptococcus pneumoniae

b (Formation of a cast matrix is not uncommon following strenuous exercise, due primarily to dehydration resulting in decrease urine flow. Increased metabolism by the renal tubular cells results in excess excretion of lysosomes that become attached to the cast matrix, resulting in the appearance of granular casts.)

98) Granular casts found in the urine of a football player admitted to the hospital with a broken leg occurring during the game can be the result of: a. excessive bruising b. strenuous exercise c. excess power drink ingestion d. bone fracture

a (In an indirect ELISA, patient antibody to an antigen [eg, rubella antigen] is detected by addition of an enzyme-labeled antibody to human immunoglobulin, which, in turn, binds to the patient's IgG.)

98) In an indirect ELISA method designed to detect antibody to the rubella virus in patient's serum, the conjugate used should be: a. anti-human IgG conjugated to an enzyme b. anti-rubella antibody conjugated to an enzyme c. rubella antigen conjugated to an enzyme d. anti-rubella antibody conjugated to a substrate

d (The correct answer is d, change gloves and wash hands with antiseptic soap and water. The other distractors are inappropriate practices.)

98) While processing patient specimens, a technologist splashes a few small drops of a bronchial wash specimen in his/her gloves. The first action should be to: a. wash the gloves with antiseptic/soap and water b. continue to wear the gloves until grossly contaminated or leaving the area c. wash the gloves with an appropriate disinfectant d. change gloves and wash hands with antiseptic/soap and water

a (The Gram stain and culture growth describe a Staphylococcus species. Catalase production confirms that the organism belonged to the genus Staphylococcus and coagulase is used to differentiate S aureus from coagulase-negative staphylococci.)

99) Gram stain examination from a blood culture bottle show dark blue, spherical organisms in clusters. Growth on sheep blood agar shows small, round, pale yellow colonies. Further tests should include: a. catalase production and coagulase test b. bacitracin susceptibility and serological typing c. oxidase and deoxyribonuclease reactions d. Voges-Proskauer and methyl red reactions

c (Transport function of transferrin.)

99) Total iron binding capacity measures the serum iron transporting capacity of: a. hemoglobin b. ceruloplasmin c. transferrin d. ferritin

d (The overall incidence of the e antigen is 98%. The overall incidence of c is 80%, D is 85% and E is 30%.)

99) Which of the following Rh antigens has the highest frequency in Caucasians: a. D b. E c. c d. e

c (Waxy casts are seen with extreme stasis of urine flow, indicating they have remained in the tubules for an extended time. These aging casts are more retractile, and often contain notches and jagged edges as the result of granular disintegration.)

99) Which of the following casts most frequently appears to have a brittle consistency? a. hyaline b. granular c. waxy d. fatty


संबंधित स्टडी सेट्स

evaluacion conceptos basicos sobre atencion primaria en salud

View Set

Family, Culture, and Spirituality

View Set

Econ Final Study Guide, Chapter 20

View Set

Lesson 2 - Fotonovela- Identificar

View Set

Abeka 5th Grade Science Quiz 18 Study Guide

View Set